Anda di halaman 1dari 184

- membentuk kata benda

Rumus :
MORFOLOGI - 1 bukan kata kerja dan sifat

(e) Simulfiks (gabungan awalan dan akhiran


yang melekat bertahap)
I. AFIKS
me-kan, di-kan, per-kan, me-i, di-i, ber-kan,
Afiks adalah bentuk yang mampu melekat pada suatu
memper-kan, diper-kan, ber-an, dll.
kata.
Contoh :
(2) Berdasarkan Asal
me-, -el-, -an, ke-an, memperkan
(a) Afiks Asli
Afiks asli yakni afiks yang berasal dari bahasa
II. AFIKSASI
Indonesia.
Afiksasi adalah proses melekatnya afiks terhadap suatu
 me-, di-, ber-, per-, pe-, ter-, se
kata.
Contoh :  -i, -kan, -an
me- + makan = memakan (b) Afiks Asing
ke-an + adil = keadilan Afiks asing yakni afiks yang berasal dari
gembung + -el- = gelembung bahasa asing.
 Inggris : -if, -logi, -isme, isasi
Dalam afiksasi ada dua hal yang perlu diperhatikan  Belanda : -us, -or, -al
yaitu:  Arab : -i, -wi, -iah, -in, -at
(A) Jenis Afiks  Sansekerta : -wan, -man, -wati
Jenis afiks dapat dibedakan atas tiga dasar tinjauan (3) Berdasarkan Keproduktifan
yaitu: (a) Produktif
(1) Berdasarkan Posisi Afiks produktif adalah afiks yang sering
(a) Prefiks (awalan) dipakai. Jenis afiks produktif, antara lain :
me-, di-, ber-, per-, pe-, ke-, ter-, se-  Semua awalan kecuali ke-
Fungsi :  Semua akhiran
 me-, di-, ber- membentuk kata kerja  Semua konfiks
 per-, pe- membentuk kata benda  Semua simulfiks
 ter- membentuk kata kerja pasif dan kata  Prefiks asing -wan
sifat.
 ke- membentuk kata benda dan kata (b) Improduktif
bilangan. Afiks improduktif adalah afiks yang jarang
(b) Infiks (sisipan) dipakai. Jenis afiks improduktif, antara lain :
-el-, -em-, -er-  Awalan ke-
(c) Sufiks (akhiran)  Sisipan -el-, -em-, -er-
-i, kan-, -an  semua imbuhan asing kecuali –wan
Fungsi :
 -i, -kan membentuk kata kerja III. ALOMORF
 -an membentuk kata benda Alomorf adalah variasi morfem yang terjadi pada
(d) Konfiks (gabungan awalan dan akhiran awalan me(N)–, pe(N)–, per–, ber–.
yang melekat serentak)  ber–  be  ber + kerja  bekerja
ke-an, per-an, pe-an, se-nya, ber-an bel  ber + ajar  belajar
Fungsi :  per–  pe  per + kerja  pekerja
 per-an, pe-an membentuk kata benda pel  per + ajar  pelajar
 ber-an membentuk kata kerja  me(N)–  men– pada kata berawalan c,d,t,j
 se-nya membentuk kata sifat dan mem– pada kata berawalan b,p,f
keterangan meng– pada kata berawalan k,g,h,i,u,e,o
 ke-an mempunyai 3 fungsi yaitu : meny– pada kata berawalan s
- membentuk kata kerja menge – pada satu suku kata
Rumus :  pe(N)–  perubahannya sama dengan awalan
dapat di… me(N)–.

- membentuk kata sifat Aturan :


Rumus :  kata dasar yang diawali dengan huruf K,T,S,P jika
bertemu dengan awalan me- luluh/lebur
terlalu/menderita…
 Konsonan beriring tidak luluh
Contoh : C. Penyaringan air itu terbuat dari sabut kelapa dan
me- + produksi = memproduksi arang.
me- + kritik = mengkritik D. Pendapatan pengamen itu bisa seratus ribu per hari.
 Bentuk be- digunakan untuk kata dasar yang E. Pendidikan anak itu hanya sampai bangku SMP.
berawalan huruf r dan kata yang suku kata awalnya
berakhir dengan bunyi er 6. Makna imbuhan pada kata bergaris bawah dalam kalimat
Contoh : berenang, beternak “Ia melukai hati kami” sama dengan pada kata…
A. menggarami
B. membumbui
SOAL PENGANTAR C. menyusui
D. menulisi
1. Konstruksi berafiks di bawah ini yang bertransposisi E. melebarkan
menjadi kata kerja ialah…
A. lari – berlari 7. Di antara kata-kata turunan berikut yang tidak
B. lebar – melebarkan mengandung imbuhan jenis konfiks adalah …
C. remas – meremas A. kehidupan
D. lempar – melempari B. berguguran
E. hias – menghiasi C. perapian
D. perwujudan
2. Kata-kata berikut mengandung imbuhan yang E. diamankan
improduktif adalah…
A. bangsawan 8. Alomorf prefiks pe(N)– terdapat pada proses afiksasi
B. terangkat berikut, kecuali …
C. kedatangan A. pengebom
D. jemari B. pelari
E. mempertemukan C. penembak
D. pemburu
3. Pemakaian akhiran –wan yang benar terdapat pada E. penyerang
kalimat…
A. Para ilmuan sedang meneliti penyakit yang 9. Kata dasar yang menimbulkan nasalisasi /ny/ bila
mematikan itu. mendapat prefiks me(N)– atau pe(N)– adalah …
B. Dia adalah seorang tokoh sejarahwan di negeri ini. A. suruh
C. Gempa di Padang diteliti oleh para Geologiwan. B. daftar
D. Industriawan Indonesia sudah memakai teknologi C. ubah
canggih. D. junjung
E. Pria itu adalah seorang rohaniawan E. gali

4. Pengimbuhan me(N)-kan yang tepat terdapat dalam 10. Imbuhan yang diserap dari bahasa Arab terdapat pada
kalimat… deretan kata ….
A. Pak guru mencek kehadiran siswa di kelas. A. insani – jauhi – prakasa
B. Televisi itu mentayangkan pertandingan sepak bola B. rohaniah – duniawi – ilmiah
secara langsung. C. karyawan – karyawati – muslimat
C. Para terdakwa berusaha memputarbalikkan fakta D. alami – alamiah – hadiah
yang terjadi. E. muslimin – sastrawan – budiman
D. Mereka mengkaitkan krisis ekonomi dengan daya
beli masyarakat. 11. Kalimat–kalimat berikut ini mengandung kata berawalan
E. Kita jangan sampai salah menafsirkan kejadian itu se–. Awalan se– yang menyatakan makna satu terdapat
pada hal-hal yang negatif. pada kalimat …
A. Hargailah sesama manusia.
1. B. Jangan bertindak sekehendak hati saja.
2. C. Wanita setengah baya itu tampak masih sangat
3. bersemangat.
4. D. Sepulang sekolah ia terus pergi ke rumah kakeknya.
5. Pembangunan rumah mewah itu tidak cukup enam E. Sepandai–pandainya tupai melompat akhirnya jatuh
bulan. Imbuhan pe–an yang semakna dengan imbuhan juga.
pe–an pada kata pembangunan tersebut adalah…
A. Karya tulis ilmiah harus ditulis berdasarkan 12. Akhiran –i pada kata mengelilingi dalam kalimat “Setiap
penelitian. minggu pagi saya mengelilingi Masjid Agung dengan
B. Penangkapan penjahat itu dilakukan dengan berjalan kaki” mempunyai makna yang sama dengan
kekerasan.
akhiran –i pada kata yang bercetak miring pada 17. Ketika ayahnya dating, bukunya masih berserakan di
kalimat… lantai.
A. Seorang ibu yang bijak harus menyusui anaknya. Makna imbuhan ber-an pada kata berserakan dalam
B. Polisi memasuki sarang penyamun dengan bersenjata kalimat ini memiliki makna yang sama dengan makna
api. imbuhan ber-an pada kata bercetak miring dalam
C. Sering kita temui pelajar yang tidak menghormati kalimat…
gurunya. A. Kegiatan gelar seni yang akan datang
D. Iwan pantang menyakiti hati kekasihnya. diselenggarakan bersamaan dengan kegiatan bulan
E. Ibu menggulai teh untuk tamu. bahasa.
B. Karena sama-sama kurang memperhatikan rambu
13. Kata berawalan me- yang tidak menyatakan kata kerja lalu lintas, kedua mobil tersebut bertabrakan.
terdapat dalam kalimat… C. Karena sudah lama tidak turun hujan, taman kota
A. Dia tidak mengakui perbuatannya yang salah. banyak yang gersang karena daun dan bunga
B. Penduduk desa itu banyak yang merotan. berguguran.
C. Para penerjun telah mendarat dengan selamat. D. Walaupun berpenghasilan tinggi, ia tetap saja
D. Kami mengontrak rumah di Pulomas. berpenampilan seperti orang yang tidak punya.
E. Kami dating menjelang pesta dimulai. E. Kedua kelompok itu tidak pernah saling mengerti.
Oleh karena itu, mereka terus bermusuhan.
14. Lokasinya sangat kurang menguntungkan, tetapi
keindahannya sangat memukau. 18. Bentuk kata yang dicetak miring dalam kalimat-kalimat
Arti ke-an pada kata keindahan dalam kalimat di atas di bawah ini baku, kecuali…
terdapat juga pada kata yang bercetak miring dalam A. Dukungan masyarakat sangat diperlukan untuk
kalimat… menyukseskan program itu.
A. Masalah kegunungapian merupakan suatu ilmu B. Acara itu sangat menarik dikarenakan dipersiapkan
tersendiri. dengan baik.
B. Tidak menutup kemungkinan kalau areal itu juga C. Banyak permukiman penduduk yang rusak akibat
akan dikelola menjadi objek wisata. gempa.
C. Untuk memecahkan masalah itu diperlukan kearifan D. Pemukiman para korban bencana alam berlangsung
yang tinggi. sangat cepat.
D. Kehadirannya sangat diharapkan. E. Indonesia sudah dapat memproduksi barang-barang
E. Keberhasilannya mengelola lembaga pendidikan itu yang bermutu tinggi.
menjadi pertimbangan utama.
19. Pada hari pertama sebelum ujian dimulai, semua peserta
15. Proses pembentukan kata berkedudukan dalam kalimat tes latihan mengisi lembar jawaban ujian. Setiap peserta
Kantor Pusat kami berkedudukan di Palembang pada ujian mendapatkan satu set soal dan lembar jawaban
duduk > kedudukan > berkedudukan. untuk berlatih. Dalam pengisian lembar tersebut, mereka
Proses yang sama terdapat pula pada bentukan kata dipandu oleh pengawas ujian. Perlatihan ini
dalam kalimat berikut, kecuali… dimaksudkan agar mereka dapat mengerjakan dengan
A. Mereka berkesempatan menginap selama dua malam. lancar tanpa mengalami banyak kesalahan.
B. Bencana berkepanjangan menimpa daerah mereka. Dalam kutipan tersebut, terdapat bentukan kata yang
C. Polisi berkesimpulan bahwa mereka memang salah, yakni…
bersalah. A. Latihan-mendapatkan-mengerjakan
D. Program Melati berkesinambungan dengan program B. Mendapatkan-berlatih-perlatihan
sebelumnya. C. Latihan-berlatih-perlatihan
E. Setiap warga berkewajiban membersihkan D. Latihan-berlatih-mengerjakan
lingkungan masing-masing. E. Berlatih-perlatihan-mengerjakan

16. Kata berimbuhan (bercetak miring) digunakan secara 20. Ulama itu mempertemukan Hasan dan Azizah.
tepat dalam kalimat… Proses pembentukan kata mempertemukan dalam kalimat
A. Mandor di pabrik itu membawahi sepuluh orang kuli tersebut sejalan dengan proses pembentukan kata
yang rajin-rajin. berimbuhan memper-kan dalam kalimat…
B. Kepala sekolah menugasi para guru untuk A. Patih Gadjah Mada berupaya mempersatukan
menghadiri rapat. kerajaan-kerajaan Nusantara.
C. Anita telah mengirimi surat kepada bapaknya. B. Atasan Anda tidak mempersalahkan Anda atas
D. Tujuan kegiatan ini adalah mengolahragakan para peristiwa yang terjadi kemarin.
karyawan. C. Mengapa Andi selalu mempertanyakan nilai-nilai
E. Tini pergi ke dokter untuk mencabutkan giginya yang kuliahnya?
sakit. D. Saya mohon Anda tidak mempersamakan saya
dengan dia.
E. Wartawan memperjuangkan orang yang tertindas D M
dengan kata-kata. kursi malas
D M

 M–D
sepak bola
M D

beberapa contoh
M D
MORFOLOGI - 2
(2) Eksosentris/Kopulatif
Kata majemuk eksosentris atau kopulatif adalah
I. KATA MAJEMUK kata majemuk yang mempunyai dua inti.
A. Pengertian Contoh:
Kata majemuk adalah gabungan dua kata atau lebih hancur lebur
yang membentuk satu arti baru. D D
Contoh: sunyi sepi
rumah makan D D
rumah sakit umum merah putih
kereta api D D

B. Ciri – Ciri Kata Majemuk : D. Kata Majemuk Idiomatis


 Gabungan 2 kata/lebih Kata majemuk idiomatis adalah kata majemuk
 Membentuk satu arti baru bermakna kiasan.
 Umumnya berupa kata dasar Contoh:
 Strukturnya tetap panjang tangan = suka mencuri
 Strukturnya padu artinya tidak dapat disisipi kata sempit hati = cepat tersinggung
tugas yang serta dan. buah hati = anak kesayangan
lapang dada= sabar
Catatan : naik daun = terkenal/populer
Ada beberapa gabungan kata yang dapat disisipi oleh
kedua kata tugas di atas, tetapi apabila terbentuk arti Catatan :
(1) Bahasa Indonesia umumnya mengikuti hukum D –
baru gabungan kata tersebut termasuk kata majemuk.
Misalnya : M.
(2) Kata majemuk idiomatis umumnya mengikuti
(1) Orang tua dapat disisipi kata yang, tetapi bila
artinya orang yang tua memang bukan kata hukum M – D.
(3) Menentukan inti kata majemuk dapat dilakukan
majemuk. Orang tua berbentuk kata majemuk bila
artinya menyatakan bapak/ibu. dengan memasukkan kata majemuk tersebut ke
dalam kalimat dan kata yang paling dapat mewakili
Bedakan dalam kalimat berikut :
itulah sebagai inti.
 Saya seberangkan orang tua itu. (orang yang
Contoh:
tua)
kursi malas
 Orang tua saya sedang tidak berada di rumah.
D M
(bapak/ibu)
Ibu membeli kursi malas
(2) Siang malam dapat disisipkan dan, tetapi bila
Ibu membeli kursi bukan membeli malas. Jadi
artinya siang dan malam bukan kata majemuk.
intinya adalah kursi
Kata siang malam berbentuk kata majemuk bila
bermakna 24 jam.
sepak bola
Bedakan dalam kalimat berikut :
M D
 Ayah bekerja siang malam. (siang dan malam) Mereka bermain sepak bola
 Apotek itu buka siang malam. (24 jam) Mereka bermain bola bukan bermain sepak. Jadi
intinya adalah bola
C. Jenis Kata Majemuk
(1) Endosentris/Tatpurusa E. Kata Majemuk Sepola
Kata majemuk endosentris atau tatpurusa adalah Untuk menentukan kata majemuk sepola dapat dilihat
kata majemuk yang mempunyai satu inti. dengan cara:
Contoh: (1) Jenis sama
 D–M (2) Struktur sama
kereta api (3) Kelas kata sama
Contoh: mobil mobil-mobilan
jaksa agung = rumah sakit buah buah-buahan
D(KB) M(KS) D(KB) M(KS) kuda kuda-kudaan
merah kemerah-merahan
kamar mandi = kapal terbang
D(KB) M(KK) D(KB) M(KK) (5) Kata Ulang Sebagian
Kata ulang sebagian adalah kata ulang yang bentuk
II. KATA ULANG dasarnya diulang hanya sebagian.
Kata ulang/reduplikasi adalah kata yang mengalami Contoh:
proses perulangan, baik sebagian ataupun seluruhnya Bentuk dasar Bentuk
disertai dengan perubahan bunyi ataupun tidak. perulangan
berjalan berjalan-jalan
A. Jenis Kata Ulang berkata berkata-kata
(1) Perulangan Seluruh Bentuk Dasar bermain bermain-main
Perulangan ini disebut juga perulangan utuh atau memukul pukul-memukul
dwilingga. Perulangan utuh terdiri atas dua macam.
Pertama, perulangan terhadap kata dasar. Kedua,
B. Nosi/Makna Kata Ulang
perulangan terhadap kata berimbuhan.
(1) Menyatakan meskipun
Contoh:
Contoh: panas-panas, hujan-hujan
Bentuk Bentuk perulangan
dasar (2) Menyatakan menyerupai
orang orang-orang Contoh: mobil-mobilan, orang-orangan
baik baik-baik
kejadian kejadian-kejadian (3) Menyatakan hal
malam malam-malam Contoh: karang-mengarang,
pelajar pelajar-pelajar jahit-menjahit

(2) Dwipurwa (4) Menyatakan agak


Dwipurwa adalah perulangan yang terjadi pada Contoh: kemalu-maluan, kemerah-merahan
suku awal.
Contoh: (5) Menyatakan bermacam/berjenis
Bentuk Bentuk Contoh: tanam-tanaman, sayur-mayur
dasar perulangan
daun dedaunan (6) Superlatif/menyatakan paling
luhur leluhur Contoh: sebaik-baiknya, setinggi-tingginya
pohon pepohonan
saji sesaji (7) Menyatakan melakukan kerja dengan santai
tamu tetamu Contoh: makan-makan, duduk-duduk

(3) Dwilingga Salin Suara (8) Menyatakan resiprok/saling


Dwilingga salin suara adalah bentuk perulangan Contoh: tarik-menarik, pukul-memukul
yang disertai perubahan bunyi.
Contoh: (9) Intensitas/menegaskan arti
(a) Kualitatif = menguatkan sifat
Bentuk dasar Bentuk
Contoh: baik-baik, kuat-kuat
perulangan
(b) Kuantitatif = menyatakan banyak
lauk lauk-pauk Contoh: orang-orang, tamu-tamu
sayur sayur-mayur (c) Frekuentatif = menyatakan sering/
warna warna-warni berulang-ulang
serba serba-serbi Contoh: memukul-mukul, menetes-netes
gerak gerak-gerik
Catatan:
(4) Kata Ulang Berimbuhan Kata ulang pleonasme yaitu pemakaian kata ulang
Kata ulang berimbuhan adalah perulangan yang yang tidak tepat.
bentuk dasarnya diulang dan disertai dengan proses Contoh:
pengimbuhan. (a) Banyak tamu-tamu mulai berdatangan.
Contoh: (Pemakaian banyak tamu-tamu tidak tepat sebab
Bentuk Bentuk perulangan tamu-tamu sudah menyatakan banyak)
dasar (b) Kedua anak itu saling pukul-memukul.
(Pemakaian saling pukul-memukul tidak tepat sebab C. Serah terima
pukul-memukul sudah menyatakan saling). D. Cuci tangan
E. Sepak terjang

7. Kalimat yang menggunakan kata ulang adalah…


A. Dedaunan seakan berbisik mengingatkanku.
B. Badannya besar tetapi dia kekanak-kanakan.
SOAL PENGANTAR C. Semerbak harum bunga memanjakan kupu-kupu.
D. Masalah utang piutang almarhum sudah menjadi
1. Pasangan kata merupakan kata ulang adalah… tanggung jawab ahli waris.
A. Putra-putri E. Kuda-kudanya bagus sehingga ia tidak mudah
B. Saudara-saudari dijatuhkan musuh.
C. Agar-agar
D. Kenang-kenangan 8. Penggunaan kata-ulang yang tidak tepat terdapat pada
E. Kunang-kunang kalimat, kecuali…
A. Sejak SMA mereka saling cinta-mencintai.
2. Penulisan kalimat-kalimat di bawah ini tidak baku B. Pipinya agak kemerah-merahan saat pertama sekali
karena pemakaian kata ulang yang salah adalah… kuucapkan cinta.
A. Setiap orang tua pasti ingin memberikan C. Anak itu selalu menggeleng-gelengkan kepalanya
pendidikan terbaik untuk anak-anaknya. bila ditanya.
B. Relawan itu bahu-membahu mengevakuasi korban D. Para atlet-atlet Sea Games dipusatkan di Gedung
pesawat jet 100 sukhoi. serbaguna itu.
C. Menjelang lebaran harga lauk-pauk dan berbagai E. Kakekku sering terbatuk-batuk di malam hari.
sayur-mayur naik dari harga biasanya.
D. Ibu Karina mondar-mandir mencari anaknya yang 9. Makna kata ulang yang sama terdapat dalam kalimat-
terlambat pulang dari sekolah. kalimat di bawah ini, kecuali…
E. Ia belajar setekun-tekunnya untuk meraih A. Kedua pembalap kejar-mengejar di lintasan berliku.
impiannya. B. Pak Ruki bekerja di bagian surat-menyurat.
C. Anak-anak bekerjar-kejaran di bibir pantai.
3. Makna gabungan kata merdeka berdaulat sepola D. Masyarakat tolong-menolong dalam menghadapi
dengan gabungan kata… bencana.
A. Mereka mengobrol sambil sesekali bersenda gurau. E. Intip-mengintip kekuatan dilakukan kedua
B. Kami merasa senasib-sepenanggungan. kandidat.
C. Orang itu malu-malu kucing ketika dipersilahkan
makan oleh tuan rumah. 10. …. semangat berkoprasi merupakan salah satu cara
D. Dia menjadi hilang semangat setelah mengetahui efektif untuk ….. rentenir. Apalagi bila semangat itu
pembalap unggulannya kalah. …. oleh perbankan, misalnya dengan memberikan
E. Pelari itu bersiap-siap sambil pasang kuda-kuda. kredit …. tanpa …. dengan proses …. Yang mudah
dan cepat.
4. Kata-kata di bawah ini adalah gabungan kata yang Kata-kata yang tepat untuk mengisi bagian yang
benar penulisannya, kecuali… rumpang di atas adalah…
A. Simpang empat, sumber daya, mata kuliah A. Tumbuh kembangnya, memerangi, ditanggapi,
B. Olah raga, sapu tangan, kaca mata murah, jaminan, pencairan.
C. Matahari, halalbihalal, bumiputra B. Menumbuhkembangkan, menghapus, didukung,
D. Ekstrakulikuler, dasawarsa, acapkali lunak, agunan, pencairan.
E. Buku catatan, tembok pemisah, buku harian C. Membudidayakan, memusuhi, ditopang, besar,
agunan, perealisasian.
5. Kalimat di bawah ini di dalamnya tidak terdapat kata D. Membudidayakan, menghentikan, terdukung,
ulang berimbuhan adalah… lunak, agunan, administrasi.
A. Orang-orangan di sawah itu mengejutkanku. E. Menumbuh dan mengembangkan, menghapus,
B. Di hari besar kami sekeluarga bersalam-salaman. disambut, cepat, birokrasi, pengurusan.
C. Di sekolah kami mempelajari karang-mengarang.
D. Sidikalang penghasil buah-buahan di Sumut. 11. Akibat erupsi Merapi, seluruh anggota keluarganya
E. Gadis itu kemalu-maluan memandangku. tidur berhari-hari di pengungsian.
Bentuk ulang yang bermakna sama dengan bentuk
6. Aksi jual beli saham di bursa efek Jakarta kembali ulang yang terdapat pada kalimat tersebut adalah…
bergairah. Pasangan kata di bawah ini yang setipe A. Meskipun telah dibacanya berhalaman-halaman, dia
dengan jual beli adalah… belum menemukan juga apa yang dicarinya.
A. Pantang mundur B. Kakek terkekeh-kekeh mendengar ceritaku.
B. Terus terang
C. Ketika kami beristirahat di lembah itu, terdengar
suara burung bersahut-sahutan. 16. Kalimat berikut semuanya tidak mengandung kata
D. Setelah bersalam-salaman kami makan bersama majemuk, kecuali…
sambil melanjutkan obrolan. A. Dua hari yang lalu ibu membeli sepeda baru untuk
E. Sebelum mengajar, mereka terlebih dahulu adik.
mengikuti diklat selama dua bulan berturut-turut. B. Dia mondar mandir di kota tersebut karena tidak
memperoleh pekerjaan.
C. Meja hijau di sudut ruangan tersebut sudah di cat
12. Di antara lima kalimat berikut, terdapat kata ulang karena warnanya sudah usang.
yang menyatakan jumlah jamak yang dipakai secara D. Ketika matahari terbit ayah buru-buru bangun
tepat, yaitu… karena akan segera berangkat.
A. Banyak ruko-ruko bermunculan menyertai E. Susi sedang membaca buku baru yang dibelinya
kemudahan perizinan. dari toko kemarin.
B. Akibatnya, tidak ada keseragaman pemahaman
antar-pihak terkait dengan pelaksana program. 17. Gabungan kata di bawah ini yang sepola dengan
C. Butir-butir dalam notula rapat perlu dikelompokkan gabungan kata pucat pasi adalah…
berdasarkan topic bahasan. A. Biru terong
D. Permasalahan-permasalahan demokratisasi di B. Kuning telur
tingkat bawah belum diikuti dengan pengelolaan C. Terang benderang
pemerintah lokal yang lebih baik. D. Perdana menteri
E. Tulisan ini mengulas pelbagai pandangan- E. Sepeda baru
pandangan teoritis mengenai prinsip kerja sama
dalam komunikasi. 18. Berhjari-hari lamanyaia berjalan mondar-mandir keluar
masuk kantor. Dilihatnya banyak orang di depan loket.
13. Kata ulang yang mempunyai makna sama dengan kata Ada yang sedang bercakap-cakap, tetapi tidak seorang
ulang pernak-pernik adalah… pun yang tertawa-tawa, tanya-menanya dan pandang-
A. Kupu-kupu memandang.
B. Lintang-pukang Kata ulang yang menyatakan makna resiprok dalam
C. Bolang-baling paragraf di atas adalah…
D. Awan-gemawan A. Berhari-hari, mondar-mandir, bercakap-cakap
E. Pucat-pasi B. Bercaka-cakap, tanya menanya, pandang-
memandang
14. Makna gabungan kata merdeka berdaulat dapat juga C. Berhari-hari, bercakap-cakap
dilihat pada gabungan kata… D. Tertawa-tawa, tanya menanya
A. Mereka mengobrol sambil sesekali bersenda gurau. E. Tanya-menanya, berhari-hari
B. Kami merasa senasib sepenangungan.
C. Orang itu malu-malu kucing ketika dipersilakan 19. Pengunaan kata ulang dalam kalimat berikut yang
makan oleh tuan rumah. menunjukkan intensitas adalah…
D. Dia menjadi hilang semangat setelah mengetahui A. Janganlah sekali-sekali kamu datang ke rumahku.
pembalap unggulannya kalah. B. Di sana anak-anak bermain-main sepuas hatinya.
E. Pelari itu bersiap-siap sambil pasang kuda-kuda. C. Sesama warga kita harus selalu hormat-
menghormati.
15. Pengunaan kata majemuk dapat ditemukan dalam D. Sesama bus kota dilarang dahulu-mendahului.
kalimat berikut, kecuali… E. “Tenang, sebentar lagi mereka dating!” katanya
A. Akibat jatuh dari sepeda motor, kaki tangan agus sambil mengetuk-ngetukkan tangannya pada meja.
menderita patah tulang dan terpaksa harus dirawat
beberapa minggu di rumah sakit. 20. Gabungan kata di bawah ini yang sepola dengan
B. Masyarakat Desa Makmur Sari mencurigai sepak konstruksi rumah sakit adalah…
terjangnya karena ia memiliki kemiripan wajah A. kursi malas
dengan penjahat yang menjadi buronan polisi. B. kepala batu
C. Untuk memperoleh kredit perumahan rakyat, para C. patah hati
konsumen harus membayar uang muka sebesar 25 D. siang malam
persen dari harga tunai. E. ruang tamu
D. Setelah musim tanam usai, kebanyakan para petani
yang berasal dari desa pergi ke kota untuk mencari
pekerjaan sambilan, seperti tukang angkut barang,
penarik becak, dan sebagainya.
E. Warga Desa Kayu Wangi yang tanah
pekarangannya terkena pelebaran jalan memperoleh
ganti rugi dari pemerintah.
(b) Kekayaannya bernilai miliaran rupiah.
(c) Anak itu kedapatan merokok.

(4) Objek pelaku (O2)


Objek pelaku dapat dijumpai dalam kalimat pasif
umumnya predikat berawalan di- atau ter-
Contoh:
Adik dicubit kakak.
S P O2

SINTAKSIS-1 Adik tertimpa tangga.


S P O2
(5) Objek penyerta/tak langsung (O3)
I. SINTAKSIS Objek penyerta dalam sebuah kalimat harus
Sintaksis adalah ilmu yang mempelajari tentang memiliki pasangan. Apabila bentuk kalimat
kalimat. tersebut termasuk kalimat aktif maka yang disertai
(O3) adalah (O1) sedangkan apabila kalimat tersebut
II. JABATAN KALIMAT termasuk kalimat pasif maka yang disertai (O3)
Jabatan kalimat dalam bahasa Indonesia dapat adalah (O2).
dibedakan atas 6 jabatan pokok yaitu :
(1) Subjek Contoh:
Subjek adalah unsur pokok yang terdapat pada Ayah membeli sepeda untuk adik
sebuah kalimat, di samping unsur predikat. S P O1 O3
Dalam pola kalimat bahasa Indonesia, subjek (6) Keterangan
biasanya terletak sebelum predikat, kecuali jenis Keterangan adalah unsur fungsi yang menerangkan
kalimat inversi. Subjek umumnya berwujud seluruh fungsi yang ada dalam suatu kalimat.
nomina. Perhatikan contoh kalimat di bawah ini Ciri-ciri fungsi keterangan
 Peserta audisi itu puluhan ribu orang.  kehadirannya bersifat mana suka
 Dia datang dari Bogor  letaknya bebas
 Agnes Monica adalah seorang penyanyi  umumnya didahului oleh kata depan, seperti di,
terkenal dari, ke, ketika, jika, meskipun, dll.
 Pak Aldy pergi ke Malaysia.
Jenis keterangan dalam bahasa Indonesia cukup
(2) Predikat banyak antara lain:
Predikat juga merupakan unsur utama suatu kalimat (a) Keterangan tempat
di samping subjek yang merupakan inti dari Contoh:
kalimat. Unsur pengisi predikat suatu kalimat dapat Di sana telah terjadi kecelakaan sepeda motor.
berupa Kata, misalnya verba, adjektiva, atau (b) Keterangan waktu
nominal, numeral dan preposisional. Selain itu Contoh:
dapat pula berupa Frasa, misalnya frasa verbal, Kemarin kami sedang ujian matematika di
frasa adjektival, frasa nominal, frasa numeralia sekolah.
(bilangan). Perhatikan beberapa contoh kalimat di (c) Keterangan alat
bawah ini: Contoh:
Adik menjolok mangga dengan galah.
 Qiqi belajar di kamar. (d) Keterangan cara
 Ibu memasak tumis kangkung Contoh:
 Aldy sedang membaca Koran. Ayah memanggil kakak dengan suara keras.
(e) Keterangan tujuan
Contoh:
(3) Objek penderita/objek langsung (O1)
Pak guru membeli kapur untuk keperluan
Objek penderita dapat di jumpai dalam kalimat
sekolah.
aktif yang umumnya predikat berawalan me-
(f) Keterangan sebab akibat
Contoh:
Contoh:
(a) Setiap kios menjajakan barang dagangannya.
Anton tidak sekolah karena sakit.
(b) Natalia sedang membaca majalah.
(c) Semua warga melakukan gotong royong di
Kalimat Sepola
desa.
Cara menentukan kalimat yang sepola adalah:
Contoh kalimat yang berpelengkap:  Susunan jabatan kalimat sama
(a) Rumah pejabat itu berjumlah lima belas buah.  Jenis kata predikat sama
 Tentukan aktif atau pasif

Contoh:
Kami akan membicarakan hal itu besok.
S P O K

Kalimat di atas sepola dengan:


Ayah memukul batu dengan palu SOAL PENGANTAR
S P O K
1. Pasal 128 Undang-Undang Nomor 35 Tahun 2009
III. MAKNA STRUKTURAL KALIMAT tentang Narkotika menyebutkan, pecandu narkotik yang
Makna struktural kalimat berfungsi untuk mengubah telah cukup umur dan tengah menjalani rehabilitasi
makna gramatikal suatu kalimat yang meliputi : medis di rumah sakit dan/atau lembaga rehabilitasi
medis yang ditunjuk pemerintah tidak dituntut pidana.
UBIK Kalimat yang memiliki pola kalimat yang sama dengan
(1) Urutan Kata kalimat di atas adalah ....
Contoh: A. Dari sekian banyak kabar penggerebekan kelas teri
Harimau mengejar pemburu. itu, untunglah terbetik kabar pembongkaran
1 2 3 sindikat sabu internasional oleh BNN.
Pemburu mengejar Harimau. B. Lembaga ini menangkap Wong Ching Ping,
3 2 1 penyelundup 800 kilogram sabu-sabu lewat
Kepulauan Seribu, DKI Jakarta.
(2) Bentuk Kata C. Memenjarakan para pengguna narkoba bukanlah
Contoh: pilihan terbaik karena di penjara justru kian
Andi mencium Ani terbuka pergaulan para korban itu dengan
produsen dan pengedar yang ada di sana.
Andi dicium Ani D. Menteri Koordinator Perekonomian mengatakan,
pemerintah akan menaikkan angka penyertaan
(3) Intonasi modal negara (PMN) ke BUMN.
Intonasi meliputi tekanan dan nada. E. Merehabilitasi korban merupakan salah satu cara
Contoh: memutus mata rantai pasar obat-obatan terkutuk
(a) Para siswa baru/memakai seragam putih biru. itu.
(b) Para siswa/baru memakai seragam putih biru.
(4) Kata Tugas 2. (1) Ketika pertama kali digunakan, uang emas dan uang
Contoh: perak dinilai berdasarkan nilai intrinsiknya, yaitu kadar
Tina pergi dari Bandung dan berat logam yang terkandung di dalamnya.(2)____
Tina pergi ke Bandung (3) Namun, saat ini, uang logam tidak dinilai dari berat
emasnya, namun dari nilai nominalnya.
IV. KLAUSA Kalimat (2) dapat diisi dengan kalimat ...
Klausa adalah satuan gramatikal predikatif yang baik A. Kedua logam itu memiliki nilai yang cenderung
diikuti oleh S – O – K. Klausa identik dengan predikat, tinggi dan stabil.
dimana jumlah klausa suatu kalimat ditentukan B. Emas dan perak dinilai berdasarkan beratnya.
berdasarkan jumlah predikatnya. Lebih mudahnya ingat C. Semakin besar kandungan emas atau perak di
saja: dalamnya, semakin tinggi nilainya.
D. Kedua logam itu juga tidak mudah hancur dan
tahan lama.
Klausa = Predikat E. Emas dan perak dapat dibagi menjadi satuan yang
Contoh: lebih kecil tanpa mengurangi nilai.
(a) Adik menangis tersedu-sedu.
S P 3. Kalimat yang polanya tidak sama dengan kalimat yang
Jumlah klausanya = satu klausa lain adalah ....
(b) Kami akan pergi saat A. banyak buku dan tulisan mampu mengubah nasib
S P sebuah bangsa
matahari tenggelam. B. kegiatan membaca akan dapat membangun
S P karakter masyarakat
Jumlah klausanya = dua klausa C. setiap pelajar diharuskan menghasilkan sebuah
tulisan pendek
D. membaca tidak cukup dijadikan sebagai ajakan
atau imbauan
E. mereka pasti akan menemukan buku yang tepat
E. hormat
4. Dalam Konferensi Internasional Kebudayaan Papua itu
tidak diputuskan t empat penyelenggaraan konferensi 10. Kuserahkan segala hidupku ini untukmu.
berikutnya. Kalimat di atas berpola…
Kalimat yang memiliki pola yang sama dengan kalimat A. O2 – P – O3 – S
di atas adalah .... B. S – P – O3 – O2
A. Untung Subagyo, pejabat yang terpilih itu, C. O2 - S - P – O3
mengawali kariernya dari bawah D. O2 – P – S – O3
B. seperti telah kita ketahui, perekonomian Amerika E. P – S – O2 – O3
Serikat sangat berpengaruh
C. telah kami bicarakan dalam rapat terdahulu bahwa 11. Sebagian besar rakyat Indonesia masih menginginkan
kenaikan gaji pegawai baru bisa dilaksanakan tahun negara kesatuan.
depan Kalimat berikut yang polanya sama dengan kalimat di
D. kita sadari bahwa bencana alam dapat datang kapan atas adalah…
saja tanpa kita duga A. Para pemimpin negara harus mengayomi
E. berdasarkan pengarahan pimpinan, pembangunan masyarakat.
kampus baru itu dapat dilakukan secara bertahap B. Di setiap daerah akan diberikan otonomi seluas-
luasnya.
5. Kata murah-murah yang berfungsi sebagai predikat C. Perselisihan antarwarga sudah diselesaikan
terdapat dalam kalimat … kemarin.
A. Paman saya menjual yang murah-murah. D. Bahwa kekurangan gizi anak itu urusan
B. Barang murah-murah dibeli paman. pemerintah.
C. Barang dagangannya murah-murah. E. Sebagian anak-anak senang bermain.
D. Dia menjual barang yang murah-murah.
E. Yang murah-murah saja dibelinya. 12. Dalam kunjungannya ke panti jompo, diperolehnya
. informasi yang sangat berharga.
6. Berikut ini adalah alat kalimat yang dapat membedakan Pola urutan unsur fungsi dalam kalimat di atas terdapat
makna kalimat, kecuali … juga dalam kalimat…
A. Urutan kata A. Ketika mengerjakan tugas-tugas, buku-buku paket
B. Bentuk kata sangat diperlukan.
C. Kata tugas B. Sambil mengerjakan lagu-lagu, Sari menyelesaikan
D. Intonasi pekerjaan rumahnya.
E. Jumlah kata C. Meskipun sudah ditegur oleh gurunya, tetap saja
dia mengganggu temannya.
7. Kakak membelikan ayah kami sepeda baru. Kata D. Bagi yang suka sastra tersedia buku-buku yang
bergaris bawah pada kalimat di atas memiliki fungsi bermutu.
yang sama dengan kata paman pada kalimat berikut E. Dalam menyampaikan laporannya, Murni
ini… menggunakan bahasa Inggris.
A. Paman menjual mobil baru.
B. Dia adalah pamanku. 13. Kalimat di bawah ini yang menyatakan bahwa bantuan
C. Buku itu dibeli paman. untuk para korban baru datang adalah…
D. Kami sangat mencintai paman. A. Ban/ tuan baru/ datang tadi pagi.
E. Ibu mengirimkan paman surat. B. Ban/ tuan/ datang tadi pagi.
C. Bantuan baru/ datang tadi pagi.
8. Kalimat-kalimat berikut ini berpredikat kata kerja, D. Bantuan/ baru/ datang tadi pagi.
kecuali… E. Bantuan/ baru datang/ tadi pagi.
A. Yang sudah dibeli tidak boleh dikembalikan.
B. Tersenyum dia mendengar berita itu. 14. Bukunya ditulis dan diterbitkan di Amerika .
C. Pemuda yang menyampaikan berita itu temanku. Predikat kalimat di atas adalah…
D. Yang kuat harus menolong yang lemah. A. ditulis
E. Bekerja memerlukan ketekunan. B. diterbitkan
C. diterbitkan di Amerika
9. Subjek kalimat Dalam hal serupa ini saya lebih hormat D. ditulis dan diterbitkan
lagi kepada orang kampung yang terus terang E. ditulis dan diterbitkan di Amerika
memegang upacara yang dianggap bersangkutan
dengan agama adalah… 15. Pemakaian intonasi yang menekankan pelaku terdapat
A. orang kampung dalam kalimat…
B. saya A. Julia mengirimi ibunya surat kemarin.
C. agama B. Julia mengirimi ibunya surat kemarin.
D. upacara C. Julia mengirimi ibunya surat kemarin.
D. Julia mengirimi ibunya surat kemarin.
E. Julia mengirimi ibunya surat kemarin.

16. Kalimat berikut yang termasuk kalimat berpelengkap


adalah…
A. Generasi muda harus menguasai komputer dan
bahasa Inggris.
B. Sumber daya manusia menjadi modal
pembangunan.
C. Penghuni asrama itu mengadakan kerja bakti
kemarin.
D. Manajemen pemasaran harus dikuasai oleh pelaku
bisnis.
E. Pengelola Bank harus membangun kepercayaan
pada nasabah.

17. Kalimat-kalimat yang mempunyai pola S-P adalah…


A. Bacaan populer masih dianaktirikan.
B. Hadiah Nobel Sastra tahun 1994 dimenangkan oleh
sastrawan Jepang.
C. Pemerintah Jepang mengecam pernyataan
pemerintah Libya belum lama ini.
D. Tahun ini merupakan tahun ke-50 kemerdekaan
Indonesia.
E. Gesang sangat terkenal di Jepang.

18. Para ahli memperkirakan bahwa hari ini berawan.


Berdasarkan unsur fungsinya, kalimat di atas berpola
sama dengan kalimat…
A. Paman menyuruh saya beristirahat.
B. Kami telah memperbaiki sepeda yang rusak.
C. Bibi menghadiahi Julia kado ulang tahun.
D. Hal itu akan menjadi tanggung jawab kami.
E. Kegiatan ini merupakan program akhir tahun.

19. Ibunya menjahit ketika anaknya belajar di kamar.


Kalimat yang berpola sama dengan kalimat di atas
adalah…
A. Ayahnya datang ketika ibunya ke pasar.
B. Dokter itu memberitahukan bahwa pasiennya
berobat jalan.
C. Dokter itu menceritakan bahwa pasiennya sedang
sakit keras.
D. Ibu anak itu berdagang saat suaminya belajar ke
luar negri.
E. Murid itu berkata bahwa temannya sedang makan
di kantin.

20. Kalimat berpelengkap terdapat dalam kalimat…


A. Kami menyudahi pembicaraan setelah lama
berdebat.
B. Pohon duren di depan rumahku banyak buahnya.
C. Sepasang kekasih berpegangan tangan dengan
mesra.
D. Silakan Saudara duduk di kursi nomor dua dari
depan.
E. Stasiun televisi swasta menayangkan iklan promosi
dan layanan masyarakat.
Frase ini disebut bertingkat karena ada
unsur pokok/inti yaitu D dan ada
penjelas/atribut yaitu M.
Contoh:
ikan hias
D M
sepeda baru
D M
SINTAKSIS - 1 Buku catatan = frase atributif berimbuhan
b. Frase Koordinatif/Frase Setara
Frase koordinatif adalah frase yang unsur
I. FRASE pembentuknya semua merupakan pokok
Frase adalah gabungan dua kata/lebih yang bukan frase. Frase ini disebut juga frase setara.
merupakan rangkaian subjek – predikat – objek – Cara mengenalinya selalu dapat disisipkan
keterangan, tetapi mempunyai potensi menduduki fungsi kata dan serta atau.
jabatan kalimat tersebut. Untuk lebih mudahnya dapat Contoh:
disebut bahwa frase adalah gabungan kata/kelompok suami istri
kata yang dapat menduduki satu jabatan kalimat. D D
Contoh: tua muda
Aparat keamanan sedang menangkap D D
S P ayah ibu
kepala penjahat itu D D
O1 c. Frase Apositif
di sebuah pertokoan Frase apositif adalah frase yang berfungsi
K sebagai penjelas jabatan subjek.
Kalimat di atas terdiri atas 4 frase, yakni: Contoh:
aparat keamanan (1 frase) Jakarta, Ibu kota Republik Indonesia,
sedang menangkap (1 frase) S
kepala penjahat itu (1 frase) diguncang bom.
di sebuah pertokoan (1 frase) P O2
Ibu kota Republik Indonesia (frase apositif)
II. INTI FRASE
Inti frase merupakan pokok dari sebuah frase. Inti frase Orang tua yang memiliki anak gadis
dilambangkan dengan (D) yang dapat ditentukan S
dengan cara memasukkannya dalam kalimat lalu ambil hendaknya lebih waspada.
1 kata yang paling dapat mewakili, kata yang mewakili P
itulah sebagai intinya. Yang memiliki anak gadis.(Frase apositif)
Contoh:
rumah baru (2) Frase Eksosentris/Frase Preposisi
cara mengambil intinya, masukkan frase tersebut ke Frase eksosentris adalah frase yang tidak
dalam kalimat: memiliki inti. Frase ini ditandai pemakaian kata
Kami membeli rumah baru depan (preposisi). Oleh karena itu, frase
Lalu ambil satu kata yang dapat mewakili: eksosentris disebut juga frase preposisi.
Kami membeli rumah Contoh:
Kami membeli baru di sebuah pertokoan
Kata yang dapat mewakili adalah kata rumah maka dari Bandung
rumah adalah inti (D) sedangkan baru adalah atribut
(M). B. Ditinjau Berdasarkan Jenis Kata Inti
a. Frase Verbal
III. JENIS FRASE Frase verbal adalah frase yang memiliki inti
Jenis frase dalam bahasa Indonesia dapat dibedakan atas kata kerja.
dua dasar tinjauan yaitu : Contoh:
A. Ditinjau Berdasarkan Inti sedang menangkap
(1) Frase Endosentris M D=KK
a. Frase Atribut/Frase Bertingkat sedang membaca
Frase Atribut adalah frase yang M D=KK
mempunyai penjelas/atribut yang dapat b. Frase Nominal
dikenali dengan cara tidak dapat disisipi Frase nominal adalah frase yang memiliki
kata tugas dan serta atau. inti kata benda.
Contoh:
mobil mewah
D=KB M
jaksa agung
D=KB M

c. Frase Adjektif SOAL PENGANTAR


Frase adjektif adalah frase yang memiliki
inti kata sifat. 1. Gadis cantik yang berambut panjang yang selalu
Contoh: tersenyum itu.
sangat bagus Kalimat tersebut terdiri atas…
M D=KS A. 1 frase
merah jambu B. 2 frase
D=KS M C. 3 frase
d. Frase Numeral D. 4 frase
Frase numeral adalah frase yang memiliki E. 5 frase
inti kata bilangan.
Contoh: 2. Frase di bawah ini terdiri dari inti fase nomina dan frase
enam biji verba, kecuali…
D=K.bil M A. Rumah saya sangat rapi.
tiga buah B. Suara mereka tidak terdengar.
D=K.bil M C. Jaringan telkomsel belum aktif.
D. Ibu Ani adalah guru favorit.
e. Frase Adverbia E. Papan tulisnya sudah tiba.
Frase adverbia adalah frase yang memiliki
inti kata keterangan. 3. Frase di bawah ini yang memiliki frase atributif
Contoh: berimbuhan adalah…
tadi malam A. Di kantornya dia menerima piagam penghargaan.
D=Ket M B. Sepeda baru mereka hilang di pantai.
sore nanti C. Acara spektakuler itu dihadiri undangan istimewa.
D=Ket M D. Pernikahan itu dihadiri artis ibu kota.
E. Pemerintah daerah mengadakan pameran
Frase Idiomatis pendidikan.
Frase Idiomatis adalah frase yang mengandung makna
kiasan. 4. Susunan yang sejenis dengan binatang itu buas dan
Contoh: bukannya binatang buas adalah…
 naik daun = kariernya sedang A. Yang sedang membaca
meningkat B. Tanah milik negara
 buah tangan = oleh-oleh C. Anak kucing angora
 sempit hati = mudah tersinggung D. Armada kapal laut
 tangan kanan = kepercayaan E. Yang membaca berbahagia
 buah bibir = bahan
pembicaraan 5. Cermati paragraf berikut!
Walaupun ayahnya seorang pengusaha hebat dan
Frase Sepola terkenal di bidang ekspor-impor. Julia tetap bersikap
Frase sepola dapat ditentukan dengan cara : rendah hati. Ia sangat memamerkan kekayaan yang
(1) jumlah kata sama dimiliki orang tuanya. Ia tidak mau hanya menadahkan
(2) jenis/struktur sama tangan. Kini ia akan menyelesaikan
(3) kelas kata sama pendidikan dari jerih payahnya sendiri dan memperoleh
Contoh: nilai sangat memuaskan.
lapangan parkir Frasa yang tepat untuk mengganti frasa yang bercetak
D=KB M=KK miring tersebut adalah…
sepola dengan: A. Pengusaha tekun, akan memamerkan, telah
menyelesaikan
pesawat terbang B. Seorang yang berkuasa telah memamerkan, akan
D=KB M=KK menyelesaikan
C. Pengusaha kaya, tidak memamerkan, sudah
menyelesaikan.
D. Pengusaha rajin, ingin memamerkan, sudah
menyelesaikan.
E. Orang yang berkuasa, tidak memamerkan, ingin C. Lima ribuan
menyelesaikan D. Ban mobil baru
E. Lukisan affandi
6. Frase di bawah ini yang berupa frasa bertingkat atau
artributif adalah… 12. Kesinambungan pembangunan hanya mungkin
A. Tanah air dilaksanakan jika ketahanan nasional dan kualitas
B. Demi nusa bangsa trilogi pembangunan meningkat.
C. Drama televise Kalimat di atas menggunakan frase nomina berikut,
D. Pendidikan dan pelatihn kecuali…
E. Ke sekolah A. Kesinambungan pembangunan
B. Ketahanan nasional
7. Cermati kalimat-kalimat berikut! C. Kualitas trilogi pembangunan
1) Berwisata di Anyer sangat menyenangkan. D. Trilogi pembangunan
2) Chairil sudah mencintai seni sejak kecil. E. Mungkin dilaksanakan
3) Setahun yang lalu ayah bekerja sebagai pilot.
4) Masyarakat menikmati kebebasannya dalam 13. Makna frase Percobaan penginderaan jarak jauh
berkampanye. melalui satelit ditentukan oleh inti frasenya. Inti frase
5) Museum Bahari perlu mendapat perhatian itu adalah…
pemerintah. A. percobaan
Frasa nominal terdapat dala kalimat nomor… B. penginderaan
A. (1) D. (4) C. jarak jauh
B. (2) E. (5) D. melalui
C. (3) E. satelit

8. Cermati kalimat-kalimat berikut! 14. Frase yang berpola sama dengan frase ketua rapat
1) Koperasi dan bulog membeli gabah dengan murah. penutupan adalah…
2) Baik gitar maupun suling dominan dalam tarling. A. tidak terlalu panjang
3) Aneka kerajinan dari daerah sedang dipamerkan di B. gaji pegawai bank
PRJ. C. dua tiga orang
4) Pimpinan memperlihatkan ketegasan kepada rakyat. D. belum pernah pergi
5) Mereka menerima bingkisan dengan riang gembira. E. sudah sangat kritis
Kalimat yang menggunakan frasa bertingkat terdapat
pada kalimat .... 15. Karena pengaruh globalisasi dan tuntutan kehidupan,
A. (1) D. (4) masyarakat akan mengalami pergeseran apresiasi dan
B. (2) E. (5) cara pandang terhadap berbagai aspek kehidupan.
C. (3) Frase verbal dalam pernyataan di atas adalah..
A. pengaruh globalisasi
9. Frase bermakna ganda terdapat pada kalimat… B. akan mengalami
A. Mobil baru paman dicongkel pencuri tadi malam. C. tuntutan kehidupan
B. Sakit yang ringan pada tenggorokan sering terjadi D. pergeseran apresiasi
karena polusi udara. E. cara pandang
C. Sakit pada mulut dan tenggorokan yang ringan
sering terjadi pada anak-anak. 19. Untuk menjamin kelancaran bagi penumpang umum
D. Obat0obatan modern mampu menghilangkan rasa mudik lebaran, pemerintah menambah jumlah armada
sakit dalam waktu singkat. angkutan umum.
E. Pada tahun ajaran ini, SPP siswa baru dinaikkan. Berdasarkan konteks kalimat tersebut, kelompok kata
jumlah armada angkutan umum terbentuk melalui
10. Apabila yang dimaksud adalah “pengusaha garmen tahapan proses…
yang berjenis kelamin wanita”, rumusan frase yang A. armada  armada angkutan  jumlah armada
benar adalah… angkutan  jumlah armada angkutan umum
A. Pengusaha garmen wanita B. armada  armada angkutan  armada angkutan
B. Pengusaha wanita garmen umum  jumlah armada angkutan umum
C. Garmen pengusaha wanita C. jumlah  jumlah armada  jumlah armada
D. Wanita pengusaha garmen angkutan  jumlah armada angkutan umum
E. Wanita garmen pengusaha D. angkutan  armada angkutan armada angkutan
umum  jumlah armada angkutan umum
11. Di antara frase-frase berikut yang mempunyai tiga
E. Jumlah  jumlah angkutan  jumlah angkutan
tafsiran makna adalah …
A. Anak guru yang pandai umum  jumlah armada angkutan umum
B. Penanam padi karawang
1.
2.
3.
4.
5.
6.
7.
8.
9.
10.
11.
12.
13.
14.
15.
16.
17.
18.
19.
20. Frase benda yang salah satu unsurnya merupakan kata
ulang terdapat pada kalimat...
A. Ketika ditanya ia menggeleng-gelengkan kepala.
B. Dengan tetangga kita harus tolong-menolong.
C. Pekerjaan sulam-menyulam memerlukan ketelitian.
D. Sampai kini pekerjaan Arif dan aries masih surat–
suratan.
E. Kubiarkan ia mengelus-elus bayiku sepuas-
puasnya.
P O3
sebuah hadiah.
O1
Inti kalimat: Paman memberi hadiah (S-P-O)
Kalimat inti: Paman memberi (S-P)

Catatan :
Meski kalimat inti dan inti kalimat berbeda, tetapi dalam
soal-soal testing ke PTN keduanya sering disamakan.

POLA DASAR KALIMAT


Pola dasar kalimat bahasa Indonesia diambil dari
kalimat inti. Pola dasar disebut juga pola inti yang
terbagi atas 4 yaitu :
(1) KB - KK
Adik belajar
S=KB P=KK

(2) KB - KB
Ayahku dokter
S=KB P=KB

(3) KB - KS
Gunung tertinggi
SINTAKSIS - 2 S=KB P=KS

I. KALIMAT INTI (4) KB - K. Bil


Kalimat inti adalah kalimat yang terdiri atas inti subjek Mobilnya dua
dan inti predikat. S=KB P=K. Bil

Ciri-ciri : III. KALIMAT TRANSFORMASI


 terdiri atas dua kata Kalimat transformasi/kalimat luas adalah
 pola S–P perubahan/perluasan kalimat inti tanpa mengubah
 intonasi netral (.) makna dasar kalimat itu.
Ciri-ciri dan contoh:
 mempunyai pola dasar

Contoh: Kalimat Inti Kalimat


Andika yang tampan itu Transformasi
S 1.Terdiri dari inti 1.Jika terjadi
mampu berlari 100 m fungsi penambahan atau
P S–P atau S–P–O pengurangan fungsi
dalam 10 detik. Dia Pergi. kalimat.
K. waktu Dia akan pergi ke
Jakarta.
Kalimat intinya: 2. Intonasi netral 2.Terjadi perubahan
Andika berlari Dia pergi. intonasi (susunan)
S=KB P=KK Dia pergi?
3. Susunan normal 3.Terjadi perubahan
II. INTI KALIMAT Dia pergi. susunan
Inti kalimat adalah kalimat yang terdiri atas inti-inti S P (diinversikan)
kalimat S – P – O Pergi dia.
P S
Contoh: 4. Bentuknya 4. Terjadi
Paman yang dari Bandung positif penegatifan
S Dia pergi. Dia tidak pergi.
akan memberi kami
Contoh: Kalimat luas yang polanya masih sama dengan kalimat
Kakak menelepon. inti di atas adalah …
Kalimat transformasi dari kalimat inti di atas adalah: A. Adik saya tinggal di Banyuwangi .
 Kakak sedang menelepon. B. Adik saya, yang tinggal di Banyuwangi, belajar
 Kakak saya menelepon ayah. menelepon.
 Kakak sedang menelepon temannya. C. Adik saya, yang tinggal di Banyuwangi,
 Kakak saya yang tinggal di Semarang menelepon menelepon kemarin.
kami. D. Adik saya yang menelepon saya tinggal di
 Kakak sering menelepon dari tempat kostnya. Banyuwangi.
 Kakak tidak akan menelepon, dll. E. Adik saya tinggal di Banyuwangi dan kemarin
menelepon.

20. Setelah rangkaian panjang dari petisi-petisi dan


dengar pendapat, 150 penduduk daerah itu memenangi
kasus mereka dan memaksa perusahaan itu membayar
ganti rugi atas berbagai kerusakan ekonomi senilai
hampir setengah miliar dolar AS.
Inti kalimat dari kalimat panjang tersebut adalah…
A. Penduduk memaksa perusahaan.
B. Penduduk memenangi kasus.
C. Perusahaan membayar ganti rugi.
D. Nilai ganti rugi setengah miliar dolar.
E. Perusahaan mendapat petisi.

SOAL PENGANTAR 21. Sejak lahirnya konsep pemikiran baru dalam ilmu
kedokteran, yang dicetuskan oleh Profesor Linus
16. Setelah peristiwa pengeroyokan itu, para wartawan Pauling, yakni tentang Ortomolecular medicine yang
yang tergabung dalam Persatuan Wartawan Indonesia dasarnya adalah studi biologi molekuler sebagai sains
Medan memprotes tindakan brutal petugas keamanan dasar, penelitian medis diarahkan pada molekul-
di kantor KPUD Sumatera Utara. molekul yang secara normal biologis fisiologis ada
Kalimat di atas merupakan kalimat transformasi dari dalam tubuh manusia.
kalimat… Inti kalimat panjang tersebut adalah…
A. Wartawan dikeroyok. A. Konsep pemikiran baru dicetuskan oleh Profesor
B. Wartawan tergabung. Linus Pauling.
C. Wartawan memprotes. B. Ortomolecular medicine adalah sains dasar.
D. Tindakan brutal petugas. C. Ortomoleculer medicine dasarnya adalah studi
E. Petugas bertindak brutal. biologi.
D. Penelitian medis diarahkan pada molekul.
17. Dalam perkembangannya, perbedaan paham yang E. Biologi molekuler merupakan sains dasar.
sudah mengakar pada diri setiap insan tumbuh menjadi
konflik sehingga menodai pertemuan yang sudah di 22. Pola kalimat bahasa Indonesia yang terdiri dari kata
bangun bertahun- tahun. benda dan kata sifat terdapat pada …
Kalimat inti kalimat luas tersebut adalah… A. Wanita cantik itu menangis di kantor polisi.
A. Paham menjadi konflik B. Buruh pabrik dipecat karena malas bekerja.
B. Paham sudah mengakar C. Ujian nasional dilaksanakan bulan April tahun
C. Perbedaan tumbuh 2010.
D. Perbedaan menodai pertemuan D. Pria yang mengajarkan bahasa Indonesia itu sangat
E. Perbedaan sudah mengakar rapi.
E. Pemuda tampan berbaju hitam itu seorang pekerja
18. Yang pola dasar kalimatnya sama dengan pola dasar keras.
kalimat Usaha operasi plastik itu gagal adalah…
A. Perusahaan mengalami kerugian. 1.
B. Kartunya dua. 2.
C. Kepala jawatan itu seorang jutawan. 3.
D. Perusahaan itu memberikan bonus kepada 4.
karyawannya. 5.
E. Para sandera sakit di rumah sakit. 6.
7.
19. Adik menelepon. 8. Rencana anggaran merupakan hal yang sangat
penting di dalam suatu perusahaan sehingga harus
memahami cara mencari sumber dana. Ia juga harus Inti kalimat tersebut adalah...
mampu membuat rencana biaya baik, sehingga pada A. untuk memberikan pelayanan bagi para wisatawan
suatu saat nanti dapat dipertanggungjawabkan yang umumnya adalah keluarga.
sebagai laporan peminjaman modal di bank. B. Penduduk setempat membangun dan menyewakan
Isi beberapa kalimat berikut sesuai dengan maksud puluhan penginapan sederhana yang terbuat dari
kalimat di atas, kecuali... kayu beratap rumbia.
A. Setiap usahawan harus mampu membuat rencana C. Untuk memberikan pelayanan bagi para wisatawan
biaya operasional secara baik. yang umumnya adalah keluarga, penduduk
B. Perusahaan dapat memanfaatkan rencana biaya setempat membangun dan menyewakan puluhan
operasional untuk peminjaman modal. penginapan sederhana.
C. Dalam rencana anggaran perusahaan, terdapat D. Penduduk setempat membangun dan menyewakan
uraian tentang sumber dana dan rencana. puluhan penginapan sederhana
D. Rencana anggaran suatu perusahaan dapat E. Para wisatawan menyewa penginapan sederhana.
dimanfaatkan sebagai laporan pertanggungjawaban.
E. Di dalam membuat rencana anggaran perusahaan, 12. Anda ingin berwisata alam pedesaan sambil menikmati
diperlukan pemakahan tentang cara mencari dana. hidangan istimewa, memandang hamparan sawah luas
terbentang dan deretan pohon kelapa yang membingkai
9. Berdasarkan hasil penelitian, pengunaan varietas padi tepian desa, menedengar suara gemercik air dan
ketan unggul di Indonesia lambat. Hal ini ditunjukkan menyaksikan ikan-ikan yang berenang bebas di kolam
oleh rendahnya angka serapan pemanfaatan benih padi alam, sambil menikmati masakan udang galah yang
ketan ini. Kondisi ini tidak lepas dari lambatnya lezat?
penyampaian informasi ketersediaan varietas padi A. Anda ingin pergi ke desa?
kepada petani. Selain itu, iklim usaha beras ketan B. Anda ingin menikmati pemandangan alam?
tampaknya juga belum mendukung. C. Anda ingin menikmati udang galah yang lezat?
Paragraf tersebut dikembangkan dari kalimat inti... D. Anda ingin berwisata?
A. Perkembangan pengunaan varietas padi ketan E. Anda ingin berwisata alam dan berwisata kuliner?
unggul lambat.
B. Banyak petani yang belum mengetahui varietas 13. Jika data perusahaan Anda diakses lebih dari satu
padi ketan. orang, dari setiap orang dapat diberi hak akses data
C. Lambatnya penyampaian informasi dan iklim usaha yang berbeda-beda, password ini akan sangat berguna.
beras ketan belum mendukung. Kalimat inti dari kalimat luas tersebut adalah...
D. Rendahnya angka serapan pemanfaatan pengunaan A. Jika data perusahaan Anda diakses lebih dari satu
varietas padi ketan unggul. orang.
E. Hasil penelitian menunjukkan lambatnya B. Setiap orang dapat diberi hak akses data yang
perkembangan penggunaan varietas padi ketan berbeda-beda.
unggul. C. Password ini sangat berguna.
D. Data perusahaan Anda diakses.
10. Vitamin D dikenal sebagai salah satu unsur penting E. Setiap orang dapat diberi hak akses data.
bagi kesehatan. Jika kekurangan vitamin ini, Anda
berisiko menderita beragam penyakit mulai dari kropos 14. Sambil menikmati salak pondoh yang rasanya
tulang, kanker usus, dan problem kekebalan tubuh. ditanggung manis, para wisatawan sekaligus dapat
Selain itu, para ahli juga mengindikasikan bahwa menikmati pemandangan alam.
vitamin D berkaitan dengan penyakit rematik. Indikasi Kalimat inti dari kalimat di atas berpola sama
tersebut terlihat dengan tingginya kasus defisiensi dengan…
vitamin D pada pasien yang dirawat di klinik A. Sukiman mahasiswa.
rematologi. B. Slamet menulis laporan.
Inti kutipan tersebut adalah... C. Uangnya seribu.
A. Vitamin D sangat penting bagi kesehatan tulang. D. Gino menangis.
B. Mengonsumsi vitamin D membuat tulang menjadi E. Uangnya di Bank.
sehat.
C. Kekurangan vitamin D menimbulkan berbagai 15. Pencemaran air sungai, sumur penduduk, dan udara
problem kekebalan tubuh. oleh sejumlah pabrik industri merupakan masalah
D. Kekurangan vitamin D menimbulkan berbagai utama yang diderita masyarakat yang tinggal di lokasi
penyakit. pabrik atau daerah yang dilalui sungai tercemar ini.
E. Vitamin D berhubungan dengan penyakit tulang. Kalimat luas tersebut dibangun dari kalimat yang
unsur-unsurnya…
11. Untuk memberikan pelayanan bagi para wisatawan A. pencemaran, air sungai, sumur penduduk, udara
yang umumnya adalah keluarga, penduduk setempat B. merupakan pabrik industri, masalah, utama
membangun dan menyewakan puluhan penginapan C. masalah, masyarakat diderita
sederhana yang terbuat dari kayu beratap rumbia. D. lokasi, pabrik, tinggal
E. pencemaran, merupakan, masalah A. Pernyataan itu mengemuka dalam jumpa pers yang
diselenggarakan di sebuah stasiun televisi swasta.
16. Siti bekerja membantu ibunya sesudah ia pulang dari B. Pernyataan kontroversi presiden mengemuka saat
sekolah. sidang kabinet berlangsung.
Inti kalimat di atas adalah… C. Karena wartawan terus memancing akhirnya
A. Siti bekerja ….. Ia pulang. pernyataan artis cantik tentang rencana
B. Siti membantu ….. Ibunya pulang. pernikahannya itu mengemuka juga.
C. Siti membantu ….. Sesudah ia pulang. D. Pada kenyataannya, pernyataan itu, yang terlontar
D. Siti membantu ….. Ia pulang. secara tak sengaja dari mulut pejabat perusahaan
E. Siti bekerja ….. Ia pulang dari sekolah. kami, terus mengemuka dari rapat ke rapat.
E. Akhirnya, pernyataan yang mengemuka dalam
17. Pengembangan obat-obatan herbal yang bisa dilakukan rapat koordinasi bidang pertahanan dan keamanan
masyarakat dengan sangat mudah juga tidak dipermasalahkan hingga saat ini.
memerlukan perizinan dan persyaratan yang rumit
sehingga masyarakat lebih mudah memanfaatkannya
untuk obat-obatan dan untuk meningkatkan
kesejahteraan.
Inti kalimat tersebut adalah...
A. Pengembangan obat-obatan herbal dilakukan
masyarakat.
B. Pengembangan obat-obatan herbal sangat mudah
dilakukan masyarakat.
C. Pengembangan obat-obatan herbal tidak
memerlukan perizinan dan persyaratan yang rumit.
D. Pengembangan obat-obatan tidak memerlukan
perizinan sehingga masyarakat lebih mudah
memanfaatkannya.
E. Pengembangan obat-obatan mempunyai perizinan
yang memudahkan masyarakat untuk
memanfaatkannya.

18. Kalimat berikut yang merupakan perluasan kalimat


“Setiap bangsa memiliki bahasa” adalah…
A. Setiap bangsa yang ada di dunia ini memiliki dan
menggunakan bahasa yang terus berubah sejalan
dengan perubahan zaman.
B. Setiap bangsa yang memiliki bahasa akan selalu
menggunakan bahasanya itu untuk berkomunikasi
dalam kehidupan bermasyarakat.
C. Setiap bangsa memiliki bahasa dan setiap bahasa
selalu dimiliki dan dijunjung tinggi oleh
masyarakat.
D. Dalam kehidupan berbangsa dan bernegara, setiap
bangsa harus memiliki bahasa sebagai sarana
pengembangan budaya.
E. Hampir dapat dipastikan bahwa tidak ada bangsa di
dunia ini yang tidak memiliki bahasa.

19. Panen kali ini gagal akibat serangan hama tikus.


Kalimat di atas dapat ditransformasikan menjadi…
A. Kegagalan panen kali ini akibat serangan hama
tikus.
B. Panen kali ini akibat serangan hama tikus gagal.
C. Akibat serangan hama tikus, panen kali ini gagal.
D. Serangan hama tikus gagal akibat panen kali ini.
E. Gagal akibat diserang hama tikus panen kali ini.

20. Perluasan kalimat inti pernyataan itu mengemuka


menghasilkan kalimat berikut, kecuali …
Kalimat aktif yang predikatnya tidak
membutuhkan objek penderita.
Contoh:
Ibu menjahit di kamar
S P K

B. Kalimat Pasif
Kalimat pasif adalah kalimat yang subjeknya
dikenai pekerjaan. Kalimat pasif ditandai oleh
predikatnya yang berawalan di- atau ter-.
Contoh:
(a) Pameran itu akan dibuka oleh Pak Camat.
(b) Batu besar itu terangkat oleh kedua orang
itu.

Kalimat pasif dibagi dua yaitu:


(1) Kalimat Pasif Biasa
Berasal dari aktif transitif.
Contoh:
Ibu membaca buku = aktif transitif
S P O1
Buku dibaca Ibu = Pasif biasa
S P O2
(2) Kalimat Pasif Zero/Nol
Syarat pasif zero yang benar:

… O2 + P …

Contoh:
Saya antarkan Dona ke sekolah.
O2 P S Kt

II. BERDASARKAN PREDIKAT


A. Kalimat Verbal
Kalimat yang predikatnya adalah berkelas kata
SINTAKSIS-3 kerja.
Contoh:
Ibu membaca buku.
I. BERDASARKAN SUBJEK S P(KK) O1
A. Kalimat Aktif B. Kalimat Nominal
Kalimat aktif adalah kalimat yang subjeknya Kalimat yang predikatnya berkelas kata benda,
melakukan pekerjaan. kata sifat, dan kata bilangan.
Ciri penting menandai kalimat aktif, predikat Contoh:
kalimat itu berupa kata kerja yang berawalan (a) Ibunya seorang dokter.
me(N), ber-, dan kalimat aktif dapat pula S P (KB)
predikatnya berupa kata kerja aus. (b) Sangat kosong
Contoh: P(KS)
(a) Adikku sedang membaca. sorot matanya
(b) Mereka sedang bernyanyi di panggung. S
(c) Kami pergi ke sekolah. (c) Kerbau Pak Madi tiga ekor lagi.
Kk.aus S P(K.Bil)

Kalimat aktif dibagi dua yaitu : III. BERDASARKAN INTONASI


(1) Kalimat aktif transitif A. Kalimat Deklaratif
Kalimat aktif yang predikatnya membutuhkan Kalimat ini dikenal dengan kalimat berita.
objek penderita. Contoh:
Contoh: Tadi pagi ada tabrakan mobil di depan sekolahku.
Ibu membaca novel di kamar. B. Kalimat Imperatif
S P O1 Kt Kalimat ini disebut juga kalimat perintah/
(2) Kalimat Aktif Intransitif permintaan.
Contoh:
Silakan menunggu sebentar!
Jangan membaca di tempat gelap!
C. Kalimat Interogatif
Kalimat ini dikenal dengan kalimat tanya.
Kalimat tanya dibagi dua yaitu :
(a) Kalimat tanya biasa adalah kalimat tanya
yang memerlukan jawaban.
Contoh:
Siapa nama Tuan?
Jawab : Handoko
(b) Kalimat tanya retoris adalah kalimat tanya
yang tidak memerlukan jawaban.
Contoh:
Dapatkah manusia hidup seorang diri?

IV. BERDASARKAN BENTUK


A. Kalimat Mayor/Sempurna
Kalimat mayor adalah kalimat yang sekurang-
kurangnya mengandung dua unsur pusat, bisa
terdiri dari subjek dan predikat (S-P) atau subjek,
predikat, objek (S-P-O), ataupun lebih dari itu,
misalnya disertai dengan keterangan (S-P-O-K)
Contoh:
(a) Pekerjaannya menulis.
S P
(b) Julia membeli boneka
S P O
(c) Saya membaca buku
S P O
di perpustakaan
K
B. Kalimat Minor/Tak Sempurna SOAL PENGANTAR
Kalimat minor adalah kalimat yang mengandung 1. Dusun Bone-Bone di Enrekang, 5 jam berkendaraan
satu unsur pusat. Unsur pusat yang sering mobil dari Makasar, Sulawesi Selatan lebih hebat lagi.
digunakan dalam kalimat minor berupa predikat. Kepala Dusun berpenduduk 542 jiwa ini berinisiatif
Kalimat minor umumnya digunakan sebagai menjadikan seluruh bagian dusunnya sebagai kawasan
jawaban atas suatu pertanyaan, sebagai perintah, tidak merokok. Inisiatif ini dipicu oleh adanya warga
ataupun seruan. dusun yang pulang kampung karena sakit paru-paru
Contoh: kronis yang diakibatkan menghirup asap rokok ditempat
(a) Besok pagi. (Sebagai jawaban kapan kalian kerjanya. Pertanyaan yang menggelayuti Pak Idris saat
berangkat?) itu,”Kalau perokok pasif saja bisa terkena penyakit,
(b) Ke pasar (Sebagai jawaban pertanyaan Ibu apalagi yang merokok, ya?” Maka ditetakanlah aturan
pergi ke mana?) untuk tidak merokok di Bone-Bone. Untuk secara
(c) Kerjakan! langsung menyampaikan terima kasih atas partisipasi
(d) Lari! mandirinya menyehatkan masyarakat,Depkes
mengundang Kepala Dusun Bone-Bone untuk menerima
peghargaan di Jakarta.
Jika kita menyusun pertanyaan atas wacana artikel di
atas, maka kalimat pertanyaan yang paling tepat
adalah…
A. Dari manakah asal kepala dusun Bone-Bone hingga
berinisiatif menjadikan seluruh bagian dusunnya
sebagai kawasan tidak merokok?
B. Apa yang mendasari kepala dusun Bone-Bone
berinisiatif menjadikan seluruh bagian dusunnya
sebagai kawasan tidak merokok?
C. Dimanakah warga dusun yang pulang kampung
karena sakit paru-paru kronis yang diakibatkan
menghirup asap rokok bekerja?
D. Siapakah orang dari Depkes yang secara langsung pada Ibu, ya Nak!
menyampaikan terima kasih kepada Pak Idris atas (4) Suhita : Tidak, Bu!
partisipasi mandirinya menyehatkan masyarakat (5) Mardilah : Baiklah, sampaikan
Dusun Bone-Bone? surat itu secepatnya.
E. Setujukah Departemen kesehatan dengan inisiatif Pak Kalimat minor dalam dialog di atas terdapat pada
Idris memberlakukan larangan warganya untuk tidak kalimat nomor…
merokok? A. (1) dan (2)
B. (2) dan (3)
2. Pengulangan subjek kalimat dapat ditemukan dalam C. (2) dan (4)
kalimat… D. (3) dan (4)
A. Peserta seminar yang datang dari Singapura E. (4) dan (5)
menginap di hotel ketika berada di Jakarta.
B. Pengunjung segera meninggalkan arena pameran 8. Kalimat berikut yang merupakan kalimat sempurna,
setelah mereka melihat-lihat berbagai hasil kecuali …
kerajinan. A. Ia di dapur sedang menangis.
C. Simposium itu diadakan di Yogyakarta dan B. Wanita itu sangat cantik.
membicarakan persoalan pemilu. C. Rumahnya terbakar semalam.
D. Pegawai itu segera mengubah rencananya setelah D. Ibu ke luar negeri kemarin.
bertemu dengan kepala kantor. E. Pergi ibu ke pasar.
E. Masyarakat tidak hanya diingatkan pada
kewajibannya, tetapi juga diberi kesempatan untuk 9. Kalimat-kalimat berikut termasuk kalimat inversi,
memperjuangkan haknya. kecuali…
A. Dalam kebun tanah airku, tumbuh bunga mawar.
3. Kalimat berikut yang termasuk kalimat pasif adalah... B. Kuserahkan segala hidupku untukmu.
A. Pembalap itu merajai lomba pada etape pertama. C. Tak terkabul permintaannya.
B. Penceramah itu berceramah tentang refleksi diri. D. Yang sudah dibeli tidak boleh dikembalikan.
C. Guru yang sedang bercerita itu seorang penulis E. Berdiri aku di sunyi senyap.
cerita pendek.
D. Telah kukirimkan kembali buku itu kemarin ke 10. Yang berjenis kalimat verbal di bawah ini adalah...
pemiliknya. A. Ayahnya seorang guru sekolah dasar.
E. Diskusi yang sedang berlangsung membicarakan B. Baju biru adik ternyata kekecilan.
peningkatan etos kerja. C. Halaman rumahnya sangat kotor.
4. Kalimat aktif di bawah ini yang bisa dipasifkan D. Kakaknya dosen di USU.
adalah… E. Suaranya kedengaran dari sini.
A. Negara Indonesia berdasarkan Pancasila.
B. Adik suka mandi di Sungai Musi. 11. Penggunaan bentuk pasif yang tepat terdapat dalam
C. Nenek menangis tersedu – sedu. kalimat…
D. Dia memukul adiknya yang nakal itu dengan sapu. A. Setelah makan, Ia akan kerjakan tugas itu.
E. Amran pergi ke Lampung hari Sabtu. B. Ku telah kirimkan paket itu ke Jakarta.
C. Dalam mimpinya dia selalu sebutkan namamu.
5. Yang tergolong kalimat aktif intransitif, kecuali… D. Dalam kehidupan sehari-hari harus kita junjung
A. Karung berisi beras. tinggi bahasa Indonesia yang baik dan benar.
B. Pakaian putih raja itu bertahtakan permata. E. Kau telah berikan segalanya untuknya.
C. Saya bermaksud baik kepada mereka.
D. Kami bertanya kepadamu. 12. Pergantian mengajar harap dilaporkan setiap saat.
E. Ibu menangisi adik ketika mau berangkat Pernyataan yang tepat untuk kalimat di atas, kecuali…
merantau. A. Kalimat majemuk
B. Kalimat berita
6. Yang duduk itu adikku. C. Kalimat sempurna
Bila ditinjau dari predikatnya kalimat di atas adalah D. Kalimat pasif
kalimat… E. Kalimat verbal
A. verbal
B. intransitif 13. Kalimat sapaan terdapat dalam kalimat…
C. nominal A. Pak Rudi mempunyai lima orang saudara.
D. aktif B. Saya memiliki hubungan khusus dengan saudara
E. tunggal Luna.
C. Luar biasa! Anda memang layak mendapat
7. (1) Mardilah : Engkau anakku Suhita… bintang.
(2) Suhita : Ya, Bu ! D. Akulah yang bertanggung jawab bukan dia.
(3) Mardilah : Kau tidak boleh marah E. Mereka berunjuk rasa di bundaran HI.
14. Kalimat yang tidak dapat dipasifkan adalah…
(1) AS menyerang Irak.
(2) Adik main bola.
(3) SBY mengunjungi korban tsunami.
(4) Mereka berdialog serius.

15. Kalimat-kalimat berikut yang merupakan kalimat


imperatif adalah…
A. Siapa nama Saudara ?
B. Rumah itu milik saudagar kaya.
C. Pergilah sekarang juga !
D. Di mana ada gula di situ ada semut.
E. Aku tidak mengenal siapa ayahnya.

16. Kalimat di bawah ini yang tergolong kalimat masdar


adalah…
A. Ia memberikan hadiah.
B. Dipukulnya adik dengan kayu.
C. Menangis ia dengan keras.
D. Berolah raga sangat baik bagi kesehatan.
E. Bermain bola kami di lapangan.

17. Kalimat di bawah ini yang bukan merupakan kalimat


mayor adalah…
A. Negara Indonesia adalah negara kesatuan.
B. Pekerjaannya itu tidak selesai.
C. Anton kemarin ke diskotik.
D. Dia di Bandung kuliah.
E. Ia mempelajari masalah itu.

18. Nelayan menangkap ikan di laut.


Pola kalimat ini sama dengan kalimat berikut,
kecuali…
A. Siswa membaca buku di perpustakaan.
B. Kami bergembira di pesta ulang tahunnya.
C. Kemarin polisi menangkap pencopet di pasar itu.
D. Kami harus mengembalikan buku itu pada
tempatnya semula.
E. Mereka menonton pesta rakyat di alun-alun.

19. Kalimat berikut yang merupakan kalimat inversi


adalah…
A. Pak tani menanam jagung di sekeliling rumahnya.
B. Berdiri aku di sunyi senyap.
C. Permintaannya sejak tahun lalu belum terkabul.
D. Ibu mendudukkan adik di jok mobil.
E. Sahabat kami akan berangkat ke luar negeri.

20. Suara itu sudah kedengaran dari sini walaupun masih


agak pelan.
Pernyataan yang tepat untuk kalimat di atas adalah,
kecuali…
A. Kalimat aktif
B. Kalimat berita
C. Kalimat sempurna
D. Kalimat pasif
E. Kalimat verbal
A. KALIMAT MAJEMUK SETARA/
KOORDINATIF
Kalimat majemuk setara atau disebut juga kalimat
koordinatif adalah gabungan dua pokok pikiran
atau lebih yang kedudukannya setara atau
sederajat.
Contoh:
(a) Sebenarnya ia tidak bodoh, tetapi ia malas.
(b) Ibu membaca buku sedangkan Ayah
membersihkan kebun.

Berdasarkan kata hubung yang digunakan Kalimat


majemuk setara terbagi atas empat jenis yaitu:
1. Kalimat Majemuk Setara Menggabungkan
Kalimat majemuk setara meng-gabungkan
umumnya menggunakan kata penghubung:
dan, lalu, serta, kemudian, lagipula, malah,
bahkan.
Contoh:
Mereka akan bermain sebaik-baiknya dan
memberi perlawanan ketat.

2. Kalimat Majemuk Setara Memilih


Kalimat majemuk setara memilih
menggunakan kata penghubung atau
Contoh:
Engkau tinggal di rumah atau engkau ikut
dengan membawa bungkusan ini.

3. Kalimat Majemuk Setara Berlawanan/


Mempertentangkan.
Kalimat majemuk setara berlawanan/
mempertentangkan biasanya meng-gunakan
kata penghubung tetapi, sedangkan,
melainkan, namun.
SINTAKSIS-4 Contoh:
(a) Tetangga kami kaya, tetapi hidupnya
tidak bahagia.
I. KALIMAT BERDASARKAN ISI (b) Pria itu bukan pamanku, melainkan ayah
Jenis kalimat ditinjau berdasarkan isi ataupun temanku.
berdasarkan pokok pikiran terbagi atas dua jenis
kalimat yaitu : 4. Kalimat Majemuk Setara Sebab Akibat
(1) Kalimat Tunggal Kalimat majemuk setara sebab akibat
Kalimat tunggal adalah kalimat yang mempunyai umumnya menggunakan kata penghubung:
satu pokok pikiran. Pokok pikiran kalimat bahasa sebab itu, sebab, karena itu, karena.
Indonesia adalah subjek atau predikat. Dengan Contoh:
kata lain kalimat tunggal adalah kalimat yang Kakek sudah sangat tua oleh karena itu ia
memiliki satu subjek dan satu predikat serta dapat tinggal bersama kami.
saja diikuti objek maupun keterangan.
(2) Kalimat Majemuk 5. Kalimat Majemuk Setara Menegaskan
Kalimat majemuk adalah kalimat yang Kalimat majemuk setara menegaskan
mempunyai dua/lebih pokok pikiran. Dengan kata umumnya menggunakan kata penghubung:
lain kalimat majemuk adalah kalimat yang bahkan, malah, lagipula
sekurang-kurangnya terdiri atas dua subjek dan
dua predikat. B. KALIMAT MAJEMUK BERTINGKAT
Kalimat Majemuk Bertingkat atau kalimat
JENIS – JENIS KALIMAT MAJEMUK kompleks atau disebut juga kalimat subordinatif
Kalimat majemuk dapat dibagi lagi menjadi tiga jenis, adalah kalimat tunggal yang salah satu
yaitu : jabatannya atau lebih diperluas sehingga terbentuk
kalimat baru.
Dalam kalimat majemuk bertingkat kita mengenal Kalimat majemuk bertingkat perluasan sebab
: umumnya menggunakan kata penghubung karena
(a) Induk kalimat dan sebab.
Induk kalimat adalah jabatan kalimat yang
bersifat tetap atau tidak mengalami (10) Kalimat Majemuk Bertingkat Perluasan
perubahan. Keterangan Syarat
(b) Anak kalimat Kalimat majemuk bertingkat perluasan
Anak kalimat adalah jabatan kalimat yang keterangan syarat umumnya meng-gunakan kata
diperluas sehingga terbentuk kalimat baru. penghubung: andai, jika, kalau, bila, asal.
Anak kalimat ditandai pemakaian kata
penghubung dan bila letaknya mendahului (11) Kalimat Majemuk Bertingkat Perluasan
induk kalimat dipisah dengan tanda baca Keterangan Berlawanan
koma. Kalimat majemuk bertingkat perluasan
keterangan perlawanan umumnya menggunakan
Jenis-Jenis Kalimat Majemuk Bertingkat kata penghubung: walaupun, meskipun, biarpun.
(1) Kalimat Majemuk Bertingkat Perluasan Subjek
Kalimat majemuk bertingkat perluasan subjek C. KALIMAT MAJEMUK RAPATAN
umumnya ditandai kata peng-hubung yang atau Kalimat majemuk rapatan pada dasarnya
bahwa. merupakan KMS yang salah satu/lebih unsurnya
dilepaskan atau dirapatkan karena sama.
(2) Kalimat Majemuk Bertingkat Perluasan Predikat Ibu membeli ayam lalu
Kalimat majemuk bertingkat perluasan predikat S P O1
menggunakan kata penghubung yang. Ibu memotong nya
S P O1
(3) Kalimat Majemuk Bertingkat Perluasan O 1  KMS
(Penderita) Ibu membeli ayam lalu
Kalimat majemuk bertingkat perluasan O1 S P O1
biasanya menggunakan penghubung yang dan memotong nya
bahwa. P O1
 KMR Subjek
(4) Kalimat Majemuk Bertingkat Perluasan O 2
(Pelaku)
Kalimat majemuk bertingkat perluasan O2 SOAL PENGANTAR
biasanya menggunakan kata penghubung yang.
(5) Kalimat Majemuk Bertingkat Perluasan O 3 1. Kalimat luas berikut yang efektif adalah …
(penyerta) A. Meskipun pernah tinggal di Indonesia pada tahun
Kalimat majemuk bertingkat perluasan O3 1961-1963 untuk melakukan riset tentang minoritas
(Penyerta/tidak langsung) biasanya menggunakan Tionghoa, saya belum pernah menginjakkan kaki di
kata penghubung yang dan bahwa. kawasan Bangka Belitung, sebuah kawasan yang
merupakan pemukiman etnik Tionghoa.
(6) Kalimat Majemuk Bertingkat Perluasan B. Saya pernah tinggal di Indonesia pada tahun 1961-
Keterangan Waktu 1963 untuk melakukan riset tentang minoritas
Kalimat majemuk bertingkat perluasan keterangan Tionghoa, tetapi saya belum pernah menginjakkan
waktu umumnya meng-gunakan kata penghubung: kaki di kawasan Bangka Belitung, yaitu kawasan
ketika, saat, semenjak, sewaktu, sesudah, setelah, pemukiman etnik Tionghoa.
sebelum. C. Sebenarnya, saya pernah tinggal di Indonesia pada
tahun 1961-1963 untuk mela-kukan riset tentang
(7) Kalimat Majemuk Bertingkat Perluasan minoritas Tionghoa, tetapi saya belum pernah
Keterangan Tujuan menginjakkan kaki di kawasan Bangka Belitung,
Kalimat majemuk bertingkat perluasan keterangan padahal kawasan ini merupakan pemukiman etnik
tujuan umumnya meng-gunakan kata Tionghoa.
penghubung: agar, supaya, untuk. D. Meskipun pernah tinggal di Indonesia pada tahun
1961-1963 untuk melakukan riset tentang minoritas
(8) Kalimat Majemuk Bertingkat Perluasan Tionghoa, tetapi saya belum pernah menginjakkan
Keterangan Akibat kaki di kawasan Bangka Belitung, kawasan yang
Kalimat majemuk bertingkat perluasan keterangan merupakan pemukiman etnik Tionghoa.
akibat umumnya menggunakan kata penghubung: E. Bahkan, saya pernah tinggal di Indonesia pada
karena dan sebab. tahun 1961-1963 untuk mela-kuka riset tentang
minoritas Tionghoa, tetapi saya belum pernah
(9) Kalimat Majemuk Bertingkat Perluasan Sebab menginjakkan kaki di kawasan Bangka Belitung,
kawasan yang merupakan pemukiman etnik
Tionghoa. 6. (1) Anak pengusaha itu masih muda.
(2) Ia sudah pandai berbisnis.
2. Lingkungan fisik Indonesia yang terdiri atas daerah- Kedua kalimat di atas dapat digabungkan menjadi
daerah yang terbentang dari Sabang sampai Merauke kalimat majemuk dengan menggunakan konjungsi…
yang bertabur beribu-ribu pulau di seluruh Nusantara A. akibat
berpengaruh padap praktik sosial, antara lain berupa B. sebab
perilaku masyarakat. C. atau
Kalimat luas tersebut dibangun oleh beberapa kalimat D. tetapi
tunggal berikut ini, KECUALI E. supaya
A. Lingkungan fisik Indonesia terdiri atas daerah-
daerah 7. Kalimat-kalimat berikut merupakan kalimat majemuk,
B. Lingkungan fisik Indonesia terentang dari Sabang kecuali…
sampai Merauke A. Indonesia berjuang melawan kemiskinan agar lebih
C. Daerah-daerah tertabur menjadi beribu-ribu pulai banyak orang jadi sejahtera.
D. Lingkungan fisik Indonesia berpengaruh pada B. Udara di Bogor dingin; udara di Jakarta panas.
praktik sosial C. Kami menempuh ujian SNMPTN agar kami dapat
diterima di PTN.
E. Praktik sosial berupa perilaku bermasyarakat
D. Surat kabar memberitakan semua kejadian di tanah
air kita ini.
3. Udara yang kotor karena debu ataupun asap sisa
E. Surat kabar yang terbit pagi ada tiga, sedangkan
pembakaran menyebabkan kadar oksigen berkurang
surat kabar yang terbit sore ada dua.
sehingga sangat membahayakan kelangsungan hidup
setiap organisme.
8. Kalimat di bawah ini yang merupakan kalimat majemuk
Kalimat di atas merupakan perluasan dari kalimat
campuran adalah…
dasar…
A. Dia sudah melamar pekerjaan kemana-mana, tetapi
A. Udara kotor berkurang.
belum juga berhasil mendapatkan pekerjaan.
B. Udara kotor menyebabkan oksigen berkurang.
B. Selang beberapa waktu masuklah mereka.
C. Asap menyebabkan oksigen.
C. Saya menjaga rumah sedangkan ibu pergi ke pasar.
D. Udara kotor membahayakan.
D. Adik mendapat nilai buruk padahal dia sudah
E. Asap sisa pembakaran membahayakan.
belajar giat.
E. Ketika ayah datang, adik bermain bola dan kakak
4. Kalimat majemuk bertingkat dengan anak kalimat membaca buku.
pengganti subjek adalah….
A. Ayah seorang yang berjuang pada masa perang 9. Kalimat-kalimat di bawah ini sepola, kecuali…
kemerdekaan. A. Antoni memancing ikan di kolam, kemudian
membantu ibunya.
B. Ibu mengunjungi keluarga yang membesarkannya.
B. Saya dan Anita membaca lalu kami berdiskusi.
C. Nenek tinggal di Ibu Kota Negara Republik C. Alex tidak datang karena sakit.
Indonesia. D. Negara tetangga akan memberikan sumbangan dan
D. Yang berpakaian seragam SMP itu,adik saya. memberi dukungan.
E. Eli berlari ke rumah yang baru dibangun tersebut. E. Anak kecil itu dipukul temannya dengan kayu
karena itu badannya sakit.
5. Penggunaan konjungsi korelatif yang benar terdapat
dalam kalimat… 10. Kalimat majemuk bertingkat dengan klausa bawahan
A. Penerapan nilai Pancasila tidak hanya melalui pengganti keterangan adalah...
pemahaman, tetapi juga melalui pengalaman secara A. Yang mengantarkan kami tadi adalah adik ayah.
pribadi. B. Mereka sudah mengetahui bahwa saya lulus.
B. Penerapan nilai Pancasila bukan hanya melalui C. Ketika matahari telah tinggi, ia baru terbangun.
pemahaman, tetapi juga melalui pengalaman secara D. Saya akan segera melaksanakan tugas yang
pribadi. diberikan ayah.
C. Penerapan nilai Pancasila tidak hanya melalui E. Mereka akan berangkat dengan kereta api malam.
pemahaman, melainkan juga melalui pengalaman
secara pribadi. 11. Bu Karta memilih kain itu dan Pak Karta
D. Penerapan nilai Pancasila bukan hanya melalui membayarnya.
pemahaman, melainkan juga melalui pengalaman Pola kalimat majemuk tersebut sama dengan pola
secara pribadi. kalimat majemuk berikut, kecuali ...
E. Penerapan nilai Pancasila bukan melalui A. Adik tidur saja di sini atau adik ikut pulang juga.
pemahaman saja, tetapi juga melalui pengalaman B. Hasan tidak mau belajar serius, melainkan bermain-
secara pribadi. main sepanjang hari.
C. Para mahasiswa aktif mengikuti, hanya Andi yang A. dan
pasif. B. bahkan
D. Kakak makan di rumah atau kakak makan di C. padahal
warung. D. sedangkan
E. Miryati dan kepala regu penyiar mengetahui hal itu. E. Lalu

12. Kalimat majemuk bertingkat dengan anak kalimat yang 16. Kalimat di bawah ini yang termasuk kalimat majemuk
berpola S – P adalah … setara mempertentangkan adalah…
A. Rusli belum bangun meskipun hari sudah terang. A. Dia ini bukan adikku, melainkan dia itu sahabatku.
B. Gadis yang memetik bunga itu sudah pergi. B. Kamu ikut saya atau ikut mereka.
C. Meskipun ia belum membaca surat itu, ia sudah C. Walaupun dia kaya, dia kikir.
tahu apa isinya. D. Ayah mengatakan bahwa kau akan datang.
D. Ketika ayah pulang dari kantor, adik sedang E. Kau boleh berangkat bila hujan berhenti.
menangis.
E. Karena permintaannya tidak dikabulkan ibunya, ia 17. Kalimat majemuk di bawah ini yang menggunakan
marah – marah. relasi implisit adalah…
A. Adik cantik, tetapi dia suka menangis.
13. (1) Serangga memfosil di dalam batu. B. Sewaktu kakak makan, kami pergi.
(2) Serangga itu diperkirakan telah ada C. Shania tidak pergi, tetapi dia tidur.
55 juta tahun sebelum munculnya dinosaurus. D. Saat Dina tidur, ibu memasak di dapur.
(3) Serangga itu ditemukan pada E. Adik membaca buku, kakak menulis surat.
sebuah tambang batu bara di Ohio.
Rangkaian yang tepat dari ketiga kalimat di atas 18. Saya terharu ketika Elton Jhon menyanyikan lagu
adalah… “candle in the wind” yang dipersembahkan kepada
A. Serangga yang ditemukan pada sebuah tambang Princes of Wales. Kalimat di atas termasuk kalimat
batu bara di Ohio itu diperkirakan telah ada 55 juta majemuk koordinatif
tahun sebelum munculnya dinosaurus dan kini SEBAB
memfosil di dalam batu. Salah satu unsur kalimat di atas diperluas
B. Serangga itu diperkirakan telah ada 55 juta tahun
sebelum munculnya dinosaurus yang ditemukan
pada sebuah tambang batu bara di Ohio yang 19. Mereka belajar dengan tekun ketika saya datang.
memfosil di dalam batu. Pola induk kalimat dan anak kalimat di atas adalah...
C. Serangga yang memfosil di dalam batu (1) Induk Kalimat : S-P-K
diperkirakan telah ada 55 juta tahun sebelum (2) Anak Kalimat : S-P-K
munculnya dinosaurus dan ditemukan pada sebuah (3) Anak Kalimat : S-P
tambang batu bara di Ohio. (4) Induk Kalimat : S-P
D. Serangga yang memfosil di dalam batu ditemukan
pada sebuah tambang batu bara di Ohio dan O3
diperkirakan telah ada 55 juta tahun sebelum S − P − O1 −
munculnya dinosaurus. 20. Kalimat yang berpola S −P −K adalah …
E. Serangga yang memfosil di dalam batu itu (1) Paman memberikan saya hadiah sepatu yang baru.
diperkirakan telah ada 55 juta tahun sebelum (2) Anton meminjamkan buku kepada Aminah kawan
munculnya dinosaurus yang ditemukan pada sekelasnya.
sebuah tambang batu bara di Ohio. (3) Pedagang itu memberikan diskon kepada pembeli
yang sudah lama menjadi langganannya
14. Ketika guru itu datang, murid - murid ternyata belum (4) Kakak membelikan boneka untuk adik yang
lengkap. menangis di kamar.
Kalimat di atas tergolong…
A. Kalimat sederhana.
B. Kalimat luas setara.
C. Kalimat luas tak setara bersubjek sama.
D. Kalimat luas tak setara dengan klausa bawahan
pengganti keterangan.
E. Kalimat luas tak setara dengan klausa bawahan
pengganti objek.

15. Paket pertama mengambil tema puasa dan krisis moral.


Paket kedua mengambil tema prospek ekonomi rakyat.
Kedua kalimat di atas dapat dijadikan kalimat majemuk
setara mempertentangkan dengan konjungsi…
 Banyak rumah baru dibangun di daerah
kami. (√)
 Rumah-rumah baru dibangun di daerah kami.
(√)

(2) Analogi
Analogi adalah pembentukan kata-kata baru dari
contoh yang sudah ada sebelumnya.
Contoh:
(a) dewa/dewi
saudara/saudari
siswa/siswi
putra/putri

(b) tunanetra
tunarungu
tunawisma
tunaaksara

(3) Hiperkorek
Hiperkorek adalah usaha memperbaiki yang benar
sehingga salah. Gejala bahasa hiperkorek
menyebabkan dalam bahasa Indonesia terbentuk
kata baku sebagai bentuk yang benar dan kata
nonbaku sebagai bentuk yang mengalami
hiperkorek (salah).
Kata Baku (√) Kata Nonbaku (x)
aktif Aktiv
aktivitas Aktifitas
apotek Apotik
analisis Analisa
asas Azas
atlet atlit
SEMANTIK - 1
cenderamata cinderamata
definisi difinisi
februari pebruari
I. GEJALA BAHASA
Gejala bahasa adalah peristiwa-peristiwa yang terjadi Hakikat hakekat
dalam bahasa. Gejala bahasa dalam bahasa Indonesia hipotesis hipotesa
terbagi lagi dalam lima kelompok besar yaitu : Hierarki hirarki
Ijazah ijasah
P A H A K Izin ijin
Jadwal jadual
(1) Pleonasme Jenazah jenasah
Pleonasme adalah pemakaian kata secara Kaidah kaedah
berlebihan/mubazir. Karier karir
Contoh: konduite kondite
(a) Hanya ini saja yang dapat saya berikan. (x) Konkret kongkrit
 Hanya ini yang dapat saya berikan.(√) Kuitansi kwitansi
 Ini saja yang dapat saya berikan. (√) Kata Baku (√) Kata Nonbaku (x)
(b) Sejak dari kecil ia sakit-sakitan. (x) Lembap lembab
 Sejak kecil ia sakit-sakitan. (√) Lubang lobang
 Dari kecil ia sakit-sakitan. (√) mengubah merubah
(c) Para tamu-tamu mulai datang. (x) menerjemahkan menterjemahkan
 Para tamu mulai datang. (√) Metode metoda
 Tamu-tamu mulai datang. (√) Nasihat nasehat
(d) Banyak rumah-rumah baru dibangun di Rezeki rejeki
daerah kami (x) Saksama seksama
spiritual spirituil
Silakan silahkan (b) Kontaminasi frase
Sintesis sintesa Kontaminasi frase atau gabungan kata adalah dua
System sistim bentukan frase berbeda yang mengandung
standardisasi standarisasi pengertian sama dijadikan satu frase.
Teoretis teoritis Contoh:
tradisional tradisionil  dan lain sebagainya (x)
Zaman jaman - dan lain-lain (√)
Foto photo - dan sebagainya (√)
Subjek subyek
Ekspor eksport  kadang kala (x)
Tim team - kadang-kadang (√)
Film filem - ada kalanya (√)
Manajer manager
Kualitas kwalitas (c) Kontaminasi kalimat
Ekstrem ekstrim Kontaminasi kalimat adalah dua bentukan kalimat
Insaf insyaf yang benar dijadikan satu kalimat yang kurang
Trotoar trotoir tepat. Kalimat tersebut dikatakan kurang tepat
karena gagasan kalimatnya menjadi kabur atau
menyukseskan mensukseskan
tidak jelas.
Risiko resiko
Contoh:
 Dalam perutnya mengandung racun. (x)
(4) Adaptasi/penyesuaian
K P O1
Adaptasi atau penyesuaian adalah usaha memasukkan
unsur bahasa asing ke dalam bahasa Indonesia. - Dalam perutnya terkandung racun. (√)
Adaptasi ini terjadi melalui dua cara : K P S
(a) Adaptasi fonologis yaitu pengadaptasian bunyi - Perutnya mengandung racun. (√)
bahasa. Bagaimana bunyi dalam bahasa asingnya, S P O1
demikianlah dalam bahasa Indonesianya.
Contoh :
 pencil menjadi pensil
 machine menjadi mesin
(b) Adaptasi morfologis yaitu peng-adaptasian bentuk
SOAL PENGANTAR
kata.
Contoh:
1. Kalimat yang mengandung hiperkorek di bawah ini,
 prakara menjadi perkara
kecuali …
 post card menjadi kartu pos A. Siswa diharap membawa kwitansi setiap hari.
B. Anak itu mempunyai kreatifitas yang patut
(5) Kontamimasi/kerancuan dicontoh.
Kontaminasi atau disebut juga kerancuan adalah C. Dia membeli obat ke apotik.
penurunan dua bentuk yang benar menjadi satu bentuk D. Dia salah menganalisis masalah itu.
yang salah. E. Metoda yang diberikan cukup jelas.
Kontaminasi terbagi atas tiga bentuk yaitu:
(a) Kontaminasi afiks 2. Penulisan unsur serapan berikut benar, kecuali…
Kontaminasi afiks terjadi bila suatu kata terbentuk A. manajemen
dari : B. standardisasi
C. frekuensi
memper - kan D. ekuivalen
diper - kan + kata sifat
E. sistimatis

Contoh: 3. Kalimat yang baku terdapat pada…


 memperlebarkan (x) A. “Kurangi minum kalau ingin berat badan turun, dan
- melebarkan (√) jangan minum air es,” begitu nasehat yang kerap
- memperlebar (√) dilontarkan awam kepada orang yang sedang diet.
Selain bentukan kata dari rumus di atas ada kata B. Sebuah pendapat yang kurang bijak, tapi banyak
lain yang merupakan kontaminasi afiks yaitu : diyakini kebenarannya.
 mempelajarkan (x) C. Mengingat peran air yang esensial inilah makan
- mempelajari (√) banyak pengobatan alternative yang mengandalkan
- mengajarkan (√) air.
D. Tidak ada salahnya juga Anda mencoba membantu E. Untuk belajar fotografi yang baik hanya
diet dengan suplemen air, baik untuk menurunkan memerlukan kamera digital yang bisa dipaki
dan mempertahankan berat badan. manual dan sekaligus otomatis.
E. Biasakan minum satu atau dua gelas air dingin
sebelum makan. 7. Ia tidak peduli jika karyanya mulai banyak ditiru, tetapi
konsumennya tetap bisa membedakan mana buatannya.
4. Banyak pedagang yang meninginkan keuntungan yang Kalimat tersebut dapat diperbaiki dengan…
sebesar-besarnya, tanpa peduli kesehatan pembeli A. Mengganti kata tetapi dengan supaya serta
dengan menggunakan bahan yang lebih murah, tetapi menghilangkan tanda koma.
tidak menjamin bahan tersebut layak untuk di konsumsi. B. Menambahkan kata meskipun pada awal kalimat.
Kalimat tersebut tidak baku ; bentuk baku kalimat C. Mengganti kata ia dengan dia.
tersebut adalah… D. Mengganti kata tetapi dengan karena serta
A. Banyak pedagang yang menginginkan keuntungan menghilangkan tanda koma.
besar, tetapi tanpa peduli kesehatan pembeli, E. Menghilangkan kata tetapi serta mengganti tanda
sehingga menggunakan bahan yang lebih murah tapi koma dengan titik koma.
tidak menjamin bahan tersebut layak untuk
dikonsumsi. 8. Kalimat di bawah ini yang bermakna ambigu,
B. Banyak pedagang yang menginginkan keuntungan kecuali…
sebesar-besarnya tidak peduli pada kesehatan A. Menurut kabar burung Song Joongki meninggal.
pembeli, lalu menggunakan bahan yang lebih murah B. Ayah dan Ibu pergi ke Surabaya selama 3 minggu.
tetapi tidak menjamin bahan tersebut layak untuk C. Wendy adik seorang model yang cantik dan
dikonsumsi. berprestasi.
C. Banyak pedagang menginginkan keuntungan besar, D. Ibu presiden yang baru dilantik adalah mantan artis
tetapi tidak peduli pada kesehatan pembeli dengan terkenal.
menggunakan bahan yang lebih murah namun tidak E. Sejarah dunia baru dipelajari bersama guru sejarah
menjadi bahan tersebut layak untuk dikonsumsi. baru.
D. Banyak pedagang menginginkan keuntungan besar,
tetapi tidak peduli pada kesehatan pembeli sehingga 9. Kata serapan yang berasal dari bahasa asing digunakan
mereka menggunakan bahan yang lebih murah, tetapi secara tepat dalam kalimat…
tidak layak untuk dikonsumsi. A. Para wakil rakyatyang duduk di DPR harus dapat
E. Banyak pedagang yang menginginkan keuntungan menyampaikan isnpirasi masyarakat kepada
besar tidak peduli pada kesehatan pembeli sehingga pemerintah.
menggunakan bahan yang lebih murah, akan tetapi B. Tampaknya isi pasal-pasal dalam perundangan itu
tidak menjamin atau layak untuk dikonsumsi. tidak efektif untuk menanggulangi kejahatan.
5. Kalimat yang memenuhi kaidah bahasa Indonesia baku C. Kami tidak suka membeli barang-barang yang
adalah… kuantitasnya kurang baik.
A. Tida ada yang menyangkalnya, salah satu daya D. Kita dapat menjangkau desa terkecil itu dengan
tarik Aceh adalah kopi bercita rasa khas. menggunakan sarana komunikasi sepeda motor.
B. Walaupun ada pejalan yang mengaku berpenyakit E. Orang itu dapat memanfaatkan waktu secara
jantung, namun penyakit itu dilupakan saat selektif sehingga sukses dalam usahanya.
meminum kopi khas Aceh.
C. Sesudah itu, rasa sakit harus ditanggung sebagai 10. Terdapat penulisan kata yang salah dalam kalimat-
bentuk konsekuensi. kalimat berikut, kecuali…
D. Pascatsunami, warung kopi semakin menjamur di A. Seseorang dapat dipastikan menderita suatu
seantero Banda Aceh. penyakit tertentu setelah melalui hasil analisa
E. Dari yang semula hanya bangunan kecil sampai pemeriksaan laboratorium.
yang memiliki banyak pintu, semuanya dipadati B. Setiap perusahaan akan selalu berusaha untuk
pengunjung. meningkatkan produktifitas para pegawainya agar
perusahaan itu dapat meningkatkan layanan kepada
6. Kalimat yang memenuhi kaidah kebakuan adalah… konsumen.
A. Ketika membeli kamera, kita bukan mencari yang C. Setelah menyelesaikan program strata satu di
terbaik, tetapi yang paling sesuai dengan Universitas Udayana. Guntur Saputra pergi ke
kebutuhan. Tokyo untuk mengambil program pascasarjana di
B. Perbedaan antara kamera bergaransi dengan tidak kota itu.
bergaransi memang tidak banyak. D. Fotokopi ijazah yang telah dilegalisir oleh kepala
C. Walaupun memakai kamera DSLR memang sekolah harus disertakan pada formulir pendaftaran
membuka peluang untuk hasil yang baik, tetapi kita untuk ke universitas.
tidak perlu membeli kamera secangih itu. E. Hanya akibat dari persoalan yang sangat sederhana,
D. Dalam dunia fotografi saat ini, Anda dapat sekadar pertikaian – bahkan perkelahian – antar wilayah
belajar merekam kejadian dengan baik atau sampai sering kali terjadi pada tahun-tahun terakhir.
ke teknik terdalamnya.
11. Kalimat-kalimat berikut ini tidak baku, kecuali… B. Akhlak, ikhlas, amendemen
A. Di era globalisasi seperti sekarang ini, yakni jarak, C. Akidah akhir, lembap
ruang, dan waktu menjadi begitu relatif karena D. Akseptabel, akseptor, akselerasi
dalam hitungan detik pelbagai arus informasi dapat E. Faksimili, aktifitas, piranti
menyebar ke seluruh dunia.
B. Bagi calon kepala desa yang berusia di atas 54 15. (1) Pernahkah Anda bertanya-tanya, mengapa
tahun sampai dengan 60 tahun diharuskan rambut hanya tumbuh di kepala dan beberapa tempat
mendapatkan rekomendasi dari camat dan lainnya? (2) Sementara itu, pada telapak tangan dan
dinyatakan sehat oleh rumah sakit daerah setempat. telapak kaki tak ada satu pun rambut yang
C. Memperingati Sumpah Pemuda, Kominte Nasional tumbuh? (3) Menurut para ilmuwan, ini dikarenakan
Pemuda Indonesia (KNPI) selain mengadakan oleh adanya molekul khusus di tubuh kita yang
upacara, juga mengadakan kampanye antinarkoba. menghalangi pertumbuhan rambut. (4) Molekul khusus
D. Untuk mengembangkan semua desa pada sekitar ini protein bernama Dickkopf 2 (DKK2) yang secara
460 kabupaten di Indonesia, dibutuhkan efektif merusak jalur WNT, bagian yang bertanggung
perencanaan dan waktu yang panjang. jawab untuk memicu pertumbuhan rambut di tubuh
E. Membicarakan tentang Mafia Camora tidak dapat kita. (5) Hal ini diungkapkan oleh para ilmuwan dari
dilepaskan dari peran serta Raffaele Cutolo (65 University of Pennsylvania School of Medicine yang
tahun) yang saat ini mendekam di penjara percaya bahwa dalam konsep evolusi, hewan-hewan
maksimum Asinara, Sardinia. tertentu telah mengembangkan protein DKK2 ini pada
bagian-bagian tertentu di tubuh mereka dengan tujuan
12. Yang merupakan kalimat baku adalah… membantu makhluk bertahan hidup.
A. Buku ini terdapat istilah-istilah kekerabatan pada
orang Jawa dan Batak dalam susunan masyarakat (1) "Dalam penelitian ini, kami telah
yang ditulis berdasarkan ilmu antropologi. menunjukkan bahwa kulit di daerah yang secara alami
B. Dari beberapa pokok persoalan yang diberikan tanpa rambut memproduksi inhibitor yang
untuk membandingkan dua atau lebih dialek, antara menghentikan WNT dari melakukan tugasnya,"
lain dalam bidang fonetik atau semantic.
ungkap Profesor Sarah E. (2) Millar seorang penulis
C. Adapun yang akan saya uraikan di sini ialah
studi yang dipublikasikan dalam jurnal “Cell
kebersihan dan kesehatan, saya terdorong untuk
Reports”. (3) Millar, yang juga profesor dermatologi di
mengemukakannya, karena sering dilalaikan orang
University of Pennsylvania School of Medicine,
dan sesungguhnya kebersihan dan kesehatan itu
berkata bahwa WNT sangat penting untuk
perlu, sebab dengan bersih tentu akan menjadi
perkembangan kantung rambut dan menyebabkan
sehat.
D. Karena nilai yang didapatkan lebih besar dari rambut tumbuh lebih banyak.
penolakan maka hipotesis nihil ditolak.
E. Jika data yang diolah tidak memenihi syarat, baik Penulisan kata yang kurang tepat pada teks di atas
dari segi kuantitas maupun kualitasnya, hasil adalah...
penelitian di lapangan itu tidak dapat A. Kalimat pertama paragraf pertama.
dipertanggungjawabkan secara ilmiah. B. Kalimat kedua paragraf pertama
C. Kalimat ketiga paragraf pertama.
13. Kalimat-kalimat berikut tidak baku, kecuali… D. Kalimat pertama paragraf kedua.
A. dengan naiknya angka pengganguran ini bisa saja E. Kalimat kedua paragraf kedua.
membuka kemungkinan untuk menambah jumlah
penduudk miskin. 16. Ragam baku terdapat pada kalimat…
B. Dalam bidang perbangkan, khususnya, saat ini A. Konon, rakyat Timtim putuskan untuk berpisah
sudah memberikan kemudahan kepada setiap dengan Indonesia.
nasabahnya. B. Hasil daripada pertandingan itu belum diumumkan.
C. Dampak positif dari teknologi perbankan lebih C. Gubernur perintahkan para bupati memperhatikan
dapat dirasakan oleh para pengusaha di mana masalah banjir.
mereka dapat bertransaksi keuangan dalam jumlah D. Sangat kami butuhkan peralatan ini.
besar hanya melalui internet. E. Ini hari mereka akan datang.
D. Dengan perekonomian yang baik, maka akan dapat
menyejahterakan seluruh lapisan masyarakat. 16. Kalimat berikut yang merupakan kalimat baku
E. Dengan berwirausaha, para lulusan perguruan adalah…
tinggi tidak hanya menjadi pribadi yang lebih A. Partai politik yang tidak menaati peraturan akan
mandiri, tetapi juga akan membantu pemerintah mendapat sanksi yang tegas.
dalam mengurangi pengganguran pada usia B. UGM memelopori aktivitas itu dengan mengadakan
produksi. KKN pengamat pemilu.
C. Dealer menyeediakan beberapa kemudahan dalam
14. Penulisan kata-kata berikut benar, kecuali… pembayaran sehingga konsumen dapat menicil
A. Akhirat, akta, akbar angsuran sesuai waktu yang diinginkan.
D. Meski tidak diakui, beberapa waktu lalu Golkar seharusnya:
pernah menginstruksikan kantor-kantor pemerintah - Yang tidak berkepentingan
untuk mencat pagar mereka dengan warna kuning. S
E. Dengan adanya status otonomi pemerintah pusat dilarang masuk. ()
tidak lagi sepenuhnya mensubsidi daerah. P
 Dalam bahasa Indonesia tidak
17. Kata serapan yang penulisannya salah terdapat pada K
kalimat… mengenal sistem kala. (x)
A. Dia adalah anggota badan eksekutif yang sangat P O1
disegani. seharusnya:
B. Gunakan rasio Anda untuk memecahkan soal ini. - Dalam bahasa Indonesia
C. Kita harus memilih seorang formatur. K
D. Ritme bunyi-bunyian mengesankan kengerian. tidak dikenal
E. John J. Little adalah anggota konggres dari P
Massachusetts.
sistem kala. ()
S
19. Banyak para karyawan yang di-PHK karena berulang
- Bahasa Indonesia
kali melakukan gerakan untuk menuntut segera
S
disyahkan kenaikan UMR.
tidak mengenal
Dalam kalimat di atas terdapat gejala bahasa, kecuali…
P
A. adaptasi
sistem kala. ()
B. pleonasme
C. kontaminasi O1
D. analogi (2) Koherensi/kepaduan
E. hiperkorek Koherensi atau kepaduan yaitu hubungan timbal
balik yang tepat antarunsur kalimat. Koherensi atau
20. Di samping kata dewa-dewi dibentuk juga kata yang kepaduan menyangkut pemakaian kata tugas yang
baru yaitu siswa-siswi, putra-putri, muda-mudi yang tepat.
disebut gejala bahasa analogi. Contoh :
SEBAB  Kami telah membicarakan tentang hal itu. (x)
Kata warna-warni juga termasuk analogi dari kata seharusnya:
dewa-dewi. Kami telah membicarakan hal itu.()
 Untuk supaya berhasil dalam ujiannya nanti, dia
belajar mati-matian. (x)
seharusnya:
Supaya berhasil dalam ujiannya nanti, dia
belajar mati-matian. ()
 Saya sudah selesaikan tugas itu. (x)
SEMANTIK-2 seharusnya:
Sudah saya selesaikan tugas itu.()
 Saya pergi ke sekolah supaya belajar.(x)
KALIMAT EFEKTIF seharusnya:
Kalimat efektif adalah kalimat yang dengan tepat mampu Saya pergi ke sekolah untuk belajar.()
menyampaikan gagasan dari seorang penulis/pembicara
sehingga menimbulkan gagasan yang sama tepatnya di (3) Kesejajaran atau keparalelan
benak pembaca/pendengar. Kesejajaran atau kepararelan adalah pemakaian
bentuk gramatikal yang sama untuk bagian-bagian
Syarat-syarat kalimat efektif : kalimat tertentu.
(1) Kesatuan gagasan Contoh :
Kalimat efektif harus memperlihatkan kesatuan  Kami telah merencanakan membangun pabrik,
gagasan. Unsur-unsur dalam kalimat itu harus saling membuka hutan, pelebaran jalan desa, dan
mendukung sehingga membentuk kesatuan ide yang pembuatan tali air. (x)
padu. seharusnya:
Kesatuan gagasan dalam kalimat dapat dilihat dari Kami telah merencanakan membangun, pabrik,
jabatan (S – P). membuka hutan, melebarkan jalan desa, dan
Contoh : membuat tali air. ()
 Bagi yang tidak berkepentingan
K (4) Kehematan
dilarang masuk (x)
P
Kalimat efektif menggunakan kata-kata secara A. Pada mulanya, masyarakat belum mengenal
efisien, tidak berlebih-lebihan. Setiap kata yang tentang pertukaran karena setiap orang berusaha
digunakannya memiliki fungsi yang jelas. memenuhi kebutuhannnya dengan usaha sendiri.
Contoh : B. Kesulitan-kesulitan itu antara lain karena benda-
 Sejak dari kecil dia sakit-sakitan. (x) benda yang dijadikan alat tukar belum mempunyai
seharusnya: pecahan sehingga penentuan nilai uang,
-Sejak kecil dia sakit-sakitan. () penyimpanan, dan pengangkutan menjadi sulit
-Dari kecil dia sakit-sakitan. () dilakukan.
 Hanya ini saja yang dapat saya berikan. (x) C. Mula-mula uang kertas yang beredar yaitu
seharusnya: merupakan bukti-bukti pemilikan emas dan perak
-Hanya ini yang dapat saya berikan.() sebagai alat atau perantara untuk melakukan
-Ini saja yang dapat saya berikan. () transaksi.
D. Uang kartal adalah alat bayar yang sah dan wajib
 Para ibu-ibu PKK mengadakan rapat. (x)
digunakan oleh masyarakat dalam melakukan
seharusnya:
transaksi jual-beli sehari-hari.
-Para Ibu PKK mengadakan rapat. ()
E. Sedangkan yang dimaksud dengan uang giral
-Ibu-ibu PKK mengadakan rapat. () adalah uang yang dimiliki masyarakat dalam
bentuk simpanan (deposito) yang dapat ditarik
(5) Kelogisan sesuai kebutuhan.
Kelogisan maksudnya penalaran atau alur berfikir
yang harus masuk akal. Supaya efektif, kata-kata 3. Menurut Wakil Menteri Perdagangan, Bayu
dalam sebuah kalimat tidak boleh menimbulkan Krisnamurthi, mengatakan bahwa masih tingginya
makna ambigu (ganda) atau tidak boleh harga daging di pasaran mengindikasikan paso
mengandung dua pengertian. kandaging tersendat.
Contoh : Kalimat tersebut menjadi logis dengan cara ...
 Hari Kemerdekaan Republik Indonesia A. menghilangkan kata menurut.
ke – 57 . (x) B. menghilangkan kata bahwa.
alasan : C. menggantikan masih tingginya dengan kata
Seolah-olah ada 57 negara Republik Indonesia. naiknya.
Hari kemerdekaan ke–57 Republik Indonesia D. mengubah kata di pasaran menjadi di pasar.
E. menghilangkan tanda baca koma sebelum kata
 Kepada Bapak Gubernur waktu dan tempat kami mengatakan.
persilakan. (x)
alasan : 4. Di antara kalimat berikut, kalimat yang tidak baku
Waktu dan tempat tidak mungkin kami
adalah ...
persilakan.
A. Akan tetapi, tidak sekadar mengubah bahasa
 Bapak Gubernur kami persilakan. () tersebut, melainkan melibatkan/berkonsultasi
SOAL PENGANTAR kepada policy makers atau pihak-phak lain yang
1. Dalam kaitan Jakarta sebagai pusat pemerintahan dan lebih memahami proses pembuatan kebijakan
ekonomi, pemerintah seyogianya lebih tegas dalam terkait isu-isu kebijakan yang akan diteliti.
memberikan izin membangun kawasan bisnis seperti B. Seharusnya stakeholders di Kemenristek memiliki
ruko, kantor, apartemen, dan mal. Masyarakat boleh data base para ahli yang telah melakukan riset-
membangun, tetapi mesti dengan kriteria ketat. riset kebijakan iptek dan riset lain yang
Halaman yang sempit tidak bolehseluruhnya ditutup. mendukung.
Harus ada sumur resapan dan tanaman. Sumur resapan C. Stakeholders dapat melibatkan kelompok
sebetulnya sudah diatur,tetapi karena tidak ada kepentingan, seperti konsultan kebijakan, lembaga
pengawasan, siapa punberani melanggar. Kita tentu swadaya masyarakat, masyarakat
sepakat bahwa siapapun berhak berbisnis, siapa pun terpelajar/cendekiawan Indonesia, dan kelompok
dapat membangunusaha, tetapi aturan tetap harus lain yang concern terhadap perkembangan iptek di
ditaati. Indonesia.
Kata-kata dalam paragraf di atas bercirikan ragam D. Di Indonesia reliabilitas temuan riset kebijakan
bahasa formal, kecuali ... yang dilakukan oleh lembaga litbang, terutama
A. kaitan. yang dapat mendukung kebijakan iptek,sebaiknya
B. seyogianya. dinilai sebelum diserahkan keKemenristek.
C. ruko. E. Peran Dewan Riset Nasional (DRN) sebagai
D. mesti. institusi yang mampu menilai dan menyaring
E. tetapi. temuan-temuan riset kebijakan yang memiliki
reliabilitas tinggi untuk dijadikan input dalam
2. Kalimat yang memenuhi kaidah penulisan kalimat baku proses penyusunan kebijakan strategis
adalah ... pembangunan iptek dibutuhkan.
A. Di Kalimantan, macan dahan dapat ditemui di
5. Menyadari bahwa tes HIV mempunyai peran penting hutan hujan dataran rendah.
dalam penanggulangan AIDS, pemeriksaan gratis B. Anggrek hitam tidak hanya berkembang biak
diselenggarakan Pemerintah kepada siapa pun yang dengan biji, tetapi juga dapat dikembangbiakkan
ingin memeriksakan diri. dengan cara memisahkan umbinya.
Kalimat di atas perlu diperbaiki dengan cara ... C. Suku Tengger di Bromo dikenal sangat berpegang
A. menghilangkan kata bahwa. teguh pada adat dan istiadat Hindu lama.
B. menambahkan karena sebelum menyadari D. Karena hidup di pepohonan, beruang madu
danmenambahkan oleh setelah diselenggarakan. bertelapak kaki tidak berbulu.
C. mengubah urutan pemeriksaan E. Beruang madu dikategorikan sebagai binatang
gratisdiselenggarakan Pemerintah menjadi yang mudah diserang.
Pemerintah menyelenggarakan pemeriksaan 9. Penderita yang mengalami kerusakan hati, ditandai
gratis. selaput mata berwarna kuning, tingkat kematian lebih
D. menghilangkan kata bahwa; menambahkan tinggi lagi.
katakarena sebelum menyadari; menambahkan Kalimat tersebut akan efektif jika diperbaiki menjadi ...
olehsetelah diselenggarakan. A. Penderita yang mengalami kerusakan hati, ditandai
E. mengganti siapa pun dengan anggota selaput mata berwarna kuning mempunyai tingkat
masyarakatdan peran dengan peranan. kematian yang lebih tinggi lagi.
B. Tingkat kematian penderita yang mengalami
6. Di antara lima kalimat berikut, terdapat kata ulang yang kerusakan hati lebih tinggi, yang ditandai selaput
menyatakan jumlah ’jamak’ yang dipakai secara tepat, mata berwarna kuning.
yaitu ... C. Penderita yang mengalami kerusakan hati dengan
A. Banyak ruko-ruko bermunculan menyertai ditandai selaput mata berwarna kuning, tingkat
kemudahan perizinan. kematiannya lebih tinggi lagi.
B. Akibatnya, tidak ada keseragaman pemahaman D. Tingkat kematian lebih tinggi lagi pada penderita
antarpihak-pihak terkait dengan pelaksanaan yang mengalami kerusakan hati, ditandai selaput
program. mata berwarna kuning.
C. Butir-butir dalam notula rapat perlu dikelompokkan E. Tingkat kematian penderita yang mengalami
berdasarkan topik bahasan. kerusakan hati, yang ditandai selaput mata
D. Permasalahan-permasalahan demokratisasi di berwarna kuning, lebih tinggi lagi.
tingkat bawah belum diikuti dengan pengelolaan
pemerintahan lokal yang lebih baik. 10. Kalimat yang efektif di antara kalimat-kalimat berikut
E. Tulisan ini mengulas pelbagai pandangan- adalah ...
pandangan teoretis mengenai prinsip kerja sama A. Dari hasil survei, diperoleh data bahwa rata-rata
dalam komunikasi. penghasilan masyarakat sekitar
Rp3.000.000,00/tahun atau berkisar
7. Untuk mengelola air perlu teknologi ramah lingkungan Rp250.000,00/bulan.
yang penerapannya akan meresapkan air sebanyak- B. Dari wawancara mendalam terungkap untuk
banyaknya ke dalam tanah. membiayai kehidupan sehari-hari, penduduk (yang
Kalimat tersebut menjadi efektif jika diperbaiki diwakili oleh responden) harus mengeluarkan
menjadi… hingga mencapai 87% dari penghasilan mereka.
A. Untuk mengelola air memerlukan teknologi ramah C. Dari sempitnya kepemilikan lahan dengan tingkat
lingkungan yang penerapannya akan meresapkan produktivitas yang juga rendah, berakibat pada
air sebanyak-banyaknya ke dalam tanah. tingkat penghasilan masyarakat yang rendah.
B. Untuk mengelola air diperlukan teknologi ramah D. Namun karena rendahnya kapasitas petani,
lingkungan yang meresapkan air ke dalam tanah. sebagian besar dari mereka hanya sebagai buruh
C. Untuk mengelola air, kita perlu teknologi ramah tani, yang hasilnya juga belum mencukupi
lingkungan yang penerapannya akan meresapkan kebutuhan hidup mereka.
air sebanyak-banyaknya ke dalam tanah. E. Demikian pula bagi sebagian kecil petani yang
D. Pengelolaan air memerlukan teknologi ramah memilih berwirausaha sebagai pekerjaan
lingkungan yang dalam penerapannya akan sampingan akan menambah kekurangan dalam
meresapkan air sebanyak-banyaknya ke dalam upaya menutupi kebutuhan sehari-hari.
tanah.
E. Diperlukan teknologi ramah lingkungan yang 11. Di antara kalimat berikut, kalimat yang tidak efektif
penerapannya meresapkan air sebanyak- adalah ...
banyaknya ke dalam tanah untuk mengelola air. A. Dari hasil pengolahan menunjukkan bahwa 43%
responden teridentifikasi telah memanfaatkan
8. Kalimat yang tidak baku di antara kalimat berikut teknologi meskipun responden yang
adalah ... menggunakan teknologi tradisional lebih banyak
(57%).
B. Terdapat berbagai cara mendapatkan teknologi B. Gubernur Jenderal Abraham Van Riebeeck
yang dipergunakan petani saat ini. membangun Staadhuis yang cukup besar dan
C. Dari survei teridentifikasi bahwa sebagian besar megah itu.
petani mendapatkan teknologi yang mereka C. Selesainya pembangunan Staadhuis yang cukup
gunakan dari pedagang keliling. besar dan megah itu pada masa pemerintahan
D. Jika dilihat dari jenis teknologinya, yang Gubernur Jenderal Abraham Van Riebeeck.
didominasi oleh teknologi tradisional, sangatlah D. Pembangunan Staadhuis yang cukup besar dan
wajar mereka dapat membelinya dari pedagang megah itu selesai pada masa pemerintahan
keliling saja. Gubernur Jenderal Abraham Van Riebeeck.
E. Jika dikelompokkan ke dalam tingkatan teknologi, E. Staadhuis yang cukup besar dan megah itu selesai
petani lebih banyak menggunakan peralatan pembangunanya pada masa pemerintahan Gubernur
manual, seperti cangkul dan gebotan, yang Jenderal Abraham Van Riebeeck.
dikombinasi dengan mekanik, seperti traktor.
15. Untuk mengelola air perlu teknologi ramah lingkungan
12. Kalimat yang memenuhi kaidah penulisan kata baku yang penerapannya akan meresapkan air sebanyak-
adalah ... banyaknya ke dalam tanah.
A. Orang yang berbahagia adalah orang yang faham Kalimat tersebut menjadi efektif jika diperbaiki menjadi
untuk apa hidup di dunia ini. ...
B. Pada hakekatnya, orang membangun rumah adalah A. Untuk mengelola air memerlukan teknologi ramah
sebagai upaya untuk menciptakan lingkungan lingkungan yang penerapannya akan meresapkan
buatan utuh yang mampu menampung kebutuhan air sebanyak-banyaknya ke dalam tanah.
hidup. B. Untuk mengelola air diperlukan teknologi ramah
C. Tidak dapat dimungkiri bahwa industri tembakau lingkungan yang meresapkan air ke dalam tanah.
memberikan kontribusi cukai sebesar Rp73,252 C. Untuk mengelola air, kita perlu teknologi ramah
triliun ke dalam kas negara. lingkungan yang penerapannya akan meresapkan
D. Aborsi memiliki resiko yang tinggi terhadap air sebanyak-banyaknya ke dalam tanah.
kesehatan maupun keselamatan seorang wanita. D. Pengelolaan air memerlukan teknologi ramah
E. Jika melihat perilakunya, kelihatannya dia lingkungan yang dalam penerapannya akan
menganggapmu lebih dari sekedar teman. meresapkan air sebanyak-banyaknya ke dalam
tanah.
13. (1) Total spesies yang diketahui berjumlah hampir E. Diperlukan teknologi ramah lingkungan yang
10.000, dengan perkiraan 3.000 di antaranya tumbuh di penerapannya meresapkan air sebanyak-banyaknya
Indonesia. (2) Paku-pakuan cenderung ditemukan pada ke dalam tanah untuk mengelola air.
kondisi tumbuh marginal, seperti pada lantai hutan yang
lembab dan tebing perbukitan, merayap pada batang 16. Di antara lima kalimat berikut, kalimat yang logis
pohon atau batuan, di dalam kolam/danau, daerah adalah ...
sekitar kawah vulkanik, serta di sela-sela bangunan A. Kapolres menjelaskan pengeroyokan mayat laki-
yang tidak terawat. (3) Tumbuhan paku tersebar di laki yang ditemukan di kolong jembatan dicurigai
seluruh bagian dunia, kecuali daerah bersalju abadi dan terjadi dua—tiga hari yang lalu.
daerah kering (gurun). (4) Meskipun demikian, B. Penonton bersorak-sorai ketika pengembalian bola
ketersediaan air yang mencukupi pada rentang waktu dari Lin Dan menyangkut di jaring.
tertentu diperlukan karena salah satu tahap hidupnya C. Kemiskinan adalah ketika seseorang tidak sanggup
tergantung pada keberadaan air. (5) Sebagian besar memenuhi kebutuhan pokok dalam kehidupan
anggota paku-pakuan tumbuh di daerah tropika basah sehari-harinya.
yang lembab. D. Bantuan modal ini bisa saja dilakukan oleh
Agar terbentuk paragraf efektif, urutan kalimat-kalimat pemerintah maupun swasta (LSM).
di atas yang benar adalah ... E. Jasad korban kecelakaan lalu lintas itu telah
A. 3, 1, 5, 4, 2. dimakamkan di tempat pemakaman umum.
B. 3, 1, 5, 2, 4.
C. 3, 2, 5, 1, 4. 17. Di antara kalimat-kalimat berikut, kalimat yang efektif
D. 3, 1, 2, 5, 4. adalah ....
E. 3, 2, 4, 5, 1. A. Pemukiman manusia di sekitar Bukhara setidaknya
berasal dari masa 5000 tahun lalu.
14. Staadhuis yang cukup besar dan megah itu B. Kekaisaran Persia Kuno adalah pemukiman
pembangunannya selesai pada masa pemerintahan pertama di Bukhara bertepatan dengan periode
Gubernur Jenderal Abraham Van Riebeeck. migrasi Arya.
Jika dibakukan, kalimat di atas menjadi ... C. Asal-usul nama Bukhara yang berarti tanah subur
A. Pada masa pemerintahan Gubernur Jenderal berasal dari bahasa Persia.
Abraham Van Riebeeck, Staadhuis yang cukup D. Pada abad ke-8, nama kota ini juga dikenal dengan
besar dan megah itu pembangunannya selesai. sebutan Buqar.
E. Di kalangan umat Islam, kota tua ini masyhur C. Pekerjaan merupakan cara untuk mencapai tujuan
karena salah satu perawi sunah terkemuka lahir di walaupun tidak sepenting kewajiban spiritual dan
sini. hal-hal lain yang menyangkut jiwa.
D. Mereka yang menganggap pekerjaan sebagai
18. Kalimat yang penulisannya tidak sesuai dengan EYD sesuatu yang harus dilakukan. Selain itu, pekerjaan
adalah …. dipandang sebagai cara untuk mencapai tujuan
A. Masyarakat DKI Jakarta masih perlu bersabar yang tidak sepenting kewajiban spiritual dan hal-
untuk memiliki sistem transportasi massal yang hal lain yang menyangkut jiwa.
bebas macet. E. Mereka manganggap pekerjaan sebagai sesuatu
B. Jika bandara-bandara itu tetap tutup, lini perakitan yang harus dilakukan untuk dapat bertahan hidup,
di Jepang dan di China akan kehabisan akan tetapi pekerjaan dipandang sebagai cara untuk
semikonduktor, disk drive, dan komponen- mencapai tujuan dan tidak sepenting kewajiban
komponen lainnya yang diproduksi di Thailand. spiritual dan hal-hal lain yang menyangkut jiwa.
C. Protes dari penumpang yang terlantar akibat
pembekuan tersebut tidak ada karena ini masalah
kondisi politik di thailand.
D. Sebelumnya, Bandara Suvamabhumi yang di tutup
membuat jadwal penerbangan Cengkareng
Bangkok dibatalkan.
E. Dari semua jenis polusi yang dihadapi, polusi
cahaya yang memungkinkan merupakan polusi
yang paling mudah dibenahi.

19. Pemberantasan narkoba adalah paling utama karena


dapat mengancam generasi muda yang menjadi penerus
bangsa.
Kalimat di atas tidak baku. Kalimat tersebut dapat
diperbaiki menjadi ….
A. Pemberantasan narkoba adalah merupakan hal
utama karena dapat mengancam generasi muda
yang menjadi penerus bangsa.
B. Pemberantasan narkoba merupakan hal utama
karena dapat mengancam generasi muda penerus
bangsa.
C. Pemberantasan narkoba adalah yang terutama
karena dapat mengancam generasi muda yang
menjadi penerus bangsa.
D. Pemberantasan narkoba merupakan hal yang utama
karena narkoba dapat mengancam generasi muda
yang menjadi penerus bangsa.
E. Pemberantasan narkoba merupakan yang utama
karena mengancam generasi muda yang menjadi
penerus bangsa.

20. Mereka tidak hanya menganggap pekerjaan suatu yang


harus dilakukan untuk dapat bertahan hidup, tetapi
pekerjaan juga dipandang sebagai cara untuk
mencapai tujuan dan tidak sepenting kewajiban
spiritual dan hal-hal lain yang menyangkut jiwa.
Kalimat di atas kurang efektif. Perbaikannya adalah ….
A. Mereka menganggap pekerjaan sebagai suatu yang
harus dilakukan untuk dapat bertahan hidup.
Pekerjaan juga dipandang sebagai cara untuk
mencapai tujuan walaupun tidak sepenting
kewajiban spiritual dan hal-hal lain yang
menyangkut jiwa.
B. Mereka menganggap pekerjaan sebagai sesuatu
yang harus dilakukan untuk dapat bertahan hidup.
b. Gagasan Penjelas
Gagasan penjelas atau sering disebut sebagai
pikiran penjelas adalah gagasan yang
fungsinya menjelaskan gagasan utama.
Gagasan penjelas umumnya dinyatakan oleh
lebih satu kalimat.

(C) Ciri-Ciri Paragraf yang Baik


(1) Mempunyai satu pokok pikiran
(2) Mempunyai beberapa pikiran jelas
(3) Mengandung koherensi/kepaduan
(4) Mengandung kesatuan ide
(5) Taat asas atau sesuai dengan EYD (ejaan yang
disempurnakan)

(D) Jenis-Jensi Paragraf


1. Berdasarkan Letak Kalimat Utama
a. Paragraf Deduktif
Paragraf deduktif adalah paragraf yang
letak kalimat utamanya berada di awal
paragraf.

b. Paragraf Induktif
Paragraf Induktif adalah paragraf yang
letak kalimat utamanya berada di akhir
paragraf.

c. Paragraf Campuran
Paragraf campuran adalah paragraf yang
kalimat utamanya terletak di awal dan di
akhir paragraf.

d. Paragraf Pararel
Paragraf pararel adalah paragraf yang
kalimat utamanya terletak di seluruh isi
paragraf.

2. Berdasarkan pola pengembangan paragraf


PRAGMATIK-1 dapat dibedakan atas :
a. Paragraf Umum – Khusus
Paragraf yang diawali dengan pernyataan
I. PARAGRAF umum lalu diikuti pernyataan khusus.
(A) Definisi Paragraf
Paragraf atau alinea adalah rangkaian kalimat yang b. Paragraf Khusus – Umum
saling berhubungan dan membentuk satu kesatuan Paragraf yang diawali dengan pernyataan
pokok pembahasan. khusus lalu diikuti pernyataan umum.

(B) Unsur-Unsur Paragraf c. Paragraf Sebab – Akibat


Secara umum, paragraf dibentuk oleh dua unsur, Paragraf yang terdiri dari kalimat
yakni gagasan utama dan beberapa gagasan hubungan sebab akibat.
penjelas.
a. Gagasan Utama d. Paragraf Akibat – Sebab
Gagasan utama atau sering disebut pikiran Paragraf yang terdiri dari kalimat hubungan
utama adalah gagasan yang menjadi dasar akibat sebab.
pengembangan sebuah paragraf. Gagasan
utama paragraf mungkin terletak di awal II. SILOGISME
ataupun di akhir paragraf. Ada pula gagasan (A) Defenisi Silogisme
utama yang berada di awal dan di akhir Silogisme adalah salah satu penarikan
paragraf sekaligus. kesimpulan dari umum ke khusus.
Contoh :
(B) Jenis-Jenis Silogisme PU = Semua anggota OSIS sedang melakukan
a. Silogisme Positif rapat.
Silogisme positif adalah silogisme yang kedua PK = Reza adalah anggota OSIS.
premisnya yakni Premis Umum (PU) dan K = Reza sedang melakukan rapat.
Premis Khusus (PK) bersifat positif dan E = Reza sedang melakukan rapat karena ia
kesimpulan yang dihasilkan pun bersifat anggota OSIS.
positif.

Rumus :

PU = semua A = B
PK = C = A
K =C=B

Contoh :
PU = Semua mahasiswa harus memiliki KTP.
PK = Andi adalah seorang mahasiswa.
K = Andi harus memiliki KTP.

b. Silogisme Negatif
Silogisme negative adalah silogisme yang
salah satu premisnya negative atau ditandai
dengan kata pengingkat seperti bukan atau
tidak.

Rumus :

PU = semua A tidak = B
PK = C = A
K = C tidak = B

Contoh :
PU = Semua guru tidak boleh absen.
PK = Sukirman adalah seorang guru. SOAL PENGANTAR
K = Sukirman tidak boleh absen.
1. Semua siswa mengikuti senam pagi.
Beberapa siswa memakai sepatu putih.
c. Silogisme Salah Kesimpulan dari kedua premis tersebut adalah…
Silogisme salah adalah silogisme yang dimana A. Ada siswa yang tidak mengikuti senam pagi.
kesimpulan yang ditarik dari kedua premis B. Semua siswa memakai sepatu putih.
memunculkan kesalahan. C. Beberapa siswa peserta senam pagi bersepatu putih.
Rumus : D. Ada siswa bersepatu putih tidak mengikuti senam
pagi.
PU = semua A = B E. Semua siswa peserta senam pagi bersepatu putih.
PK = C = B
K =C=A 2. Semua sekolah suka mengadakan pertunjukan musik.
Sekolah yang suka mengadakan pertunjukan musik
Contoh : suka mengadakan bakti sosial.
PU = Semua pegawai tamatan SMA. Sebagian sekolah yang suka mengadakan bakti sosial
PK = Andre adalah tamatan SMA. tidak suka mengikuti kompetisi olahraga antarsekolah.
K = Andre adalah tamatan SMA. Kesimpulan dari premis di atas adalah…
A. Sebagian sekolah suka mengikuti kompetensi antar
d. Entimen sekolah.
Entimen adalah silogisme yang disingkat. B. Semua sekolah tidak suka mengikuti kompetensi
olahraga tetapi suka mengadakan pertunjukan
Rumus : musik.
C. Semua sekolah suka mengadakan pertunjukan
C = B karena/sebab C = A musik tetapi tidak suka mengadakan bakti sosial.
D. Sebagian sekolah suka mengadakan bakti sosial dan E. Secara sosial bahasa dapat menimbulkan
tidak suka mengadakan pertunjukan musik. penafsiran.
E. Semua sekolah tidak suka mengikuti kompetensi
olahraga tetapi suka mengadakan pertunjukan 7. Cady dan para koleganya meneliti 6.997 pasien kanker
musik. payudara. Sebagian dari mereka rutin menjalani
mamografi pada periode 1990−1999 dan sebagian
3. Jika laut pasang. Dermaga tenggelam. lainnya tidak. Perkembangan para pasien tersebut lalu
Jika dermaga tenggelam sebagian kapal tidak dapat dicatat hingga tahun 2007. Setelah sekitar 12,5 tahun
merapat. periode follow-up, 461 dari mereka meninggal karena
Simpulan hyang tepat adalah… kanker payudara dan hamper 75% dari mereka tidak
A. Jika laut pasang, semua kapal tidak dapat merapat. menjalani pemeriksaan mamografi secara teratur.
B. Jika laut pasang, semua kapal dapat merapat. Simpulan yang dapat diambil dari paragraph diatas
C. Jika laut tidak pasang, semua kapal dapat merapat. adalah...
D. Jika laut tidak pasang, sebagian kapal dapat A. Pemeriksaan mamografi yang teratur dapat
merapat. menghindari risiko kematian pada pasien kanker
E. Jika laut tidak pasang, semua kapal tidak dapat payudara.
merapat. B. Mamografi adalah metode pengobatan kanker
payudara bagi perempuan guna menghindari
4. Peserta UTBK SBMPTN 2020 mengikuti Tes Potensi kematian.
Skolastik (TPS). C. Perempuan pada umumnya mendapat gangguan
Made tidak lulus UTBK SBMPTN 2020. kanker payudara karena tidak menjalani
Simpulan yang tepat tentang Made adalah… pengobatan secara dini.
A. Tidak mengikuti TPS dalam UTBK SBMPTN D. Pada tahun 1990−1999 Cady dan rekan-rekan
2020. mengadakan penelitian terhadap perempuan yang
B. Tidak mengikuti UTBK SBMPTN 2020 dan TPS. terkena kanker payudara.
C. Bukan peserta UTBK SBMPTN 2020 yang E. Tidak semua pasien perempuan terkena penyakit
mengikuti TPS. kanker payudara karena rutin menjalani mamografi.
D. Peserta UTBK SBMPTN 2020 yang mengikuti tes
bukan TPS. 8. Bayi yang dilahirkan tidak dengan berat badan rendah
E. Telah mengikuti TPS dalam UTBK 2020. pada umumnya mempunyai status gizi saat lahir yang
kurang lebih sama dengan status gizi bayi di Amerika.
5. Semua penipu pandai bicara dan ramah. Akan tetapi, seiring dengan bertambahnya umur
Tuan M tidak ramah tetapi pandai bicara. ditambah faktor-faktor lainnya, sebagian besar bayi
Kesimpulan yang tepat adalah… tersebut terus mengalami penurunan status gizi. Puncak
A. Tuan M seorang penipu yang pandai bicara. penurunan terjadi pada umur kurang lebih 18−24
B. Tuan M seorang penipu yang tidak ramah. bulan. Pada kelompok umur inilah prevalensi balita
C. Tuan M seorang penipu yang pandai bicara dan kurus (wasting) dan balita pendek (stunting) mencapai
tidak ramah. titik tertinggi (Hadi, 2001). Setelah melewati umur 24
D. Tuan M bukan seorang penipu meskipun pandai bulan, status gizi balita umumnya mengalami
bicara. perbaikan meskipun tidak sempurna.
E. Tuan M bukan seorang penipu yang ramah. Topik yang dibicarakan dalam bacaan tersebut adalah...
A. Status gizi balita.
6. Secara social bahasa mempunyai makna yang dapat B. perbandingan gizi balita di Indonesia dan Amerika.
menimbulkan berbagai penafsiran. Membelokkan C. Kasus gizi buruk pada balita.
makna simbolis ke arah pengertian tertentu justru bisa D. Prevalensi balita kurus dan balita pendek.
berdampak negative yang berjangka panjang dan E. Perbaikan status gizi balita.
turun-temurun. Pembelokan makna simbolis dari
agama, budaya, dan tradisi melalui bahasa, misalnya, 9. Miliaran burung dari belahan bumi utara berpindah
akan bisa mendatangkan malapetaka turun-temurun dalam kelompok besar ke bagian selatan setiap musim
yang menyandera generasi. Bisa dibayangkan dingin sebagai fenomena alamiah. Perpindahan
dampaknya jika penafsiran yang salah disebarluaskan tersebut dipicu oleh perubahan cuaca yang tajam dan
ke masyarakat. berkurangnya makanan. Burung-burung tersebut
Gagasan utama teks di atas adalah... mengembara dari tempat asal mereka di Eropa Utara,
A. Dampak penafsiran makna bahasa yang salah dapat Asia Utara, bahkan Alaska menuju wilayah yang
dibayangkan. sangat hangat, daerah tropika atau belahan bumi
B. Membelokkan makna simbolis berdampak negatif. selatan akan tetapi tidak semua jenis burung
C. Makna bahasa dapat menimbulkan banyak melakukan perjalanan panjang dan berbahaya ini. Ada
penafsiran. sebagian burung yang tidak melanjutkan migrasinya
D. Pembelokan makna simbolis mendatangkan setelah menempuh jalur yang singkat. Burung-burung
malapetaka. tersebut memasuki Indonesia dari dua arah, yaitu utara
dan selatan khatulistiwa. Dari bagian utara, burung- A. Walaupun dianggap negatif,
burung tersebut masuk melalui Semenanjung Malaya, primordialisme mempunyai nilai positif juga.
Sumatra, dan biasanya langsung ke bagian timur serta B. Jadi, primordialisme sebenarnya bukanlah
dari Jepang, Taiwan lalu Filipina dan masuk wilayah sesuatu yang perlu dipikirkan terlalu serius.
Indonesia. Ringkasa dari kutipan adalah … C. Orang yang mempertahankan primordialisme
A. Miliaran burung dari belahan bumi utara berpindah merupakan salah satu perusak kelangsungan
dalam kelompok besar ke bagian selatan setiap hidup suatu bangsa.
musim dingin sebagai fenomena alamiah. Burung D. Kekuatan suatu bangsa tidak ditentukan oleh
yang datang dari arah selatan berasal dari Australia. primordialisme.
B. Perpindahan burung dari belahan bumi utara E. Kekuatan primordalisme akan dapat memicu
merupakan sebuah fenomena alamiah. Burung- potensi konflik antara kebudayaan suku-suku
burung tersebut berpindah untuk mencari wilayah bangsa yang ada.
yang hangat.
C. Burung-burung tersebut memasuki wilayah Bacalah teks berikut untuk menjawab soal No. 12 dan 13!
Indonesia. Melalui dua arah, yaitu utara dan (1) Penyakit stroke adalah gangguan fungsi otak akibat
selatan. aliran darah ke otak mengalami gangguan. (2) Akibatnya,
D. Miliaran Burung dari belahan bumi utara berpindah nutrisi dan oksigen yang dibutuhkan otak tidak terpenuhi
dalam kelompok besar ke bagian selatan setiap dengan baik. (3) Penyebab stroke ada dua macam, yaitu
musim dingin sebagai fenomena alamiah. Burung- adanya sumbatan di pembuluh darah dan adanya pembuluh
burung tersebut memasuki Indonesia melalui dua darah yang pecah. (4) Umumnya stroke diderita oleh orang
arah, yaitu utara dan selatan. tua karena proses penuaan yang menyebabkan pembuluh
E. Tidak semua burung melakukan migrasi dengan darah mengeras dan menyempit serta lemak yang
jalur yang panjang dan berbahaya. Ada burung menyumbat pembuluh darah. (5) Pada beberapa kasus
yang menyudahi migrasinya setelah menempuh terakhir menunjukkan peningkatan kasus stroke yang
jalur yang singkat. terjadi pada usia remaja dan produktif (15-40 tahun).
(6)Pada golongan ini, penyebab utama stroke adalah stres,
10. Kiper nomor satu Qatar dapat menangkap bola dari faktor keturunan dan gaya hidup yang tidak sehat, seperti
penyerang Vietnam. Akan tetapi, tangkapannya lemah penyalahgunaan narkoba dan alkohol. (7) Pada kasus stroke
sehingga bola pun menyusur masuk ke dalam gawang usia remaja, faktor keturunan merupakan penyebab utama
sendiri. Pada penghujung babak pertama, sebenarnya terjadinya stroke. (8) Sering ditemukan kasus stroke yang
Qatar memiliki peluang untuk menyamakan kedudukan disebabkan oleh pembuluh darah yang mudah pecah atau
melalui kaki Adel. Namun, tendangannya masih lemah. kelainan sistem darah, seperti penyakit hemofilia dan
Tendangan itu sempat mengenai bek Vietnam sehingga talasemia yang diturunkan oleh orang tua penderita. (9) Jika
hanya menghasilkan tendangan penjuru. Hingga babak ayah atau ibu menderita diabetes, hipertensi, atau penyakit
pertama berakhir, kedudukan tetap tidak berubah. Usai jantung, kemungkinan anak terkena stroke menjadi lebih
turun minum, tempo permainan berjalan kian menarik. besar.
Penyerang Qatar kembali mengancam gawang
Vietnam. Tendangan bebas dari Waleed Abdulla 12. Gagasan utama yang tepat untuk melanjutkan teks
sangat terukur ke arah gawang, namun Duong lebih tersebut adalah…
cepat dalam mengantipasi gerakan bola. A. penanganan stroke
Gagasan utama pada bacaan di atas adalah… B. risiko stroke
A. Penyerang sering mengancam gawang Vietnam C. ancaman stroke
Tendangan bebas penyerang Qatar mampu D. gejala stroke
diantisipasi Duong. E. penelitian stroke
B. Tendangan bebas Walled Abdulla sangat terukur.
C. Tempi permainan berjalan kian menarik. 13. Simpulan yang tepat dari teks tersebut adalah…
D. Duong lebih cepat mengantisipasi gerakan bola. A. penyakit stroke kebanyakan dialami oleh orang usia
E. Duong lebih cepat mengantisipasi gerakan bola. lanjut
B. penyakit stroke disebabkan gangguan aliran darah
11. Primordialisme merupakan faktor penting untuk ke otak
memperkuat ikatan kelompok kebudayaan ketika ada C. sumbatan atau pecahnya pembuluh darah
ancaman dari luar terhadap kelompok kebudayaan mengakibatkan stroke
tersebut. Namun primordialisme dipandang sangat D. stroke dapat terjadi di usia remaja, produktif,
negatif karena mengganggu kelangsungan hidup suatu dan usia lanjut
bangsa. Primordialisme sering dianggap bersifat E. penyebab stroke adalah stress, penuaan, gaya
primitif regresif dan merusak. Primordialisme hidup, dan keturunan
dianggap akan menghambat modernisasi, proses
pembangunan dan merusak integritas nasional. Bacalah teks berikut untuk menjawab soal No. 14 dan 15!
Kalimat yang dapat ditambahkan sesudah kalimat Ethiopia adalah negara tertua di dunia. Negeri itu telah
terakhir kutipan di atas adalah… terbentuk sejak sebelum 4900 tahun yang lalu. Ethiopia
terletak di Afrika Timur, tepatnya di sebelah selatan pintu Pernyataan-pernyataan berikut sesuai dengan isi
masuk Laut Merah (Terusan Suez). Negara tetangganya bacaan di atas, kecuali…
adalah Kongo, Sudan, Somalia, dan Djiboti. Wilayah A. Hasil tangkapan lobster berkisar antara 2 s.d. 7 kg
Ethiopia cukup luas, hampir dua kali luas Pulau Sumatera. sekali melaut.
Negeri ini sering dilanda cuaca buruk dan musim kering B. Panen lobster terjadi pada saat gelombang laut
berkepanjangan. Ethiopia adalah negara peternakan kelas 1 tinggi.
dunia. Jumlah sapi dan kudanya menempati urutan ke-10 C. Nelayan beralih mencari lobster karena hasil
dunia dengan 2,6 juta ekor sapi dan 1,5 tangkapannya lebih banyak daripada ikan.
juta kuda. Ethiopia juga merupakan negara D. Selain lobster, nelayan dapat juga mencari
pengekspor kopi terbesar keenam dunia. Pada rajungan dan ubur-ubur di daerah pinggir pantai.
tahun 1985, akibat kemarau yang sangat panjang, negeri ini E. Mencari lobster membutuhkan bahan bakar tidak
menggemparkan dunia karena dilanda krisis sebanyak untuk mencari ikan laut
pangan dan penduduknya menderita kelaparan hebat.
Bacalah teks berikut untuk menjawab soal No. 17 dan 18!
14. Ide pokok teks di atas adalah… Teks 3A
A. Berdirinya negara Ethiopia... (1) Bioteknologi merupakan teknologi dengan
B. Letak negara Ethiopia. pemanfaatan mikroorganisme, tanaman, atau hewan melalui
C. Usia negara Ethiopia. modifikasi proses seluler untuk menghasilkan produk yang
D. Ciri-ciri geografis negara Ethiopia. bermanfaat. (2) Banyak negara, khususnya negara-negara
E. keadaan alam negara Ethiopia. maju, menjadikan bioteknologi sebagai penahaaan terdepan
ketahanan pangan. (3) Penelitian bioteknologi mencakup
15. Pernyataan berikut berhubungan, baik secara eksplisit berbagai bidang, yaitu pertanian, peternakan,
maupun implisit, dengan bacaan di atas, kecuali... farmakoseutika, kimia, pemrosesan makanan, dan
A. Ethiopia menempati peringkat 10 dunia dalam hal fermentasi. (4) Di Indonesia sumber daya manusia yang
ternak karena kualitasnya yang baik. berkompetensi di bidang bioteknologi masih sedikit dan
B. Kekeringan merupakan salah satu masalah serius terbatas. (5) Padahal, perkembangan bioteknologi global dan
yang dihadapi Ethiopia. bisnis yang terkait sangat menjanjikan. (6) Tenaga terampil
C. Ethiopia merupakan negara penghasil kopi dunia. dan ahli yang kompeten di bidang bioteknologi sangat
D. Krisis pangan yang terjadi di Ethiopia pernah diperlukan. (7) Pendidikan dan pengembangan SDM di
menggemparkan dunia. bidang bioteknologi harus mendapat prioritas dan dukungan,
E. Jumlah ternak di Ethiopia cukup banyak. baik dari pemerintah, universitas, lembaga penelitian,
maupun perusahaan swasta terkait. (8) Semua pihak harus
16. (1) Nelayan di pantai selatan Kabupaten Kebumen mendukung agar SDM bidang bioteknologi semakin banyak.
memilih mencari lobster selama cuaca buruk melanda
perairan itu. (2) Selain harganya mahal, biaya Teks 3B
operasional nelayan juga tidak besar karena area (9) Dewasa ini perkembangan bioteknologi tidak hanya
tangkapan lobster hanya terletak di sekitar bukit- didasari oleh biologi semata, tetapi juga oleh ilmu-ilmu
bukit karang tepi pantai. (3) Muslimin (45), nelayan terapan dan murni lain, seperti biokima, komputer, biologi
Pantai Pasir, mengatakan bahwa masa panen lobster molekular, mikrobiologi, genetika, kimia, matematika, dan
berlangsung pada saat gelombang laut tinggi lain sebagainya. (10) Dengan kata lain,
menyebabkan nelayan tidak bisa melaut. (4) Dalam bioteknologi adalah ilmu terapan yang mengabungkan
sekali tangkapan, paling sedikit nelayan membawa berbagai cabang ilmu dalam proses produksi barang dan
lobster 2 kg. (5) “Untuk sekali melaut, satu perahu bisa jasa. (11) Banyak negara menjadikan bioteknologi sebagai
mendapat 4-5 kg, tetapi jika sepi hanya mendapat 2 kg. pertahanan terdepan ketahanan panganannya, khususnya di
(6) Bahkan, beberapa perahu bisa mendapat 7 kg," negara-negara maju. (12) Akan telapi, tidak semua pihak
ujarnya. (7) Dia menuturkan bahwa pendapatan dari dapat menerima bioteknologi karena dianggap bertentangan
menangkap lobster juga lebih besar daripada ikan dengan kodrat alam. (13) Bioteknologi memunculkan
laut. (8) Satu kilogram bisa terjual Rp250.000,00 kontroversi, misalnya bayi tabung, pengklonan manusia, dan
hingga Rp450.000,00. (9) Lobster merah dihargakan transplantasi organ. (14) Kemajuan di bidang
Rp250.000,00 - Rp350.000,00 per kg, lobster hijau bioteknologi tidak terlepas dari berbagai kontroversi yang
Rp250.000,00 - Rp450.000,00 per kg, dan lobster melingkupi perkembangan teknologinya.
mutiara Rp600.000,00 - Rp1.000.000,00 per kg. (10)
Eko Wardi (33), nelayan di Pantai Menganti, 17. Perbedaan tujuan penulisan Teks 3A dengan Teks 3B
mengatakan bahwa selain nilai ekonomi lebih tinggi, adalah…
jarak dan waktu tangkap lobster lebih pendek. (11) A. Teks 3A menjelaskan definisi bioteknologi; Teks
Biasanya untuk melaut, nelayan membutuhkan 20 liter 3B memaparkan ilmu -ilmu yang mendasari
solar, tetapi untuk mencari lobster, mereka hanya bioteknologi.
membutuhkan 3-4 liter. (12) Selain lobster, kata Eko, B. Teks 3A menjelaskan pentingnya pengembangan
beberapa komoditi laut lainnya yang dapat dicari di SDM bioteknologi; Teks 3B menjelaskan
daerah pinggir pantai adalah rajungan dan ubur-ubur. penolakan pemanfaatan bioteknologi.
C. Teks 3A menguraikan peran bioteknologi di bidang menyemarakkan pesta-pesta rakyat, meresmian
pangan; Teks 3B memaparkan peran bioteknologi gedung baru, atau menyambut tamu terhormat.
di bidang kesehatan manusia. D. Dalam perkembangan selanjutnya, pertunjukan
D. Teks 3A memaparkan keunggulan bioteknologi: ondel-ondel juga digunakan untuk menyemarakkan
Teks 3B memaparkan kelemahan bioteknologi. pesta-pesta rakyat, meresmikan gedung baru, atau
E. Teks 3A menjelaskan kendala pengembangan SDM menyambut tamu terhormat.
bioteknologi; Teks 3B menjelaskan kemajuan E. Pertunjukan ondel-ondel juga digunakan untuk
bidang bioteknologi. pesta-pesta rakyat, peresmian gedung baru, atau
menyambut tamu terhormat.
18. Informasi yang ada di dalam Teks 3B, tetapi tidak
dimuat dalam Teks 3A adalah… 20. Vietnam gagal mencetak diri sebagai tim pertama yang
A. Definisi bioteknologi lolos ke babak perempat final setelah pada
B. Pengembangan SDM bioteknologi pertandingan kedua dipaksa bermain imbang 1-1 oleh
C. Penolakan terhadap bioteknologi Qatar di Stadion My Dinh Hanoi. Vietnam yang
D. Cakupan bidang bioteknologi memiliki nilai sempurna setelah mengalahkan Uni
E. Komitmen pemerintah di bidang bioteknologi Emirat Arab 2-0 di partai perdana harus tertekan oleh
Qatar sejak 10 menit paruh pertama. Pada menit ke-20,
19. Saat ini makin banyak kebudayaan Betawi yang tidak penyerang naturalisasi Qatar. Sebastian Andres
dapat dinikmati masyarakat globalisasi. (2) Contohnya Quitana mencetak gol pembuka. Namun sayangnya,
adalah seni pertunjukkan ondelondel. (3) Seni tendangan itu masih dapat di baca dengan baik oleh
pertunjukkan ondel-ondel adalah seni pertunjukkan Duong Hong Son. Delapan menit kemudian. Quitana
dengan boneka besar setinggi dua meter. (4) Pada kembali menjadi momok diserang, justru Vietnam
awalnya, boneka ini berfungsi sebagai penolak bala yang mampu membuka keunggulan lebih dulu. Mereka
dari gangguan roh halus. (5) Dalam perkembangan mencetak gol 10 menit kemudian.
selanjutnya, pertunjukan ondel-ondel juga digunakan Pernyataan yang sesuai dengan bacaan tersebut
untuk menambah semarak pesta-pesta rakyat, adalah….
peresmian gedung baru, atau menyambut tamu A. Vietnam dikalahkan Uni Emirat Arab 20 sehingga
terhormat. (6) Contoh lain kebudayaan Betawi yang harus mengalahkan Qatar.
terancam tergerus globalisasi adalah Lenong. (7) B. Pertandingan antara Vietnam dan Qatar berakhir
Lenong merupakan teater tradisional Betawi yang seri.
diiringi musik gambang kromong. (8) Gambang C. Vietnam terus menyerang Qatar dalam
kromong adalah musik tradisional Betawi yang pertandingan tersebut.
mendapat pengaruh dari suku Tionghoa. (9) Lakon D. Qatar berhasil mencetal gol lebih dahuludari pada
atau scenario lenong umumnya mengandung pesan Vietnam.
moral, seperti membela kaum lemah. (10) Pertunjukan E. Qatar lebih banyak menyerang Vietnam.
itu biasanya diadakan di lapangan terbuka tanpa
panggung. (11) Ketika pertunjukan tersebut
berlangsung, salah seorang aktor atau aktris mengitari
penonton untuk meminta sumbangan secara sukarela.
(12) Pada saat ini, lenong mulai dipertunjukkan atas
permintaan pelanggan dalam acara-acara di panggung
hajatan, seperti resepsi pernikahan. (13) Selain kedua
seni pertunjukan tersebut, budaya arsitektur dan
pertanian Betawi juga mengalami kemunduran. (14)
Contohnya rumah-rumah asli Betawi di kawasan
Condet saat ini terabaikan. (15) Tambahan lagi, rumah-
rumah baru orang Betawi saat ini tidak menggunakan
gaya arsitektur Betawi.
Kalimat perbaikan dari kalimat (5) pada teks di atas
adalah…
A. Dalam perkembangan selanjutnya, pertunjukkan
ondel-ondel yang juga digunakan untuk
menyemarakkan pestapesta rakyat, meresmikan
gedung baru, atau menyambut tamu terhormat.
B. Selanjutnya, pertunjukan ondel-ondel juga
digunakan untuk menyemarakkan pesta-pesta
rakyat, peresmian gedung baru, atau menyambut
tamu terhormat.
C. Dalam perkembangan selanjutnya,
mempertunjukkan ondel-ondel adalah untuk
Contoh :
- sakit  sa – kit
kv kvk
- rajin  ra – jin
kv kvk
- perawan  pe – ra – wan
kv kv kvk

(3) Bila ada 2 konsonan berurut ataupun lebih


maka pisah K1 dan K2
Contoh :
- kursi  kur – si
kvk kv
- pulpen  pul – pen
kvk kvk
- terbang  ter – bang
kvk kvk
- kompromi  kom – pro – mi
kvk kkv kv
- instrumen  in - stru – men
vk kkkv kvk

(B) Kata Berimbuhan


Bila kata berimbuhan pisahkan imbuhan dari kata
dasar kecuali bila berbentuk sisipan. Bila kata itu
dilekati sisipan maka perlakukan seperti kata dasar.
Contoh :
- gemetar  dilekati sisipan - em
ge – me - tar
kv kv kvk
- seruling  dilekati sisipan – er
se – ru – ling
kv kv kvk
- menguasai  kata dasar kuasa
me – ngu – a – sa – i
kv kv v kv v
- menuliskan  kata dasar tulis
EJAAN BAHASA INDONESIA (EBI) - 1 me – nu – lis – kan
kv kv kvk kvk
- mengalami  kata dasar alam
meng – a – lam – i
I. PERSUKUAN kvk v kvk v
Pemisahan suku-suku kata dalam bahasa Indonesia - persatuan  kata dasar satu
haruslah mematuhi ketentuan yang ada dalam Pedoman per – sa - tu – an
Umum Ejaan Yang Disempurnakan. kvk kv kv vk
(A) Kata Dasar
Persukuan kata dasar bila di tengah kata dilakukan II. PENULISAN KATA
dengan tiga cara : (A) Kata Maha
(1) Bila ada 2 vokal berurut maka pisah V1 dan V2 (1) Kata maha bila diikuti kata dasar ditulis
Contoh : serangkai kecuali kata Maha Esa
- baik  ba – ik Contoh :
kv vk - mahaadil
- kuasa  ku – a – sa - mahasiswa
kv – v – kv
- saat  sa – at (2) Kata maha bila diikuti kata berimbuhan ditulis
kv vk terpisah
Contoh :
(2) Bila ada 1 konsonan diapit 2 vokal maka pisah - maha pengasih
sebelum konsonan tersebut. - maha mengetahui
Partikel pun umumnya ditulis terpisah kecuali bila
(B) Gabungan Kata Berimbuhan tergabung dalam :
(1) Gabungan kata yang dilekati awalan dan
akhiran sekaligus, ditulis serangkai AB3 + KMS2 + W
Contoh : Kata – kata tersebut adalah :
- pertanggungjawaban - adapun
- memberitahukan - andaipun
- ketidakadilan - ataupun
- disatupadukan - biarpun
- menyebarluaskan - bagaimanapun
- betapapun
(2) Gabungan kata yang dilekati awalan atau - kalaupun
akhiran saja, ditulis terpisah : - kendatipun
- bertanggung jawab - maupun
- memberi tahu - meskipun
- garis bawahi - sekalipun
- sungguhpun
(3) Gabungan kata yang berupa kata-kata : - walaupun
antar
anti Di luar ketiga belas kata di atas, partikel pun
non ditulis terpisah.
panca Contoh :
pasca dituliskan serangkai - Apa pun yang ia lakukan kami tidak mau
tuna perduli.
tata - Siapa pun yang datang tolong melapor
sub terlebih dahulu kepada piket.
peri
(D) Kata Depan
(C) Penulisan Partikel Kata depan digunakan untuk menunjukkan
(1) Partikel lah , kah ditulis serangkai tempat, menjelaskan keadaan ataupun
Contoh : membandingkan beberapa hal. Kata depan dalam
- apakah bahasa Indonesia ditulis terpisah dengan kata yang
- siapakah mengikutinya.
- marilah (1) Kata depan di dan ke digunakan menunjukkan
tempat
(2) Partikel per Contoh :
Partikel per biasanya ditulis serangkai kecuali - Ibu memasak di dapur.
bila bermakna : - Adik berangkat ke sekolah.
(a) mulai - Di sana mereka bertempat tinggal.
Contoh : - Anak itu berlari ke sana.
 Tahun anggaran 2010/2011 (2) Kata depan dari digunakan untuk :
dilaksanakan per 1 April 2010. (a) menyatakan asal tempat
 Kenaikan gaji pegawai negeri akan Contoh :
dibayar per 1 Mei 2011. - Ia berasal dari Manado.
(b) demi - Paman datang dari Merauke.
Contoh : (b) Menyatakan asal bahan
 Pengunjung harap masuk satu per Contoh :
satu. - Cincin itu terbuat dari emas
(c) tiap - Ember itu dibuat dari plastik
Contoh : (3) Kata depan daripada digunakan untuk
 Pengunjung yang hendak masuk arena membandingkan dua hal.
permainan dikenakan biaya masuk Rp Contoh :
2.000,00 per kepala. - Jakarta lebih ramai daripada Bandung.
 Mangga itu dijual Rp 5.000,00 per - Pemain lawan lebih tangguh daripada
kilogram. pemain kita.
 Peserta pertandingan dikenakan biaya - Daripada duduk melamun lebih baik
pendaftaran Rp 30.000,00 per grup. membaca buku.

(3) Partikel pun


III. PENULISAN BILANGAN
 Satu sampai dua kata ditulis dengan huruf.
 Tiga kata atau lebih ditulis dengan angka. 3. Penulisan gabungan kata yang tidak tepat terdapat pada
 Di awal kalimat menggunakan huruf kalimat ...
 Angka untuk ukuran panjang, berat, luas, isi, satuan A. Menurut pengakuannya, tidak pernah sekali pun ia
menggunakan obat terlarang itu.
waktu, nilai uang, jumlah.
B. Sungguh pun sudah larut malam, keramaian di
 Penulisan bilangan tingkat
kota tersebut masih tetap terasa.
C. Akhirulkalam, kami mengucapkan terima kasih
kepada semua pihak yang telah membantu.
D. Kami menyampaikan ikut belasungkawa atas
wafatnya Bapak Hasan Basri.
E. Adakalanya, pengemudi taksi tersebut membawa
pulang uang komisi yang hanya cukup untuk naik
angkutan umum.

4. Penulisan kata bergaris bawah berikut ini yang tidak


tepat terdapat pada kalimat…
A. Warga komplek membangun gedung pertemuan
secara swadaya.
B. Pada saat pasca panen, harga gabah sering melorot.
C. Setiap bus antarpropinsi harus dalam kondisi laik
jalan
D. Banyak pelajar melakukan kerja sosial untuk
mengisi liburan.
E. Para tunawisma di jembatan lama akan ditertibkan.

5. Penulisan kata yang semuanya BENAR terdapat pada…


A. Kuitansi, kwalitas, jadual
B. Kuitansi, kualitas, jadwal
C. Kuitansi, kualitas, jadual
D. Kwitansi, kualitas, jadwal
E. Kwitansi, kwalitas, jadwal

6. Penulisan gabungan kata berikut yang benar semuanya,


kecuali ….
SOAL PENGANTAR
A. tanggung jawab, kerjasama, tandatangan, terima
kasih
1. Penulisan lambang bilangan yang tepat terdapat pada
B. semipermanen,mancanegara, pascasarjana,
kalimat ...
saptakrida
A. 17 orang, termasuk 4 orang artis di dalamnya,
dicokok oleh BNN dalam penggerebekan Minggu C. acapkali, adakalanya, barangkali, bagaimana
pagi itu. D. mata ajar, meja tulis, orang tua, simpang empat
B. 252 tenaga ahli kebidanan dilatih secara intensif di E. kacamata, dukacita, sapu tangan, halalbihalal
balai pelatihan ini.
C. Di masa tahun 60-an itu, ada istilah khusus untuk 7. Makna gabungan kata merdeka berdaulat dapat juga
berpacaran di tempat-tempat yang sepi dari dilihat pada gabungan kata dalam kalimat ….
perhatian orang, misalnya di taman yang rimbun A. Mereka mengobrol sambil sesekali bercanda
atau di gedung bioskop yang gelap. gurau.
D. Sebanyak 2 per 3 anggota organisasi ini menolak B. Kami merasa senasib sepenanggungan.
pencalonannya menjadi ketua umum. C. Orang itu malu-malu kucing ketika dipersilakan
E. Dari 39 spesies burung cendrawasih, ada yang makan oleh tuan rumah.
berukuran 16 sentimeter, dan ada pula yang D. Dia menjadi hilang semangat setelah mengetahui
berukuran 125 sentimeter. pembalap unggulannya kalah.
E. Pelari itu bersiap-siap sambil pasang kuda-kuda.
2. Gabungan kata yang sesuai dengan EBI terdapat pada ...
A. halal bihalal, manca negara, pascasarjana. 8. Penulisan kata yang benar terdapat pada kalimat di
B. asalamualaikum, kerjasama, bea siswa. bawah ini adalah….
C. sapu tangan, tanda tangan, bagaimana. (1) Indonesia mendapatkan peringkat ke dua pada
D. acapkali, adakalanya, olah raga. olimpiade fisika internasional.
E. mata kuliah, meja tulis, orang tua. (2) Siapa pun tidak boleh mengubah laporan itu.
(3) Pelaku korupsi harus mempertanggung-jawabkan D. Selama ini, setiap penelitian besar, hanya tersedia
tindakannya. dana sekitar Rp250 juta.
(4) Istilah lifestyle berarti gaya hidup E. Wakatobi berjarak lebih dari 2.000 kilo-meter dari
Yogyakarta.
9. Penulisan gabungan kata berikut baku, KECUALI ….
A. mitrabestari; matahari; tatabahasa 15. Kalimat berikut ini yang semua ejaannya ditulis sesuai
B. bilamana; saputangan; darmabakti dengan aturan EYD, adalah ....
C. peribahasa; sumber daya; syahbandar A. Jaringan infrastruktur di negara Ginseng Korea
D. halalbihalal; acapkali; citarasa Selatan berkembang pesat.
E. dukacita; sukaria; tunarungu B. Bus antarkota itu terguling sehingga menewaskan
sepuluh penumpang
10. Penulisan kata dalam kalimat-kalimat berikut benar, C. Organisasi itu dilengkapi dengan sejumlah sub
KECUALI … seksi yang berjenjang
A. Banyak pengguna computer sudah memahami D. Segitiga itu sama sisi, dengan panjang sisi 3 meter
copy dan paste E. Alat pandang dengat itu dibelinya dengan harga
B. Ibu baru membeli baju bayi di took My Baby mahal di Semarang
C. Saat melakukan drag, teks yang dikopu akan
berwarna menyala (highlight) Bacalah soal dengan seksama kemudian kerjakan soal
D. Tiras Harian ‘Kompas’ paling tinggi nomor 16 s.d. 17!
E. Berikutnya. Klik kanan di area yang sudah disorot (1) Dalam ilmu gizi diyakini bahwa pati dicerna dengan
sempurna di dalam usus halus manusia. (2) Akan tetapi,
11. Cara penulisan kata berikut sesuai dengan EBI, teori tersebut sekarang dikoreksi setelah banyak penelitian
KECUALI … baik in vitro maupun in vivo menemukan bahwa tidak semua
A. khazanah pati yang dikonsumsi dapat dicerna dengan sempurna. (3)
B. konkret Kebanyakan makanan berkarbohidrat tinggi seperti: sereal,
C. kanker padi-padian, dan umbi-umbian diolah dengan pemanasan
D. jadwal (dengan atau tanpa adanya air) sebelum dikonsumsi. (4)
E. sejarahwan Pemanasan pati akan mengakibatkan pati mengalami
gelatinisasi, suatu proses yang meliputi hidrasi dan pelarutan
12. Dengan demikian, maka sasaran pemerintah granula pati (Wursch, 1989:234). (5) Pemanasan kembali
menurunkan harga BBM dapat terwujud untuk serta pendinginan pati yang telah mengalami gelatinisasi
mengurangi aksi-aksi kontroversial apa pun yang tersebut dapat mengubah struktur pati yang mengarah pada
mengarah pada sikap anarkis masyarakat Kalimat terbentuknya kristal baru yang tidak larut, yaitu berupa pati
tersebut tidak baku karena .... teretrogradasi yang sering terjadi selama pengolahan bahan
A. Setelah penggunaan kata dengan demikian tidak berpati dapat mempengaruhi ketercernaan pati di dalam usus
diperlukan kata maka. halus. (7) Sebagai contoh sejumlah pati pada kentang,
B. Setelah penggunaan kata dengan demikian tidak pisang, dan kacang-kacangan serta berbagai produk olahan,
diperlukan tanda koma misalnya roti tawar dan corn flakes ditemukan tidak dicerna
C. Penulisan BBM seharusnya B.B.M. dengan sempurna di dalam usus halus manusia dan hewan
D. Penulisan kata apa pun salah. yang ditandai dengan adanya pati dalam digesta (isi usus) di
E. Penulisan kata pada tidak tepat. usus besar.
(8) Pati ini sebut pati tahan cerna atau resistant strach (RS)
13. Penulisan bilangan secara tepat terdapat dalam (9) secara fisiologis, RS didefinisikan sebagai jumlah dari
kalimat… pati dan hasil pencernaan pati yang tidak diserap di dalam
A. Perjalanan itu memakan waktu 5 jam. usus halus individu yang sehat. (10) Secara analitis, RS
B. Sejumlah 21 orang tewas dan 5 orang terluka parah didefinisikan sebagai pati yang tahan terhadap disperse di
dalam kecelakaan itu. dalam air mendidih dan hidrolisis amilase pankreas.
C. Besok saya membeli tiga puluh lima ekor ayam.
D. Di kelas itu ada tiga puluh peserta ujian, yakni 16 16. Pada kalimat manakah kesalahan penggunaan ejaan
pria dan 14 wanita. (tanda baca, penulisan huruf, dan penulisan kata)
E. Sekarang bacalah penjelasannya pada halaman ditemukan ?
1.150. A. 1 dan 3
B. 3 dan 7
14. Kalimat yang di dalamnya dijumpai penulisan bilangan C. 4 dan 7
yang salah adalah .... D. 4 dan 8
A. Dia memanfaatkan lahan 2,5 ha untuk membuat E. 1 dan 8
hutan kecil.
B. Petani singkong Imogiri, mengolah singkong rata- 17. Penulisan kata yang tidak sesuai dengan pedoman ejaan
rata 25 kilogram per dua hari. terdapat dalam kalimat ....
C. Satu proyek membutuhkan 20 tenaga terlatih.
A. buah dan sayuran nonorganik tidak selalu berarti  Dia masih keturunan Sultan Hasanuddin.
tidak sehat (4) Awal nama gelar keagamaan diikuti nama orang
B. puting beliung telah memporakporandakan Contoh :
kawasan perumahan yang baru saja dibangun  Dia tinggal di rumah Haji Jeffry.
C. buku ini memberi tahu para pembaca tentang  Saya sudah lama berkenalan dengan Pendeta
pemikiran brilian sang proklamator Belferick.
D. rompi antipeluru ini selalu mereka pakai ketika (5) Awal nama khas geografi
melakukan operasi Contoh :
E. la dapat menyelesaikan tugas itu dengan baik  Selat Sunda
karena jalan pikirannya sangat sistematis  Pulau Bali
 Sungai Musi
18. Penulisan partikel berikut yang tidak tepat adalah…  Jawa Barat
A. Buah jeruk itu dijual seharga Rp 4.000,00 per  Danau Toba
kilogram.  Gunung Galunggung
B. Siapapun yang datang tolong melapor ke piket. (6) Awal petikan langsung
C. Apa pun ia lakukan asal keluarganya bisa makan. Contoh :
D. Bagaimanapun ia masih saudaramu.  Adik berkata, “Di manakah ibu menyimpan
E. Dari penjualan tanah itu ia mendapat dua perenam buku ?”
bagian.  “Ibu sangat senang,” kata ibu, “karena kamu
lulus.”
19. Penulisan gabungan kata berikut ini yang tepat adalah… (7) Awal nama dokumen resmi
A. dibebaskan tugas Contoh :
B. memutar balikkan  Piagam Jakarta
C. menganaksungai  Konfrensi Asia Afrika
D. ketidakadilan  Liga Bank Mandiri
E. pertanggungan jawab (8) Awal singkatan bila yang digunakan adalah huruf
awal tiap kata.
20. Persukuan kata berikut ini yang tepat adalah… Contoh :
A. ak – hir  KTT (Konfresi Tingkat Tingkat)
B. me – ngu – la – ngi  MPR (Majelis Permusyawaratan Rakyat)
C. meng – a – lam – i  PJPT I (Pembangunan Jangka Panjang Tahap
D. pen – du – du – kan I)
E. a – pril (9) Awal singkatan berupa akronim bila singkatan itu
merupakan singkatan nama suatu program.
Contoh :
 Repelita (Rencana Pembangunan Lima Tahun)
 Inpres (Instruksi Presiden)
 Kepres (Keputusan Presiden)
EJAAN BAHASA INDONESIA (EBI) - 2 (10) Awal tiap kata judul karangan, buku, novel kecuali
kata tugas. Namun bila kata tugas itu di awal judul,
maka ditulis juga dengan huruf kapital.
I. HURUF KAPITAL Contoh :
Huruf kapital digunakan untuk menuliskan : Dari Ave Maria ke Jalan Lain ke Roma
(1) Awal nama suku, bangsa, bahasa Cintaku di Kampus Biru
Contoh : Yang Terhempas dan yang Putus
 bahasa Melayu (11) Awal kata-kata sapaan
 suku Dayak Contoh :
 bangsa Indonesia Bolehkah saya tahu nama Adik?
(2) Awal nama hari, bulan, tahun Di manakah rumah Tuan?
Contoh : Sungguh Saudara orang yang berbudi.
 hari Rabu
II. TANDA TITIK ( . )
 bulan Januari
(A) Tanda titik digunakan pada :
 tahun Masehi
(1) Akhir kalimat berita
(3) Awal nama pangkat, jabatan, gelar bila diikuti
Contoh :
nama orang / nama daerah
 Putra sulungnya kuliah di ITB.
Contoh :
 Hasil ujian mereka sangat memuaskan.
 Ia anak Jendral Haryono.
(2) Akhir singkatan:
 Besok Gubernur Syamsul Arifin akan dilantik. (a) nama orang
 Besok Gubernur Sumatera Utara akan datang.
Contoh :  ITB (Institut Teknologi Bandung)
 H. Nasution (c) Singkatan nama lambang kimia
 Moh. Yamin Contoh :
(b) pangkat  Cu (Sulfur)
Contoh :  CO2 (Karbondioksida)
 Kol. Sutanto  Mg (Magnesium)
 Kapt. Haryadi (d) Singkatan satuan ukuran
(c) gelar keagamaan Contoh :
Contoh :  kg (kilogram)
 H. Komarudin  g (gram)
 Hj. Siti Nuraini  m (meter)
 Pdt. Marcus Federick  km (kilometer)
(d) gelar akademik  (liter)
Contoh : (e) Singkatan yang berupa Akronim
 Drs. Amir Syafudin, S.H. Akronim adalah singkatan yang sudah
 Prof. Dr. Andika, M.B.A. dianggap sebagai kata. Akronim dibaca bukan
(e) kata-kata umum dieja huruf per huruf.
Contoh : Contoh :
 a.n. (atas nama)  Puskesmas (Pusat Kesehatan Masyarakat)
 u.b. (untuk beliau)  Mendagri (Menteri Dalam Negeri)
 yth. (yang terhomat)  Inpres (Instruksi Presiden)
 dll. (dan lain-lain)  Poskamling (Pos Keamanan Keliling)
 hlm. (halaman)
(3) Memisahkan jam, menit, dan detik III. TANDA TITIK KOMA ( ; )
Contoh : Tanda titik koma dapat dipakai sebagai pengganti kata
 waktu sekarang : pukul 23.00.00 penghubung untuk memisahkan kalimat yang setara
 jangka waktu : 2.30.50 jam dalam kalimat majemuk.
(4) Memisahkan angka ribuan, jutaan, dan Contoh:
seterusnya bila menyatakan jumlah  Malam makin larut ; pekerjaannya belum selesai.
Contoh :  Kakak membaca buku di ruang tengah ; ibu membaca
 Rumah ini akan dijual seharga Rp buku di ruang tamu.
105.000.000  Ayah mengurus tanamannya di kebun itu ; ibu sibuk
 Buku itu tebalnya 1.125 halaman bekerja di dapur.

SOAL PENGANTAR
(B) Tanda titik tidak digunakan pada :
(1) Penulisan bilangan yang tidak menyatakan jumlah. 1. Penggunaan tanda titik koma (;) yang tepat terdapat
Contoh : pada kalimat ...
 Coba buka halaman 1125 ! A. Nyatanya, betul ada mutasi yang kerap muncul
 Ia lahir tahun 1993 di Medan. dalam berbagai diskusi soal ini; sebuah varian gen
bernama DRD4.
(2) Akhir singkatan yang berupa : B. Populasi satwa di taman nasional tersebut terus
(a) singkatan mata uang mengalami penurunan sejak tahun 1990; sungai-
Contoh : sungai di wilayah itu sering meluap pada musim
Rp (rupiah) hujan akibat sedimentasi.
¥ (Yen) C. Tempat hunian mereka pun berpindah dari gua ke
U$ (Dolar Amerika Serikat) tempat terbuka sembari mendomestifikasi hewan;
(b) Singkatan pada lembaga umum dan singkatan bercocok tanam; dan membuat peralatan.
yang mengambil huruf awal tiap kata. D. Menurut banyak ahli, kematian James Cook di
Contoh : Hawaii menutup era penjelajahan samudra; itu
 DPR (Dewan Perwakilan Rakyat) tidak menghentikan penjelajahan manusia pada
 MA (Mahkamah Agung) masa-masa kemudian.
 KKN (Korupsi, Kolusi, Nepotisme) E.Selama dua minggu, tim Truman Simanjuntak
 SMU (Sekolah Menengah Umum) menginap di rumah penduduk di Padangbiru;
 PT (Perseroan Terbatas) selanjutnya temuan-temuan mereka dihamparkan di
 KTP (Kartu Tanda Penduduk) lantai dengan alas kertas koran.
 SMU (Sekolah Menengah Umum)
 UI (Universitas Indonesia) 2. Dalam upaya redenominasi bank sentral akan merilis
 UGM (Universitas Gajah Mada) uang baru, tetapi ketiga nolnya dihilangkan. Misalnya
Rp1.000,00 nanti akan menjadi Rp1,00. Namun, nilai (5) Benarkah semangat perjuangan kedua perempuan
Rp1,00 ini akan sama dengan Rp1.000,00. tersebut tetap berada pada jalurnya? (6) Kali ini, dalam
Konsekuensinya, nilai rupiahN berupa sen akan muncul rangka memperingati hari lahir R.A. Kartini, kami akan
kembali. membicarakan kiprah sejumlah wanita Indonesia.
Kata konsekuensi pada kalimat di atas dapat diganti Dua tanda baca yang salah dalam paragraf di atas
dengan kata ... adalah ….
A. hasil. A. tanda koma sesudah kata lalu kalimat (1) dan
B. dampak. tanda koma sesudah kata pendidikan pada kalimat
C. efek. (2)
D. karena. B. tanda koma sesudah kata rasakan pada kalimat (3)
E.pengaruh. dan tanda titik koma sesudah kata pendidikan pada
kalimat (4)
3. Penulisan tanda baca yang sesuai dengan EYD adalah… C. tanda titik pada kata R.A. Kartini pada kalimat (1)
A. Rp 1.000.000,00 dan tanda titik pada kalimat (3)
B. Rp1.000.000,- D. tanda koma sesudah kata ini dan tanda koma
C. Rp 1.000.000,- sesudah kata Kartini pada kalimat (6)
D. Rp 1.000.000,00 E.tanda titik pada kalimat (2) dan tanda tanya pada
E.Rp1.000.000,00 kalimat (5)

4. Kalimat berikut ditulis dengan huruf kapital yang benar, 7. Penanganan masalah pendidikan diantaranya ditempuh
KECUALI … dengan membangun SD kecil untuk melayani
A. Bila berkunjung ke Makassar, Ibu Kota Provinsi kebutuhan pendidikan si daerah terpencil yang
Sulawesi Selatan, jangan lupa mampir di toko dilakukan pada pelita V, di samping SD regular di
cenderamata. wilayah-wilayah yang padat penduduk.
B. Sepanjang perjalanan, kapal yang kamu tumpangi Ejaan pada kalimat di atas menjadi benar jika diperbaiki
ke Kepulauan Krakatau terus diguncang dengan cara ...
gelombang. (A) Menulis kata diantaranya menjadi di antaranya.
C. Kebebasan dapat anda rasakan saat bersepeda di (B) Menulis kata pelita V menjadi PELITA V.
jalan Lingkar Luar Jakarta. (C) Menghilangkan tanda koma (,) setelah kata pelita
D. Perjalanan kami lanjutkan ke Desa Pandes V.
Sempokan. (D) Menulis kata di samping menjadi disamping.
E.Di antara dominasi warna terakota keramik dan (E) Menulis kata reguler menjadi regular.
gerabah yang memenuhi sisi jalan, kami
menemukan Toko Hasta Kreasi yang menjual 8. Bacalah paragraf berikut dengan saksama.
patung keramik. Sampai akhir bulan juni, badan meteorologi dan geofisika
5. Tanda titik koma digunakan secara tepat dalam mencatat curah hujan di bawah normal telah terjadi di
kalimat… sebagian besar pulau Jawa. Oleh sebab itu, disarankan
A. Suntikan obligasi dapat mendatangkan pendapatan masyarakat mengantisipasi kemungkinan terjadinya
yang pasti; penghilangan kredit menjadikan neraca kelangkaan air di sebagian wilayah pulau Jawa tersebut.
per-bankan bersih. Perbaikan yang tepat untuk penulisan yang tercetak miring
B. Kehadiran obligasi rekap memberikan tiga pada paragraf tersebut adalah ....
keajaiban; pendapatan yang pasti, kolektibilitas A. Bulan juni, badan Metearologi dan Geofisika,
yang lancar, dan pencadangan yang kecil. pulau Jawa
C. Pertama, bank memasyarakatkan produk dan B. Bulan Juni, Badan meteorologi dan geofisika,
layanan jasa kepada masyarakat; menyalurkan Pulau jawa
kredit ke sektor perdagangan, jasa, transportasi, dan C. bulan Juni, Badan meteorologi dan geofisika,
industri. pulau Jawa
D. Bank itu dinyatakan palait awal tahun ini; padahal D. bulan Juni, Badan Meteorologi dan Geofisika,
kinerjanya dianggap baik tahun lalu. Pulau Jawa
E.Karena modal di bank terbatas; tidak semua E.bulan Juni, Badan Meteorologi Dan Geofisika, Pulau
pengusaha lemah memperoleh kredit Jawa

6. (1) Hampir satu setengah abad lalu, Dewi Sartika dan 9. Penggunaan huruf kapital yang betul terdapat pada
R.A. Kartini memperjuangkan emansipasi. (2) Perjuang kalimat…
mereka terbatas pada hak perempuan terhadap akses A. Kita harus berusaha menggunakan Bahasa
pendidikan, bukan lainnya. (3) Buah perjuangan mereka Indonesia yang baik dan benar.
dapat kita rasakan, saat ini. (4) Kaum perempuan B. Pada Bulan Agustus ia akan berangkat ke Amerika.
sekarang mempunyai akses yang luas terhadap C. Di mana banyak terdapat Suku Jawa?
pendidikan; dari pendidikan dasar hingga universitas.
D. Pegunungan yang membentang di daratan D. hari Minggu
Sumatera itu bernama Bukit Barisan. E. Tahun Baru
E.Dengan gembira masyarakat menyambut hari lebaran.
10. Penggunaan huruf kapital pada kalimat-kalimat berikut 16. Pemakaian tanda titik yang sesuai dengan EYD
ini benar, kecuali… adalah…
A. Ia masih keturunan bangsawan yang bergelar A. Saya lahir tahun 1976 di Medan.
Raden Mas. B. Harga gula saat ini Rp. 4.200,00 per kg.
B. Rumah saya berdekatan dengan rumah Haji C. M.P.R sedang bersidang hari ini.
Komar. D. Setiap siswa S.M.U yang mau lulus SNMPTN
C. Rapat kerja ini dibuka oleh Gubernur Suryadi. harus rajin belajar.
D. Anak muda itu ternyata adik Kopral Jono. E. Lihat halaman 1.250 buku itu !
E.Saya pernah bertemu dengan Profesor Sumadi.
17. Penulisan singkatan yang benar menurut PUEYD
11. Penulisan kalimat di bawah ini yang sesuai dengan adalah …
EYD adalah, kecuali… A. d/a ( dengan alamat)
A. Semoga kamu berhasil. B. s. d. a ( sama dengan di atas)
B. Cintaku di Kampus Biru C. Mg. (Magnesium)
C. Tiga orang mereka datang ke pesta itu. D. an. (atas nama)
D. Bila jumlah yang hadir sudah mencapai dua pertiga E. s.d. (sampai dengan)
maka sidang akan segera dimulai.
E.Dia hanya menduduki peringkat ke – X dalam 18. Yang termasuk akronim adalah, kecuali …
kejuaraan itu. A. ABRI
B. PSSI
12. Pemakaian huruf kapital untuk huruf pertama kata C. USU
saudara dalam kalimat- kalimat berikut ini tepat, D. PASI
kecuali… E. Puskesmas
A. Terima kasih atas bantuan Saudara!
B. Sebenarnya saya mempunyai tiga orang Saudara. 19. Fungsi tanda titik yang tepat terdapat dalam kalimat,
C. “Mengapa Saudara kali ini datang terlambat?” sapa kecuali…
Anton . A. Prof. Dr. Riwandy, S.E. menghadiri acara
D. Saudaranya yang paling sulung tinggal di pertemuan itu.
Pontianak. B. Paman sudah lama tinggal di Jakarta.
E.“Sebenarnya saya belum tahu nama Saudara, C. Buku itu terdiri dari 2.112 halaman.
bolehkah saya mengetahuinya?” D. Dia berangkat ke sekolah pada pukul 06.45.30.
setiap pagi.
13. Penggunaan huruf kapital yang benar terdapat pada …. E. Pagi ini Ayah akan menemui Sdr. Anto di
A. “Siapakah nama Ibu Saudara?” tanya petugas. kantornya.
B. “Di manakah rumah Bapak?” tanya orang itu. 20. Semua penggunaan tanda titik di bawah ini benar,
C. Di warung itu tidak ada pisang Ambon. kecuali…
D. Masalah itu menjadi urusan pak lurah. A. Ratih S., S.S.
E. Mereka memakai Bahasa Jerman. B. P.T. Bahagia
C. dkk.
14. Penggunaan huruf kapital pada kalimat di bawah ini D. a.n.
benar , kecuali….. E. hlm.
A. Rumah saya berdekatan dengan rumah Haji
Fadillah.
B. Mandra pernah bertemu dengan Profesor Sumantri.
C. Rapat itu dibuka oleh Presiden Megawati Soekarno
Puteri.
D. Anak itu ternyata adik Kopral Rudiman.
E. Ia masih keturunan bangsawan yang bergelar
Pangeran.

15. Penulisan huruf kapital antara lain digunakan dalam


penulisan huruf pertama, nama hari, bulan dan tahun.
Penulisan huruf kapital yang benar terdapat dalam
pernyataan di bawah ini adalah, kecuali …
A. hari Lebaran
B. bulan Mei
C. tahun Masehi
- ke- dengan angka (peringkat ke-2);
- angka dengan –an (tahun 1950-an);
- kata atau imbuhan dengan singkatan yang
berupa huruf kapital (hari-H, sinar-X, ber-KTP,
di-SK-kan);
- kata dengan kata ganti Tuhan (ciptaan-Nya, atas
rahmat-Mu);
- huruf dan angka (D-3, S-1, S-2); dan
- kata ganti -ku, -mu, dan -nya dengan singkatan
yang berupa huruf kapital (KTP-mu, SIM-nya,
STNK-ku).
UI 2009 - dipakai untuk merangkai unsur bahasa
Indonesia dengan unsur bahasa daerah atau
bahasa asing.
Contoh :
di-sowan-i (bahasa Jawa, 'didatangi')
ber-pariban (bahasa Batak, 'bersaudara sepupu')
di-back up
me-recall
- digunakan untuk menandai bentuk terikat yang
menjadi objek bahasan.
Contoh :
Kata pasca- berasal dari bahasa Sanskerta.
Akhiran -isasi pada kata betonisasi sebaiknya
diubah menjadi pembetonan.
EJAAN BAHASA INDONESIA (EBI) - 3
II. TANDA PISAH (—)

I. TANDA HUBUNG (-) Pengunaan tanda pisah menurut PUEBI adalah sebagai
berikut
Pengunaan tanda hubung menurut PUEBI adalah sebagai (a) Dipakai untuk membatasi penyisipan kata atau
berikut kalimat yang memberi penjelasan di luar bangun
(a) Dipakai untuk menandai bagian kata yang kalimat.
terpenggal oleh pergantian baris. Contoh :
Contoh : Kemerdekaan bangsa itu—saya yakin akan tercapai
Kini ada cara yang baru untuk meng- —diperjuangkan oleh bangsa itu sendiri.
ukur panas. (b) Dipakai juga untuk menegaskan adanya keterangan
(b) Dipakai untuk menyambung unsur kata ulang. aposisi atau keterangan yang lain.
Contoh : Contoh :
Rumah-rumah, kekuning-kuningan Soekarno-Hatta—Proklamator Kemerdekaan RI—
(c) Dipakai untuk menyambung tanggal, bulan, dan diabadikan menjadi nama bandar udara
tahun yang dinyatakan dengan angka atau internasional.
menyambung huruf dalam kata yang dieja satu-satu. (c) Tanda pisah dipakai di antara dua bilangan, tanggal,
Contoh : atau tempat yang berarti 'sampai dengan' atau
12-12-2012, c-i-n-t-a 'sampai ke'.
(d) Dipakai untuk memperjelas hubungan bagian kata Contoh :
atau ungkapan. Tahun 2010—2013
Contoh : Tanggal 5—10 April 2013
ber-evolusi, meng-ukur, dua-puluh-lima ribuan (25 Jakarta—Bandung
x 1.000), 23/25 (dua-puluh-tiga perdua-puluh-lima),
mesin hitung-tangan III. TANDA KOMA (,)
(e) Dipakai untuk merangkai Pengunaan tanda koma menurut PUEBI adalah sebagai
- se- dengan kata berikutnya yang dimulai berikut
(a) Memisahkan unsur-unsur dalam suatu perincian
dengan huruf kapital (se-Indonesia, se-Jawa
atau pembilangan.
Barat); Contoh:
 Ayah membeli kertas, pena, dan tinta.
(b) Sebelum kata penghubung melainkan dan tetapi
Contoh:
 Dini bukan anak saya, melainkan anak Pak
Hasan.
 Agnes anak pintar, tetapi tidak sombong.
(c) Memisahkan anak kalimat dari induk kalimat
apabila anak kalimat mendahului induk kalimat.
Contoh:
 Kalau hujan turun, acara akan dibatalkan.
 Karena sibuk, paman tidak jadi datang.
Catatan:
Tanda koma tidak dipakai apabila anak kalimat
mengiringi induk kalimat.
Contoh:
 Acara akan dibatalkan kalau hujan turun.
 Paman tidak jadi datang karena sibuk.
(d) Memisahkan petikan langsung dari bagian lain
dalam kalimat.
Contoh:
 Kata ibu, “Saya sangat senang.”
 Ayah bertanya, “Siapa namamu, Nak?”
(e) Mengapit keterangan tambahan atau keterangan
aposisi
SOAL PENGANTAR
Contoh:
 Jakarta, ibu kota RI, berhawa panas. 1. Pemakaian tanda hubung di bawah benar, kecuali
 Guru kami, Pak Samsudin, pandai sekali. A. Ani menduduki peringkat ke-12
(f) Memisahkan antara nama dan gelar akademik. B. Bulan ini akan diadakan musyawarah gubernur se-
Contoh: Indonesia.
 Shinta Dewi, S.E. C. Ia harus segera memperpanjang KTP-nya.
 Prof. H. Rangkuti, M.B.A. D. Kami ujian di sekolah mulai hari Senin – Kamis.
(g) Di belakang kata atau ungkapan penghubung antar E. Rumah-rumahan didirikan di daerah perkotaan itu.
kalimat yang terdapat pada awal kalimat termasuk
di dalamnya oleh karena itu, jadi, lagi pula, 2. Penggunaan tanda koma yang tepat terdapat pada
meskipun begitu, akan tetapi. kalimat…
Contoh: A. Dia lupa akan janjinya, karena sibuk.
 Oleh karena itu, kita harus berhati-hati. B. Anak itu berpendapat, bahwa soal itu tidak penting.
 Jadi, soalnya tidak semudah itu. C. Anak itu malas, sehingga tidak naik kelas.
(h) Tanda koma dipakai untuk memisahkan kata seperti D. Saya tidak akan datang, kalau hari hujan.
o, ya, wah, aduh, kasihan dari kata lain yang E. Saya ingin datang, tetapi hari hujan.
terdapat dalam kalimat.
Contoh: 3. Bacalah paragraf berikut!
 O, begitu. Kios milik Pak Anto selalu ramai dikunjungi pembeli.
 Wah, bagus sekali permainanmu. Kios milik Pak Anto menjual berbagai macam buah dan
(i) Memisahkan antara nama dan alamat, bagian- sayuran. Pak Anto menjual apel, jeruk, papaya, manga,
bagian alamat, tempat dan tanggal. dan pisang kangkung, bayam, lobak.
Contoh: Tanda baca yang tepat untuk melengkapi paragraf
 Shanty tinggal di Jln. Dahlia No. 5, Medan, tersebut adalah…
Sumatera Utara. A. Tanda koma (,) setelah kata pisang pada kalimat
 Sdr. Edu, Jalan Permai 28, Bandung terakhir.
(j) Memisahkan bagian nama yang dibalik dalam daftar B. Tanda pisah (—) setelah kata pisang pada kalimat
pustaka. terakhir.
Contoh:
C. Tanda hubung (-) setelah kata pisang pada kalimat
 Keraf, Gorys. 1984. Tata Bahasa Indonesia.
Jakarta: Nusa Indah. terakhir.
D. Tanda titik dua (:) setelah kata pisang pada kalimat
terakhir.
E. Tanda titik koma (;) setelah kata pisang pada E. Baloteli men—tackle lawan sehingga ia dapat kartu
kalimat terakhir. kuning.

8. Penggunaan tanda baca secara tepat terdapat pada


4. Bacalah paragraf berikut! kalimat berikut, kecuali…
Kementrian pertanian akan meningkatkan produksi 3
komoditas pertanian. Peningkatan produksi tersebut A. Lima puluh tiga-perempat sama dengan 50 4
harus diimbangi dengan tingkat kesejahteraan petani B. Kabarnya Kabinet Indonesia Bersatu Jilid II akan di-
seIndonesia. Selama ini kesejahteraan para petani masih resuffle.
belum diperhatikan karena pemerintah hanya focus C. Rianto, putra Pak Bonar diterima di ITB.
meningkatkan produksi di sector pangan. D. Tampaknya Chelsea dapat juara ke-1.
Tanda baca yang tepat untuk melengkapi paragraf E. KTP-nya sudah tidak berlaku lagi.
tersebut adalah…
A. Tanda koma (,) pada kalimat pertama setelah kata 9. Tanda koma yang benar penggunaannya terdapat pada
pertanian. kalimat…
B. Tanda titik dua (:) pada kalimat pertama sesudah A. Aminah sedang pergi ke sekolah, ketika ayah
membelikan ibu sebuah baju baru.
kata meningkatkan.
B. Jakarta, Ibu Kota Republik Indonesia berhawa panas.
C. Tanda hubung (-) pada kalimat kedua setelah di C. Saudara Prof. Dr. Maryam Darus, S.S. sedang
antara se dan Indonesia. memberikan ceramah di gelanggang mahasiswa.
D. Tanda pisah (—) pada kalimat kedua setelah kata D. Fauzan pintar tetapi, ia tidak sombong kepada
petani dan sebelum kata oleh. kawan-kawannya.
E. Ibu, berkata “Ayah pulang larut malam.”
E. Tanda koma (,) pada kalimat ketiga setelah kata ini
dan sebelum kata pemerintah. 10. Tanda koma hanya dapat digunakan pada …
A. Mengapit keterangan aposisi.
5. Penggunaaan tanda hubung yang tepat terdapat dalam B. Memisah gelar kesarjanaan dengan nama orang.
kalimat, kecuali… C. Pada akhir kalimat berita.
D. Penulisan nomor surat resmi.
A. Medan – Jakarta E. Memisah angka jam, menit, detik yang menyatakan
B. 24-12-1986 waktu.
C. berlari-lari
D. istri lurah-baru 11. Kalimat yang ditulis dengan benar terdapat pada
E. se-Indonesia kalimat…
A. Ia benar-benar tidak menyadari, bahwa perbuatannya
itu menyinggung perasaan orang lain.
6. Perhatikan kalimat-kalimat berikut ini! B. Agar memudahkan pengiriman, jamu itu dimasukkan
1) Kesuksesan itu — saya yakin — bisa dicapai dengan dalam satu kotak karton besar.
kerja keras, tanggung jawab, dan pantang menyerah. C. Bersama para Bupati di istana, rombongan dari
2) Pameran buku sekolah dan umum akan dilaksanakan Thailand mengikuti jamuan makan malam.
pada tanggal 1—7 Oktober 2016 di Balai Kartini. D. Karena tidak memahami topic, Dina tidak dapat
3) Pancasila — yang diyakini sebagai dasar negara — memberi saran apapun.
harus diamalkan dalam kehidupan sehari-hari. E. Selain terbit di Sentra Media, buku-buku bahasa
4) Kereta api eksekutif tujuan Jakarta — Surabaya Indonesia karangannya juga terbit di C.V. Grafika
mengalami keterlambatan jadwal pemberangkatan. Indah.
5) Menteri koordinator — Bidang Kemaritiman Luhut
Binsar Panjaitan — meninjau Bandara Silangit. 12. Tanda koma digunakan secara benar dalam kalimat
Penulisan tanda baca pisah yang tidak tepat ditunjukkan berikut, kecuali…
oleh kalimat… A. Jika diperhatikan secara cermat, data penelitian ini
A. 1 ada dua, yaitu x dan y.
B. 2 B. Namun perlu disadari, bahwa ego pribadi harus
C. 3 dikendalikan agar tidak merugikan orang lain.
D. 4 C. Sebenarnya ia tidak terlibat dalam kegiatan ini, tetapi
E. 5 ia selalu mengikutinya dengan baik.
D. Walaupun semua tugas sudah dikerjakan, ia tetap
7. Penggunaan tanda pisah yang tepat terdapat dalam rajin mmembaca buku di perpustakaan.
kalimat… E. Komponen sistem pembelajaran meliputi tujuan,
A. Atlet angkat besi itu berhasil memecahkan rekor materi, media, metode, dan evaluasi.
dunia pada angkatan ke—2.
B. Medan — Ibu Kota Provinsi Sumatera Utara — 13. Penggunaan tanda koma yang salah terdapat pada
bertetangga dengan Nanggore Aceh Darusallam. kalimat…
C. Guru adik — Pak Joko menjadi guru teladan.
D. Abad ke — 21 ini cuaca sangat ekstrem.
A. Perubahan politik di Jerman Timur yang ditandai E. Orang Dayak, baik laki-laki maupun perempuan,
dengan menurunnya kekuasaan komunis, mencapai memiliki jiwa ksatria, pemberani, dan pantang
klimaks pada pembukaan tembok Berlin. menyerah.
B. Harga gula yang makin gila, disertai dengan harga
kenaikan beras di tengah keributan para elite bangsa 16. Pemakaian tanda baca yang tidak tepat terdapat dalam
ini mengenai impor beras, merupakan cermin kalimat…
bagaimana pemerintah kurang melindungi A. Jika terjadi pelanggaran kode etik, advokat yang
masyarakat. terbukti melakukan pelanggrana akan mendapat
C. Namun, terlepas dari pro dan kontra yang sanksi dari organisasi profesi.
mengemuka, forum itu menegaskan bahwa B. Apakah semua organisasi advokat PERADI, KAI,
pemberantasan korupsi di segala aspek tidak boleh dan PERADIN bersedia menjalan kode etik?
surut, siapa pun pelakunya. C. Wadah tunggal organisasi profesi advokat akan
D. Di atas meja, misalnya, dapat diamati himpunan memudahkan sosialisasi, monitoring, dan evaluasi
benda-benda seperti jarum, pena, benang, kertas, Kode Etik Advokat Indonesia — yang notabene
garpu, buku tulis, tinta, paku, dan dapat diperbanyak dirumuskan PERADI — dalam pelaksanaanya.
lagi dengan benda-benda lain. D. Secara empiris, memang ada kode etik profesi yang
E. Pasalnya, selama sepulub tahun berkiprah di dijalankan oleh beberapa organisasi profesi —
Indonesia, Jacksen termasuk pemain sepak bola asing misalnya — Kode Etik Guru Indonesia.
dengan prestasi paling bagus. E. Menjaga profesionalisme advokat merupakan
komitmen pemerintah, dan warga negara wajib ikut
14. (1) Selain pameran lukisan bertajuk Mengeja Laku mendorong penegakan hokum Indonesia.
Liku Kaliku, komunitas peduli sampah selaku
penyelenggara Festival memamerkan berbagai barang 17. Penggunaan tanda hubung yang benar terdapat pada,
yang diproduksi dari sampah sungai, seperti tas, kecuali…
keranjang, vas bunga, dan lampion. (2) Berbagai seni A. Medan – Jakarta
pertunjukan juga digelar selama Festival berlangsung, B. 17 – 08 – 2019
seperti acara tradisi palang pintu Betawi, atraksi manusia C. di-download
petasan di atas bukit, debus, karinding, dan lenong. (3) D. dukun - beranak dua = artinya dukun beranak ada dua
Bahkan, partisipasi juga dilakukan oleh warga, orang
pengusaha, aktivis, dan pelaku seni dalam lomba, seperti E. reformasi terjadi di tahun 90-an.
lomba memancing sampah dan mewarnai. (4) Festival
semakin semarak dengan sajian berbagai tari tradisional 18. Setiap hari Sabtu bis trayek Medan — Sidikalang penuh
dan kuliner. sesak. Pemakaian tanda pisah pada kalimat tersebut
(5) Mereka menggelar kegiatan budaya ini untuk sudah tepat.
meningkatkan kepedulian wargaDesa Cisadane. (6) SEBAB
Pinggiran Kali Cisadane dipilih sebagai lokasi acara,
karena kondidi sudah memprihatinkan. (7) Jadi, dengan Tanda pisah pada kalimat tersebut bermakna sampai
pameran dan pegelaran seni budaya ini, semua pihak, dengan.
baik para pengusaha, aktivitas dan pegiat lingkungan,
pelaku seni, maupun masyarakat luas diajak untuk 19. Untuk membentuk kalimat yang baku, tanda baca (,)
bersinergis menjaga, merawat, dan melestarikan sungai perlu digunakan kecuali pada kalimat…
yang selama ini menjadi sumber kehidupan warga yaitu A. Computer dan barang elektronik lainnya,
kebutuhan air baku. (8) Warga tepi Kali Cisadane mengandung bagian tidak berbahaya dalam
menyambut antusias kegiatan budaya ini. penggunaan sehari-hari tetapi jadi beracun jika tidak
Penggunaan tanda baca koma yang salah terdapat pada dibuang dengan tepat.
kalimat… B. Pabrik-pabrik yang akan dibangun harus memiliki
A. (1) D. (6) prasarana untuk pengumpulan e-waste namun ada
B. (2) E. (8) pabrik yang tidak mengindahkan syarat tersebut.
C. (3) C. Beberapa negara, termasuk Cina, telah melarang
impor sampah elektronik.
15. Pemakaian tanda baca yang tidak sesuai dengan EBI D. Daur ulang terbukti lebih ramah lingkungan daripada
terdapat dalam kalimat… penguburan atau pembakaran sampah, tetapi hal ini
A. Sebelum mengisi soal, panitia memberikan tidak disadari oleh sebagian besar masyarakat.
pengarahan kepada para peserta. E. Sejauh ini, Eropa telah memimpin dunia dalam
B. Dalam hal in, karakteristik wilayah dan kemampuan bahan-bahan kemasan daur ulang.
tanah harus sungguh-sungguh diperhatikan, agar
rekomendasi yang dihasilkan dalam penataan ruang 20. Penggunaan tanda koma pada kalimat berikut tidak
dapat sesuai dengan pengunaanya. benar, kecuali…
C. Dosa besar dalam kehidupan orang Dayak adalah A. Tak ubahnya Jakarta, jalanan Kota Boston pun ruwet,
merampas, mencuri, dan merampok. karena tak mudah dijelajahi dengan mobil.
D. Dalam ujian calon hakim, tiap-tiap peserta mendapat B. Pada tahun 2006, untuk ketiga kalinya, majalah
soal skolastik dan pengetahuan umum. Bicycling memasukkan kota ini ke dalam daftar kota
terburuk AS bagi pesepeda.
C. Menino akhirnya memahami manfaat dari budaya  Marilah kita menyanyikan lagu "Maju Tak
ramah sepeda demi lingjungan, yaitu mengurangi Gentar"!
kepadatan lalu lintas dan menyehatkan masyarakat.
D. Kota-kota, mulai dari Louisville hingga Los Angeles,  Film "Ainun dan Habibie" merupakan kisah
menawarkan kemudahan akases bersepeda, dan nyata yang diangkat dari sebuah novel.
wisatawan adalah sasran utama fasilitas tersebut.  Saya sedang membaca "Peningkatan Mutu Daya
E. Sejumlah larangan membatasi keberadaan sepeda di Ungkap Bahasa Indonesia" dalam buku Bahasa
jalanan umum, sehingga memudahkan akses ke
Indonesia Menuju Masyarakat Madani.
pedalaman, atau menggenjot naik menyusuri pesisir
sepanjang Old Highway.  Makalah "Pembentukan Insan Cerdas
Kompetitif" menarik perhatian peserta seminar.
 Perhatikan "Pemakaian Tanda Baca" dalam
buku Pedoman Umum Ejaan Bahasa Indonesia
yang Disempurnakan.

(c) Dipakai untuk mengapit istilah ilmiah yang kurang


dikenal atau kata yang mempunyai arti khusus.
Contoh :
 "Tetikus" komputer ini sudah tidak berfungsi.
 Dilarang memberikan "amplop" kepada
petugas!

II. TANDA PETIK TUNGGAL (‘…’)

Pengunaan tanda petik tunggal menurut PUEBI adalah


sebagai berikut
(a) Mengapit petikan dalam petikan
Contoh :
 “Saya masih sempat mendengar teriakan anak
saya, ‘Bu, tolong saya, Bu’ sebelum saya sendiri
pingsan,” kata wanita itu.
EJAAN BAHASA INDONESIA (EBI)N - 4  “Coba dengarkan suara ‘krek-krek-krek’ di
belakang rumah!” seru Amir.

I. TANDA PETIK GANDA (“…”) (b) Mengapit terjemahan istilah asing


Contoh :
Pengunaan tanda petik ganda menurut PUEBI adalah  point of view ‘sudut pandang’
sebagai berikut  rate of inflation ‘laju inflasi’
(a) Dipakai untuk mengapit petikan langsung yang  flash back ‘alur sorot balik’
berasal dari pembicaraan, naskah, atau bahan tertulis  master of ceremony ‘pembawa acara’
lain.
III. TANDA TITIK DUA ( : )
Contoh :
Pengunaan tanda titik dua menurut PUEBI adalah
"Merdeka atau mati!" seru Bung Tomo dalam
sebagai berikut
pidatonya.
(a) Pada akhir suatu pernyataan lengkap. Bila diikuti
"Kerjakan tugas ini sekarang!" perintah atasannya.
rangkaian atau perincian.
"Besok akan dibahas dalam rapat."
Contoh:
Menurut Pasal 31 Undang-Undang Dasar Negara
Republik Indonesia Tahun 1945, "Setiap warga  Yang kita butuhkan sekarang adalah perabot
negara berhak memperoleh pendidikan." rumah tangga: kursi, meja, lemari, dan tempat
tidur.
(b) Dipakai untuk mengapit judul sajak, lagu, film,  Fakultas itu memiliki dua jurusan ekonomi:
ekonomi makro dan ekonomi mikro.
sinetron, artikel, naskah, atau bab buku yang dipakai
dalam kalimat. Catatan :
Contoh : Tanda titik dua tidak dipakai jika rangkaian atau
 Sajak "Pahlawanku" terdapat pada halaman 125 perincian itu merupakan pelengkap yang
buku itu. mengakhiri suatu pernyataan.
Contoh:
 Kita memerlukan, kursi, meja, dan lemari. 3. Pemakaian tanda baca di bawah ini tidak benar,
 Fakultas itu memiliki jurusan ekonomi makro kecuali…
dan ekonomi mikro. A. Dia digelari “si Kribo” karena rambutnya yang keriting.
B. “Come back”  kembali
(b) Teks kepanitiaan C. Saya sudah membaca novel ‘Belenggu’ karya Armyn
Contoh: pane.
 Ketua : Rinaldy D. Al. Merupakan jenis logam berat.
 Sekretaris : Sutri E. Baju baru adik seharga Rp 57.500,-.
 Bendahara : Ribka
4. Pemakaian tanda baca dalam kalimat Ayah berkata,
(c) Naskah drama sesudah kata yang menunjukkan “Coba dengarkan suara ‘krek-krek’ itu!” adalah benar.
pelaku dan percakapannya. SEBAB
Contoh: Fungsi tanda petik ganda dan tanda petik tunggal sama
Ibu : (meletakkan beberapa kopor) “Bawa saja dalam suatu kalimat.
kopor ini, Mir!”
Amir : “Ke mana, Bu?” 5. Pemakaian tanda baca yang sempurna terdapat pada…
A. Dian bertanya: "Siapa gadis itu?".
(d) Di antara jilid atau nomor buku/majalah dan B. Rok "canda" sudah tidak terkenal lagi di Indonesia.
halaman, antara bab dan ayat dalam kitab suci, atau C. Lana tinggal di Jalan Hakim, Medan.
antara judul dan anak judul dalam sebuah karangan. D. Rate of inflation, 'laju inflasi', di Indonesia mulai
Contoh: menurun.
 Budaya Jaya, II (1975,34 : 17) E. Rina menjuarai perlombaan itu, alasannya rajin
 Surat Yasin : 27 beralatih.
 Layar Terkembang : Analisis Psikologi
6. Penulisan tanda baca titik dua di bawah ini yang sudah
tepat adalah, kecuali …
A. Surat Yasin : 19
B. Belenggu : Sebuah Roman Jiwa
C. UUD 1945 Pasal 33 : 3
SOAL PENGANTAR D. Mereka memerlukan peralatan berkemah seperti :
tenda, tikar, dan senter.
1. Cermati kalimat-kalimat berikut! E. Romeo : “Aku sungguh mencintaimu, Juliet.”
1) Donita bertanya, “Di mana kita akan belajar Juliet :“Ya, aku tahu! Tapi jangan pernah
kelompok?” meninggalkan aku.”
2) Alya dan Fina akan membacakan puisi “Karawang
Bekasi” karya Chairil Anwar. 7. Penggunaan tanda baca yang benar terdapat pada kalimat
3) Ayah berkata,” Paman akan berangkat ke Singapura …
besok pagi”. A. Mereka tidak setuju, karena gagasan itu tidak
4) Kata Ibu “Ayah sudah pesan buku Teknik Bertanam masuk akal.
Hidroponik.” B. Kami memerlukan kertas, mistar, dan lain-lain.
5) Amira suka membaca “Cerpen Senyum Karyamin” C. Di toko itu terdapat : pakaian anak-anak, alat
karya Ahmad Tohari. olahraga, dan perabot dapur.
D. Bapaknya orang Bali ; sedangkan ibunya orang
Penulisan tanda petik ganda yang tidak tepat terdapat Minang.
pada kalimat nomor… E. Hindun, sahabat Aminah menjadi juara kelas di
A. 1 D. 4 bidang seni tari.
B. 2 E. 5
C. 3 8. Ibu membeli buah-buahan, seperti : apel, melon, dan
jeruk.
2. Pemakaian tanda baca yang tepat terdapat pada Kesalahan penggunaan tanda baca dalam kalimat di atas
kalimat… adalah…
A. Celana “jengki” sudah tidak populer lagi di A. tanda koma sebelum kata “seperti”
Indonesia. B. tanda titik dua setelah kata “seperti”
B. Rate of inflation, ‘laju infasi’ di Indonesia memang C. tanda koma sebelum kata “dan”
agak tinggi pada bulan Oktober 2000. D. tanda titik di akhir kalimat
C. Surat yang dikirimkan itu No : 124/ PP/ Pes/ I/ E. tanda hubung pada kata buah-buahan
2001.
D. Ali bertanya ; “Dimana kau beli buku ini ?” 9. Puisi “Bulan” terdapat di buku kumpulan puisi “Pada
E. Ia bertempat tinggal di jalan Diponegoro IV – 5 Malam”.
Kesalahan penggunaan tanda baca dalam kalimat di atas A. “Apakah kalian mendengar suara ‘tok, tok, tok’
adalah… seperti orang mengetuk pintu?” Tanya Dafa
A. tanda petik dua yang mengapit kata “Bulan” kepadaku.
B. tanda petik dua yang mengapit “Pada Malam” B. Di masa itu, ada istilah ‘indehoi’, yaitu berpacaran di
C. tanda titik di akhir kalimat tempat-tempat yang jauh dari perhatian orang,
D. tanda petik dua setelah kata “Malam” misalnya di taman yang rimbun atau gedung bioskop
yang gelap.
10. Penggunaan tanda petik tunggal yang tepat terdapat pada C. ‘Dia telah banyak berubah sekarang’, kataku dalam
kalimat… hati.ketika bertemu dengan Ratna dalam reuni itu.
(1) Istilah feed back ‘sorot balik’ terrdapat pada alur. D. Karena ciri fisiknya yang menonjol, Laksamana
(2) Karena kecepatan serangannya, Fernando Tores diberi julukan ‘si jangkung’ oleh teman-temannya.
mendapatkan julukan ‘El Nino’. E. Dalam tahap tersebut, pasien akan mengalami gejala
(3) “Kau dengar suara ‘kring-kring’ tadi?” Tanya Susi ‘angina’ atau “nyeri dada”.
berbisik.
(4) Ia membaca sajak ‘Aku dengan penuh improvisasi. 16. Penulisan kalimat yang sesuai dengan EBI adalah…
A. “Sampah yang muncul pascabanjir di DKI Jakarta
11. Penggunaan tanda petik ganda yang tidak tepat terdapat mengalami peningkatan rata-rata 1.100 ton per hari
dalam kalimat… dengan jumlah terbesar berasal dari wilayah Jakarta
A. “Mari kita tingkatkan kualitas belajar kita,” ujar Utara dan Jakarta Barat,” Kata Kepala Dinas DKI
kepala sekolah. “agar kita dapat menggangkat citra Jakarta, Unu Nurdin.
sekolah ini.” B. Sampah yang muncul pascabanjir di DKI Jakarta
B. Karena postur tubuhnya, Galih mendapat julukan mengalami peningkatan rata-rata 1.100 ton per hari,
“Si Gendut”. dengan jumlah terbesar berasal dari wilayah Jakarta
C. Genangan air merupakan tempat Utara dan Jakarta Barat, kata Kepala Dinas DKI
berkembangbiaknya nyamuk “aedes aegepty”. Jakarta Unu Nurdin.
D. Lahar dingin itu “nggorojog” dari atas seperti C. “Sampah yang muncul pasca banjir di DKI Jakarta
ombak besar. mengalami peningkatan rata-rata 1.100 ton per hari
E. Esai Ratna yang berjudul Kesehatan Reproduksi dengan jumlah terbesar berasal dari wilayah Jakarta
dimuat di “tempo”. Utara dan Jakarta barat”, kata Kepala Dinas DKI
Jakarta Unu Nurdin.
12. Bacalah “Bab II” dalam buku Kritik Sosial Zaman D. Sampah yang muncul pasca banjir di DKI Jakarta
Reinesance. mengalami peningkatan rata-rata 1.100 ton per hari
SEBAB dengan jumlah terbesar berasal dari Wilayah Jakarta
Kata Bab II seharusnya tidak diberi tanda petik ganda. Utara dan Jakarta Barat, kata kepala DKI Jakarta,
Unu Nurdin.
13. Penggunaan tanda titik dua (:) yang benar terdapat E. “Sampah yang muncul pascabanjir di DKI Jakarta
pada… mengalami peningkatan rata-rata 1.100 ton perhari
(1) Para karyawan mempunyai tuntutan yang keras : dengan jumlah terbesar besaral dari Wilayah Jakarta
direktur mundur atau dimundurkan. Utara dan Jakarta Barat”, kata Kepala Dinas DKI
(2) Mereka ingin memiliki kebebasan dasar seperti : Jakarta, Unu Nurdin.
bebas dari rasa takut dan bebas untuk menentukan
jalan hidup. 17. Kalimat yang penulisannya sesuai dengan aturan EBI
(3) (Yani, 2019 : 125-177) adalah…
(4) Para pejabat Pemda Kalteng mengatakan : relokasi A. Studi lintas bahasa menunjukkan, bahwa jumlah dari
itu sulit dilakukan dalam waktu dekat. makna leksikon dalam ranah yang sama berbeda dari
bahasa yang satu ke bahasa yang lain.
14. Penulisan tanda baca di bawah ini yang tidak sesuai B. Berdasarkan stratifikasi sosialnya Masyarakat
dengan pengetahuan umum EBI adalah… Keraton Yogyakarta dikelompokkan menjadi dua
A. Salsa tidak suka membaca novel :Jangan Rebut golongan yakni golongan bangsawan atau ‘ningrat’
Kekasihku”. yaitu golongan yang memiliki hubungan darah atau
B. Master of ceremony ‘pembawa acara’ kekerabatan dengan raja atau sultan serta golongan
C. Ibu membeli : makanan, minuman, dan buah-buahan. ‘abdi dalem’ Keraton atau priyayi yaitu golongan
D. Tika, anak tetangga saya, lulus di Teknik Informatika yang mempunyai status sebagai pegawai Keraton.
UGM. C. Bahan atau materi tulisan ini, berupa informasi-
E. Abangnya bukan seorang guru, tetapi seorang tentor. informasi mengenai nama dan gelar di Keraton yang
diperoleh dari teks-teks lisan atau tertulis.
15. Penggunaan tanda petik tunggal (‘…’) yang tepat D. Gelar keturunan adalah gelar yang diberikan
terdapat pada kalimat… berdasarkan hubungan pertalian darah, yaitu oleh
orang tua kepada anak, cucu, cicit hingga generasi di
bawahnya.
E. Amerika dibentuk oleh kaum “imigrant” yang dating
dari berbagai negara dengan alasan antara lain
misalnya untuk memperoleh kebebasan beragama,
melepaskan diri dari penindasan politis dan mencari
kehidupan yang lebih baik.

18. Pulau Kela dalam cerita “1001 Malam” itu diasumsikan


hovig sebagai Pulau Bangka sekarang.
Ejaan yang perlu diperbaiki pada kalimat tersebut
adalah…
A. Memberikan tanda koma (,) setelah kata itu.
B. Tanda petik (“) di depan dan di belakang 1001
Malam dihilangkan.
C. Mengubah huruf K pada Kela mejadi k.
D. Mengubah kata hovig menjadi bercetak tegak.
E. Mengubah huruf h pada hovig dan b pada Bangka
menjadi H dan B.

19. Di antara lima kalimat berikut, kalimat yang


penulisannya benar dari segi kaidah EBI adalah…
A. “Siapa dia Mas?” tanyanya sambil menunjuk
perempuan molek berambut panjang yang dari tadi
berdiri mematung di dekat kios Pak Udin.
B. Makna kata fleksibel adalah “luwes”.
C. “Sewaktu pintu kelas kubuka, kudengar suara
serentak dari murid-muridku , “Selamat pagi, Bu
Guru,” kisah Bu Kiki.
D. Kata depan “di”, “ke”, dan “dari” dituliskan terpisah
dari kata yang mengikutinya, misalnya di kamar, ke
atas, dan dari belakang.
E. Harga kaos ini tidak lebih dari Rp. 30.000,-

20. Penggunaan tanda baca yang benar terdapat pada


kalimat…
A. Beberapa penumpang, segera dievakuasi dari lokasi
kecelakaan.
B. Setiap hari, para siswa kelas A belajar dengan keras,
sehingga mereka dijuluki “Kelas Rajin”.
C. Dua hal yang harus diperhatikan saat naik bus adalah:
barang-barang yang dibawa dan penumpang yang
mencurigakan.
D. “Lihat ini, Boi! Ada gambarnya!”
E. “Apakah engkau tak malu meminta uang pada
anakmu?”
(4) Menuliskan ungkapan atau kata asing yang belum
disesuaikan ejaannya.
Contoh :
 Belanda menguasai Indonesia selama tiga
setengah abad dengan politik de vide at
impera.
 Buku welthancheung telah diterjemahkan ke
dalam beberapa bahasa.

II. GARIS MIRING (/)


(1) Penulisan nomor surat resmi dan nomor pada
alamat
Contoh:
No. 0901/A.A8/KP/2009
Jalan Rajawali III/10

(2) Sebagai pengganti kata atau dan tiap


Dikirim lewat darat/laut.
Harganya Rp8.000,00/lembar

SOAL PENGANTAR

1. Kalimat yang ditulis secara baku adalah ….


EJAAN BAHASA INDONESIA (EBI) - 5 A. Kemarin saudara saya dirawat dirumah sakit
B. Sudahkah anda membaca Media Indonesia hari
ini?
I. HURUF MIRING C. Pamanku yang tinggal di bandung sudah 2 kali
Huruf miring atau garis bawah digunakan pada naik haji.
penulisan : D. P. Ari Subagio M. Hum sedang mengambil S-3 di
(1) Menuliskan judul buku, karangan, syair, dan UGM.
majalah dalam karangan E. Mereka mendaki gunung Gede, pada saat hujan
Contoh : lebat.
 Dalam bukunya Harimau - Harimau, Mochtar
Lubis memaparkan bahwa manusia adalah 2. Pemakaian tanda garis miring(/) berikut ini sesuai
harimau bagi manusia lainnya. dengan EYD, KECUALI …
 Firasat bahwa kematian akan datang A. Tahun akademik 2018/2019 akan segera kita
menjemputnya dipaparkan Chairil Anwar masuki
dalam sajak Yang Terhempas dan yang Putus. B. Acara akan berlangsung dari 13.00 s/d 14.30
(2) Menegaskan bagian kata, kata, dan kelompok kata C. Nomor surat keterangan tersebut 21/ PFL/XI/2008
Contoh : D. Honornya Rp.850.000,00/minggu
 Huruf pertama kata abad adalah a.
E. Jalan Kamboja X/4
 Dia bukan menipu, melainkan ditipu.
(3) Menuliskan nama-nama ilmiah
3. Kalimat dengan pemakaian kata berhuruf miring yang
Contoh :
tidak tepat adalah ... .
 Nama latin manggis adalah circinia
A. Penyelam gua bawah laut dapat menemukan sarang
mangostana.
ikan kerapu macan atau Epinepehelusfuscoguttatus.
 Pohon kayu putih atau eucalyptus banyak
tumbuh di depan kantor itu.
B. Para penulis untuk majalah Pujangga Baru D. Tanda pisah tidak dipakai mengapit keterangan
mencurahkan energinya untuk mengembangkan aposisi.
bahasa Indonesia. E. Tanda garis miring dipakai sebagai pengganti kata
C. Berita tentang letusan Gunung Kelud dimuat di dan.
Kompas.
D. Tak Pandai Bohong adalah salah satu tulisan yang 7. Pemakaian huruf miring atau garis bawah dibenarkan,
terdapat dalam kumpulan tulisan Orang dan Bambu kecuali untuk…
Jepang karya Ajip Rosidi. A. nama orang atau nama instansi atau lembaga
E. Kisah dalam Layar Terkembang karya Sutan B. menegaskan bagian kata, kata, atau kelompok kata
TakdirAlisyahbana berawal di gedung akuarium C. menuliskan kata nama-nama ilmiah
kota. D. menuliskan ungkapan asing, kecuali yang telah
disesuaikan ejaannya
4. Kata pensil berasal dari bahasa Prancis kuno, pincel, E. menuliskan nama buku dan majalah yang dikutip
yang berarti “kuas kecil”. Kata pensil juga berasal dari dalam karangan
bahasa Latin penicillus yang berarti ‘ekor kecil’. Alat
itu berupa sikat halus dari bulu unta. Alat itu digunakan 8. Kalimat berikut yang ditulis sesuai dengan EYD
sebelum ada kapur pensil. Biasanya, pensil ini adalah…
digunakan menulis naskah pada daun lontar. Sebelum A. Dalam mengantisipasi terjadinya demonstrasi
tahun 1560-an, Grey Knotts, menentukan endapan ribuan karyawan, yakni menuntut kenaikan upah
granit. lembur jajaran direktur di perusahaan Metal
Dua tanda baca yang salah pada paragraf di atas Perkasa mengadakan koordinasi bersama dengan
adalah… para Manajer.
A. tanda koma setelah pincel dan tanda petik tunggal B. Dalam mengantisipasi terjadinya demonstrasi
pada ekor kecil ribuan karyawan, yakni menuntut kenaikan upah
B. tanda petik tunggal pada ekor kecil dan Koma lembur, jajaran Direktur di Perusahaan Metal
setelah biasanya Perkasa mengadakan koordinasi bersama dengan
C. tanda koma setelah biasanya dan tanda koma Manajer.
setelah 1560-an C. Dalam mengantisipasi terjadinya demonstrasi
D. tanda koma setelah Knotts dan tanda petik pada ribuan karyawan, yakni menuntut kenaikan upah
kuas kecil lembur, jajaran direktur di perusahaan Metal
E. tanda koma setelah kuno dan tanda koma setelah Perkasa mengadakan koordinasi bersama dengan
biasanya para manajer.
D. Dalam mengantisipasi terjadinya demonstrasi
5. Kalimat berikut ini ditulis dengan ejaan yang benar, ribuan karyawan, yakni menuntut kenaikan upah
KECUALI… lembur, jajaran Direktur di perusahaan Metal
A. Untuk mencegah penyebarluasan penyakit, perkasa mengadakan koordinasi bersama dengan
diharapkan partisipasi masyarakat dalam para Manajer.
membrantas tempat-tempat perkembangbiakan E. Dalam mengantisipasi terjadinya demonstrasi
nyamuk. ribuan karyawan yakni menuntut kenaikan upah
B. Penelitian itu melibatkan 1.010 orang sehat, 277 lembur, jajaran direktur di Perusahaan Metal
orang pengidap pradiabetes, dan 148 orang Perkasa mengadakan koordinasi bersama dengan
pengidap diabetes. para Manajer.
C. Olahraga dapat mencegah alzhemer, penyakit
kemunduran daya pikir dan daya ingat akibat 9. Penggunaan tanda baca di bawah ini yang tepat
kematian sel-sel saraf yang berlangsung secara adalah…
tepat. A. Ayah dilantik oleh Saudara Hendra, SE. tadi siang
D. Jaringan tumor yang tumbuh, meski hanya satu di kantor.
atau dua sentimeter di bawah kulit, langsung B. Dani menyelesaikan soal itu dari nomor 1 s/d 10 di
menempel ke otot-otot dada. sekolah.
E. Usaha penurunan berat badan yang paling aman C. Kami membeli buku-sejarah baru kemarin di toko
adalah penurunan setengah hingga satu kilogram buku Gramedia.
per minggunya D. Dia tinggal dengan Ny. Dewi.S.H.
E. Ayah sekarang berpangkat Letnan
6. Pernyataan di bawah ini yang tidak benar adalah …
A. Tanda pisah dipakai mengapit penyisipan di luar 10. Dalam kalimat berikut terdapat kesalahan penerapan
bangun kalimat. EYD, KECUALI…
B. Tanda hubung digunakan merangkai angka dengan A. Nama zat kimia yang disebut thallium kembali
–an. menjadi buah bibit, setelah adanya percobaan
C. Tanda hubung digunakan untuk merangkai unsur pembunuhan terhadap mantan mata-mata Rusia,
kata ulang. Alexander Litvinenko
B. Kondisi Litvenko makin buruk, jumlah sel darah D. Kesabarannya hilang, sehingga dia langsung
putihnya nyaris nol, sehingga dokter mematikan telepon genggamnya.
menyimpulkan, bahwa racun thallium telah E. Peraturan di kantor lama, ternyata tidak berlaku di
menyerang sumsum tulang Litvinenko. kantor baru.
C. Pembangunan dalam bidang apa pun tidak akan
berjalan lancar jika tidak didukung sumberdaya 14. Orang tua disarankan, agar tidak secara langsung
manusia (SDM) yang berakhlak mulia serta membantu mereka tetapi lebih mengarah kepada
berprilaku jujur dalam melaksanakan setiap amanah pemberian petunjuk.
yang diberikan Perbaikan ejaan dalam kalimat tersebut adalah….
D. Larangan praktek pelayanan kesehatan pasien bagi 1. Di depan kata agar tanda koma (,) dihilangkan
mantri kesehatan dan perawat sudah final 2. Kata orang tua penulisannya digabung
E. Sekarang, makin banyak orang menggeluti dunia 3. Didepan kata tetapi harus memakai tanda koma (,)
fotografi, baik sebagai profesi atau sekedar hobi 4. Di depan kata disarankan ditubuhi tanda titik dua
(:)
11. Penulisan huruf miring diperkenankan dalam pedoman
umum EYD, kecuali…. 15. Penggunaan tanda baca titik dua (:) yang benar terdapat
(A) Menuliskan nama ilmiah. pada ….
(B) Menuliskan ungkapan asing yang belum 1. Para karyawan mempunyai tuntutan yang keras:
disesuaikan ejaannya. direktur mundur atau dimundurkan.
(C) Menuliskan nama orang. 2. Mereka ingin memiliki kebebasan dasar seperti:
(D) Menuliskan nama buku atau majalah dalam bebas dari rasa takut dan bebas untuk menentukan
kutipan. jalan hidup
(E) Menuliskan huruf atau kata yang mendapat 3. (Yani, 1987 : 125 - 177)
penegasan khusus. 4. Para pejabat Pemda Kalteng mengata-kan: relokasi
itu sulit dilakukan dalam waktu dekat.
12. Cermati paragraf berikut!
(1) Bagi pencinta durian, menikmati tempoyak 16. Penggunaan tanda baca yang tidak tepat terdapat pada
memberi pengalaman baru yang sulit dilupakan kalimat ….
(2) buah durian dengan rasa manis legit, dan, kadang A. Akibat proyek itu, ruas Jalan Raya Bogor tersebut
sedikit pahit berubah menjadi asin gurih, ketika hanya bisa dilalui satu jalur. Sehingga
diolah dalam wujud tempoyak menimbulkan kemacetan.
(3) tempoyak yang merupakan hasil fermentasi durian B. Karena tidak membawa ban serep, akhirnya,
ini, semakin lezat, ketika diolah dengan pangan tronton dengan nomor polisi B 9765 P itu sulit
lokal khas daerah penghasil durian, seperti, untuk dievakuasi.
Lampung C. Saat ini, petugas hanya bisa menunggu mobil
(4) di Bandar Lampung, cukup mudah untuk, perusahaan yang mengoprasikan tronton itu dari
menikmati masakan dengan bumbu olahan bandung yang akan membawa ban serep.
tempoyak D. Tahun lalu Thailand mengkspor sekitar U$$ 40
(5) tempoyak diolah beragam masakan mulai dari miliar untuk komponen elektronik dan komputer.
seruit tempoyak, brengkes baung tempoyak, E. Selain itu penutup bandara tersebut juga
hingga, udang selimut tempoyak diperkirakan akan berbahaya bagi mereka yang
Penggunaan tanda koma (,) yang tepat terdapat pada membutuhkan perawatan medis.
kalimat nomor ....
A. (1)
17. Penggunaan tanda baca secara tepat terdapat pada
B. (2)
kalimat ....
C. (3)
A. Kami akan membeli perlengkapan rumah : kursi
D. (4)
tamu, lemari pakaian, tempat tidur, dan meja
E. (5)
makan.
B. Menteri Kesehatan mengatakan bahwa penyakit flu
13. Kalimat yang memiliki penggunaan tanda koma yang
burung telah merebak ke berbagai negara.
tepat adalah ...
A. Setiba di Kebonjahe yang sejuk dan tidak C. Artikel yang berjudul "Bahasa dalam
Politik"dikarang oleh Drs. P.Ari Subagio M.Hum.
terlaluramai, kami berjalan-jalan di sepanjang jalan
utama Kota Kebonjahe. D. Mantan Presiden Republik Indonesia, Megawati
B. Aku pun tidak membukakan pintu saat dia memberikan pandangan terha-dap keadaan negara
mengetuk pintu, karena aku yakin bahwa adikku kita saat ini.
atau ayahku akan membukakan pintu itu. E. Wah, kalau caranya seperti itu banyak orang yang
C. Kereta tak selalu berjalan stabil, adakalanya kereta tidak setuju untuk memilih dia sebagai ketua partai.
berjalan pelan, dan kadang-kadang juga, kereta
berjalan cepat.
18. Di antara kalimat-kalimat berikut, kalimat yang memuat D. Pemaparan teori tentang ekonomi pembangunan
tanda koma secara tidak tepat adalah ... . ini, dimaksudkan untuk meningkatkan kepekaan
A. Dengan demikian, ada keinginan untuk dalam memahami fenomena peralihan status elit
mempelajari, mempraktikkan, ataupun mendirikan E. Dalam konteks sosialbudaya itu, citra, diri di
suatu institusi sebagai wadah belajar. tengah masyarakat dimaknai teramat utama dan
B. Padahal, menurut Baneson, salah satu penyebabnya sentral sifatrnya
adalah kurangnya perhatian pemerintah terhadap
budaya asli daerah,terutama di daerah perbatasan.
C. Keberadaan rumah panjang atau betang sebagai
tempat siswa berlatih tari, juga diperlukan sebagai
simbol kembalinya karakter masyarakat Dayak.
D. Dalam pengelolaan sumber daya alam, misalnya,
provinsi ketiga terluas di Indonesia ini dihadapkan
pada dua kepentingan pengelolaan, yakni antara
pemerintah pusat dan daerah.
E. Oleh karena itu, menurutnya, perlu dilakukan
persiapan sehingga apabila pengajuan disetujui
pada tahun 2017, hal itu bukan merupakan awal
gerak, melainkan sebagai bonus terhadap program
yang secara nasional telah dilakukan.

19. Kalimat berikut yang ditulis sesuai dengan EBI adalah



A. Dalam mengantisipasi terjadinya demonstrasi
ribuan karyawan, yakni menuntut kenaikan upah
lembur jajaran direktur di perusahaan Metal
Perkasa mengadakan koordinasi bersama dengan
para Manajer.
B. Dalam mengantisipasi terjadinya demonstrasi
ribuan karyawan, yakni menuntut kenaikan upah
lembur, jajaran Direktur di Perusahaan Metal
Perkasa mengadakan koordinasi bersama dengan
Manajer.
C. Dalam mengantisipasi terjadinya demonstrasi
ribuan karyawan, yakni menuntut kenaikan upah
lembur, jajaran direktur di perusahaan Metal
Perkasa mengadakan koordinasi bersama dengan
para manajer.
D. Dalam mengantisipasi terjadinya demonstrasi
ribuan karyawan, yakni menuntut kenaikan upah
lembur, jajaran Direktur di perusahaan Metal
perkasa mengadakan koordinasi bersama dengan
para Manajer.
E. Dalam mengantisipasi terjadinya demonstrasi
ribuan karyawan yakni menuntut kenaikan upah
lembur, jajaran direktur di Perusahaan Metal
Perkasa mengadakan koordinasi bersama dengan
para Manajer.

20. Penulisan ejaan pada kalimat-kalimat berikut


seluruhnya sesuai dengan aturan EBI, KECUALI …
A. Berpartisipasi dalam mencurahkan tenaga atau
pikiran pada kegiatan yang menjadi kepentingan
masyarakat dimaknai sebagai aktivitas kerja
B. Bahkan meluangkan waktu serta jiwa raga untuk
menunaikan kewajiban peribadatan kepada Tuhan
juga diartikan sebagai aktivitas kerja
C. Pemaknaan seperi itu berakar pada dan merupakan
pantulan dari pandangan hidup yang melembaga
secara turun-temurun
4. Tarik kesimpulan dari data yang disampaikan.

II. TABEL
Tabel adalah daftar yang berisi sejumlah informasi berupa
TPS – 1 kata-kata dan bilangan, yang tersusun berturut ke bawah
PENALARAN UMUM dalam kolom dan baris tertentu. Untuk menjelaskan isi tabel
kita perlu membekali diri dengan pengetahuan yang
berkaitan dengan informasi yang disusun dalam baris dan
I. GRAFIK kolom.
Grafik adalah gambaran pasang surutnya suatu Tujuan dibuat tabel :
keadaan atau data yang ada dengan garis atau gambar. 1. Dapat memberikan banyak informasi secara ringkas
Grafik dibedakan menjadi tiga macam, yaitu grafik batang, 2. Mempermudah pembaca dalam memahami.
grafik garis, dan grafik lingkaran. Hal-hal yang perlu diperhatikan dalam menafsir tabel :
Grafik Batang adalah lukisan naik turunnya data 1.Mencermati bagian-bagian tabel
berupa batang atau balok dan dipakai untuk menekan kan 2.Membaca informasi di sekitar tabel
adanya perbedaan tingkatan atau nilai berupa aspek. 3. Mengajukan pertanyaan tentang isi tabel
4. Menafsirkan angka-angka atau data
5. Mengaitkan antardata dengan tabel
6. Mengambil kesimpulan.

III. BAGAN
Bagan merupakan gambar yang menyampaikan informasi
dalam hubungan atasan-bawahan.
Isi bagan dapat dipahami dengan melihat hubungan antara
kata-kata yang ada di atas dengan bagian yang ada di
bawahnya atau di samping kiri dan kanannya.

Graf
ik Garis adalah lukisan naik turunnya data berupa garis yang
di hubungkan dari titik-titik data secara berurutan. Grafik ini
di gunakan untuk menggambarkan perkembangan atau
perubahan dari waktu ke waktu.

Grafik Lingkaran adalah gambaran naik turunnya data


berupa lingkaran untuk menggambarkan persentase dari IV.SIMPULAN
nilai total atau seluruhnya.
Simpulan adalah pernyataan berisi fakta,pendapat,alasan
pendukung mengenai tanggapan suatu objek. Bisa dikatakan
bahwa kesimpulan merupakan pendapat akhir dari suatu
uraian berupa informasi. Dalam soal Bahasa Indonesia
kesimpulan bisa berupa rangkaian kalimat kalimat fakta
yang diberi pendapat.

Manfaat Grafik :
1. Menunjukkan fakta dengan jelas dan mudah dipahami
2. Menjadikan proses komunikasi lebih cepat dan menarik
Langkah – langkah membaca grafik :
1. Amati judul grafik,
2. Amati lajur kanan, kiri, dan bawah,
3. Temukan benda yang mencolok pada data tersebut, dan
SOAL PENGANTAR 3. Urutan bulan dilihat dari harga gabah yang paling tinggi
ke terendah adalah…
Teks berikut digunakan untuk menjawab soal nomor 1 A. Agustus - November - September - Oktober -
sampai dengan 7. Desember - Januari
Produksi pertanian tanaman pangan di Jawa Tengah B. Januari - Desember - Oktober - November -
diperkirakan terus meningkat pada tahun ini. Agar angka September – Agustus
perkiraan tidak merosot jauh dengan realisasi, diperlukan C. Januari - Desember - Oktober - September -
pengawalan ketat. Pengawalan yang dimaksud bisa berupa Agustus – November
suplai pupuk yang cukup pada masa tanam dan informasi D. Januari - Desember - Oktober - September -
cuaca yang tepat dan sesuai dengan jenis tanaman. November – Agustus
Selain itu, pemerintah juga memberikan bantuan E. Agustus - September - November - Oktober -
stimulan bagi daerah yang terkena bencana seperti Desember – Januari
kekeringan, banjir, dan bencana alam lainnya. Namun, yang
tidak kalah pentingnya adalah kebijakan harga yang 4. Berdasaran paragraf 1, manakah di bawah ini
menguntungkan para petani. Selama ini yang paling pernyataan yang benar…
menikmati harga kenaikan beras hanya para pengepul atau A. Petani menikmati harga kenaikan beras lebih.
produsen besar, sementara petani menikmati sebagian B. Produksi pertanian tanaman pangan terus
kecilnya saja. meningkat.
C. Kebijakan harga dilakukan agar tidak ada
spekulasi.
D. Perkiraan produksi pertanian pangan di Jawa
Tengah terus meningkat.
E. Suplai pupuk yang cukup pada masa tanam dan
informasi cuaca yang tepat tidak diperlukan.

5. Berdasarkan paragraf 2, apa yang menyebabkan selisih


harga terjadi?
A. Harga terlalu tinggi yaitu Rp 4.700,00
B. Spekulasi yang dikendalikan oleh para tengkulak
C. Gabah kering hanya Rp 2.200,00
D. Margin keuntungan petani sangat tinggi
E. Harga beras selalu naik setiap bulan

6. Berdasarkan grafik, pada bulan berapa gabah kering di


Kondisi harga gabah kering panen saat itu hanya Jawa Tengah menunjukkan harga kedua tertinggi?
berkisar Rp 2.200,00 per kilogram sementara harga jual A. September
beras jenis IR-64 rata-rata telah mencapai Rp 4.700,00 per B. Agustus
kilogram. Selisih harga tersebut terjadi karena spekulasi C. Desember
dalam perdagangan beras yang dikendalikan para tengkulak D. Januari dan Desember
tanpa memikirkan margin keuntungan bagi petani produsen. E. November dan Agustus
(Sumber: Kompas, 26 Maret 2008)
7. Berdasarkan grafik, bulan ke berapakah gabah kering
1. Pada paragraf 2, pernyataan yang benar adalah… mengalami peningkatan paling rendah yang kedua?
A. Petani tidak menikmati kenaikan harga beras. A. November
B. Pemerintah tidak memperhatikan petani. B. Oktober
C. Bencana kekeringan, banjir, dan yang lainnya tidak C. Desember
diperhatikan oleh petani. D. September
D. Kenaikan harga beras dinikmati oleh para pengepul E. Agustus
beras.
E. Pemerintah memberikan kebijakan yang sangat Bacalah soal dengan saksama kemudian kerjakan soal
menguntungkan petani. nomor 8 sampai dengan 14!
(1) Tahun ini ujian nasional (UN) tetap dilaksanakan dengan
2. Dari grafik dapat disimpulkan bahwa harga gabah beberapa perubahan. (2) Salah satu perubahan adalah
kering mengalami penurunan pada bulan… kriteria kelulusan yang mengikutsertakan nilai rapor. (3)
A. Agustus Nilai rapor yang sebelumnya tidak menjadi pertimbangan,
B. September dan Oktober mulai tahun ini dipertimbangkan pencapaiannya untuk
C. Oktober dan November membantu kelulusan siswa. (4) Tentu saja, kejujuran
D. November sekolah dituntut untuk itu. (5) Kejujuran sering tidak sesuai
E. Desember dengan harapan, akhirnya sulit memprediksi kemampuan
seseorang. (6) Nilai rapor yang bagus belum tentu
mencerminkan kemampuan yang sebenarnya jika kejujuran 11. Komentar yang paling tepat dengan isi teks di atas
diragukan. (7) Dengan demikian, kualitas seseorang juga adalah ….
tidak dapat diketahui dengan tepat. (8) Sebagai gambaran, 1) Nilai mata pelajaran Bahasa Inggris lebih baik
berikut disajikan tingkat pencapaian nilai UN mata pelajaran daripada nilai Bahasa Indonesia; padahal, Bahasa
tahun 2010. Indonesia telah dikenal anak sejak kecil dan
Tabel Tingkat Pencapaian Kelulusan Mata Pelajaran menjadi bahasa percakapan.
Tahun 2010 2) Sangat ironis bahwa kelulusan mata pelajaran
Bahasa Indonesia terendah pada semua sekolah;
SM SM SM Rer padahal Bahasa Indonesia menjadi kunci
Mata Pelajaran
AX AY AZ ata keberhasilan semua mata pelajaran.
45% 55% 3) Kelulusan mata pelajaran Bahasa Indonesia hampir
Matematika 55% 65% mengalahkan pencapaian kelulusan mata pelajaran
35% 43% Matematika; padahal, Bahasa Indonesia sudah
Bahasa Indonesia 48% 46% dipelajari oleh peserta didik sejak dini.
56% 4) Rerata kelulusan mata pelajaran Bahasa Inggris
Bahasa Inggris 65% 40% 63% mengalahkan pencapaian kelulusan Bahasa
Indonesia; padahal, Bahasa Indonesia dipelajari
(9) Dari Tabel 1 dapat diketahui tingkat pencapaian oleh peserta didik sejak dini.
kelulusan di SMA X paling banyak pada mata pelajaran
Bahasa Inggris, sedangkan SMA Y dan SMA Z pada 12. Berdasarkan paragraf 2, manakah di bawah ini
Matematika. (10) Selain itu, Bahasa Indonesia mempunyai pernyataan yang BENAR?
rerata terkecil dibandingkan dengan Matematika dan Bahasa A. Rerata terkecil terdapat pada mata pelajaran
Inggris. matematika.
B. Rerata terkecil terdapat pada mata pelajaran bahasa
8. Berdasarkan paragraf 1, manakah di bawah ini Inggris.
pernyataan yang BENAR? C. Rerata terkecil terdapat pada mata pelajaran bahasa
A. Nilai rapor tahun sebelumnya menjadi Indonesia dan bahasa Inggris.
pertimbangan. D. Rerata terbesar terdapat pada mata pelajaran
B. Pencapaian kelulusan bahasa Inggris paling matematika.
banyak. E. Rerata terkecil terdapat pada mata pelajaran bahasa
C. UN membantu kelulusan siswa tahun ini. Indonesia.
D. Kejujuran sekolah sangat diperlukan dalam
menghadapi UN. 13.Dari tabel tersebut maka pernyataan yang benar adalah…
E. Nilai rapor menjadi pertimbangan kelulusan tahun A. Rata-rata SMA X > rata-rata SMA Z
ini. B. Rata-rata SMA Y > rata-rata SMA Z
C. Rata-rata SMA X > rata-rata SMA Y
9. Berdasarkan tabel 1, SMA manakah pencapaian nilai D. Rata-rata SMA X = rata-rata SMA Y
bahasa Indonesia kedua tertinggi? E. Rata-rata SMA Y = rata-rata SMA Z
A. SMA Z
B. SMA Y 14. Urutan nilai rata-rata ketiga SMA dari yang tertinggi ke
C. SMA X terendah adalah…
D. SMA X dan Y A. SMA X – SMA Y – SMA Z
E. SMA Z dan X B. SMA X – SMA Z – SMA Y
C. SMA Y – SMA X – SMA Z
10. Pernyataan berikut ini yang paling sesuai dengan isi D. SMA Z – SMA X – SMA Y
tabel di atas adalah …. E. SMA Z – SMA Y – SMA X
1) Urutan tingkat pencapaian kelulusan mata pelajaran
dari yang terbesar ke yang terkecil pada SMA Y Bacalah teks berikut!
sama dengan pada rerata. (1) Pembentukan Masyarakat Ekonomi ASEAN 2015
2) Urutan tingkat pencapaian kelulusan mata pelajaran membuka peluang sekaligus tantangan bagi Indonesia dalam
dari yang terbesar ke yang terkecil pada SMA X meningkatkan kesejahteraan bangsa melalui integrasi
sama dengan pada rerata. ekonomi dan keuangan di kawasan. (2) Luasnya wilayah
3) Urutan tingkat pencapaian mata pelajaran dari yang Indonesia dengan komoditas unggulan ekspor relatif
terbesar ke yang terkecil pada SMA Z sama dengan bervariasi antarprovinsi membuka peluang peningkatan
pada rerata. diversifikasi ekspor Indonesia, baik dalam rangka
4) Urutan tingkat pencapaian kelulusan mata pelajaran meningkatkan ekspor ke kawasan ASEAN, Asia lainnya
dari yang terkecil ke yang terbesar pada ketiga maupun dunia. (3) Secara spesifik, ekspor Indonesia
sekolah tidak ada yang sama. ditentukan oleh kinerja ekonomi dan perdagangan provinsi.
(4) Perdagangan internasional di suatu negara dapat
dibangun oleh perdagangan dari setiap provinsi.
(5) Unggulan ekspor yang relatif bervariasi antar provinsi A. Komoditas ekspor Indonesia yang sangat bervariasi
dapat didayagunakan untuk memacu pertumbuhan memacu peningkatan disversifikasi ekspor ke
ekspornya. (6) Hingga saat ini, provinsi yang paling dunia.
dominan melakukan kegiatan ekspor adalah Jawa Barat B. Setiap wilayah di Indonesia memiliki kekhususan
diikuti Riau, Jawa Timur, dan Jakarta. (7) Sebanyak 26,2% komoditas utama yang diekspor ke kawasan
ekspor Jabar ditunjukkan ke ASEAN dengan negara tujuan ASEAN.
utama Malaysia, Filipina, Thailand, dan Vietnam. (8) C. Pembentukan masyarakat ekonomi ASEAN 2015
Provinsi Riau melakukan kegiatan ekspor/terutama ke memacu pertumbuhan ekspor Indonesia.
kawasan Singapura. D. Sebagian besar provinsi di Indonesia hanya
(9) Jika dilihat secara lebih detail, maka setiap wilayah di memiliki komoditas ekspor utama yang terbatas.
Indonesia memiliki kekhususan komoditas utama yang E. Peningkatan kesejahteraan bangsa dapat dilakukan
diekspor, misalnya Sumatera: minyak sawit dan karet melalui pengintegrasian ekonomi dan keuangan
mentah; Kalimantan: batu bara; Sulawesi: coklat, minyak kawasan.
sawit dan metalliferous: Jawa dan Bali : tekstil, garmen,
kertas, furniture dan produk kayu; Nusa Tenggara: 17. Penulis teks tersebut bertujuan agar pembaca …
metalliftrous; serta Irian : metalliferous. (10) Dari data A. meyakini bahwa komoditas ekspor nonmigas di
pertumbuhan investasi dan Kinerja Ekonomi Makro Indonesia penting untuk diperhatikan lebih
Indonesia diketahui sebagian besar provinsi di Indonesia seksama kedepannya.
hanya memiliki komoditas ekspor utama terbatas pada satu B. memiliki gambaran bahwa potensi ekspor
hingga dua komoditas. (11) Hanya beberapa provinsi saja di nonmigas Indonesia lebih besar daripada ekspor
pulau Jawa yang memiliki komoditas ekspor unggulan yang nonmigas.
lebih terdiverifikasi. C. menyimpulkan bahwa beberapa provinsi Jawa
memiliki komoditas ekspor unggulan yang lebih
terdiverifikasi.
Provinsi Tabel Kontribusi Provinsi terhadap Ekspor D. menyadari bahwa kevariasian unggulan produk
provinsi harus didayagunakan untuk meningkatkan
Ekspor pertumbuhan ekspor.
Provinsi E. menyadari bahwa pada tahun 2015 anggota
2005 2006 2007 ASEAN siap menghadapi kerja sama ekonomi
Jawa Barat 23,2 21,4 14,4 antaranggota.
Riau 13,6 13 13,5
Jawa Timur 9,9 10,4 11,5 18. Dari tabel di atas provinsi yang mengalami penurunan
Jakarta 8,1 7,9 7,8 ekspor yang konstan adalah…
Sumut 6,6 6,4 6,5 A. Jawa Barat, Jakarta, Banten, dan Irian
Kaltim 5,9 6,2 5,9 B. Banten, Jakarta, Jawa Barat, dan Jawa Tengah
Banten 7,7 7,6 6,6 C. Banten, Jawa Tengah, Irian, dan Kalimantan
Jawa Tengah 4,2 3,9 3,8 Selatan
Irian 3,8 3,9 4,8 D. Jawa Timur, Banten, Irian, dan Kalimantan Selatan
Kalsel 3,1 3,7 3,8 E. Banten, Jakarta, Jawa Barat, dan Irian

15. Pernyataan manakah yang paling sesuai dengan isi tabel 19. Dari tabel di atas provinsi yang mengalami peningkatan
tersebut? ekspor yang konstan adalah…
A. Hanya Provinsi Jawa Timur yang menunjukan A. Jawa Barat, Riau, Jakarta, Kalimantan Timur, dan
perkembangan positif ekspor nonmigas. Banten
B. Hanya Provinsi Jawa Barat yang menunjukan B. Jakarta, Banten, Jawa Barat, Jawa Timur, dan
persentase ekspor menurun dari tahun 2005 hingga Kalimatan Timur
tahun 2007. C. Kalimantan Selatan, Jawa Timur, dan Irian
C. Provinsi yang paling dominan melakukan kegiatan D. Jawa Timur, Banten, Irian, Kalimantan Selatan
ekspor adalah provinsi-provinsi di Pulau Jawa. E. Jawa Timur, Jakarta, Banten, dan Jawa Tengah
D. Provinsi Riau, Sumut, dan Kaltim mengalami
penurunan ekspor pada tahun 2006 dan bangkit 20. Apabila diurutkan rata-rata jumlah ekspor sepuluh
kembali pada tahun 2007. provinsi tersebut selama 3 tahun dari terbesar sampai
E. Kontribusi ekspor nonmigas Provinsi Riau terkecil adalah…
melebihi akumulasi provinsi-provinsi di pulau A. 2005 – 2006 – 2007
Kalimantan. B. 2006 – 2005 – 2007
C. 2005 – 2007 – 2006
16. Simpulan manakah yang paling tepat untuk teks D. 2006 – 2007 – 2005
tersebut? E. 2007 – 2006 – 2005
i) Logis,
j) Sesuai dengan isi.
Judul dibagi menjadi dua,yaitu : 
TPS-2 1. Judul langsung
PEMAHAMAN BACAAN DAN MENULIS Judul yang erat kaitannya dengan bagian utama
berita, sehingga hubugannya dengan bagian utama
nampak jelas.
I. JUDUL 2. Judul tak langsung
Judul adalah nama yang dipakai untuk buku, dalam Judul yang tidak langsung hubungannya dengan
buku, kepala berita, dan lalin-lain, identitas atau cermin bagian utama berita tapi tetap menjiwai seluruh isi
dari jiwa seluruh karya tulis, bersifat menjelaskan diri karangan atau berita.
dan menarik perhatian. Dalam artikel, judul sering
disebut juga kepala tulisan. Ada juga yang II. MAKNA KATA
mendefenisikan judul sebagai lukisan suatu artikel atau Makna adalah hubungan antara lambang bunyi ujaran
juga disebut miniature isi bahasan. Judul dapat dikatakan dengan hal atau benda yang dimaksud
sebagai jabaran topik atau tema. Karena itu, judul harus
mampu mencerminkan topik atau tema, tidak boleh A. JENIS MAKNA
menyimpang dari isinya. Jenis makna dalam bahasa Indonesia dapat dibedakan
menjadi empat jenis yaitu :
Ciri-Ciri Judul (1)Leksikal
Ciri-ciri judul antara lain : Makna leksikal adalah arti kata seperti apa yang
1. Relevan dengan tema cerita tersebut, atau ada terdapat dalam kamus. Makna leksikal suatu kata
keterkaitan dengan beberapa bagian penting dari kita dapat melihatnya di dalam kamus. Karena
tema tersebut. dalam kamus itu kita menemukan makna kata
2. Bisanya judul harus provokatif dengan menarik si berbentuk kata dasar, maka lebih mudahnya
pembaca dan menimbulkan keingintahuan pembaca makna leksikal adalah makna kata berupa kata
terhadap isi cerita tersebut. dasar. Bila kata itu berdiri sendiri sudah pasti
3. Judul terdiri dari lima kata dan diusahakan tidak bermakna leksikal.
boleh lebih. Contoh:
4. Judul tidak boleh mengambil bentuk kalimat atau  makan : memasukkan sesuatu ke dalam
frasa yang panjang tetapi berbentuk kata yang mulut mengunyah lalu menelannya.
singkat.  uang : alat pembayaran, alat tukar-
menukar.
Syarat-Syarat Judul  bunga : bagian tumbuhan yang akan menjadi
Berikut adalah syarat untuk menentukan judul karangan buah, biasanya elok warnanya dan
yang baik yaitu:  harum baunya.
a) Asli
Jangan menggunakan judul yang sudah pernah ada, (2)Gramatikal/Struktural
bila terpaksa dapat dicarikan sinonimnya. Makna gramatikal/struktural adalah arti yang
b) Relevan muncul setelah adanya proses tatabahasa. Proses
Setelah menulis,baca ulang karangan anda, lalu tatabahasa dalam bahasa Indonesia meliputi :
carilah judul yang relevan dengan karangan anda  afiksasi : pengimbuhan
( harus mempunyai pertalian dengan temanya, atau  reduplikasi : pengulangan
ada pertalian dengan beberapa bagian penting dari
 kompositum = pemajemukan
tema tersebut).
c) Provokatif
Jadi, bila kata dasar yang bermakna leksikal itu
Judul tidak boleh terlalu sederhana, sehingga(calon)
mendapat imbuhan, menjadi kata ulang, atau
pembaca sudah dapat menduga isi karangan anda,
menjadi kata majemuk maka makna itu menjadi
kalau(calon) pembaca sudah dapat menebak isinya
makna gramatikal.
tentu karangan anda sudah tidak menarik lagi.
d) Singkat
Contoh:
Judul tidak boleh bertele-tele, harus singkat dan
 makanan : sesuatu yang dimakan
langsung pada inti yang ingin dibicarakan sehingga
maksud yang ingin disampaikan dapat tercermin  makan-makan : melakukan kerja dengan
lewat judul. santai.
e) Harus bebentuk frasa  rumah makan : kedai tempat makan
f) Awal kata harus huruf kapital kecuali preposisi dan  bunga-bungaan : berjenis bunga
konjungsi,  berbunga : mempunyai bunga
g) Tanpa tanda baca di akhir judul karangan,
h) Menarik perhatian, Catatan :
Dalam bahasa Indonesia kita mengenal jenis kata raja laki
tugas monovalen yaitu kata tugas yang hanya semua anak
berfungsi memperluas kalimat seperti yang, dan, Putri anak wanita raja
wanita
tetapi, bahwa, dari, di, ke, kepada, dll. Kata tugas teman nasi tidak
monovalen ini selalu bermakna gramatikal. Ikan lauk-pauk terbatas pada ikan
saja
(3) Denotasi/Lugas/Polos
Makna denotasi adalah arti yang sebenarnya. Arti (2) Menyempit
sebenarnya ini kita lihat dalam konteks kalimat. Perubahan makna menyempit (spesialisasi), terjadi
Bila kata-kata dalam kalimat tersebut merujuk karena cakupan makna suatu kata sekarang lebih
pada pengertian sesungguhnya, maka disebut sempit daripada makna asalnya atau dengan kata
bermakna denotasi. lain ruang lingkup/ sasaran pemakaiannya sekarang
Contoh: lebih sedikit daripada dulu.
 Andini suka makan bakso. Contoh
 Beni jatuh di kamar mandi. Makna asal Makna baru
kata
 Ayuning rajin menanam bunga. orang yang pemuka agama
ulama
 Baju sekolah adik sudah kotor. berilmu Islam
orang yang pemuka agama
pendeta
(4) Konotasi/Idomatis pandai kristen
Makna konotasi adalah arti yang tidak sebenarnya sarjana cendekiawan gelar universitas
atau sering juga disebut arti kiasan. Arti kiasan ini puisi, prosa,
boleh berupa gabungan kata, tetapi boleh juga sastra tulisan
drama
berupa konteks kalimat, namun lazimnya kita lihat semua alat yang
dalam kalimat. Bila ada satu kata atau beberapa pesawat mempermudah kapal terbang
kata dalam satu kalimat tidak merujuk pada arti pekerjaan
sesungguhnya, maka kalimat tersebut disebut
bermakna konotasi. (3) Ameliorasi
Contoh: Amelioratif adalah perubahan makna kata yang
 Andini suka makan hati melihat tingkah Dewi. nilai rasanya lebih tinggi/halus daripada asalnya.
 Beny jatuh hati pada gadis tetangga rumahnya. Contoh
 Ningsih adalah bunga kelas kami. Makna Asal Makna Baru
Kata
 Kekayaan keluarganya diperoleh dari lebih rendah lebih tinggi
pekerjaan kotor. wanita daripada daripada
 Sungguh hitam hidupnya di masa lalu. perempuan perempuan
lebih tinggi
lebih rendah
B. PERUBAHAN MAKNA Pria daripada laki-
daripada laki-laki
Dalam penggunaannya, suatu kata sering mengalami laki
perubahan makna. Perubahan makna dalam bahasa lebih halus lebih kasar
Indonesia dapat dibagi atas enam bentuk yaitu : tunanetra
daripada buta daripada buta
(1) Meluas lebih halus Lebih kasar
Perubahan makna meluas (generalisasi), terjadi Istri
daripada bini daripada bini
karena cakupan makna sekarang lebih luas
daripada makna asalnya. Cakupan yang dimaksud (4) Peyorasi
adalah ruang lingkup atau tepatnya sasaran Peyoratif adalah perubahan makna karena makna
pemakaian suatu kata lebih banyak sekarang sekarang dianggap lebih rendah nilai rasanya
daripada dulu. daripada dulu.
Contoh
Contoh Makna Asal Makna Baru
Makna asal Makna baru Kata
kata orang yang
mengarungi mengarungi kelompok
gerombolan berjalan secara
berlayar lautan dengan lautan dengan pengacau
bergerombol
kapal layar berbagai kapal kawan dari
setiap perempuan kroni sahabat seorang
Ibu Emak
dewasa, nyonya penjahat
setiap laki-laki Bini istri yang sah istri simpanan
bapak Ayah
dewasa, tuan buruh karyawan pekerja kasar
orang yang seibu orang yang sama
saudara
sebapak kedudukannya
putra anak laki-laki semua anak laki- (5) Sinestesia
Sinestesia adalah perubahan makna kata akibat Jawaban : A
pertukaran tanggapan dua indra yang berbeda.
Contoh III. FAKTA DAN OPINI
Makna Asal Makna Baru
Kata A. FAKTA
kata-katanya indra indra Fakta merupakan sebuah pernyataan memiliki
pedas pengecap pendengaran bentuk dan yang menampilkan situasi yang akurat
indra tentang sebuah masalah atau kejadian, Karena hal
berwajah dingin indra peras yang demikian bisa dikatakan kebenaran dalam
penglihatan
suaranya sangat indra indra sebuah fakta yang sudah teruji. Sementara ada
indah penglihatan pendengaran juga yang berpendapat bahwa Fakta adalah Salah
(6) Asosiasi satu ungkapan yang menyatakan tentang sebuah
Asosiasi adalah perubahan makna yang terjadi karena persoalan atau keadaan yang pernah terjadi dan
adanya persamaan sifat dari hal sesungguhnya yang sedang terjadi.
dengan maksud lain. B. OPINI
Contoh Makna Opini merupakan sebuah pernyataan tentang suatu
Makna Asal ide dan pikiran serta pendapat yang bersifat tidak
Kata Baru
wadah untuk objektif, sehingga belum di sahkan tentang
amplop suap kebenarannya.
memberi uang
buaya binatang buas orang jahat
organ tubuh paling Ciri-Ciri Fakta dan Opini
kepala atasan
atas FAKTA OPINI

Catatan : Belum teruji tentang


Bila diperhatikan baik-baik, kalimat yang mengalami Sudah teruji tentang
kebenaran nya dan
perubahan makna asosiasi sekaligus juga bermakna kebenaran nya di depan
masih bersifat
konotasi karena tidak lagi merujuk pada makna serta bersifat objektif.
subjektif.
sebenarnya. Namun bukan berarti langsung dapat
disimpulkan bahwa makna konotasi sudah pasti Memiliki data yang akurat Tidak memiliki data
mengalami perubahan makna asosiasi. atau pendukung pendukung karena
Dengan kata lain, Perubahan makna asosiasi sudah kebenarannya. bukti yang akurat.
pasti berjenis makna konotasi, tetapi makna konotasi
belum tentu mengalami perubahan makna asosiasi.
Pernah dilihat oleh
Contoh: Merupakan peristiwa
manusia serta telah
(a) Tiga rekor nasional ditumbangkan perenang yang belum terjadi,
dilakukan pengujian dan
berusia 13 tahun itu. karena merupakan
pemastian pada tempat
 mengalami perubahan makna asosiasi yang suatu pendapat saja
umum.
menyatakan makna dilampaui.
 Sekaligus bermakna konotasi karena sudah
tidak merujuk pada arti sebenarnya. Kebenarannya
(b) Sebentar lagi bulan muda kalimat Bersifat objektif kalimat belum dapat
dibuktikan
 berjenis makna konotasi
 tidak mengalami perubahan makna asosiasi
Pernyataan merupakan Pernyataan memiliki
Bila ada soal dimana kalimatnya bermakna konotasi sumber dari peristiwa yang sumber dari saran
sekaligus berasosiasi maka perhatikan apa yang nyata, atau pengamatan atau gagasan pribadi
ditanya. Bila yang ditanya jenis makna, maka jawabnya
konotasi. Bila yang ditanya perubahan makna, maka Pada umumnya pendapat Pendapat setiap
jawabnya asosiasi. dari setiap orang yang orang memiliki
Contoh Soal: sama berbedaan
Pemerintah menyapu bersih pedagang kaki lima.
Kalimat di atas mengalami perubahan makna : Data tersebut kurang
Kalimat Bersifat pasti
A. asosiasi D. leksikal akurat
B. denotasi E. konotasi
C. gramatikal Kalimat Bersifat
Memiliki Data akurat
subjektif
Pembahasan:
Kalimat tersebut sebenarnya bermakna konotasi, tetapi
karena yang ditanya adalah perubahan makna maka SOAL PENGANTAR
jawabnya menjadi asosiasi.
Bacalah teks berikut untuk menjawab soal no 1-6! E. Dinas perhubungan melakukan penataan angkutan
Dinas Perhubungan Gunung Kidul telah melakukan kajian di daerah.
terhadap penataan angkutan di daerah tersebut. Armada
angkutan perkotaan yang ada, ternyata menambah beban 6. Berdasarkan teks tersebut, pengunaan tanda baca yang
konsumen, terutama yang berasal dari berbagai daerah- salah terdapat pada…
daerah sehingga angkutan perkotaan (Angkot) akan dilebur A. Kalimat 1
menjadi satu dalam angkutan pedesaan (angkudes). Para B. Kalimat 2
penumpang yang berasal dari desa terpaksa berganti C. Kalimat 3
angkutan untuk sampai ke kota, baik pusat pemerintahan D. Kalimat 4
kabupaten, pasar, maupun tempat lainnya sehingga E. Kalimat 5
menambah biaya. Di samping itu, jumlah kendaraan,
khususnya angkudes dan angkot, juga semakin menyusut Paragraf Berikut untuk menjawab soal no 7-13!
karena dijual ke luar daerah atau tidak diperpanjang izin
trayeknya akibat semakin sepinya penumpang. (1) Sebuah studi menunjukkan bahwa anak yang
dibiasakan mendengarkan cerita sejak dini dan dikenalkan
1. Pernyataan yang tidak berhubungan dengan isi bacaan di dengan kebiasaan membaca memiliki perkembangan
atas adalah .... jaringan otak yang lebih awal. (2) Sebaliknya, anak yang
A. Angkutan perkotaan ternyata menambah beban tidak dikenalkan dengan kebiasaan membaca memiliki
konsumen perkembangan yang kurang pada jaringan tersebut. (3)
B. Penumpang dari desa harus naik angkutan lebih dari Anak-anak balita dengan orang tua yang rutin membacakan
satu kali untuk menuju pusat pemerintahan. buku untuk mereka mengalami perbedaan perilaku dan
C. Biaya perpanjangan trayek yang mahal membuat prestasi akademik antara anak-anak dengan orang tua yang
pemilik angkutan tidak memperpanjang izin cenderung pasif dalam membacakan buku. (4) Menurut
trayeknya. sebuah studi baru yang diterbitkan dalam jurnal Pediatrics
D. Terjadi perubahan jumlah kendaraan di Gunungkidul menemukan perbedaan yang juga terjadi pada aktivitas otak
E. Penumpang semakin berkurang sehingga banyak anak.
angkutan yang tidak beroperasi (5) Peneliti mengamati perubahan aktivitas otak anak-
anak usia 3 sampai dengan 5 tahun yang mendengarkan
2. Ide pokok bacaan di atas adalah… orang tua mereka membacakan buku melalui scanner otak
A. Hasil kajian terhadap penataan angkutan di yang disebut functional magnetic resonance imaging
Gunungkidul (FMRI). (6) Orang tua menjawab pertanyaan tentang berapa
B. Penyatuan angkot dan angkudes banyak mereka membacakan cerita untuk anak-anak serta
C. Alasan keberatan penumpang angkutan seberapa sering melakukan komunikasi. (7) Para peneliti
D. Menyusutnya jumlah angkutan di Gunungkidul melihat bahwa ketika anak-anak sedang mendengarkan
E. Sepinya penumpang angkutan di Gunungkidul orang tua bercerita, sejumlah daerah di bagian kiri otak
menjadi lebih aktif. (8) Ini adalah daerah yang terlibat dalam
3. Berdasarkan teks di atas kalimat yang tidak efektif memahami arti kata, konsep, dan memori. (9) Wilayah otak
terdapat pada… ini juga menjadi aktif ketika anak-anak bercerita atau
A. Kalimat 1 membaca. (10) Pada studi ini menunjukkan bahwa
B. Kalimat 2 perkembangan daerah ini dimulai pada usia yang sangat
C. Kalimat 3 muda. (11) Yang lebih menarik adalah bagaimana aktifitas
D. Kalimat 4 otak di wilayah ini lebih sibuk pada anak-anak yang orang
E. Kalimat 5 tuanya gemar membaca. (12) Membacakan buku untuk anak
membantu pertumbuhan neuron di daerah ini yang akan
4. Berdasarkan paragraf tersebut, terdapat penulisan kata mengakomodasi anak di masa depan dalam hal kebiasaan
yang salah yaitu… membaca. (13) Membacakan buku dengan cara berhadapan
A. Kalimat 1 langsung dengan si anak juga akan lebih meningkatkan
B. Kalimat 2 pemahaman pada anak.
C. Kalimat 3
D. Kalimat 4 7. Judul yang tepat untuk teks tersebut adalah...
E. Kalimat 5 A. Pengenalan Kebiasaan Membaca sejak Dini
B. Balita dan Kebiasaan Mendengarkan Cerita
5. Berdasarkan teks tersebut, inti dari kalimat pertama C. Pembiasaan Anak dalam Mendengarkan Cerita
adalah… D. Peningkatan Kinerja Otak melalui Membaca
A. Dinas telah melakukan kajian. E. Peran Orang Tua dalam Membacakan Cerita
B. Dinas Perhubungan telah melakukan kajian.
C. Dinas Perhubungan Gunung Kidul telah melakukan 8. Kalimat yang tidak efektif dalam teks tersebut adalah...
kajian. A. 1 dan 7
D. Dinas Perhubungan telah melakukan penaatan B. 2 dan 8
angkutan. C. 3 dan 9
D. 4 dan 10 D. Anak yang dibiasakan mendengarkan cerita sejak
E. 5 dan 12 dini memiliki perkembangan jaringan otak yang
lebih awal.
9. Imbuhan ber-an pada kata berhadapan yang terdapat E. Otak kiri menjadi aktif ketika anak membaca atau
di kalimat (13) mempunyai makna yang sama dengan bercerita.
imbuhan ber-an di dalam kalimat berikut, kecuali...
A. Rumah Adit berdekatan dengan rumah Sari Teks Berikut Untuk Menjawab Soal No 14-20!
B. Kami berkenalan sepuluh tahun yang lalu.
C. Kedua pasangan itu bertatapan muka dengan penuh (1)Dalam beberapa tahun terakhir, mulai banyak pasien
rasa cinta. anak-anak dan remaja-remaja yang memiliki keluhan pada
D. Ayah saya berpandangan sebaliknya. tulang belakang. (2) Padahal, rasa sakit seperti ini biasanya
E. Orang-orang berjalan berpegangan tangan. dialami orang yang berusia 40 tahun ke atas. (3) Anak dan
remaja ini ternyata gemar menggunakan gadget, seperti
10. Pernyataan berikut yang merupakan ringkasan teks telepon pintar, komputer, dan tablet. (4) Banyak dari mereka
tersebut adalah... yang bermain gadget sambil tengkurap, membungkuk, atau
A. Anak-anak yang belajar membaca pada usia 3-5 lehernya ke bawah untuk menatap layar monitor, sehingga
tahun akan mempercepat perkembangan otaknya. kepala membebani leher. (5) Ada pula yang badannya
B. Semakin awal kebiasaan membaca buku bersandar di kursi dengan posisi layar lebih tinggi dari pada
diperkenalkan, semakin aktif otak anak bekerja. mata. (6) Di samping itu, juga ada yang memiringkan kepala
C. Anak-anak yang mulai belajar membaca sejak ke satu sisi untuk menjepit gadget di antara telinga dan
balita akan menjadi anak-anak yang otak kirinya pundak ketika menelepon. (7) Apabila ini dilakukan terus
lebih aktif. menerus, tulang belakang akan “protes” dengan
D. Kebiasaan mendengarkan cerita dan membaca sejak mengirimkan sinyal nyeri. (8) Ada berbagai macam sensasi
usia balita berdampak positif untuk perkembangan nyeri pada tulang belakang, seperti ditusuk-tusuk,
otak. kesemutan, tersetrum, dan nyeri cenat cenut seperti sakit
E. Otak kiri bertanggung jawab terhadap proses gigi. (9) Pada nyeri yang diakibatkan gadget, biasanya
pemahaman seperti kata dan konsep. sensasinya seperti tersetrum. (10) Nyeri ini memang tak
berbahaya. (11) Tetapi, jika dibiarkan terus menerus, nyeri
11. Kalimat manakah yang maknanya sejajar dengan kata ini bisa merusak postur tulang. (12) Ada tiga tahapan nyeri
mengakomodasi pada kalimat (12) teks di atas? pada tulang belakang : rasa nyeri yang dialami otot,
A. Panitia menyediakan fasilitas penginapan secara kemudian menjalar ke sendi, dan terakhir mengenai tulang.
gratis bagi peserta luar kota.
B. Sudah menjadi kewajiban guru untuk memenuhi 14. Kesalahan penggunaan tanda baca ditemukan pada
kebutuhan ilmu pengetahuan para siswa. kalimat...
C. Peserta kemah remaja telah memanfaatkan sarana A. (2)
yang tersedia di lokasi. B. (3)
D. Moderator secara maksimal telah mengatur C. (4)
jalannya sidang dengan baik. D. (6)
E. Pimpinan sidang mengambil keputusan berdasarkan E. (8)
saran-saran anggota.
15. Berdasarkan teks tersebut, kata sambung yang salah
12. Kata serapan yang tidak mengikuti kaidah penulisan adalah ...
kata baku terdapat pada... A. sambil pada kalimat (4)
A. Kalimat (1) B. sehingga pada kalimat (4)
B. Kalimat (3) C. apabila pada kalimat (7)
C. Kalimat (4) D. seperti pada kalimat (8)
D. Kalimat (5) E. tetapi pada kalimat (11)
E. Kalimat (11)
16. Kata 'ini' pada kalimat (7) di atas merujuk pada ....
13. Fakta-fakta berikut terdapat pada teks di atas, A. Posisi menjepit gadget
kecuali.... B. Posisi tengkurap dan membungkuk
A. Anak yang tidak dikenalkan dengan kebiasaan C. Posisi bersandar di kursi
membaca memiliki perkembangan yang kurang D. Posisi menggunakan gadget
pada jaringan otak. E. Posisi menatap layar gadget
B. Otak kiri berfungsi untuk memahami arti kata,
konsep, dan memori. 17. Manakah pertanyaan yang jawabannya tidak terdapat
C. Membacakan buku akan mengakomodasi anak di dalam teks?
masa depan dalam hal kebiasaan membaca. A. Bagaimana rasa nyeri yang disebabkan oleh
penggunaan gadget?
B. Apa akibat dari rasa nyeri yang tidak diobati?
C. Mengapa banyak anak dan remaja menderita nyeri
pada tulang belakang?
D. Bagaimana kebiasaan remaja memainkan gadget?
E. Kapan para anak dan remaja biasanya
menggunakan gadget?

18. Kalimat pertama pada teks tersebut merupakan kalimat


yang tidak efektif, hal yang mengakibatkan
ketidakefektifan tersebut adalah....
A. Ketidaklengkapan unsur kalimat berupa subjek.
B. Ketidaktepatan penempatan unsur kalimat
predikat
C. Ketidaktepatan penempatan unsur kalimat subjek.
D. Penggunaan unsur kalimat yang pleonasme.
E. Ketidaklengkapan unsur kalimat berupa
keterangan.

19. Pada teks tersebut terdapat kalimat yang mengalami


gelaja hiperkorek yaitu...
A. Kalimat 2
B. Kalimat 4
C. Kalimat 6
D. Kalimat 8
E. Kalimat 10

20. Pengunaan tanda baca titik dua (:) pada kalimat (12)
tersebut tidak tepat, hal tersebut terjadi karena...
A. Tanda titik dua dipakai sesudah ungkapan yang
memerlukan pemerian.
B. Tanda titik dua hanya dipakai di antara nomor
atau halaman.
C. Tanda titik dua dipakai pada akhir suatu
pernyataan lengkap yang diikuti pemerincian atau
penjelasan.
D. Tanda titik dua tidak dipakai jika perincian atau
penjelasan itu merupakan pelengkap yang
mengakhiri pernyataan.
E. Tanda titik dua tidak digunakan untuk
menjelaskan tahapan tentang beberapa hal.
Sekarang kita sudah memasuki musim penghujan.
TPS-3 Terdapat banyak sampah yang menumpuk karena
PEMAHAMAN UMUM kita sering membuang sampah sembarangan.
Apalagi dangkalnya permukaan sungai saat ini
bertambah. Sehingga tidak mengherankan apabila
I. PARAGRAF banjir datang setiap hari.
Paragraf atau alinea adalah rangkaian kalimat yang
saling berhubungan dan membentuk satu kesatuan pokok c. Campuran
pembahasan. Paragraf campuran merupakn paragraf yang ide
pokoknya ada pada kalimat awal dan akhir
II. UNSUR-UNSUR PARAGRAF paragraf.
Secara umum, paragraf dibentuk oleh dua unsur, yakni
gagasan utama dan beberapa gagasan penjelas. Contoh :
Pendidikan yang paling utama untuk anak adalah
c. Gagasan Utama pendidikan karakter. Karena dengan pendidikan
Gagasan utama atau sering disebut pikiran utama karakter yang baik, anak memiliki pondasi mental
adalah gagasan yang menjadi dasar pengembangan dan karakter yang kuat. Orang tua dan guru bisa
sebuah paragraf. Gagasan utama paragraf mungkin bekerja sama untuk membentuk karakter anak.
terletak di awal ataupun di akhir paragraf. Ada pula Ketika berhasil, maka ilmu dan nasihat akan lebih
gagasan utama yang berada di awal dan di akhir mudah di transfer ke anak. Jadi, pendidikan
paragraf sekaligus. karakter harus diutamakan dalam pendidikan.

d. Gagasan Penjelas d. Deskriptif


Gagasan penjelas atau sering disebut sebagai pikiran Paragraf deskriptif merupakan paragraf yang ide
penjelas adalah gagasan yang fungsinya menjelaskan pokoknya terlerak di semua kalimat dalam paragraf
gagasan utama. Gagasan penjelas umumnya tersebut.
dinyatakan oleh lebih satu kalimat.
Contoh :
III. CIRI-CIRI PARAGRAF YANG BAIK
Ngarai Sianak Lembah yang sangat indah terlerak
(6) mempunyai satu pokok pikiran
dekat jantung Kota Bukit Tinggi. Kedalaman
(7) mempunyai beberapa pikiran jelas
lembahnya lebih kurang 100 m dan panjangnya
(8) mengandung koherensi/kepaduan
sekitar 15 km. Sepanjang tebing Ngarai Sianok
(9) mengandung kesatuan ide
terlihat tebing-tebing pasir yang curam. Di bawah
(10) taat asas atau sesuai dengan EYD (ejaan yang
tebing itu mengalir sungai yang disebut Batang
disempurnakan)
Sianok. Sungai ini mengalir sepanjang Ngarai itu,
IV. JENIS PARAGRAF seakan menyusuri Lembah Ngarai.
i. Berdasarkan Letak Ide Pokoknya ii. Berdasarkan Tujuan
a. Deduktif a. Narasi
Deduktif merupakan paragraf yang ide pokonya Narasi merupakan karangan yang bertujuan
terletak di kalimat pada awal paragraf. menceritakan suatu peristiwa.
Dalam narasi umumnya kita akan menemukan
Contoh :
 Latar tempat (tempat kejadian)
Ada beberapa varian dendeng di dalam masakan
 Latar waktu (waktu kejadian)
khas Padang, salah satunya adalah dendeng balado.
Makanan ini terbuat dari daging sapi yang dipotong  Tokoh/pelaku (pelaku kejadian)
tipis dan melebar. Daging sapi tersebut kemudian
dijemur dibawah sinar matahari sampai kering. Narasi terbagi dua yaitu :
Setelah dijemur sampai kering, daging sapi (a) Fiksi yaitu berisi khayalan
digoreng, kemudian diberi bumbu berbahan dasar Cerpen, Novel, Roman, Dongeng
cabai atau biasa dikenal dengan bumbu balado. (b) Nonfiksi yaitu berisi kenyataan
Biografi, Otobiografi, Ulasan
b. Induktif
Induktif merupakan paragraf yang ide pokoknya b. Deskripsi
terletak di kalimat pada akhir paragraf. Deskripsi merupakan karangan yang bertujuan
menggambarkan atau melukiskan satu keadaan.

Contoh :
c. Eksposisi
Eksposisi atau paparan merupakan karangan yang - Argumentasi : mempengaruhi
memaparkan atau menjelaskan informasi secara  Fungsi Data:
efisien. - Eksposisi : menjelaskan
Karangan eksposisi sebagai karangan ilmiah - Argumentasi : membuktikan
membutuhkan :  Penutup:
 data - Eksposisi : penegasan
 analisis - Argumentasi : kesimpulan
 sintesis
bila judul karangannya ada, dapat ditentukan
berjenis eksposisi atau bukan dengan rumus. iii. Berdasarkan pola pengembangan paragraf
dapat dibedakan atas :
Bagaimana + Judul
(a) Paragraf Umum – Khusus
Bila judul karangan itu sendiri diawali kata cara, Paragraf yang diawali dengan pernyataan umum
maka itu pasti berbentuk eksposisi. lalu diikuti pernyataan khusus.
Contoh :
- Prosedur Mengikuti UN (b) Paragraf Khusus – Umum
Paragraf yang diawali dengan pernyataan
d. Argumentasi khusus lalu diikuti pernyataan umum.
Argumentasi merupakan karangan yang bertujuan
(c) Paragraf Sebab – Akibat
untuk mempengaruhi pembaca dengan alasan yang
Paragraf yang terdiri dari kalimat hubungan
kuat. Karangan argumentasi yang juga berjenis
sebab akibat
karangan ilmiah membutuhkan :
 Data (d) Paragraf Akibat – Sebab
 Analisis Paragraf yang terdiri dari kalimat hubungan
 Sintesis akibat sebab
 Alasan

bila judul karangannya jenis argumentasi, dapat


ditentukan dengan rumus.

Mengapa + Judul

Contoh :
Mutu Pendidikan di Indonesia Harus Ditingkatkan

e. Persuasi
Persuasi merupakan karangan yang bertujuan
merangsang pembaca agar hanyut dalam isi
karangan.
Persuasi mirip dengan argumentasi, cuma bedanya
argumentasi merangsang pembaca dengan alasan
dan bukti yang kuat, sementara persuasi
merangsang pembaca agar mengikuti apa yang
dikehendaki karena memang pembaca
membutuhkannya.

Contoh :
Deterjen Pemutih Pakaian

Perbedaan dan persamaan Eksposisi dan Argumentasi


Persamaan
 Membutuhkan data, analisis, sintesis
 Sumber ide dari observasi (pengamatan) dan
pengalaman.

Perbedaan
 Tujuan:
- Eksposisi : menginformasikan
SOAL PENGANTAR D. Terapi seni terbukti dapat menurunkan kualitas
hidup penderita sakit secara keseluruhan
Teks berikut untuk menjawab soal No 1 s,d, 6! E. Terapi seni merupakan bentuk penyembuhan
Mendengar kata terapi, boleh jadi kita akan segera dengan menggunakan proses kreasi seni
mengasosiasikannya dengan proses penyembuhan. Tetapi
pada kenyataannya terapi tak hanya menggunakan langkah- 3. Tema yang cocok untuk bacaan di atas adalah ....
langkah medis, tetapi juga memanfaatkan ranah seni. A. Terapi Air Seni
Banyak orang tak menyadari besarnya peranan seni dalam B. Seniman Terapi
kehidupan. Meskipun sering dipandang sebelah mata dan C. Seni untuk Terapi
tidak dianggap bernilai, berbagai penelitian dalam dunia D. Terapi untuk Seni
psikologi di abad ke-20 justru membuktikan bahwa seni E. Terapi bagi Seniman
dapat menjadi salah satu media terapi yang tepat untuk
penyembuhan pascatrauma. Terapi seni dianggap mampu Teks Berikut Untuk Menjawab Soal No 4 s.d. 9!
memberikan pelayanan psikologi bagi yang tengah
mengalami masalah dan tekanan hidup. Terapi seni Teks 3A
diterapkan untuk korban tsunami di Aceh, khususnya bagi
anak-anak dengan pemutaran film keliling dari satu desa ke (1) Bioteknologi merupakan teknologi dengan pemanfaatan
desa, serta lewat melukis dan menggambar. Demikian pula mikroorganisme, tanaman, atau hewan melalui modifikasi
untuk menghadapi anak-anak korban bencana erupsi Merapi proses seluler untuk menghasilkan produk yang bermanfaat.
di Yogyakarta. Mengapa bisa demikian? American Art (2) Banyak negara, khususnya negara-negara maju,
Therapy Association mendeskripsikan terapi seni sebagai menjadikan bioteknologi sebagai penahan terdepan
bentuk penyembuhan yang menggunakan proses kreasi seni ketahanan pangan. (3) Penelitian bioteknologi mencakup
untuk meningkatkan kemampuan fisik, mental, serta berbagai bidang, yaitu pertanian, peternakan,
emosional seseorang dari segala usia. Proses kreatif yang farmakoseutika, kimia, pemrosesan makanan, dan
mengungkapkan ekspresi jiwa dipercaya dapat membantu fermentasi. (4) Di Indonesia sumber daya manusia yang
seseorang dalam menghadapi persoalan hidupnya, berkompetensi di bidang bioteknologi masih sedikit dan
meningkatkan keterampilan diri, mengontrol diri, terbatas. (5) Padahal, perkembangan bioteknologi global dan
mengurangi stres, meningkatkan kepercayaan diri, serta bisnis yang terkait sangat menjanjikan. (6) Tenaga terampil
mendapatkan pencerahan. Melalui gambar, misalnya, dapat dan ahli yang kompeten di bidang bioteknologi sangat
terungkap emosi atau perasaan terdalam seseorang yang diperlukan. (7) Pendidikan dan pengembangan SDM di
terlalu menyakitkan jika diungkapkan dengan kata-kata. Di bidang bioteknologi harus mendapat prioritas dan dukungan,
sisi lain, berbagai penelitian terkini juga mengatakan terapi baik dari pemerintah, universitas, lembaga penelitian,
seni dapat menjadi bentuk terapi yang tepat bagi penderita maupun perusahaan swasta terkait. (8) Semua pihak harus
sakit kronis, seperti kanker. Terapi musik, terapi tari, dan mendukung agar SDM bidang bioteknologi semakin banyak.
terapi seni lainnya membantu pasien kanker mengatasi
kecemasan, depresi, serta meningkatkan kualitas hidup Teks 3B
mereka secara keseluruhan. Tidak hanya itu, terapi seni juga (9) Dewasa ini perkembangan bioteknologi tidak hanya
dianggap efektif untuk menangani penyakit degradasi didasari oleh biologi semata, tetapi juga oleh ilmu-ilmu
memori seperti Alzheimer, orang yang mengalami terapan dan murni lain, seperti biokima, komputer, biologi
penurunan fungsi kognitif, penderita stroke, posttraumatic molekular, mikrobiologi, genetika, kimia, matematika, dan
stress disorder, dan penuaan. lain sebagainya. (10) Dengan kata lain, bioteknologi adalah
ilmu terapan yang mengabungkan berbagai cabang ilmu
1. Kesimpulan yang dapat diambil dari bacaan di atas dalam proses produksi barang dan jasa. (11) Banyak negara
adalah .... menjadikan bioteknologi sebagai pertahanan terdepan
A. Seni bermanfaat sebagai sarana terapi ketahanan panganannya, khususnya di negara-negara maju.
B. Terapi air seni penting untuk kesehatan (12) Akan tetapi, tidak semua pihak dapat menerima
C. Ajakan bagi setiap orang untuk menikmati seni bioteknologi karena dianggap bertentangan dengan kodrat
D. Penyakit cacat fisik dapat sembuh melalui terapi alam. (13) Bioteknologi memunculkan kontroversi,
seni misalnya bayi tabung, pengklonan manusia, dan
E. Seni teater berguna untuk menghibur anak korban transplantasi organ. (14) Kemajuan di bidang bioteknologi
bencana tidak terlepas dari berbagai kontroversi yang melingkupi
perkembangan teknologinya.
2. Pernyataan di bawah ini yang sesuai dengan bacaan di
atas adalah .... 4. Apa makna kata kompeten pada kalimat (6) Teks 3A?
A. Menikmati dunia seni dengan melihat pertunjukan A. Ahli
teater bukan wujud terapi seni B. Pintar
B. Terapi seni kurang bermanfaat untuk mengobati
C. Pakar
orang yang sakit degradasi memori
D. Hebat
C. Terapi seni bermanfaat untuk menghasilkan orang
dengan tingkat emosionalitas tinggi E. Cakap
E. Bioteknologi merupakan teknologi dengan
5. Berdasarkan isi Teks 3A, kepada siapa penulis pemanfaatan mikroorganisme, tanaman, atau
berpihak? hewan.
A. Tenaga terampil bidang bioteknologi
B. Peneliti bidang bioteknologi Teks Berikut Untuk Menjawab Soal No. 10 s.d. 14!
C. Lembaga penelitian bioteknologi Kecepatan komputer mengolah informasi sangat
D. Pengusaha bidang bioteknologi ditentukan oleh prosesornya. Dalam teknologi digital silikon
(konvensional), untuk meningkatkan kecepatan prosesor,
E. Sumber daya manusia bioteknologi
kerapatan transistor dalam cip prosesor harus ditingkatkan.
Upaya meningkatkan kerapatan transistor ini tidak mungkin
6. Apa perbedaan tujuan penulisan Teks 3A dengan Teks
dilakukan terus-menerus tanpa batas karena suatu saat pasti
3B? akan maksimum, yaitu ketika ukuran transistor sudah tidak
A. Teks 3A menjelaskan definisi bioteknologi; Teks dapat diperkecil lagi. Pada keadaan ini perlu ditemukan
3B memaparkan ilmu -ilmu yang mendasari teknologi baru, misalnya teknologi kuantum, untuk
bioteknologi. meningkatkan kecepatan prosesor.
B. Teks 3A menjelaskan pentingnya pengembangan Istilah kuantum (quantum) belakangan ini mulai
SDM bioteknologi; Teks 3B menjelaskan populer dan sering digunakan dalam berbagai konsep yang
penolakan pemanfaatan bioteknologi. memperkenalkan paradigma baru, misalnya quantum
C. Teks 3A menguraikan peran bioteknologi di bidang learning, quantum teaching, quantum business, dan
pangan; Teks 3B memaparkan peran bioteknologi sebagainya. Kiranya tidak berlebihan jika istilah kuantum
di bidang kesehatan manusia. pertama kali diperkenalkan oleh Max Planck, seorang
D. Teks 3A memaparkan keunggulan bioteknologi: fisikawan Jerman yang melahirkan fisika kuantum yang
Teks 3B memaparkan kelemahan bioteknologi. mempunyai  efek dominan pada sistem dalam skala atomik.
E. Teks 3A menjelaskan kendala pengembangan SDM Sejalan dengan perkembangan ilmu fisika dan
bioteknologi; Teks 3B menjelaskan kemajuan informasi, belakangan ini telah dimulai dikembangkan
bidang bioteknologi. komputasi kuantum yang menggunakan prinsip-prinsip
fisika kuantum. Komputasi ini nantinya diharapkan dapat
melahirkan teknlogi kuantum yang memungkinkan
7. Informasi apa yang ada di dalam Teks 3B, tetapi
terobosan teknologi untuk mewujudkan komputer masa
TIDAK dimuat dalam Teks 3A?
depan (komputer kuantum) yang bekerja dengan cara yang
A. Definisi bioteknologi
sama sekali berbeda dengan komputer konvesional yang
B. Pengembangan SDM bioteknologi dikenal saat ini.
C. Penolakan terhadap bioteknologi
D. Cakupan bidang bioteknologi 10. Kecepatan pengolahan informasi masa depan dalam
E. Komitmen pemerintah di bidang bioteknologi sistem komputer yang berteknologi digital silikon, 
seperti sekarang tidak akan lagi diharapkan maksimum
8. Apa kelemahan isi teks? karena....
A. Teks 3A tidak memuat secara terperinci contoh A. Cip prosesornya bermasalah
bidang bioteknologi. B. Ukuran transistornya tidak dapat lagi diperkecil
B. Teks 3B tidak memuat manfaat bioteknologi bagi C. Transistornya sudah lemah
kehidupan. D. Teknologi bersifat konvesional
C. Teks 3B tidak menjelaskan latar tentang produksi E. Ilmu fisika dan informasi sudah berkurang
barang dan jasa.
D. Teks 3A tidak memuat alasan pentingnya 11. Untuk meningkatkan kemampuan kecepatan prosesor
pengembangan SDM bioteknologi. pada masa yang akan datang diperlukan     teknologi
E. Teks 3B tidak memuat komitmen negara maju baru yang disebut teknologi komputer     kuantum yang
mendukung bioteknologi. cara kerjanya sama sekali berbeda     dengan komputer
konvensional saat ini.
9. Apakah gagasan utama paragraf pertama? Pernyataan di atas terdapat pada paragraf .....
A. Bioteknologi dijadkan sebagai penahan terdepan A. Pertama
ketahanan pangan. B. Kedua
B. Bioteknologi merupakan teknologi yang C. Ketiga
bermanfaat. D. Pertama dan kedua
C. Bioteknologi memanfaatkan mikroorganisme, E. Kedua dan ketiga
tanaman, atau hewan.
D. Bioteknologi melalui proses seluler untuk 12. Kata kuantum yang terdapat dalam teks di atas
menghasilkan produk yang bermanfaat. bermakna .....
A. Banyaknya (jumlah) sesuatu
B. Energi yang tak dapat dipecah-pecah lagi
C. Kecepatan C. Konsep hutan larangan dari pengalaman
D. Lompatan yang tergesa-gesa D. Komunitas masyarakat untuk menjaga lingkungan
E. Satuan ilmu hidup
E. Konsep hutan larangan
13. Max Planck , seorang fisikawan Jerman yang bergerak 18. Kesimpulan teks di atas adalah…
di bidang… A. Masyarakat menerapkan norma pengendali sikap
A. Komputansi kuantum B. Kearifan lokal yang ditujukan untuk menjaga
B. Teknologi kuantum kelestarian fungsi lingkungan hidup
C. Cahaya kuantum C. Konsep yang dihasilkan yaitu konsep larangan
hutan
D. Fisika kuantum
D. Pentingnya menerapkan konsep larangan hutan
E. Kecepatan kuantum
E. Hutan sebagai habitat warisan yang perlu
dipertahankan
14. Harapan penulis dari teks di atas tertuang pada ...
A. Paragraf 1 19. Lawan makna kata menjaga pada di atas adalah…
B. Paragraf 2 A. Merawat
C. Paragraf 3 B. Mengasuh
D. Paragraf 1 dan 2 C. Mengampu
E. Paragraf 2 dan 3 D. Melindungi
E. Membiarkan
Tulisan berikut diikuti oleh satu butir pertanyaan.
Pertimbangkan apakah kata atau kalimat pada setiap nomor 20. Digitalisasi saat ini telah menjadi bagian kehidupan
bercetak tebal TIDAK PERLU DIPERBAIKI (A) atau masyarakat dunia. Ancaman terhadap dunia digital
diganti dengan pilihan lain yang tersedia (B,C,D, atau E). muncul. Untuk mencegah masalah tersebut, diperlukan
proteksi terhadap kehidupan digital. Proteksi awal
15. Dia hanya memanfaatkan statusnya sebagai bintang dalam mencegah masalah itu adalah menghapus surat
olahragawan sebagai alat tawar, yaitu pertarungan yang elektronik yang meragukan, mengetik langsung alamat
dia inginkan. Alhasil, dia menghadapi tarung ulang situs internet ketika mengunjungi sebuah situs untuk
dengan (35)nate diaz pada UFC 202, Agustus 2016. pertama kali, menyimpan catatan transaksi online, dan
A. Tidak perlu diperbaiki mengubah kata sandi secara berkala. Selain itu,
B. Nate Diaz perubahan kebiasaan buruk juga perlu dilakukan,
C. NATE Diaz seperti menggunakan sandi yang sama saat login online
D. Nate diaz dan tidak mengubahnya secara berkala. Proteksi juga
E. Nate DiaZ dapat dilakukan dengan menggunakan sistem operasi
yang asli. ...
Teks berikut untuk menjawab soal No. 16 s.d. 19! Kalimat yang dapat menjadi kalimat penutup pada teks
Di antara kearifan lokal yang dihasilkan dari pengalaman di atas adalah ...
adaptasi masyarakat dengan lingkungannya adalah adanya A. Dengan demikian, pengguna dapat memanfaatkan
konsep hutan larangan. Konsep tersebut merupakan sistem digital dengan baik.
pandangan yang bersumber pada pengetahuan masyarakat B. Akhirnya, proteksi terhadap kehidupan digital
dalam upaya pengelolaan lingkungan secara tradisional. dapat tercipta.
Melalui konsep hutan larangan, masyarakat menerapkan C. Sistem operasi asli saat ini sudah tersedia dengan
norma pengendali sikap dan perilaku hidup dalam harga murah.
pengelolaan hutan. Hutan dianggap sebagai habitat warisan D. Dengan menggunakan sistem operasi yang asli,
yang perlu dipertahankan. Oleh karena itu, pada komunitas perangkat elektronik akan terhindar dari virus,
masyarakat tertentu berkembang kearifan lokal yang seperti trojan dan worm.
ditujukan untuk menjaga kelestarian fungsi lingkungan E. Dengan demikian, kehidupan digital dapat
hidup. berlangsung tanpa masalah.

16. Makna konsep yang sesuai dengan bacaan di atas


adalah…
A. Mental dari objek
B. Surat
C. Rancangan
D. Konkret
E. Inti

17. Gagasan utama dari bacaan di atas adalah…


A. Kearifan yang dihasilkan dari pengalaman
B. Kearifan yang dihasilkan dari adaftasi masyarakat
Hasil penelitian menunjukkan bahwa rerata skor sebelum
terapi lebih rendah daripada setelah terapi, baik pada anak-
laki-laki maupun perempuan. Hal tersebut menujukkan
bahwa jika anak-anak mendapatkan terapi bermain, mereka
Bacalah teks berikut! lebih bersikap kooperatif. Sikap kooperatif misalnya
ditunjukkan dalam bentuk kepatuhan dan kemampuan
Kesehatan anak selalu menjadi perhatian utama bagi orang berkomunikasi dengan orangtua. Sikap kooperatif membuat
tua. Bagaimanapun, semua anak kecil berpotensi mengalami anak merasa lebih nyaman saat menjalani perawatan di
sakit. Sakit yang biasa dialami anak-anak adalah flu, batuk, rumah sakit. Dalam kondisi ini terapi diibaratkan sebagai
dan cacar. Sebagian anak yang berpotensi mengalami sakit obat yang berfungsi untuk menyembuhkan atau mengurangi
pada akhirnya harus berobat ke rumah sakit. Banyak anak sakit.
menolak diajak ke rumah sakit. Sayangnya, tidak semua
anak mau dibawa berobat ke rumah sakit. Banyak anak 1. Berdasarkan paragraf 1, manakah simpulan di bawah ini
menolak diajak ke rumah sakit, terutama anak laki-laki, yang benar?
apalagi jika harus menjalani rawat inap. Banyaknya A. sebagian anak kecil yang sakit sering ke rumah
pemandangan orang sakit menjadi salah satu alasan mereka sakit
menolak ke rumah sakit, begitu juga dengan bau obat yang B. sebagian anak kecil yang tidak sakit tidak senang
menyengat dan penampilan staf rumah sakit dengan baju ke rumah sakit
putihnya. Mereka mengira bahwa jika ke rumah sakit, C. sebagian anak kecil yang ke rumah sakit tidak
mereka akan disuntik. Membayangkan sakitnya jarum berpotensi mengalami sakit
suntik menjadi alasan yang paling banyak ditemui D. sebagian anak kecil pada akhirnya harus berobat ke
mengapa anak-anak tidak suka ke rumah sakit. rumah sakit
E. sebagian anak kecil yang sakit flu di rawat inap di
Untuk mengatasi hal ini seorang peneliti melakukan riset rumah sakit
pada anak-anak yang dirawat di rumah sakit yang bertujuan
untuk mengetahui apakah terdapat pengaruh terapi bermain 2. Simpulan apa yang dapat ditarik dari paragraf 3?
terhadap tingkat kooperatif anak prasekolah selama A. anak mendapatkan terapi bermain apabila mereka
menjalani perawatan di ruang pediatri. Tujuan terapi lebih bersikap kooperatif
bermain adalah agar saat menjalani perawatan, mereka tetap B. anak mendapatkan terapi bermain apabila mereka
mendapatkan pembelajaran atau kegiatan menyenangkan. patuh dan dapat berkomunikasi dengan orang tua
Penelitian dilakukan pada anak-anak usia 3-5 tahun. C. anak lebih bersikap kooperatif apabila anak
Prosedur penelitian dilakukan dengan cara memberikan nyaman dalam menjalani perawatan di rumah
terapi bermain pada sejumlah anak. Terdapat dua kelompok sakit
terapi yaitu kelompok laki-laki dan perempuan dengan D. anak mendapatkan terapi bermain apabila
jumlah masing-masing 50 anak, serta terdapat 6 jenis menjalani perawatan di rumah saki
permainan yang diberikan. Sebelum terapi, anak-anak E. anak merasa lebih nyaman saat menjalani
tersebut diukur perilaku kooperatifnya, kemudia dihitung perawatan di rumah sakit apabila mendapatkan
reratanya. Demikian pula setelah terapi, perilaku kooperatif terapi bermain
anak-anak tersebut diukur kembali. Setelah riset, diperoleh
hasil seperti tercantum pada Tabel 1. 3. Berdasarkan paragraf 1, manakah simpulan di bawah ini
Tabel 1. Jumlah Anak yang Kooperatif Setelah Terapi yang paling mungkin benar?
A. anak laki-laki lebih berpotensi sakit dibandingkan
anak perempuan
B. anak perempuan lebih berani dibandingkan anak
Jenis Laki-laki Perempuan laki-laki
permaina C. anak-anak sering mendapatkan pengalaman buruk
n di rumah sakit
D. anak-anak merasa takut pergi ke rumah sakit
E. anak-anak sering dirawat inap di rumah sakit
Tali 20 25
4. Berdasarkan paragraf 1, apabila anak-anak tidak
Bola 30 20 disuntik, manakah simpulan di bawah ini yang benar?
A. anak-anak tidak dapat pengalaman buruk di
rumah sakit
Boneka 15 25 B. tidak ada anak-anak ke rumah sakit
C. anak-anak merasa takut pergi ke rumah sakit
Monopoli 35 35 D. anak-anak tidak dirawat inap di rumah sakit
E. anak-anak tidak suka ke rumah sakit
Ular 12 15
tangga

Rumah- 20 20
rumahan
5. Manakah pernyataan di bawah ini teknologi komunikasi mengalami kemajuan, orang
yang tidak mendukung kalimat bercetak tebal pada mudah dalam berkomunikasi. Kemudahan orang dalam
paragraf 1? berkomunikasi menyebabkan suatu peristiwa yang
A. banyak anak tidak suka ke rumah sakit karena takut dapat diketahui orang di seluruh dunia pada saat yang
disuntik sama. Misalnya, hanya dengan berada di depan
B. banyak anak mendapatkan pengalaman tidak komputer orang bisa melakukan berbagai aktivitas,
menyenangkan ketika disuntik seperti melakukan pembicaraan dengan orang lain,
C. banyak anak belum pernah disuntik di rumah sakit mengirimkan informasi, serta melakukan transaksi
D. banyak anak belum pernah ke rumah sakit karena seperti pembelian, pembayaran, dan penjualan.
takut dokter yang memegang jarum suntik Meskipun demikian, kemajuan teknologi bagi
E. banyak anak tidak suka ke rumah sakit walaupun kehidupan manusia dapat diibaratkan sebuah pisau
tidak pernah tahu jarum suntik bermata dua.
(diadaptasi dari dailysocial.id)
6. Berdasarkan tabel di atas, jenis permainan yang  
menempati keberhasilan kedua tertinggi setelah terapi
adalah ....
A. Tali
B. Bola
C. Boneka
D. Monopoli
E. rumah-rumahan

7. Berdasarkan Tabel di atas, jika hasil tersebut tidak


berubah pada masa mendatang, jenis permainan yang
paling mungkin dipertahakan untuk diberikan pada
anak-anak adalah .... 
A. tali
B. Bola
C. Boneka
D. Monopoli
E. rumah-rumahan  
Gambar 1. Pengguna Internet DKI Jakarta Berdasarkan
8. Pengguna internet di Indonesia mengalami peningkatan menggunakan Internet Tahun 2011-2014
yang sangat tajam. Perkembangan teknologi seperti (dalam persentase)
telepon yang semakin canggih merupakan faktor utama (Sumber: Badan Pusat Statistik)
penyebab melonjaknya pengguna internet di Indonesia.  
Jika teknologi berkembang, pengguna internet Berdasarkan paragraf 1, manakah simpulan di bawah ini
meningkat. Sementara itu, jika seseorang mengikuti yang benar?
perkembangan zaman, seseorang bisa berkomunikasi A. semua orang yang memiliki telepon canggih tidak
dengan berbagai media. Dalam hal ini, tidak ada gagap teknologi
pengguna internet yang gagap teknologi. Sebagian dari B. sebagian orang yang memiliki telepon canggih
pengguna internet tersebut memiliki telepon canggih. gagap teknologi
Jumlah pengguna internet di Indonesia tahun 2017 C. sebagian orang yang tidak memiliki telepon
adalah 143,26 juta. Angka ini adalah sebesar 54,68% canggih gagap teknologi
bila dibandingkan dengan jumlah seluruh penduduk di D. sebagian orang yang memiliki telepon canggih
Indonesia. Hasil survei APJII (Asosiasi Penyelenggara tidak gagap teknologi
Jasa Internet Indonesia) menyatakan bahwa E. semua penggua internet yang gagap teknologi tidak
perkembangan pengguna internet di Indonesia dari memiliki telepon canggih
tahun ke tahun mengalami peningkatan.Kominfo
Indonesia memprediksi peningkatan yang tinggi di 9. Berdasarkan paragraf 3, manakah simpulan di bawah ini
tahun 2018, bahkan peningkatannya lebih besar 50% yang benar?
dari peningkatan di tahun 2017. Para pengguna internet A. suatu peristiwa tidak dapat diketahui orang di
ini mengakses internet di berbagai lokasi, seperti di seluruh dunia pada saat yang sama apabila
warnet, kantor, dan sekolah. Gambar 1 menyajikan data teknologi komunikasi tidak mengalami kemajuan
pengguna internet di DKI jakarta berdasarkan lokasi B. suatu peristiwa dapat diketahui orang di seluruh
pada tahun 2011-2014 dunia pada saat yang sama apabila teknologi
Penerapan teknologi komunikasi dan informasi komunikasi mengalami kemajuan
ditunjukkandengan  berkembangnya nilai-nilai di C. suatu peristiwa tidak dapat diketahui orang di
negara-negara maju. Nilai-nilai tersebut di antaranya seluruh dunia apabila orang tidak mudah
adalah efesiensi, efektivitas, dan rasionalitas. Jika berkomunikasi
D. orang tidak mudah berkomunikasi apabila D. tahun 2012, persentase pengguna internet di warnet
peristiwa tidak dapat diketahui orang di seluruh lebih tinggi daripada gabungan pengguna internet
dunia pada saat yang sama di kantor dan warne
E. orang mudah berkomunikasi apabila peristiwa E. persentase pengguna internet gabungan kantor dan
dapat diketahui orang di seluruh dunia pada saat warnet tahun 2014 lebih rendah daripada tahun
yang sama 2012

10. Berdasarkan paragraf 1, apabila pengguna internet tidak 14. Berdasarkan Gambar 1, apa yang paling mungkin
meningkat atau seseorang tidak bisa berkomunikasi terjadi pada tahun 2015?
dengan berbagai media, manakah simpulan di bawah ini A. selisih persentase pengguna internet antara yang
yang benar? berlokasi di warnet dan kantor semakin kecil
A. teknologi tidak berkembang atau seseorang tidak B. selisih persentase pengguna internet antara yang
mengikuti perkembangan zama berlokasi di kantor dan sekolah semakin besar
B. teknologi  berkembang atau seseorang mengikuti C. selisih persentase pengguna internet antara yang
perkembangan zama berlokasi di warnet dan sekolah semakin kecil
C. teknologi tidak berkembang dan seseorang tidak D. selisih persentase pengguna internet antara yang
mengikuti perkembangan zaman berlokasi di warnet dan kantor semakin besar
D. teknologi berkembang dan seseorang mengikuti E. selisih persentase pengguna internet antara yang
perkembangan zaman berlokasi di warnet dan sekolah semakin besar
E. teknologi berkembang tetapi seseorang tidak
mengikuti perkembangan zaman 15. Kata-kata berikut tidak mengandung imbuhan yang
improduktif, kecuali …
11. Berdasarkan paragraf 3, apabila sebuah negara telah A. manisan (D) serabut
mengembangakan nilai-nilai efisiensi dan efektivitas, B. berjamur (E) terpasang
manakah simpulan di bawah ini yang paling mungkin C. kesehatan
benar?
A. negara tersebut adalah negara maju yang 16. Pengimbuhan me(N)–kan yang tepat terdapat dalam
memberikan fasilitas teknologi kepada warganya kalimat…
B. negara tersebut adalah negara maju yang memiliki A. Televisi swasta acapkali mentayangkan sinetron
penelitian terbaik di bidang teknologi picisan.
C. negara tersebut adalah negara maju yang
B. Aparat pemerintah harus mensukseskan pemilu
memiliki teknologi canggih terbaru
tahun 2004.
D. negara tersebut adalah negara maju yang
memasukkan nilai budaya dalam berkomunikasi C. Pemerintah segera mensosialisasikan kurikulum
E. negara tersebut adalah negara maju yang KBK kepada masyarakat.
menerapkan teknologi komunikasi dan informasi D. Mereka mengkaitkan krisis ekonomi dengan daya
beli masyarakat.
12. Berdasarkan paragraf 3, jika kemajuan teknologi E. Dengan mudah ia dapat menafsirkan harga barang
diibaratkan pisau bermata dua, manakah simpulan di yang dilelang.
bawah ini yang paling mungkin benar?
A. kemajuan teknologi memudahkan pekerjaan 17. Pembangunan rumah mewah itu tidak cukup enam
manusi bulan. Imbuhan pe–an yang semakna dengan imbuhan
B. kemajuan teknologi digunakan untuk melakukan pe–an pada kata pembangunan tersebut adalah…
tindak kejahat A. Karya tulis ilmiah harus ditulis berdasarkan
C. kemajuan teknologi memudahkan manusia dalam penelitian.
berkomunikas B. Penangkapan penjahat itu dilakukan dengan
D. kemajuan teknologi membuat manusia malas kekerasan.
melakukan pekerjaan berat C. Penyaringan air itu terbuat dari sabut kelapa dan
E. kemajuan teknologi dapat memberikan dampak arang.
baik maupun dampak buruk D. Pendapatan pengamen itu bisa seratus ribu per
hari.
13. Berdasarkan Gambar 1, manakah simpulan di bawah ini E. Pendidikan anak itu hanya sampai bangku SMP.
yang benar mengenai pengguna internet di DKI jakarta?
A. persentase pengguna internet gabungan kantor dan 18. Pemakaian akhiran –wan yang salah terdapat pada
sekolah tahun 2011 lebih tinggi daripada tahun 201 kalimat …
B. persentase pengguna internet gabungan kantor dan A. Para ilmuwan sedang mengadakan rapat di
warnet tahun 2013 lebih rendah daripada tahun Bandung.
2014 B. Wartawan pasti ikut meliput perebutan piala dunia
C. tahun 2014, persentase pengguna internet di kantor 1994 di Amerika Serikat.
lebih tinggi daripada gabungan sekolah dan warn C. Gempa di Lampung diteliti oleh para geologiawan.
D. Industriwan Indonesia sudah memakai teknologi B. pesilat, petatar, pemain
canggih. C. pemohon, penatar, pelukis
E. Ia sudah lama mengabdikan diri sebagai D. pedagang, petinju, pejalan
rohaniwan. E. pelari, peminum, pemasak

19. Konstruksi berafiks di bawah ini yang bertransposisi 26. Begitu melihat polisi datang, mereka yang
menjadi kata kerja adalah … bergerombolan itu berlari berhamburan.
A. duduk – menduduki Afiks yang sama dengan kata berhamburan adalah …
B. tinggi – meninggikan A. berjualan
C. remas – meremas B. bertubrukan
D. angkat – mengangkat C. berpandangan
E. pukul – memukuli D. berjatuhan
E. berhalangan
20. Makna imbuhan pada kata bergaris bawah dalam
kalimat “Ia melukai hati kami” sama dengan pada 28. Dunia perbukuan kita. Kalimat tersebut terdiri atas…
kata … A. 3 morfem
A. menggarami B. 4 morfem
B. membumbui C. 5 morfem
C. menyusui D. 8 morfem
D. menulisi E. 9 morfem
E. melebarkan
29. Kata dasar yang menimbulkan nasalisasi /ny/ bila
21. Di antara kata-kata turunan berikut yang tidak mendapat prefiks me(N)– atau pe(N)– adalah …
mengandung imbuhan jenis konfiks adalah … A. suruh
A. kehidupan B. daftar
B. berguguran C. ubah
C. perapian D. junjung
D. perwujudan E. gali
E. diamankan
30. Imbuhan yang diserap dari bahasa Arab terdapat pada
22. Alomorf prefiks pe(N)– terdapat pada proses afiksasi deretan kata ….
berikut, kecuali … A. insani – jauhi – prakasa
A. pengebom B. rohaniah – duniawi – ilmiah
B. pelari C. karyawan – karyawati – muslimat
C. penembak D. alami – alamiah – hadiah
D. pemburu E. muslimin – sastrawan – budiman
E. penyerang

23. Imbuhan me(N)– membentuk kata kerja transitif


terdapat pada kata …
(1) menari, menangis
(2) menyisiki, menolak
(3) menyanyi, melamun
(4) mengambil, membawa

24. Kata-kata berikut ini dibentuk dengan konfiks ber–an,


kecuali …
A. berpakaian
B. bermunculan
C. berlarian
D. berhamburan
E. berlawanan

25. Kata pejuang diturunkan dari kata berjuang. Kata


benda berikut yang dibentuk berdasarkan pola di atas
adalah …
A. petunjuk, pembawa, pemotret
E. Perusahaan itu telah memberikan kepada beberapa
orang karyawan penghargaan.
4. Kalimat berikut yang tidak efektif. Kecuali …
A. Dalam membangun kelompok tertinggal ini,
biasanya banyak kendala sehingga perlu
1. Walaupun petugas kemanan sering merazia, tetapi dirumuskan model pendekatan yang tepat
pedagang kaki lima tetap bermain kucing-kucingan. B. Kajian ini masih berupa catatan sehingga masih
Pemakaian kata walaupun dan tetapi pada kalimat di perlu disempurnakan lagi baik mengenai materi dan
atas tidak tepat. Kalimat tersebut dapat diperbaiki cara pendekatannya
menjadi …. C. Maka orang-orang desa itu jumlahnya tidak di
A. Walaupun petugas kemanan sering melakukan hitung sesuai dengan jumlah mereka tetapi dihitung
razia, namun pedagang kaki lima tetap bermain jumlah tenaganya yang digunakan untuk
kucing-kucingan. mengerjakan tanah
B. Meskipun petugas kemanan sering melakukan D. Kontrol tenaga kerja pada bidang desa dan
razia, tetapi pedagang kaki lima tetap bermain pertanian menuntut prasyarat terselenggaranya
kucing-kucingan. kontrol atas tanah
C. Petugas kemanan sering melakukan razia, E. Supermal Karawachi beroperasi setiap hari mulai
meskipun pedagang kaki lima tetap bermain pukul 09.00 sampai dengan pukul 22.00
kucing-kucingan.
D. Walaupun petugas kemanan sering melakukan 5. Seorang ahli hukum tata negara menyebut hal ini
razia, tetapi pedagang kaki lima tetap bermain sebagai bentuk "kudeta redaksional". Makna kata
kucing-kucingan. kudeta pada kalimat di atas adalah…
E. Petugas kemanan sering melakukan razia, sehingga A. Penggalangan kekuasaan
pedagang kaki lima tetap bermain kucing- B. Penggulingan kekuasaan
kucingan. C. Pengambilalihan kekuasaan
D. Perebutan kekuasaan dengan paksa
2. Karena wawasan masyarakat mengenai antibiotik E. Pengakuan kedaulatan
terbatas, menyebabkan mereka menentukan sendiri
kapan memakai antibiotik yang dijual tanpa resep 6. Kalimat yang tidak efektif adalah …
dokter. A. Tindak kekejaman, kekerasan, dan menindas orang
Kalimat di atas tidak baku. Perbaikannya adalah sebagai kecil merupakan perbuatan tidak terpuji.
berikut …. B. Ketika saya datang, mereka sudah berkumpul di
A. Karena wawasan mengenai antibiotik terbatas, halaman sekolah untuk menanti inspektur upacara.
masyarakat menentui sendiri kapan memakai C. Tata tertib ini tidak boleh diubah sampai ada tata
antibiotik yang dijual tanpa resep dokter. tertib baru yang disahkan oleh pimpinan.
B. Karena wawasan masyarakat mengenai antibiotik D. Sebagai mahasiswa, Anda diharapkan dapat
terbatas, peredaran antibiotik dijual tanpa resep memberi keteladanan yang baik
dokter. E. Diperlukan orang yang sanggup berpikir kritis dan
C. Peredaran antibiotik yang dijual tanpa resep dokter tidak ekstrem.
menyebabkan mereka menentukan sendiri kapan
memakai obat tersebut karena wawasan 7. Di antara kalimat-kalimat berikut yang merupakan
masyarakat mengenai antibiotik terbatas. kalimat baku adalah …………..
D. Peredaran antibiotik yang dijual tanpa resep dokter A. Film ini menceritakan tentang seorang pria yang
menyebabkan wawasan masyarakat mengenai telah beristri tetapi berselingkuh dengan seorang
antibiotik terbatas. model cantik yang pernah dipacarinya dulu
E. Karena wawasan masyarakat mengenai antibiotik sebelum menikah dengan isterinya.
terbatas, peredaran antibiotik ditentukan oleh B. Perekonomian yang kurang baik menjadi faktor
mereka sendiri tanpa resep dokter. utama dari pergaulan bebas yang dilakukan.
C. Akibat dari perbuatan tersebut sangat lah fatal, ada
3. Kalimat yang baku adalah …. yang dikucilkan dari keluarganya, mendapatkan
A. Pemeriksaan terhadap barang bukti membuktikan kesan yang buruk dari masyarakat.
bahwa tersangka bersalah D. Karena penyesalan selalu datang terlambat setelah
B. Seminar itu membicarakan tentang upaya kita mengalami kenyataan-kenyataan yang pahit.
penanganan terhadap bencana alam gempa bumi. E. Akhirnya, model tersebut berhubungan lagi dengan
C. Pihak sekolah menjelaskan kepada orang tua siswa pria-pria yang telah menyakiti hatinya di masa
mengenai peraturan sekolah. lampau tetapi dengan maksud memeras harta
D. Para siswa mendengarkan tentang, penjelasan kekayaannya.
bahaya narkoba.
8. Alokasi anggaran untuk bidang pertahanan dan 13. (1) Partisipasi perempuan dalam bidang politiksaat ini
keamanan tahun ini dirasakan belum memadai bagi terus dituntut untuk ditingkatkan. (2)Namun sayangnya,
Polri untuk memelihara keamanan dan ketertiban di Indonesia, partisipasi politik perempuan ini belum
masyarakat. diikuti pendidikan dan bekal pengetahuan yang
Alokasi dalam kalimat tersebut mengandung maksud .... memadai. (3) Bahwa terdapat beberapa hambatan yang
A. Penempatan dihadapi perempuan dalam partisipasinya dalam bidang
B. Penjatahan politik, antara lain hambatan politik, terutama masalah
C. Penyediaan partai politik dan sistem politik yang tidak mendukung.
D. Penetapan (4) Perempuan selama ini tidak mendapatkan akses
E. Pemenuhan yang cukup untuk memperoleh informasi mengenai
siapa saja calon legislator dan calon pemimpin sehingga
9. Kalimat yang efektif di bawah ini adalah .... mereka sering terjebak dengan memilih calon hanya
A. Korban banjir itu telah diberikan bantuan berupa berdasarkan penampilan. (5) Selain itu, tanpa
beras, selimut, bahan makanan dan bahan darurat pendidikan dan bekal pengetahuan yang memadai,
lainnya untuk membantu mereka yang terkena perempuan seringkali tidak memiliki pertimbangan
musibah tersebut. yang jelas dalam memilih calon anggota legislatif dan
B. Banjir bandang merendam sebelas desa di calon pemimpin lainnya dalam Pemilu. (6) Dalam
Kecamatan Jeruk Legi, Jawa Tengah. mengatasi hal ini,para aktivis perempuan, yaitu
C. Banjir itu terparah selama 25 tahun terakhir. perempuan pemilih yang sadar akan proses dan
D. Banjir menyebabkan seorang warga tewas pentingnya politik, harus memberikan sosialisasi dan
terhanyut di Sungai Jambu. terus mengedukasi mengenai pentingnya mengetahui
E. Luapan air itu berasal dari beberapa sungai-sungai informasi tentang calon pemimpin.
yang terdapat di Cilacap bagian barat. Kalimat yang bentuknya tidak baku dalam paragraf
diatas adalah ... .
10. Kalimat yang mengandung kata yang penulisannva A. kalimat (1) dan (2)
tidak baku adalah … B. kalimat (3) dan (4)
A. Dia membasuh wajahnya dengan sapu-tangan. C. kalimat (5) dan (6)
B. Anggota organisasi harus bekerjasama. D. kalimat (3) dan (6)
C. Pemimpin berprilaku buruk tidak akan mendapat E. kalimat (2) dan (4)
dukungan.
D. Gerakan tubuh penari itu lemah gemulai. 14. (1) Beberapa penelitian mutakhir menemukan peranan
E. Pertikaian antarkelompok dilerai oleh aparat polisi. bau dalam navigasi. (2) Bau berperan sebagai salah satu
mekanisme penting untuk memberi tahu keberadaan
11. Kalimat yang baku di antara kalimat berikut adalah … dan memimpin kemana arah pulang.
A. Berdasarkan survei International Educa-tional Hasil penggabungan terbaik dari kedua kalimat di atas
Achievement (IEA) menempatkan kemampuan adalah ... .
membaca siswa SD Indonesia di urutan ke-38. A. Beberapa penelitian mutakhir menemukanperanan
B. Di sekitar kita banyak penjual makanan dan yang bau untuk memberi tahu keberadaan dan
berharga mudah hingga harga cukup mahal. memimpin kemana arah pulang.
C. Pada zaman yang serba mahal ini, se-bagian besar B. Dalam beberapa penelitian mutakhir, terlihat bahwa
orang pasti ingin mencari jajanan yang murah. peranan bau dalam navigasi sebagai salah satu
D. Dan pedagang makanan asongan sampai pedagang mekanisme penting untuk memberi tahu
makanan kaki lima di pinggir jalan biasa menjual keberadaan dan memimpin kemana arah pulang.
makanan dengan harga yang relatif murah. C. Beberapa penelitian mutakhir menemukan peranan
E. Tanpa memedulikan bahan yang dipakai layak bau dalam navigasi sebagai salah satu mekanisme
untuk dikonsumsi atau mengandung zat-zat yang penting untuk memberi tahu keberadaan dan
berbahaya untuk dimakan. memimpin kemana arah pulang.
D. Menurut beberapa penelitian mutakhir, perananbau
12. Kalimat yang efektif di antara kalimat berikut adalah ... merupakan salah satu mekanisme pentingdalam
A. Untuk mengatasi krisis ekonomi, pemerih dan navigasi untuk memberi tahu keberadaandan
rakyat bahu-membahu satu sama lain. memimpin kemana arah pulang.
B. Ketinggian air kawah d Gunung Kelud yang E. Dalam beberapa penelitian mutakhir peranan
hampir selalu meluap saat hujan. bausebagai salah satu mekanisme penting
C. Makin banvak pendatang-pendatang dari luar dalamnavigasi untuk memberi tahu keberadaan
Jakarta yang diminta untuk mengurus surat izin. danmemimpin ke mana arah pulang ditemukan.
D. Para penonton berlari-lari ketika tembok stadion
roboh. 15. Padahal, akibat aksi unjuk rasa yang berlangsung
E. Mereka saling menghibur ketika dilanda kesedihan. hampir empat bulan tersebut perekonomian di Negara
yang, salah satunya, mengandalkan pemasukan dari
pariwisata itu mulai terganggu.
Kalimat tersebut menjadi efektif jika diperbaiki dengan zaman Barok. (8) Manfaat musik lainnya, menurut
cara ... . sebuah studi pada jurnal Nature Neuroscience pada
A. menghilangkan salah satunya tahun 2011, adalah meningkatkan performa olahraga
B. menghilangkan itu dan memerbaiki suasana hati.
C. menambahkan kata menyebabkan sesudah kata Kalimat yang tidak efektif dalam bacaan di atas adalah
tersebut kalimat ….
D. menambahkan kata dari sesudah kata akibat A. (2)
E. menambahkan kata adalah sesudah kata tersebut B. (4)
C. (5)
16. Kalimat yang merupakan kalimat efektif terdapat pada D. (6)
kalimat .... E. (8)
A. Hutan Ujung Kulon yang semakin rimbun yang
berkontribusi terhadap kelestarian salah satu satwa 19. (1) Nelayan di pantai selatan Kabupaten Kebumen
paling langka di dunia, yaitu badak jawa. memilih mencari lobster selama cuaca buruk melanda
B. Pemilik sawah, pengelola kebun, dan penggembala perairan itu. (2) Selain harganya mahal, biaya
diprioritaskan untuk dilibatkan dalam Newtrees. operasional nelayan juga tidak besar karena area
C. Kawasannya itu umumnya beralih fungsi menjadi tangkapan lobster hanya terletak di sekitar bukit-bukit
sebagai lahan tambak udang dan bandeng. karang tepi pantai. (3) Muslimin (45), nelayan Pantai
D. Walaupun diawasi, namun jalur pantura tetap Pasir, mengatakan bahwa masa panen lobster
terancam. berlangsung pada saat gelombang laut tinggi
E. Dari sekitar 8000 hektare di kawasan mangrove menyebabkan nelayan tidak bisa melaut. (4) Dalam
yang tersisa di Indramayu. sekali tangkapan, paling sedikit nelayan membawa
lobster 2 kg. (5) “Untuk sekali melaut, satu perahu bisa
17. Menurut. Wakil Ketua Eksekutif ASEAN Disability mendapat 4-5 kg, tetapi jika sepi hanya mendapat 2 kg.
Forum menambahkan isu masyarakat inklusif perlu (6) Bahkan, beberapa perahu bisa mendapat 7 kg,"
terus digaungkan agar berbagai pihak ujarnya.
mempertimbangkan hak-hak penyandang disabilitas (7) Dia menuturkan bahwa pendapatan dari menangkap
sebagai bagian dari masyarakat. lobster juga lebih besar daripada ikan laut. (8) Satu
Untuk mengefektifkan kalimat tersebut, dilakukan kilogram bisa terjual Rp250.000,00 hingga
dengan cara .... Rp450.000,00. (9) Lobster merah dihargakan
A. menambahkan kata bahwa setelah kata Rp250.000,00- Rp350.000,00 per kg, lobster hijau
menambahkan Rp250.000,00-Rp450.000,00 per kg, dan lobster
B. menambahkan kata tentang setelah kata isu mutiara Rp600.000,00- Rp1.000.000,00 per kg.
C. menambahkan kata bahwa setelah kata (10) Eko Wardi (33), nelayan di Pantai Menganti,
menambahkan dan menghapus kata terus mengatakan bahwa selain nilai ekonomi lebih tinggi,
D. menghapus kata menurut dan mengganti kata agar jarak dan waktu tangkap lobster lebih pendek. (11)
dengan sehingga Biasanya untuk melaut, nelayan membutuhkan 20 liter
E. menghapus kata menutut solar, tetapi untuk mencari lobster, mereka hanya
membutuhkan 3-4 liter. (12) Selain lobster, kata Eko,
18. (1) Musik, bagi banyak orang, telah menjadi bagian dari beberapa komoditi laut lainnya yang dapat dicari di
kehidupan sehari-hari. (2) Tidak hanya dapat membuat daerah pinggir pantai adalah rajungan dan ubur-ubur.
seseorang bersenandung dan menggerakkan badan, juga Kalimat yang tidak efektif dalam bacaan di atas adalah
dapat membantu kita mengatasi masalah kesehatan. (3) kalimat ….
Terdapat beberapa manfaat musik yang telah terbukti A. (3)
melalui penelitian terkait hubungan musik dengan B. (4)
kesehatan yang dimuat dalam berbagai jurnal ilmiah C. (5)
internasional. (4) Sebuah studi yang dimuat dalam
D. (10)
jurnal Frontiers in Psychology menemukan bahwa
ketika pasien Fibromialgia mendengarkan musik yang E. (11)
mereka sukai selama 10 menit, detak jantung mereka
menjadi lebih lambat dari 120 denyut per menit dan rasa 20. Kalimat berikut ini yang termasuk kalimat yang benar
sakit mereka berkurang. (5) Selain itu, mobilitas para menurut kaidah bahasa, tepat penggunaan kata, dan
pasien juga meningkat dengan mendengarkan musik. jelas isinya adalah ....
(6) Selanjutnya, jurnal Ergonomics pada tahun 2013 A. Mengubah kebiasaan hidup seseorang tidaklah
mempublikasikan sebuah studi yang mununjukkan pekerjaan mudah
bahwa musik dapat membantu pikiran seseorang lebih B. Bupati menghadiakan piala kejuaraan kepada Lurah
terfokus dalam bekerja dan juga membantu Ngampel
meningkatkan kemampuan mengemudi kendaraan. (7) C. Upacara keagamaan di dalamnya bersama-sama
Musik yang sesuai untuk keduanya adalah yang tampil berbagai bentuk kesenian
mempunyai irama pelan dan tenang. seperti musik pada
D. Kebutuhan akan makanan oleh manusia tidak dapat
menunggu sampai besok 24. Bacalah kutipan berikut dengan saksama!
E. Di pabrik itu,setiap mandor membawakan sepuluh (1) Sekarang Bambang telah menjadi atlit sepak bola.
orang kuli (2) Ia menjadi team nasional. (3) Mereka berlatih di
komplek Senayan. (4) Mereka berlatih secara teori dan
21. Menjawab permasalahan yang kompleks dalam tatanan praktik. (5) Pelatih menasehati setiap pemain dengan
dunia yang berorientasi pasar dan diwarnai dengan sa-bar.
kekerasan, sangat mendesak dalam menguak batas- Perbaikan penulisan kata-kata bercetak miring pada teks
batas keilmuan yang memagari suatu perspektif agar tersebut adalah ....
dapat mengembangkan disiplin yang terus-menerus A. atelit, teem, kompleks, manasehati
saling memperkaya satu sama lain. B. atliet, tim, komplex, menasihati
Kalimat tersebut tidak baku dan dapat diubah menjadi C. atlet, tim, kompelek, menasihati
kalimat baku dengan cara berikut ini, KECUALI… D. atlet, team, komplex, mensehati
A. menambahkan kata maka sebelum kata sangat E. atlet, tim, kompleks, menasihati
B. menambahkan kata dalam sebelum kata menjawab
C. mengganti kata dalam dengan kata untuk 25. Bacalah kutipan artikel berikut dengan cermat!
Senam erobik banyak manfaatnya untuk kesehatan,
D. menambahkan kata pada sebelum kata pasar
misalnya: kesehatan jantung, peredaran darah, dan
E. mengganti kata komplek dengan kata kompleks
menghilangkan stress. Apalagi erobik diikuti dengan
teratur dan sistimatik ....
22. Kalimat berikut ini termasuk kalimat tidak baku, Perbaikan kata tidak baku dalam paragraf tersebut
KECUALI… adalah ....
A. Pola kecendrungan itu yang berlaku untuk A. erobik, setres, sistematic
menjelaskan fenomena perubahan budaya kerja B. aerobic, stres, sistimatick
antarepisode, yakni episode 1920—1950an dan C. aerobik, stress, sistematis
episode 1960—1970an. D. aerobik, stres, sistematis
B. Preferensi pada episode pertama adalah budaya E. erobik, setres, sistematik
kerja produktif asketis, dikarenakan petani merasa
tertantang oleh ukuran ekonomi (sebagai taruhan) 26. Cermati kalimat-kalimat berikut!
bagi kehormatan. (1) Bagi siswa-siswi peserta lomba debat di-minta
C. Pada episode kedua, kebanyakan perani relatif berkumpul di aula.
merasa tebebas dari tantangan yang bermuatan (2) Bagi siswa-siswi peserta pelatihan perpusta-kaan
ekonomi atas citra diri mereka di tengah diharap segera daftar ulang.
masyarakat. Alasan kedua kalimat tersebut salah karena
D. Perubahan budaya kerja antarepisode lebih menggunakan ….
mencerminkan fenomena artikulasi pandangan A. kata pelatihan dan perpustakaan
hidup beserta etos budaya yang mempribadi pada B. kata depan bagi pada awal kalmia
diri para petani C. kata siswa-siswi untuk menunjuk jamak
E. Meskipun terlihat perubahan budaya kerja, ternyata D. kalimat pasif untuk mengumumkan
tetap bertumpu pada dan merupakan penjelmaan E. kata peserta setelah subjek
dari pertimbangan strategik yang diwarnai oleh
citra diri. 27. Cermati teks berikut!
Setelah pembelajaran di kelas berakhir, kepala sekolah
23. Suasana hati seseorang sesungguhnya berhubungan bersama guru-guru mengadakan rapat. Dalam rapat itu
dengan makanan yang baru dikonsumsi. Mengkon- membicarakan kelulusan siswa kelas XII.
sumsi makanan yang banyak mengandung karbohidrat Perbaikan kalimat tidak efektif yang bercetak miring
akan memberikan perasaan nyamaan dan pada paragraf tersebut dengan cara ....
meningkatkan kemampuan tubuh menahan rasa sakit. A. mengganti kata membicarakan dengan kata
Tetapi jika berlebihan, akan membuat kita mengantuk. dibicarakan
Sementara itu, mengkonsumsi makanan berprotein B. membubuhkan tanda koma di belakang kata itu
tinggi cenderung membuat badan segar dan penuh C. kata membicarakan diganti dengan kata berbicara
konsentrasi, Makan makanan tinggi protein juga akan D. menghilangkan kata itu
membuat seseo-rang tidak mudah ngantuk. E. menambahkan tanda koma di belakang kata siswa
Dalam paragraf tersebut, terdapat kalimat yang tidak
baku,yaitu kalimat .... 28. Cermati kalimat berikut!
A. pertama Riri malu untuk bilang bahwa dia ndak bisa ikut pergi
B. kedua ke Kebun Raya Bogor pada hari Minggu yang akan
C. ketiga datang.
D. keempat Perbaikan kata yang bercetak miring pada paragraf
E. kelima tersebut adalah ....
A. kata, nggak, dapet
B. katakan, enggak, dapet
C. katakan, tidak, dapat
D. mengatakan, nggak, dapat
E. mengatakan, tidak, dapat

29. Cermati teks berikut!


Perubahan ketetapan melalui peraturan pemerintah
secara berkala lumrahnya mengundang pertanyaan di
benak pelaku industri akan interperetasi undang-undang
seperti apa yang dimulai paling ideal oleh pemerintah.
Bukan apa-apa, inplikasi yang dibawa sebagai dampak
dari kewajiban yang digarisbawahi oleh kata “dan” dan
“atau” ini dapat menimbulkan perbedaan nilai inves
hingga jutaan dolar AS yang harus ditanamkan pelaku
industri.
Kata serapan yang tepat untuk memperbaiki kata yang
bercetak miring adalah ....
A. berintepretasi, berimplikasi, investor
B. menginterpretasi, diimplikasi, investor
C. interfretasi, imflikasi, inpestasi
D. interpretasi, implikasi, investasi
E. berinterpretasi, implikasi, berinvestasi

30. Cermati kalimat berikut!


Keberhasilan pasukan koalisi pemerintah membuktikan,
bahwa persatuan seluruh komponen menjadi syarat
mutlak bagi keberlanjutan Irak.
Kalimat tersebut menggunakan ejaan bahasa Indonesia
yang salah karena ....
A. kata pemerintah seharusnya Pemerintah
B. kata syarat seharusnya sarat
C. tidak menggunakan tanda koma sebelum kata bagi
D. di belakang kata pemerintah tidak menggunakan
tanda koma
E. menggunakan tanda koma sebelum kata bahwa
E. Akibatnya, untuk mengukur tingkat inflasi
diperlukan du acara, yaitu CPI dan GDP deflator.
Bacalah teks berikut untuk menjawab soal No. 4 s.d.
6!
Fungsi Pendidikan adalah serangkaian tugas
ataupun misi yang diemban dan dilaksanakan oleh
Pendidikan. Dari pengertian ini dapat dijelaskan bahwa
sesungguhnya kegiatan atau praktik pendidikan di
1. Manakah kalimat yang tidak efektif? manapun bukanlah kegiatan tanpa makna atau tujuan
A. Setelah lahan dibuka, ditanami padi dan palawija yang jelas. Dalam kegiatan Pendidikan tersirat suatu
hingga tiga kali musim tanam. tugas atau misi yang harus diwujudkan. Oleh karena itu,
B. Akan tetapi, tanah memiliki tingkat kesuburan yang para pendidik seharusnya selalu menyadari akan tugas
berbeda-beda di setiap lahan. atau misi Pendidikan yang dikelolanya.
C. Karena kesuburan tanah akan berkurang, harus Dari segi kultural dan sosial, fungsi Pendidikan
dipulihkan kembali kondisinya. adalah menumbuhkan kreativitas subjek didik; menjaga
D. Hal demikian dilakukan untuk menjaga tingkat kelestarian nilai-nilai insaniah dan ilahiah; serta
kestabilan lahan dan menghindari erosi. menyiapkan tenaga kerja produktif. Sementara itu,
E. Setelah 10-15 tahun, ketika pepohonan kembali Bock mengatakan bahwa peran Pendidikan adalah
tumbuh, lokasi itu dimanfaatkan lagi. memasyarakatkan ideologi dan nilai sosio-kultural
bangsa; menyiapkan tenaga kerja untuk mengurangi
2. Kalimat berikut ini termasuk kalimat baku, kecuali… kemiskinan, kebodohan, dan mendorong perubahan
A. Pola kecendrungan itu yang berlaku untuk sosial; serta memeratakan kesempatan dan pendapatan.
menjelaskan fenomena perubahan budaya kerja Dari sumber tersebut jelas bahwa kegiatan atau praktik
antarepisode, yakni episode 1920-1950-an dan pendidikan memiliki misi atau tugas bukan hanya
episode 1960-1970-an. menjadikan peserta didik bertumbuh dan berkembang
B. Preferensi pada episode pertama adalah budaya potensi atau kemmapuannya, melainkan juga (1)
kerja produktif asketis, dikarenakan petani merasa menjaga dan melestarikan nilai-nilai yang dianggap
tertantang oleh ukuran ekonomi (sebagai taruhan) baik dan benar oleh masyarakat dan bangsa serta (2)
bagi kehormatan. menyiapkan peserta didik menjadi tenaga kerja
C. Pada episode kedua, kebanyakan petani relative produktif. Di pihak lain, fungsi pendidikn nasional
merasa terbebas dari tantangan yang bermuatan menurut Undang-Undang Nomor 20 Tahun 2003 adalah
ekonomis atas citra diri mereka di tengah mengembangkan kemampuan dan membentuk watak
masyarakat. serta perdaban bangsa yang bermartabat dalam rangka
D. Perubahan budaya kerja antarepisode lebih mencerdaskan bangsa…
mencerminkan fenomena artikulasi pandangan
hidup beserta etos budaya yang mempribadi pada 4. Gagasan utama paragraf ke-2 teks di atas adalah…
diri para petani. A. Fungsi pendidikan adalah memasyarakatkan
E. Meskipun terlihat perubahan budaya kerja, ternyata ideologi dan nilai sosio-kultural, meniapkan tenaga
tempat bertumpu pada dan merupakan penjelmaan kerja produktif.
dari pertimbangan strategik yang diwarnai oleh B. Fungsi pendidikan dari segi kultural dan sosial
citra diri. menumbuhkan kreativitas subjek didik serta
meniapkan tenaga kerja produktif.
3. Dalam ilmu ekonomi, inflasi adalah suatu proses C. Fungsi pendidikan adalah menumbuhkan
meningkatna harga secara umum dan terus-menerus kreativitas, melestarikan nilai-nilai, serta
bekaitan dengan mekanisme pasar. Istilah inflasi juga menyiapkan tenaga kerja.
berarti peningkatan persediaan uang ang menebabkan D. Fungsi pendiidkan adalah melestarikan nilai-nilai
kenaikan harga. Inflasi terjadi jika proses kenaikan ang baik dan benar, serta meniapkan peserta didik
harga berlangsung terus-menerus dan saling menjadi tenaga kerja.
memengaruhi. E. Fungsi pendidikan melestarikan nilai-nilai bangsa
Agar menjadi paragraph yang baik, kalimat penutup dan meniapkan peserta didik menjadi tenaga kerja
yang sesuai adalah… produktif.
A. Dalam hal ini, ada dua cara untuk mengukur tingkat
inflasi, yaitu CPI dan GDP deflator. 5. Simpulan teks di atas ang paling tepat adalah…
B. Oleh karena itu, ada dua cara untuk mengukur A. Tujuan pendidikan nasional di Indonesia telah
tingkat inflasi, yaitu CPI dan GDP deflator. mencakup segi kultural dan sosial dan menjangkau
C. Jadi, ada dua cara untuk mengukur inflasi, yaitu semua lapisan masyarakat dalam hal mencerdaskan
CPI dan GDP deflator. bangsa dan menyiapkan tenaga kerja produktif.
D. Dengan demikian, ada dua cara untuk mengukur B. Pendidikan bertugas menjadikan peserta didik
tingkat inflasi, yaitu CPI dan GDP deflator. tumbuh dan berkembang potensinya, melestarikan
nilai-nilai yang benar, serta menyiapkan tenaga kebutuhan tersebbut. Kebutuhan daging per tahun
kerja produktif. dipaparkan dalam tabel berikut.
C. Fungsi pendidikan menurut Undang-Undang No.20
Tahun 2003 selaras dengan pendapat Bock yang Tabel Rata-Rata Kebutuhan Daging Tiap Hari
mengatakan bahwa peran Pendidikan adalah Per Kecamatan (kg)
memasyarakatkan ideologi.
D. Kegiatan Pendidikan memiliki misi bukan hanya Jenis 2008 2009 2010 2011
menjadikan peserta didik tumbuh dan berkembang Daging
potensina, melainkan juga menjaga dan Ayam 3.123 3.233 3.243 3.323
melestarikan nilai-nilai yang diangggap baik dan Kerbau 2.345 1.102 1.097 1.087
benar. Kambing 2.345 2.322 2.445 2.122
E. Pendidikan Nasional menurut Undang-Undang Sapi 4.567 5.568 6.569 7.569
No.20 Tahun 2003 berfungsi mengembangkan
kemampuan membentuk watak serta peradaban 7. Paragraf manakah yang paling tepat melengkapi teks di
bangsa yang bermartabat dalam rangka atas?
mencerdaskan bangsa. A. Peningkatan keutuhan daging sapi tersebut
disebabkan beberapa hal. Erdasarkan table di atas,
6. Agar menjadi paragraf yang baik, kalimat penutup daging sapi paling diminati sehingga keutuhannya
paragraf kedua yang paling tepat adalah…. meningkat. Kebutuhan daging kerbau cenderung
A. Dengan demikian, fungsi pendidikan selain untuk menurun setiap tahun, sedangkan kebutuhan daging
mencerdaskan bangsa, menyiapkan tenaga kerja ayam relatif stail.
produktif, juga membentuk watak. B. Dari tabel tersebut tampak daging sapi paling
B. Dengan demikian, fungsi Pendidikan adalah diminati sehingga kebbutuhannya meningkat. Di
menjadikan siswa terampil dan siap menjadi tenaga sisi lain, kebutuhan daging kerbau dan ayam
kerja produktif, juga membentuk watak. cenderung menurun. Keadaan demikian mendorong
C. Oleh karena itu, tugas Pendidikan adalah kebutuhan daging sapi semakin meningkat.
membentuk siswa menjadi warga negara yang C. Meningkatnya daging sapi tersebut disebabkan
berkepribadian luhur, beriman, dan bertakwa. beberapa kemungkinan. Kesadaran masyarakat
D. Karena itulah, fungsi utama pendidikan adalah untuk mengonsumsi daging semakin kuat sehingga
membentuk siswa ang cerdas dan terampil dalam kebutuhan daging semakin meningkat. Kebiasaan
menghadapi semua permasalahan. makan daging kambing dan kerbau semakin
E. Oleh sebab itu, sebagai pendidik, guru harus selalu menurun dan beralih ke daging sapi.
siap berkorban untuk membentuk anak angsa D. Tabel tersebut menunjukkan bahwa masyarakat
menjadi manusia yang berguna. Indonesia menyukai daging sapi sehingga
kebutuhan daging sapi semakin meningkat. Kondisi
Bacalah teks berikut untuk menjawab soal No. 7 s.d. tersebut tidak dapat dipenuhi dari dalam negeri
9! sehingga pemerintah terpaksa harus mengimpor
Di Indonesia produk ayam, teristimewa dagingnya, dari Australia untuk memenuhi kebutuhan tersebut.
bisa diperoleh dari tiga jenis ayam : ayam pedaging E. Tabel di atas menggungkapkan bahwa masyarakat
(broiler), ayam petelur tua, dan ayam kampung (lokal). Indonesia menyukai daging ayam dan sapi sehingga
Ayam pedaging khusus diternakkan untuk produksi kebutuhan daging ayam dan sapi semakin
daging, biasanya dipanen pada umur relatif muda meningkat dengan jumlah yang relatif sama dari
(sekitar 6 minggu). Ayam petelur sebenarnya dipelihara tahun ke tahun. Kondisi tersebut menyebabkan
untuk memproduksi telur, tetapi bila ayam tersebut tidak pemerintah mengimpor daging sapi dari Australia.
produktif (berumur antara 1,5-2 tahun) dapat juga
dikonsumsi dagingnya. Ayam kampung dijadikan petelur 8. Pernyataan manakah yang paling sesuai denagn isi tabel
dan dikonsumsi dagingnya. Kini ayam kampung semakin di atas?
langka di pasaran. Akibatnya, harga daging ayam A. Konsumsi semua jenis daging dari tahun ke tahun
kampung semakin melambung. terus meningkat.
Kondisi tersebut selayaknya menjadikan kebutuhan B. Konsumsi daging kambing dan kerbau menurun
daging sapi dalam negeri menurun. Hal itu disebabkan drastis pada tahun 2009.
kebutuhan daging dapat dipenuhi dengan mengonsumsi C. Dibandingkan dengan yang lain, konsumsi daging
daging ayam. Di sisi lain, kebiasaan masyarakat kambing paling fluktuatif.
mengonsumsi daging kambing, kerbau, dan kelinci D. Dibandingkan dengan yang lain, konsumsi daging
selayaknya juga semakin memperkecil kebutuhan daging kerbau paling fluktuatif.
sapi. Kenyataannya tidak selamanya demikian. E. Dari tahun ke tahun konsumsi daging kerbau selalu
Kebutuhan daging sapi meningkat sehingga tidak dapat paling kecil.
dipenuhi dari dalam negeri. Pemerintah terpaksa harus
mengimpor sapi dari Australia untuk memenuhi 9. Kata tersebut pada kalimat pertama paragraf kedua di
atas merujuk pada…
A. Produk ayam petelur D. Mereka yang menganggap pekerjaan sebagai
B. Tiga jenis ayam sesuatu yang harus dilakukan. Selain itu, pekerjaan
C. Ketersediaan daging ayam dipandang sebagai cara untuk mencapai tujuan
D. Konsumsi daging yang tidak sepenting kewajiban spiritual dan hal-
E. Produk ayam pedaging hal lain yang menyangkut jiwa.
E. Mereka menganggap pekerjaan sebagai sesuatu
10. (1) Beberapa waktu belakangan ini, muncul tujuan yang harus dilakukan untuk dapat bertahan hidup,
wisata baru dengan tarif murah yang dikelola secara akan tetapi pekerjaan dipandang sebagai cara untuk
mandiri oleh masyarakat di sejumlah daerah di mencapai tujuan dan tidak sepenting kewajiban
Indonesia. (2) Masyarakat pengelola tempat wisata spiritual dan hal-hal lain yang menyangkut jiwa.
mempertahankan tarif masuk agar pengunjung tidak jera
untuk dating lagi, bahkan merekomendasukan kepada 3. (1) Bagi seorang manusia primitif perbuatan mencari
orang lain untuk datang lagi, bahkan merekomendasikan buah-buahan untuk makan pada saat ia lapar merupakan
kepada orang lain untuk datang. (3) Daerah tempat sesuatu yang datang dari nalurinya. (2) Ia berubah maju
wisata alternatif itu antara lain berada di Kabupaten setingkat apabila ia mencari batu untuk melempar hewan
Banyuwangi, Kabupaten Bantul, Kabupaten Gunung yang hendak dimakannya. (3)Akan tetapi, langkah ke
Kidul, Kulon Progo, serta Kabupaten Magelang di arah sivilisasi baru terwujud apabila ia memasukkan
Klaten Jawa Tengah. (4) Salah satu paket wisata yang sebutir jagung ke dalam tanah, memelihara anak pohon
ditawarkan adalah berkunjung ke kebun kopi dan jagung, serta menunggu musim untuk panen. (4) Jadi,
peternakan kambing etawa. (5) Tujuan wisata ini dapat kemajuan nyata dalam peradaban manusia berpangkal
menjadi alternatif bagi masyarakat yang ingin mengisi pada usaha mengelola sumber daya alam, yaitu tanah,
liburan dengan biaya murah. lahan, dan air yang tersedia.
Inti dari kalimat (1) adalah… Gagasan pokok paragraf di atas adalah…
A. Tujuan wisata baru muncul. A. Bagi seorang manusia primitif perbuatan mencari
B. Tujuan wisata baru dengan tarif murah. buah-buahan untuk makan pada saat ia lapar
C. Tarif murah dikelola secara mandiri. merupakan sesuatu yang datang dari nalurinya.
D. Tujuan wisata baru dikelola masyarakat. B. Manusia primitif berubah maju setingkat apabila ia
E. Tujuan wisata baru di Indonesia. mencari batu untuk melempar hewan yang hendak
dimakannya.
1. Kalimat berikut yang merupakan kalimat baku C. Seorang manusia primitif akan melangkah ke arah
adalah… sivilisasi baru apabila ia memasukkan sebutir jagung
A. Pemeriksaan terhadap barang bukti membuktikan ke dalam tanah, memelihara anak pohon jagung, serta
bahwa tersangka bersalah. menunggu musim untuk panen.
B. Seminar itu membicarakan tentang upaya D. Langkah ke arah sivilisasi baru terwujud apabila ia
penanganan terhadap bencana alam gempa bumi. memasukkan sebutir jagung ke dalam tanah,
C. Pihak sekolah menjelaskan kepada orang tua siswa memelihara anak pohon jagung, serta menunggu
mengenai peraturan sekolah. musim untuk panen.
D. Para siswa mendengarkan tentang penjelasan E. Kemajuan nyata dalam peradaban manusia
bahaya narkoba. berpangkal pada usaha mengelola sumber daya alam,
E. Perusahaan itu telah memberikan kepada beberapa yaitu tanah, lahan, dan air yang tersedia.
orang karyawan penghargaan.
4. Primordialisme merupakan faktor penting untuk
2. Mereka tidak hanya menganggap pekerjaan sebagai memperkuat ikatan kelompok kebudayaan ketika ada
sesuatu yang harus dilakukan untuk dapat bertahan ancaman dari luar terhadap kelompok kebudayaan
hidup, tetapi pekerjaan juga dipandang sebagai cara tersebut. Namun, primordialisme dipandang sangat
untuk mencapai tujuan dan tidak sepenting kewajiban negatif karena mengganggu kelangsungan hidup suatu
spiritual dan hal-hal lain yang menyangkut jiwa. bangsa. Primordialisme sering dianggap bersifat primitif,
Kalimat di atas kurang efektif. Perbaikan yang tepat regresif, dan merusak. Primordialisme bahkan dianggap
adalah… akan menghambat modernisasi, proses pembangunan,
A. Mereka menganggap pekerjaan sebagai sesuatu dan merusak integritas nasional.
yang harus dilakukan untuk dapat bertahan hidup. Kalimat yang dapat ditambahkan sesudah kalimat
Pekerjaan juga dipandang sebagai cara untuk terakhir kutipan di atas adalah…
mencapai tujuan walaupun tidak sepenting A. Walaupun dianggap negatif, primordialisme
kewajiban spiritual dan hal-hal lain yang mempunyai nilai positif juga.
menyangkut jiwa. B. Jadi, primordialisme sebenarnya bukanlah sesuatu
B. Mereka menganggap pekerjaan sebagai sesuatu yang perlu dipikirkan terlalu serius.
yang harus dilakukan untuk dapat bertahan hidup. C. Orang yang mempertahankan primordialisme
C. Pekerjaan merupakan cara untuk mencapai tujuan merupakan salah satu perusak kelangsungan hidup
walaupun tidak sepenting kewajiban spiritual dan suatu bangsa.
hal-hal lain yang menyangkut jiwa.
D. Kekuatan suatu bangsa tidak ditentukan oleh yang selama ini berusaha dengan kukuh dan sengit
primordialisme. memerangi upaya bunuh diri.
E. Kekuatan primordialisme akan dapat memicu potensi Pernyataan yang sesuai dengan kutipan di atas adalah…
konflik antara kebudayaan suku-suku bangsa yang A. Sebagian besar penduduk di Belanda menyetujui
ada. eutanasia.
B. Sebagian besar penduduk di Belanda kecewa
5. Cara penulisan gabungan kata yang benar terdapat dalam terhadap pelaksanaan eutanasia.
kalimat… C. Sebagian besar penduduk di Belanda mentolerir
A. Hendaknya kita senantiasa bekerjasama dalam eutanasia.
menyelesaikan segala permasalahan. D. Sebagian besar penduduk di Belanda
B. Surat yang Anda kirim belum Anda tandatangani. memperdebatkan eutanasia.
C. Masyarakat di desa terpencil itu berterima kasih atas E. Sebagian besar penduduk di Belanda menerapkan
bantuan yang Anda berikan. eutanasia.
D. Dia harus mempertanggung-jawabkan perbuatannya.
E. Orang itu di-”meja hijau” –kan 9. Fenomena pekerja anak di Indonesia semula lebih
berkaitan dengan tradisi atau budaya membantu orang
6. Di antara reptil raksasa yang hidup di zaman purba, tua. Sebagian besar orang tua beranggapan bahwa
ternyata, terdapat dinosaurus yang hanya seukuran ayam. memberi pekerjaan kepada anak-anak merupakan upaya
Mungkin, inilah dinosaurus terkecil yang pernah hidup proses belajar, yaitu belajar menghargai pekerjaan, dan
di Amerika Utara jutaan tahun silam. Spesies dinosaurus belajar bertanggung jawab terhadap pekerjaan. Selain
yang diberi nama Albertonykus borealis tersebut dapat melatih dan memperkenalkan anak kepada dunia
merupakan anggota kelompok dinosaurus yang disebut kerja, mereka juga berharap dapat mengurangi beban
Alvarezsaurus. Kelompok satwa purba itu pernah kerja keluarga. Namun, sejalan dengan perkembangan
mendiami pula kawasan Asia dan Amerika Selatan. waktu, fenomena anak yang bekerja banyak berkaitan
Pernyataan yang sesuai dengan kutipan di atas adalah… erat dengan alasan ekonomi keluarga dan kesempatan
A. Albertonykus borealis adalah dinosaurus terkecil di memperoleh pendidikan. Orang tua tidak mampu lagi
dunia. menutupi kebutuhan hidup keluarga. Di pihak lain, biaya
B. Albertonykus borealis ditemukan jutaan tahun silam. pendidikan di Indonesia yang relatif tinggi telah ikut
C. Albertonykus borealis hidup di Asia dan Amerika memperkecil kesempatan mereka untuk mendapatkan
Selatan. pendidikan. Intisari kutipan di atas adalah…
D. Albertonykus borealis sama dengan alvarezsaurus. A. Selain berkaitan dengan tradisi atau budaya
E. Albertonykus borealis ditemukan di Amerika Utara. membantu orang tua, anak-anak menjadi pekerja
karena alasan ekonomi keluarga.
7. Kalimat berikut yang baku adalah… B. Orangtua mempekerjakan anak lebih karena masalah
A. Meskipun menggunakan hitungan waktu, hari, dan ekonomi.
tahun yang sama, tetapi prosesi ritual selamatan C. Masalah pekerja anak berkaitan dengan masalah
orang meninggal berbeda satu dengan yang lain. ekonomi dan pendidikan.
B. Akan tetapi, mempelajari adat budaya Jawa asli D. Pekerja anak merupakan fenomena yang berkaitan
secara lengkap tidaklah mudah karena begitu rumit dengan upaya proses belajar, yaitu belajar
dan kompleks. menghargai pekerjaan, dan belajar bertanggung
C. Dalam perkawinan antara suku Jawa dengan suku jawab terhadap pekerjaan.
lain, tata cara Jawa sering diterapkan bergantian E. Orangtua yang anaknya bekerja tidak mampu lagi
dengan tata cara suku lain tersebut. menutupi kebutuhan hidup keluarga.
D. Dari sekitar 300 suku bangsa Indonesia, suku Jawa
mencapai lebih dari separuh jumlah penduduk 10. Wisata bawah laut di perairan Wakatobi Sulawesi
Indonesia. Tenggara akan menjadi keunggulan kelautan yang
E. Dan demi keselarasan, maka sebagai konsekuensi terintegrasi. Kata keunggulan pada kalimat di atas sama
menjadi orang Jawa, ia harus akomodatif dan toleran. dengan bentuk kata yang digarisbawahi pada kalimat…
(1) Dia berharap ekspor ini akan memberikan
8. Masalah eutanasia kini sedang ramai diperdebatkan keuntungan nyata.
orang. Beberapa saat yang lalu, kita tercengang kembali (2) Hal itu diharapkan dapat meningkatkan
atas berita upaya pengesahan eutanasia dalam hukum kesejahteraan rakyat.
Belanda. Seseorang yang menderita penyakit yang tak (3) Wakatobi diharapkan dapat menjadi kekayaan
terobati berhak memilih mati untuk mengakhiri dunia.
penderitaannya dan UU yang menjamin hak tersebut (4) Pertemuan itu dihadiri tokoh keuangan dan
perlu disahkan. Angket yang disebarkan pun finansial.
menunjukkan paling sedikit tiga perempat penduduk
Belanda cenderung memilih hak individu untuk mati. 11. Masyarakat Amerika, pada umumnya, menyetujui bahwa
Persentase ini mengecewakan kaum rohaniwan di sana pendidikan merupakan faktor yang paling berpengaruh
pada perubahan hidup seseorang. Pendapat lain
menyatakan bahwa perubahan hidup seseorang sehingga makanan yang mengandung lilin akan dicerna
dipengaruhi oleh faktor keberuntungan. Hal ini terlihat dengan waktu minimal dua hari.
dari banyaknya jumlah lulusan sekolah menengah di
Amerika yang memperoleh pekerjaan bergaji lebih besar 22. Berdasarkan paragraf 1, pernyataan mana yang benar?
dibandingkan dengan lulusan perguruan tinggi. A. Persaingan makanan di Indonesia berkembang
Pernyataan-pernyataan berikut ini sesuai dengan kutipan pesat.
di atas, kecuali… B. Makanan yang lezat memiliki harga yang tidak
A. Sebagian besar orang Amerika percaya bahwa terjangkau.
pendidikan dapat mengubah hidup seseorang. C. Makanan cepat saji berkembang pesat di Indonesia.
B. Keberuntungan merupakan salah satu faktor yang D. Makanan cepat saji mengandung kontroversi.
berpengaruh dalam perubahan hidup E. Memakan makanan cepat saji berdampak buruk
C. Lulusan sekolah menengah di Amerika selalu bagi tubuh
memperoleh pekerjaan dengan gaji lebih besar
daripada lulusan perguruan tinggi. 23. Berdasarkan paragraf terakhir apa yang akan terjadi
D. Lulusan perguruan tinggi di Amerika tidak selalu apabila lilih masuk ke dalam tubuh?
memperoleh pekerjaan bergaji besar. A. Memperlambat proses pencernaan tubuh
E. Dalam pandangan banyak orang Amerika, B. Menghambat gejala stroke
pendidikan merupakan faktor yang paling C. Mengalami penyakit ginjal dengan durasi yang
berpengaruh pada perubahan hidup seseorang. lama
D. Mengurangi nafsu makan
E. Menyebabkan makanan tidak tercerna oleh tubuh

Teks berikut untuk menjawab soal No. 22 s.d. 24! 24. Berdasarkan teks di atas, pernyataan mana tentang
Dampak Buruk Junk Food untuk Kesehatan micin yang benar....
Tubuh A. Micin merupakan zat yang berbahaya.
Junk Food disebut makanan instan atau makanan B. Micin mengurangi kemampuan individu dalam
cepat saji yang kini telah berkembang pesat di persaingan berfikir.
perusahaan makanan di Indonesia. Makanan cepat saji C. Micin mengawetkan dan menyedapkan makanan.
dinilai sebagian orang lebih efektif terhadap waktu dan D. Micin hanya dikonsumsi remaja dan orang dewasa.
mudah ditemukan. Tak hanya itu saja, makanan cepat saji E. Micin tidak berdampak langsung terhadap tubuh.
juga memiliki cita rasa yang lezat ditambah lagi harganya
yang terjangkau. Teks berikut untuk menjawab soal No. 24 s.d. 26!
Makanan cepat saji sudah lama mengundang Indonesia adalah suatu negara dengan iklim tropis
kontroversi di negara kita karena terungkapnya beberapa yang terdiri atas ribuan pulau. Walaupun daratan Indonesia
dampak buruk yang ia miliki. Dampak buruk itu disebabkan tak seluas lautannya, hutan di Indonesia sangat banyak
oleh kandungan zat-zat berbahaya di dalam makanan instan mulai dari ujung Aceh yaitu Sabang hingga Merauke
seperti lilin yang ada pada mie instan. Tak berhenti di situ, (Papua). Beberapa tahun terakhir kebakaran di Indonesia
nyatanya di dalam makanan cepat saji terkandung bahan kerap terjadi, hal itu disebabkan dua faktor yaitu faktor alam
pengawet dan penyedap yang kini disebut micin. dan buatan (manusia).
Fenomena kata micin kini mendadak kerap Mengenai faktor alam memang tak ada yang dapat
digunakan para remaja hingga dewasa bila seseorang disalahkan, tetapi mengenai factor buatan yaitu manusia
mengalami hal-hal yang kurang normal. Maksud dari hal itulah hal yang perlu dievaluasi. Manusia kini telah
kurang normal itu seperti seseorang yang telat berpikir, lama kehilangan kesadarannya hingga mereka melakukan hal-hal
menjawab bila diajak bicara dan lain sebagainya. Tak yang merugikan banyak pihak di antaranya merugikan
dielakkan, makanan cepat saji memang mengandung zat lingkungan hidup contohnya hutan. Hutan adalah habitat
berbahaya seperti yang telah diungkapkan di atas. dari ribuan spesies makhluk hidup yang saling
Sejumlah penelitian telah membuktikan bahwa bergantungan.
keseringan mengkonsumsi makanan cepat saja memang Maka dari itu, aksi manusia membakar hutan untuk
tidak berdampak secara langsung ke tubuh. Namun, memenuhi maksud dari dalam dirinya sendiri memang perlu
makanan-makanan cepat saji yang dikonsumsi akan diadili. Alasan mereka melakukan pembakaran hutan
tertimbun di dalam tubuh yang kemudian hari menjadi beragam mulai dari ingin membuka lahan tanam baru hingga
penyebab penyakit mematikan seperti kanker. Tak hanya berdirinya gedung-gedung bertingkat. Namun, hal yang
kanker, penyakit berbahaya juga mengintai misalnya stroke, disayangkan yaitu betapa mereka tak memikirkan aneka
usus buntu dan penyakit ginjal. Maka bila Anda termasuk ke flora dan fauna yang tinggal di dalam hutan tersebut.
dalam orang yang hobi mengkonsumsi makanan cepat saji, Flora dan fauna di dalam hutan akan melarikan diri
kurangilah hal itu dan mulai sayangi tubuh serta diri Anda bahkan akan mati hangus terbakar api yang berkobar karena
sendiri. Perlu diketahui bahwa salah satu kandungan di ulah manusia. Mereka akan kehilangan habitat aslinya dan
dalam makanan instan yaitu lilin sulit dicerna tubuh. Lilin akibat dari hal tersebut yaitu larinya para satwa ke
itu menghancurkan prinsip kerja sistem pencernaan tubuh pemukiman penduduk. Mereka merasa tak lagi memiliki
rumah yang dapat mereka tempati sehingga jalan terakhir 30. Kehidupan masyarakat Jawa selalu diwarnai oleh
ialah lari ke pemukiman warga sekitar. kehidupan simbolis. Unsur simbolis itu sangat berperan
Tak heran bila akhir-akhir ini kasus ditemukannya dalam kehidupan sehari-hari. Dalam menjalani
hewan liar seperti macan dan singa di pemukiman warga kehidupan, masyarakat Jawa mengungkapkan perasaan
sering dikabarkan. Seperti kata pepatah bahwa apa yang kita dan perilaku dengan mengaitkannya pada hal-hal yang
lakukan akan berbalik ke diri sendiri, maka berbuatlah bersifat simbolis. Kebiasaan yang dilakukannya sering
sesuatu yang baik. Sedangkan faktor alam dari kebakaran dituangkan dalam bentuk upacara. Tak pelak lagi,
hutan yaitu musim kemarau dan adanya sambaran petir saat dalam upacara tersebut unsur simbolis sangat berperan
hujan. di dalamnya. Unsur simbolis itu sangat berkaitan
Musim memang tak dapat diprediksi manusia, dengan pandangan hidup masyarakatnya. Oleh
sehingga bila musim kemarau tiba dengan jangka waktu karenanya, unsur simbolis itu haruslah dihayati dan
yang sangat panjang itu wajar. Namun, hal itu dipahami sehingga ungkapan serta keinginan
mempengaruhi keadaan hutan karena hutan yang setiap hari masyarakatnya dapat terkuak dan menjadi pedoman
disinari matahari terik dapat menimbulkan percikan api. Hal hidupnya.
ini juga serupa bila terjadi petir lalu petir tersebut Inti kutipan di atas adalah…
menyambar suatu bagian hingga timbul A. Kehidupan masyarakat Jawa selalu diwarnai
percikan api. kehidupan simbolis.
B. Kehidupan masyarakat Jawa selalu diwarnai oleh
25. Kata berimbuhan yang salah yang digunakan dalam kehidupan simbolis yang harus selalu dihayati dan
paragraf terakhir adalah… dipahami.
A. Menyambar C. Kehidupan masyarakat Jawa yang bersifat simbolis
B. Mempengaruhi selalu dituangkan dalam upacara.
C. Menimbulkan D. Unsur simbolis dalam upacara yang dilakukan
D. Diprediksi masyarakat Jawa sangat berkaitan dengan
E. Disinari pandangan hidupnya.
E. Masyarakat Jawa menjalani kehidupan dengan
26. Pernyataan di bawah ini yang benar menurut paragraf mengaitkannya pada hal-hal yang bersifat simbolis.
kedua adalah....
A. Semua flora dan fauna tinggal di dalam hutan
B. Gedung bertingkat menyebabkan kebakaran hutan
C. Flora dan fauna dirugikan dalam kebakaran hutan
D. Flora dan fauna mati hangus dalam kebakaran
hutan
E. Salah satu faktor alam dalam kebakaran hutan
adalah munculnya petir saat hujan.
27. Berdasarkan paragraf ke dua pernyataan manakah yang
paling mungkin?
A. Hutan terbakar untuk menciptakan lahan
B. Hutan terbakar akibat aktivitas manusia
C. Hutan terbakar karena efek rumah kaca
D. Hutan terbakar di seluruh Indonesia
E. Hutan terbakar hanya karena manusia

28. Berdasarkan teks diatas mengapa banyak hewan liar


berkeliaran di pemukiman manusia?
A. Hewan liar tersebut tersesat
B. Hewan liar ingin memangsa manusia
C. Hewan liar mengalami kepanasan akibat kebakaran
hutan
D. Hewan liar tersebut mengeksplorasi rumah manusia
E. Hewan liar kehilangan habitat aslinya

29. Judul yang tepat untuk teks diatas adalah…


A. Kebakaran Hutan di Indonesia
B. Faktor Manusia yang Menyebabkan Kebakaran
Hutan
C. Dampak Kebakaran Hutan bagi Flora dan Fauna
D. Faktor-Faktor dalam Kebakaran
E. Penyebab Kebakaran Hutan
aequus yang berarti sama dan nox yang berarti malam.
Meski demikian, pada kenyataannya, waktu siang pada saat
ekuinoks sedikit lebih panjang dibandingkan dengan
malamnya. Hal itu terjadi karena saat Matahari terbit dan
terbenam lebih ditentukan berdasarkan pembiasan atau
refraksi sinar Matahari oleh atmosfer Bumi.
Ekuinoks yang terjadi pada Maret memiliki
beberapa sebutan, mulai dari ekuinoks maret, vernal (spring)
equinox, atau titik musim semi. Pada saat ini, Matahari
seolah bergerak dari belahan Bumi selatan menuju belahan
Bumi utara. Ekuinoks maret menjadi penanda datangnya
musim semi di belahan Bumi utara, tetapi jadi tanda
datangnya musim gugur di belahan Bumi selatan.
Bacalah teks berikut untuk menjawab soal no 1 s.d. 7! Sebaliknya, ekuinoks september disebut autumnal equinox
Matahari tepat berada di atas khatulistiwa pada atau ekuinoks september yang jadi penanda datangnya
Kamis, 21 Maret 2019. Pada saat itu, ketika tengah hari atau musim gugur di belahan Bumi utara dan musim semi di
saat Matahari ada di atas kepala, maka semua benda yang belahan Bumi selatan. Pada saat ini, Matahari seolah
ada di sepanjang garis khatulistiwa kehilangan bergerak dari belahan Bumi utara menuju belahan Bumi
bayangannya. Hilangnya bayang-bayang itu terjadi karena selatan. Berikut adalah data mengenai waktu titik musim
bayangan jatuh tegak lurus di bawah kaki atau sebuah semi (vernal Equinox).
benda. Namun, hilangnya bayang-bayang itu hanya terjadi
saat Matahari tepat berada di atas kepala, bukan sepanjang Tahun Titik Musim Semi “Vernal
hari. Beberapa kota di Indonesia yang ada di garis Equinox”
khatulistiwa dan mengalami fenomena hilangnya bayang- 2014 20 Maret 23 : 57 WIB
bayang itu pada 21 Maret lalu antara lain Bonjol di Pasaman 2015 21 Maret 05 : 45 WIB
(Sumatera Barat), Pontianak (Kalimantan Barat), Parigi 2016 20 Maret 11 : 30 WIB
Moutong (Sulawesi Tengah), dan Pulau Kayoa, Halmahera 2017 20 Maret 17 : 28 WIB
Selatan (Maluku Utara). 2018 20 Maret 23 : 15 WIB
Tahun 2019, Matahari tepat berada di atas 2019 21 Maret 04 : 58 WIB
khatulistiwa pada Kamis (21/3) pukul 04.58. Posisi Matahari 2020 20 Maret 10 : 49 WIB
di atas khatulistiwa pada bulan Maret itu bervariasi antara 2021 20 Maret 16 : 37 WIB
19 Maret dan 21 Maret setiap tahun.Tahun 2019, Matahari 2022 20 Maret 22 : 33 WIB
tepat berada di atas khatulistiwa pada Kamis (21/3) pukul
2023 21 Maret 04 : 24 WIB
04.58. Sementara itu, waktu Matahari tepat di atas kepala
2024 20 Maret 10 : 06 WIB
juga berubah-ubah hampir tiap hari dan berbeda untuk setiap
daerah. Namun, untuk daerah di khatulistiwa, perbedaannya
1. Berdasarkan paragraf 1, jika siang hari ini sebagian
tidak terlalu mencolok.Pada Kamis (21/3), menurut
benda di sepanjang khatulistiwa memiliki bayangan,
timeanddate.cm, Matahari tepat di atas kepala di Pontianak
maka …
terjadi pukul 11.49 WIB, Bonjol pada pukul 12.20 WIB,
A. Siang hari ini matahari tepat di atas kepala.
Parigi Moutong pada pukul 12.06 Wita, dan Pulau Kayoa
B. Siang hari ini matahari berada tidak tepat di atas
pada pukul 12.37 WIT.
kepala.
Keberadaan Matahari yang tepat di atas
C. Siang hari ini terjadi gerhana matahari.
khatulistiwa merupakan konsekuensi dari sumbu rotasi
D. Siang hari ini cahaya matahari terhalang awan.
Bumi yang miring 23,5 derajat. Sembari berotasi atau
E. Siang hari ini cuaca cerah.
berputar pada sumbunya, Bumi juga berevolusi mengelilingi
Matahari. Akibatnya, Matahari seolah-olah bergerak bolak-
2. Berdasarkan teks di atas, berikut adalah kota yang berada
balik dari belahan Bumi selatan ke belahan Bumi utara dan
di garis khatulistiwa…
sebaliknya. Dosen Astronomi Institut Teknologi Bandung,
A. Padang, Pontianak, Makassar, dan Maluku
Moedji Raharto, mengatakan, Matahari akan tepat berada di
B. Pontianak, Parigi, Halmahera dan Lombok
khatulistiwa sebanyak dua kali dalam satu tahun, yaitu
C. Halmahera, Parigi Moutong, Pontianak dan Medan
antara 19 Maret dan 21 Maret (paling sering 20 Maret) dan
D. Pontianak, Pulau Kayoa, Halmahera Barat, dan
21-24 September (paling sering 23 September) (Kompas, 20
Pasaman.
Maret 2018). Matahari akan tepat berada di khatulistiwa
E. Halmahera Selatan, Parigi Moutong, Pontianak dan
sebanyak dua kali dalam satu tahun.
Pasaman.
Jika saat Matahari berada tepat di atas khatulistiwa,
pancaran sinar Matahari akan terbagi rata ke seluruh
3. Berdasarkan paragraf 2, manakah pernyataan yang
permukaan Bumi. Maka, seluruh tempat di Bumi akan
benar tentang kondisi kota Pontianak pada tanggal 21
mengalami panjang waktu siang dan malam yang sama,
maret 2019?
yaitu sama-sama 12 jam.Karena itu, peristiwa ini disebut
ekuinoks atau equinox yang diambil dari bahasa Latin,
A. Matahari berada tepat di atas kepala pada pukul diperbincangkan secara terbuka dan hukum yang
12.37 WIB. berlaku cenderung merupakan suatu pengendalian,
B. Pada pukul 11.49 WIB , semua benda yang berada di alih-alih larangan mutlak. (3) Hal inilah yang masih
kota Pontianak kehilangan bayangannya. menjadi hambatan psikologis bagi para dokter yang
C. Kota Pontianak memiliki suhu udara yang sejuk. melakukan praktik eutanasia ini. (4) Perdebatan selama
D. Kota Pontianak memiliki suhu udara yang sama bertahun-tahun yang kemudian menghasilkan
dengan kota Bonjol . kompromi pembuatan garis-garis panduan sementara,
E. Kota Pontianak menjadi satu-satunya kota di garis tetap tidak mengesahkan penerapannya secara resmi.
khatulistiwa yang paling terik sinar mataharinya. Kalimat keempat pada kutipan di atas tidak
menyatakan hal sebagai berikut…
4. Berdasarkan paragraf 4, jika sebagian tempat di bumi A. Eutanasia sudah lama dibicarakan.
mengalami panjang waktu siang yang tidak sama dengan B. Perdebatan eutanasia menghasilkan penyusunan
panjang waktu malam, manakah pernyataan yang paling garis-garis panduan pelaksanaan euthanasia.
tepat? C. Garis-garis panduan tentang pelaksanaan eutanasia
A. Matahari berada tepat di khatulistiwa sementara sudah disusun.
B. Seluruh permukaan bumi sedang mengalami musim D. Pelaksanaan eutanasia secara sah belum dapat
panas dilakukan.
C. Sebagian permukaan bumi sedang mengalami E. Penerapan panduan pelaksanaan eutanasia
musim panas. dikompromikan.
D. Pancaran matahari terbagi rata di seluruh
permukaan bumi. 9. Kalimat berikut yang ditulis sesuai EYD adalah…
E. Matahari tidak berada tepat di khatulistiwa. 5. A. Dalam mengantisipasi terjadinya demon-strasi
ribuan karyawan, yakni menuntut kenaikan upah
5. Berdasarkan paragraf 5, jika matahari seolah bergerak lembur jajaran direktur di perusahaan Metal
dari belahan bumi selatan menuju belahan bumi utara , Perkasa mengadakan koordinasi bersama dengan
maka pernyataan di bawah ini yang paling tepat? para Manajer.
A. Di belahan bumi utara mengalami awal musim B. Dalam mengantisipasi terjadinya demon-strasi
panas. ribuan karyawan, yakni menuntut kenaikan upah
B. Di belahan bumi utara mengalami awal musim lembur jajaran Direktur di Perusahaan Metal
semi. Perkasa mengadakan koordinasi bersama dengan
C. Di belahan bumi utara mengalami awal musim para Manajer.
dingin. C. Dalam mengantisipasi terjadinya demon-strasi
D. Di belahan bumi utara mengalami akhir musim ribuan karyawan, yakni menuntut kenaikan upah
panas. lembur, jajaran direktur di perusahaan Metal
E. Di belahan bumi utara mengalami akhir musim Perkasa mengadakan koordinasi bersama dengan
semi. para manajer.
D. Dalam mengantisipasi terjadinya demon-strasi
6. Berdasarkan teks di atas, manakah di bawah ini ribuan karyawan, yakni menuntut kenaikan upah
pernyataan yang benar? lembur, jajaran Direktur di perusahaan Metal
A. Vernal Equinox merupakan penanda titik musim Perkasa mengadakan koordinasi bersama dengan
semi di belahan bumi selatan. para Manajer.
B. Pada Autumnal Equinox, matahari bergerak dari E. Dalam mengantisipasi terjadinya demonstrasi
belahan bumi utara ke belahan bumi selatan. ribuan karyawan yakni menuntut kenaikan upah
C. Terbit dan terbenamnya matahari ditentukan oleh lembur, jajaran direktur di perusahaan Metal
kemiringan sumbu rotasi bumi. Perkasa mengadakan koordinasi bersama dengan
D. Gerak rotasi bumi menyebabkan waktu siang lebih para Manajer.
lama.
E. Vernal Equinox merupakan titik awal musim panas. 10. Kesalahan penerapan EYD dijumpai dalam kalimat
berikut, kecuali ....
7. Berdasarkan data titik musim semi pada tabel di atas, A. Darah tali pusat mengandung banyak stem cells
waktu terjadinya titik awal musim semi tahun 2020 ke yang dapat digunakan untuk meng-obati berbagai
2021 bergeser sejauh … penyakit kelainan darah.
A. 3 jam 20 menit. B. Kebijakan konfrontasi agaknya lebih dipilih George
B. 4 jam 45 detik.
Bush dalam politik luar negerinya terutama pasca
C. 5 jam 48 menit.
peristiwa 11 September 2001.
D. 7 jam 11 detik.
E. 6 jam 10 menit C. Pengambilan darah tali pusat berlangsung singkat
tetapi menghabiskan biaya sekitar Rp 11 juta.
8. (1) Sebenarnya, eutanasia telah lama ditolelir di
Belanda. (2) Sejak tahun 1970-an penerapannya telah
D. Sebagai tetangga kita, Filipina dapat keluar dari A. Jumlah produksi pisang dari tahun ke
lilitan masalah internal yang dipicu oleh instabilitas tahun selalu mengalami peningkatan.
politik. B. Produksi pisang tidak selaras dengan
E. Orang superkaya di dunia sekarang ini sepertinya
jumlah tanaman pisang.
mempunyai mainan baru.
C. Luas lahan tanaman pisang selaras
11. Tanda baca dalam kalimat berikut ini mematuhi aturan dengan jumlah tanaman pisang.
EYD, kecuali .... D. Jumlah produksi pisang berhubungan
A. Jadi, persoalan pertama yang harus ditangani bupati dengan luas tanaman pisang.
adalah menyadarkan masyarakat, baik kaya E. Jumlah tanaman pisang menurun tidak
maupun miskin, untuk turut membantu konsisten dari tahun ke tahun.
pembangunan bangsa.
B. ”Mari kita tingkatkan kualitas belajar kita”, ujar 14. Dalam kalimat-kalimat berikut ini, terdapat bentukan
kepala sekolah, ”agar kita dapat mengangkat citra kata yang benar, kecuali ....
sekolah ini.” A. Ledakan bom yang dahsyat itu
C. Oleh karena itu, kebiasan hidup bersih perlu
memorakporandakan bangunan yang ada di
digalakkan pada setiap individu, sehingga kita
sekitarnya.
dapat menikmati kehidupan ini.
B. Gempa bumi yang berkekuatan besar dapat
D. Karena kebiasaan membuang sampah seenaknya,
meluluhlantahkan bangunan-bangunan megah.
baik oleh anak-anak maupun orang dewasa, banjir
terjadi di mana-mana. C. Amukan massa yang brutal dapat
E. Pendidikan ini misalnya, diharapkan da-pat menghancurkanleburkan berbagai fasi-litas umum
meningkatkan pengetahuan, keteram-pilan, dan jika tidak segera diamankan.
sikap siswa dalam menghadapi kehidupan. D. Sebenarnya dia sendiri yang bersalah, tetapi selalu
mengambinghitamkan orang lain.
12. Sumber utama masalah Pertamina adalah keterangan E. Jangan suka memutar-balikkan fakta yang sudah
Pertamina pada pemerintah dalam urusan impor minyak jelas dan sudah diketahui oleh orang banyak.
mentah. Pertamina membeli minyak mentah untuk
15. Pada era globalisasi ini, diakui oleh seluruh lapisan
menutup kebutuhan dalam negeri dengan harga pasar
masyarakat bahwa teknologi dan ilmu pengetahuan
dunia. BBM itu kemudian dijual Pertamina ke
mengalami perkembangan yang pesat sehingga
konsumen dalam negeri dengan harga lebih rendah,
berdampak besar pada lingkungan kehidupannya. Hal
yaitu dengan harga subsidi. Selisih harga itu ditanggung
ini tercermin dalam upaya masyarakat yang terus
pemerintah dengan dana subsidi BBM di APBN.
berusaha untuk belajar dan menyesuaikan diri dengan
Pernyataan berikut sesuai dengan teks, kecuali ….
perkembangan yang ada. Mereka terbius oleh
A. Pertamina membeli minyak mentah dengan harga
kecanggihan teknologi sehingga berdampak negatif
dunia dan menjualnya dengan harga subsidi.
B. Pertamina mengalami kerugian dalam yang ditimbulkan oleh teknologi tersebut sering
mengusahakan minyak untuk masyarakat dalam terabaikan. Namun, saat ini mulai disangsikan
negeri. manfaatnya karena dianggap merusak tata lingkungan
C. Biaya yang dikeluarkan pemerintah untuk dana dan terjadi banyak bencana. Karena itu, yang perlu
subsidi berasal dari anggaran negara. direnungkan adalah bagaimana manusia dapat
D. Masalah utama Pertamina adalah keti-dakmampuan mempertahankan kearifan dalam mengolah lingkungan
mengimpor minyak mentah. hidup tersebut.
E. Kebutuhan minyak mentah dalam negeri tidak Pilihan kata yang tidak tepat dalam paragraf tersebut
dapat dipenuhi oleh Pertamina. adalah ....
A. diakui (kalimat 1)
B. tercermin (kalimat 2)
13. Produksi Pisang Indonesia Tahun 1997-1999 C. terabaikan (kalimat 3)
D. terjadi (kalimat 4)
Jumlah Luas Produksi E. direnungkan (kalimat 5)
Tahu
n 16. Jumlah dana yang dialokasikan untuk pember-dayaan
tanaman (ha) (ton) masyarakat di tingkat kelurahan cukup besar sehingga
1997 281.137.644 175.616 2.192.332 tidak menutup kemungkinan peningkatan kualitas
lingkungan yang jauh lebih baik daripada yang dicapai
1998 251.750.569 169.633 2.308.378
tahun lalu.
1999 249.105.543 155.961 2.417.780 Kalimat tersebut tidak efektif sehingga peru diperbaiki
dengan cara ....
Simpulan berikut ini sesuai dengan teks A. mengganti kata dialokasikan dengan dianggarkan
tersebut, kecuali .... B. mengganti kata untuk dengan guna
C. mengganti daripada dengan dibandingkan digunakan untuk menentukan tipe kerusakan
D. menghilangkan kata cukup dan jauh terumbu karang.
E. mengganti kata menutup dengan tertutup
19. Koperasi merupakan bentuk perekonomian rakyat yang
17. Kesibukan acapkali menjadi alasan sehingga kita sistem pengelolaannya berasal dari rakyat, oleh rakyat,
”menghalalkan” segala cara untuk memperoleh dan untuk rakyat. Koperasi sebagai wadah yang tepat
kepraktisan, termasuk membeli buah potongan. Buah untuk meningkatkan hajat hidup masyarakat, terutama
tersebut dalam hal rasa, mungkin tidak akan menjadi mereka yang hidup pada tingkat ekonomi kelas bawah.
masalah, tetapi, dalam hal nutrisi, tertentu berbeda. Hal itu menjadi latar belakang pentingnya
Menurut penelitian, sinar dan oksigen biasanya akan pemberdayaan koperasi yang mempunyai prinsip dasar
mempercepat penghancuran vitamin yang terkandung menyejahterahkan seluruh anggotanya dan sebagai
dalam buah tersebut. kekuatan penyeimbang dalam system ekonomi.
Kalimat berikut ini yang tepat untuk menutup paragraf Koperasi perlu dikembangkan dan membentuk jaringan
tersebut adalah .... kerja sama antara koperasi dari berbagai negara. Tidak
A. Hal itu membuat nutrisi pada buah akan berkurang. adanya koperasi, usaha kecil menengah sulit bersaing
B. Akibatnya, kita perlu membeli buah potongan agar dengan perusahaan-perusahaan multi nasional yang
lebih memudahkan kita dalam memakannya. dikelola dengan mengedepankan penerapan prinsip
C. Oleh karena itu, untuk memaksimalkan nutrisi yang efektifitas, efisiensi, dan produktifitas.
terdapat pada buah, seharusnya kita memotong Paragraf tersebut akan menjadi paragraf yang baik jika
sesaat sebelum kita makan. diperbaiki dengan cara berikut, kecuali…
D. Itu sebabnya, mengapa buah terasa tidak segar lagi A. Mengganti kata sebagai dalam kalimat kedua
setelah dipotong dan dibiarkan beberapa saat.
dengan kata merupakan
E. Dengan kata lain, proses tersebut akan
menghilangkan nutrisi pada buah yang kita makan. B. Mengganti kata dan dalam kalimat ketiga dengan
kata untuk
18. Coremap (Coral Reef Rehabilitation and Management C. Menambahkan kata dengan pada awal kalimat
Program) akan memetakan terumbu karang di sebagian keempat
wilayah Indonesia. Dari luas lahan terumbu karang di D. Mengubah tulisan multi nasional dalam kalimat
Indonesia 4,5 juta hektar, hanya 30% yang masih bagus, kelima menjadi multinasional
45% rusak, dan 25% dalam kondisi kritis. Pemetaan E. Mengubah tulisan efektifitas dan produktifitas
pertama dilakukan di Selayar dan Pangkep, Sulawesi dalam kalimat kelima menjadi efektivitas dan
Selatan. Wilayah ini memiliki lahan terumbu karang produktivitas
seluas 5.970 kilometer persegi dengan kerusakan
mencapai 40-60%. Metode yang dilakukan adalah 20. Lembaga penyiaran publik seperti yang dimaksud dalam
pemetaan lahan berdasarkan kadar kerusakan sehingga konsep negara-negara Eropa Barat adalah lembaga
dapat diketahui wilayah terumbu karang yang masih penyiaran, baik televisi maupun radio, yang dimiliki
asli, kerusakan rendah, rusak, dan rusak berat. negara, tetapi isi pengelolaannya dilakukan secara
Ringkasan dari paragraf tersebut adalah .... independen.
A. Hasil pantauan Coremap adalah terumbu karang di Kalimat inti pada kalimat luas tersebut adalah…
Indonesia seluas 4,5 juta hektar telah banyak A. Lembaga penyiaran publik adalah lembaga
mengalami kerusakan, 2,5% dalam kondisi kritis, penyiaran.
yang utuh hanya 30% dan sisanya rusak. B. Lembaga penyiaran publik seperti konsep Negara
B. Pemetaan awal terumbu karang telah dilakukan di Eropa Barat.
Selayar dan Pangkep, Sulawesi Selatan. Hasilnya, C. Lembaga penyiaran dimiliki negara.
D. Lembaga penyiaran publik memiliki negara, tetapi
lahan seluas 5.970 kilometer persegi telah
isi pengelolaannya dilakukan secara independen.
mengalami kerusakan hingga mencapai 60%. E. Isi pengelolaan lembaga penyiaran publik dilakukan
C. Coremap melakukan pemetaan terumbu karang di secara independen.
wilayah yang mengalami kerusakan terberat.
Sasaran utama adalah Sulawesi Selatan, yakni 21. Perluasan kalimat inti Pernyataan itu mengemuka
Selayar dan Pangkep, yang kerusakannya mencapai menghasilkan kalimat berikut, kecuali ...
40-60%. A. Pernyataan itu mengemuka dalam jumpa pers yang
D. Indonesia memiliki wilayah terumbu karang yang diselenggarakan di sebuah stasiun televisi swasta
sangat luas , yakni 4,5 juta hektar. Namun lahan B. Pernyataan kontroversial presiden menge-muka saat
yang masih asli hanya 30%, yang lainnya telah sidang kabinet berlangsung.
rusak, bahkan 25% mengalami krisis. C. Karena wartawan terus memancing, akhirnya
E. Coremap akan memetakan wilayah terumbu karang pernyataan artis cantik tentang rencana
di Indonesia. Pemetaan pertama dilakukan di pernikahannya itu mengemuka juga.
Selayar dan Pangkep, Sulawesi Selatan, yang D. Pada kenyataannya, pernyataan itu, yang terlontar
mengalami kerusakan mencapai 60%. Metode secara tak sengaja dari mulut pejabat teras di
pemetaan berdasarkan kadar kerusakan akan perusahaan kami, terus mengemuka dari rapat ke
rapat.
E. Akhirnya, pernyataan yang mengemuka dalam rapat E. Kehilangan cairan tubuh sebesar 10-12 persen akan
koordinasi bidang pertahan-an dan keamanan mengakibatkan situasi. sangat kritis: kemampuan
dipermasalahkan hingga saat ini. mental berkurang, menelan menjadi sulit, dan
kerusakan fungsi badan.
22. Masyarakat petani di daerah yang jauh dari pusat
informasi biasanya bersifat ..., yakni mereka selalu 25. Pascagempa dan tsunami di Nangroe Aceh Darussalam dan
bertahan pada sistem tanam yang biasa mereka lakukan. Sumatra Utara 26 Desember 2004, beberapa negara
Mereka tidak mau melakukan atau mendapatkan ... lain membantu sistem pemantau gempa dan peringatan dini
untuk melakukan sistem pertanian yang lebih maju. tsunami kepada Indonesia. Namun, kompatibilitasnya
Untuk mengubah perilaku petani yang demikian ini, dipertanyakan karena alat dari masing-masing negara
para penyuluh pertanian memanfaatkan ... dari tokoh berbeda standar dan format.
masyarakat yang disegani di daerah tersebut. Pengertian kompatibilitas dalam teks ter-sebut adalah ...
Pilihan kata yang tepat untuk mengisi bagian rumpang A. Akurat
dari kalimat tersebut adalah ... B. cara kerja alat
A. primitif - alternasi – legalisasi C. kesesuaian
B. primitif - alternatif – legalitas D. harmonis
C. konservatif - alternasi –legalisasi E. reaksi dengan alam
D. konservasi - alternatif – legalitas
E. konservatif - alternatif – legalitas 26. Peredaran sejumlah produk makanan yang mengandung
formalin di wilayah DKI Jakarta, Banten, Bogor, dan
23. Virus polio tergolong penyakit yang ditakuti. Bekasi ternyata kian marak. Padahal, penggunaan
Sebagai bagian dari Pixcomavirus, virus polio formalin sebagai pengawet makanan sangat berbahaya
merupakan mikroorganisme yang bisa melumpuhkan bagi kesehatan manusia. Pada umumnya, formalin
tubuh. Virus polio itu tergolong virus purba. Pada relief digunakan untuk mengawetkan mayat. Oleh karena itu,
Mesir Kuno yang dipahat ribuan tahun sebelum Masehi, masyarakat diminta untuk tidak mengonsumsi produk
tertulis ada raja yang kakinya kecil sebelah. Kalangan pangan berformalin. Produk-produk makanan yang
arkeolog mengidentifikasinya sebagai korban polio.
mengandung formalin tersebut tidak hanya dijual di
Dalam paragraf tersebut terdapat kalimat yang letaknya
pasar-pasar tradisional, tetapi di pasar serba ada pun
dapat dihilangkan tanpa mempengaruhi isi paragraf.
tidak dijamin produk makanan sejenis bebas formalin.
Kalimat yang dimaksudkan adalah kalimat ....
A. Pertama D. keempat Sejumlah produk pangan, seperti ikan asin, mi basah,
B. Kedua E. kelima dan tahu, dipastikan memakai formalin sebagai
C. Ketiga pengawet.
Kalimat yang mengganggu kesatuan paragraf sehingga
24. Kehilangan cairan tubuh akan, sangat berpengaruh harus dihilangkan adalah kalimat ...
terhadap kesehatan manusia. Kehilangan cairan tubuh A. Kedua
sebesar 3 persen akan menyebabkan turunnya produksi B. Ketiga
air liur dan urine. Kehilangan cairan tubuh sebesar 5 C. Keempat
persen akan menyebabkan prestasi kerja menurun dan D. Kelima
denyut nadi serta suhu badan meningkat. ... Kehilangan E. Keenam
cairan tubuh sebesar 20 persen akan menyebabkan
kematian. 27. Walaupun direbut dengan perjuangan bersenjata dan
Kalimat yang tepat untuk mengisi titik-titik dalam teks menelan ribuan jiwa kermerdekaan ternyata tidak
tersebut adalah ... seindah yang dibayangkan rakyat Zimbabwe. Sejak
A. Kehilangan cairan tubuh sebesar 10-12 persen akan negara ini merdeka pada tahun 1980, kehidupan sehari-
mengakibatkan situasi sangat kritis: berkurangnya hari rakyat di sana belum bertambah baik.
kemampuan mental, menelan menjadi sulit, dan Perekonomian nasional terseok-seok dan tidak mampu
kerusakan fungsi badan. menampung pertumbuhan angkatan kerja. Diperkirakan
B. Kehilangan cairan tubuh sebesar 10-12 persen akan bahwa lebih dari separuh angkatan kerja di negara ini
mengakibatkan situasi sangat kritis: kemampuan menganggur. Tidak mengherankan jika dari sekitar 13
mental berkurang, menelan menjadi sulit, dan juta penduduk Zimbabwe saat ini lebih kurang satu juta
fungsi badan rusak. orang bekerja di luar negeri. Sebenarnya, Zimbabwe
C. Kehilangan cairan tubuh sebesar 10-12 persen akan bukanlah negara miskin yang kering kerontang. Negara
mengakibatkan situasi sangat kritis: kemampuan ini kaya bahan tambang seperti emas, platinum, dan
mental berkurang, kesulitan menelan, dan fungsi batu bara. Sektor pertanian dan industri pengolahannya
badan rusak. termasuk paling maju di Afrika. Selain pangan,
D. Kehilangan cairan tubuh sebesar 10-12 persen akan Zimbabwe juga mengekspor kapas, cokelat, dan
mengakibatkan situasi sangat kritis: berkurangnya tembakau. Bahkan, Zimbabwe sekarang disebut sebagai
kemampuan mental, kesulitan menelan, dan fungsi salah satu negara produsen tembakau terbesar di dunia.
badan rusak. Akan tetapi, sekitar empat ribu keluarga kulit putih
menguasai lebih dari 80 persen lahan subur di negara
ini. Mereka juga menguasai 60 persen industri
manufaktur. Kemerdekaan tidak segera
mentransformasikan kesenjangan yang sudah
berlangsung lebih dari satu abad ini sehingga jutaan
rakyat tetap berjubel di atas lahan sempit dan kering.
Kehidupan sebagian besar rakyat Zimbabwe belum
bertambah baik karena ....
A. pertumbuhan ekonominya tidak sebanding dengan
pertumbuhan angkatan kerja
B. banyak tenaga kerja Zimbabwe yang bekerja di luar
negeri
C. hasil produksi tambang dan pertanian Zimbabwe
banyak yang diekspor
D. perekonomian Zimbabwe masih dikuasai warga
minoritas kulit putih
E. kebijakan sektor pertanian tidak ber-orientasi pada
kebutuhan dalam negeri

28. Sirine sebagai bagian dari sistem peringatan dini


tsunami dijalankan dengan menggunakan teknologi
komunikasi satelit dan GSM. Begitu ada gempa, sensor
yang dipasang pada sirine akan memberikan data secara
cepat ke stasiun BMG. Berdasarkan data ini, dilakukan
analisis untuk mengetahui terjadinya tsunami atau tidak
dalam waktu 10 menit. Bila ada potensi akan terjadi
tsunami, sirine langsung dibunyikan dalam bentuk nada
tertentu, disertai pengumuman kepada penduduk
setempat agar mereka segera mempersiapkan diri dan
mengambil tindakan preventif.
Topik bacaan di atas adalah ....
A. proses penanganan tsunami secara dini
B. proses kerja sistem peringatan dini tsunami
C. proses penanggulangan terjadinya tsunami
D. proses kerja sirine sebagai sistem peringatan dini
tsunami
E. proses penanganan penduduk yang tinggal di
daerah rawan tsunami

29. Membuat produk dalam jumlah terbatas sesungguhnya


adalah trik lama dalam praktik pemasaran. Hal ini
mengacu pada hukum ekonomi, yakni permintaan
versus pemasaran. Dengan peredaran barang terbatas
dan melebihi permintaan, harga akan membumbung dan
akan diburu orang. Trik ini banyak dijumpai pada
produsen yang membuat arloji dan pena supermewah.
Trik ini sering berhasil dan manjur karena konsumen
ingin memiliki sebuah benda yang eksklusif.
Dalam paragraf tersebut terdapat kalimat yang isinya
berupa contoh bagi informasi dalam keseluruhan
paragraf. Kalimat yang dimaksudkan adalah kalimat ...
A. Pertama
B. Kedua
C. Ketiga
D. Keempat
E. kelima
4. Udara yang kotor karena debu ataupun asap sisa
pembakaran menyebabkan kadar oksigen berkurang
sehingga sangat membahayakan kelangsungan hidup
setiap organism.
Kalimat di atas merupakan perluasan dari kalimat
dasar…
A. Udara kotor berkurang.
B. Udara kotor menyebabkan oksigen berkurang.
C. Asap menyebabkan oksigen.
D. Udara kotor membahayakan.
E. Asap sisa pembakaran membahayakan

5. Karena ekploitasi yang terus-menerus berlangsung dan


tidak diimbagi dengan penanaman kembali
menyebabkan kawasan hutan menjadi rusak.
Kalimat di atas menjadi kalimat baku apabila diperbaiki
dengan cara ...
A. Menghilangkan kata karena.
1. (1) Pada setiap adanya pembaruan kurikulum, pihak B. Menghilangkan kata yang.
dinas pendidikan dituntut untuk secara cepat melakukan C. Menambahkan tanda koma (,) setelah kata
pen yesuaian terhadap para pelaksana di lapangan. (2) kembali.
Akan tetapi, kenyataan berbicara lain yakni umumnya D. Menghilangkan kata terus-menerus.
penanganan pengembangan tenaga pelaksana E. Mengubah kata tidak dengan tanpa.
dilapangan berjalan sangat lambat.
Ejaan pada kalimat (1) dan kalimat (2) di atas menjadi 6. Abrasi merupakan proses fisik yang menyebabkan
benar jika diperbaiki dengan cara ... degradasi lahan kawasan pesisir. Abrasi dipengaruhi
A. Menghilangkan tanda koma (,) setelah kata oleh tenaga gelombang air laut. Gelombang pasang air
kurikulum (kalimat 1). laut yang menghantam daratan akan menggerus bibir
B. Menulis kata dinas pendidikan (kalimat 1) dengan pantai, menghanyutkan material pantai, dan
huruf kapital. mengakibatkan karusakan lahan dan segala sesuatu di
C. Mengubah kata perihal (kalimat 1) menjadi hal. atasnya.
D. Menghilangkan tanda koma (,) setelah kata akan Ide pokok paragraf di atas adalah ...
tetapi (kalimat 2). A. Pengertian abrasi.
E. Menambahkan tanda koma (,) setelah kata lain B. faktor penyebab terjadinya abrasi.
(kalimat 2). C. Tahap-tahap terjadinya abrasi.
D. Pemicu terjadinya abrasi.
2. (1) Di mana masyarakat bermukim, di tempat itu pasti E. Proses penggerusan pantai dan abrasi.
akan terjadi dinamika sosial. (2) Dalam hal ini, sekecil
apa pun, perubahan pola hidup masyarakat juga akan 7. Sejak dulu hingga kini, ilmu bahasa mengalami
terjadi di tempat pemukiman itu dan berdampak pada perkembangan yang Sangat pesat. Perkembangan itu
perubahan bidang lainnya. (3) Sebagai contoh, terjadi berkat berbagai temuan temuan baru dalam
perubahan gaya pakaian masyarakat akan menghasilkan bidang tersebut. Kajian-kajian baru muncul karena
perubahan pada ekonomi masyarakat. ketidakpuasan terhadap hasil kajian sebelumnya yang
Dalam paragraf di atas, terdapat bentuk kata yang tidak menunjukkan ketidaktuntasan dalam menyusun
sesuai dengan konteks kalimatnya, yakni ... deskripsi bahasa. Di samping itu, perkembangan ilmu
A. Kata bermukim (kalimat 1). bahasa ini disebabkan Juga oleh adanya perbedaan
B. Kata pemukiman (kalimat 2). sudut pandang yang digunakan oIeh para ahli dalam
C. Kata berdampak (kalimat 2). meneliti bahasa. Akhirnya, perkembangan ini
D. Kata perubahan (kalimat 3). berpengaruh pada praktik pengajaran bahasa.
E. Kata pakaian (kalimat 3). Pernyataan berikut ini yang secara tersirat memiliki
kesamaan dengan isi paragraf di atas adalah ...
3. Dalam satu wilayah yang dihuni berbagai etnik, terjadi A. Pembeiajaran bahasa Indonesia mengalami
integrasi budaya sehingga menghasilkan budaya baru. perkembangan dari waktu ke waktu.
Makna istilah integrasi dalam kalimat di atas adalah ... B. Perkembangan ilmu bahasa disebabkan oleh
A. Percampuran. praktik pengajaran bahasa di sekolah.
B. Penyatuan. C. Hasil kajian bahasa selalu tidak memuaskan para
C. Pembauran. pemakai bahasa sehingga terus-menerus dilakukan
D. Pembentukan. penelitian.
E. Penyesuaian.
D. Sampai saat ini belum ada deskripsi bahasa yang C. Ketakutan masyarakat terhadap ungkapan di bidang
lengkap dan utuh yang didasarkan pada hasil kapan politik.
bahasa. D. Ketidaksepahaman daiam memilih ideologi politik.
E. Pengajaran bahasa tidak akan mengalami E. Tujuan ungkapan di bidang politik.
perkembangan jika tidak ada kajian bahasa.
12. Banyak ungkapan dalam dunia politik. Ungkapan itu
8. Setidaknya terdapat tiga hal yang perlu dilakukan dalam dipergunakan untuk menamai atau memberi label. Salah
menghadapi bencana alam. Pertama, pencegahan, yakni satu ungkapan yang masih sangat dikenal umum adalah
membuat peta potensi bencana seperti dalam atlas dari politik sebagai panglima. Ungkapan ini memliki
bakosurtanal. Yang kedua adalah mitigasi atau konotasi yang negatif. Bahkan, ungkapan tersebut
penjinakan. Jika suatu daerah termasuk rawan gempa, memiliki potensi menakutkan masyarakat. Konon,
berarti rumah yang dibangun di daerah itu harus tahan dahulu ungkapan itu mengand-ung ancaman, khususnya
gempa. Ketiga adalah peringatan dini (kesiapsiagaan). bagi pihak yang tidak sepaham dalam ideologi politik.
Hal ini perlu dilatihkan kepada masyarakat melalui Pernyataan berikut yang sesuai dengan isi bacaan di
sosialisasi dan simulasi berkelanjutan. atas adalah...
Masalah utama paragraf di atas adalah ... A. Ungkapan politik sebagai panglima berkembang
A. Aktivitas pencegahan bencana alam. pada masa silam.
B. Pentingnya rumah tahan bencana. B. Setiap politik memiliki label yang berbeda-beda.
C. Pemetaan potensi bencana alam. C. Ungkapan dalam dunia politik sering membuat
D. Perlunya sosialisasi daerah rawan bencana. masyarakat takut.
E. Kegiatan siaga dalam menghadapi bencana. D. Setiap insan memperoleh ancaman dalam masalah
politik.
9. Hutan merupakan penopang kelestarian kehidupan di
E. Ketidaksepahaman ideologi politik terjadi di
bumi karena tidak hanya menyediakan bahan pangan
masyarakat.
ataupun bahan produksi, melainkan juga menjadi
penghasil oksigen, pebahan lapisan tanah, dan penyim-
13. Lebih dari setengah dasawarsa terakhir ini, bumi
pan cadangan air.
pertiwi mengalami musibah bencana besar secara
Kalimat di atas menjadi kalimat baku apabila diperbaiki
beruntun. Faktor alam dan sejarah kebencanaan
dengan cara ...
menunjukkan bahwa Indonesia memang merupakan
A. Menambahkan tanda koma (,) sebelum kata karena.
daerah rawan bencana. Terakhir, terjadi bencana
B. Menulis kembali kata hutan setelah kata karena.
banjir di Jakarta dan sebagian wilayah lain Indonesia
C. Mengubah kata melainkan menjadi dan.
serta letusan gunung (Sinabung dan Kelud). Realitas ini
D. Mengubah kata melainkan menjadi tetapi.
menyebabkan penduduk Indonesia harus siap siaga
E. Mengubah kata penyimpanan menjadi menyimpan.
menghadapi bencana.
Simpulan yang tepat untuk paragraf di atas adalah ...
10. Media masa dalam perkembangannya telah
A. Faktor alam Indonesia menyebabkan bencana.
bertransformasi menjadi sebuah indrustri bisnis yang
B. Bumi pertiwi mengalami bencana.
menjanjikan karena kebutuhan masyarakat akan
informasi dan hiburan semakin bertambah. C. Musibah bencana beruntun akhir-akhir ini.
Makna istilah bertransformasi dalam kalimat di atas D. Indonesia merupakan daerah rawan bencana.
adalah ... E. Masyarakat harus siaga menghadapi bencana.
A. Berubah bentuk.
B. Berpindah haluan. 14. Penanganan masalah pendidikan diantaranya ditempuh
C. Berbeda tujuan. dengan membangun SD kecil untuk melayani
D. Berpindah sasaran. kebutuhan pendidikan si daerah terpencil yang
E. Berubah pemberitaan dilakukan pada pelita V, di samping SD regular di
wilayah-wilayah yang padat penduduk.
11. Banyak ungkapan dalam dunia politik. Ungkapan itu Ejaan pada kalimat di atas menjadi benar jika diperbaiki
dipergunakan untuk menamai atau memberi label. Salah dengan cara ...
satu ungkapan yang masih sangat dikenal umum adalah A. Menulis kata diantaranya menjadi di antaranya.
politik sebagai panglima. Ungkapan ini memiliki B. Menulis kata pelita V menjadi PELITA V.
konotasi yang negatif. Bahkan, ungkapan tersebut C. Menghilangkan tanda koma (,) setelah kata pelita
memiliki patensi menakutkan rnasyarakat. Konon, V.
dahulu ungkapan itu men gandung ancaman, khususnya D. Menulis kata di samping menjadi disamping.
bagi pi-hak yang tidak sepaham dalam ideology politik. E. Menulis kata reguler menjadi regular.
Ide pokok bacaan di atas adalah ...
A. Politik sebagai panglima. 15. Domestikasi hewan diduga telah dilakukan manusia
B. Ungkapan dalam bidang politik. pada saat mereka belurn mengenal budidaya dan
merupakan kegiatan pemeliharaan serta
pembudidayaan hewan yang pertama kali. Makna A. Ujian Nasional sebaiknya mendorong
istilah domestikasi dalam kalimat di atas adalah ... perkembangan kemampuan siswa.
A. Proses mengadopsi hewan liar ke dalam kehidupan B. Ujian Nasional harus dilakukan dengan prinsip
manusia. kejujuran dan efisiensi.
B. Proses menjinakkan hewan liar untuk kepentingan C. Pemerintah sebaiknya melaksanakan moratorium
hidup manusia. Ujian Nasional.
C. Proses menyeleksi dan memperbaiki keturunan D. Pemerintah harus menghapus Ujian Nasional.
hewan liar. E. Kredibilitas Ujian Nasional perlu diperhatikan.
D. Proses perubahan perilaku dari organism hewan
liar. Bacalah teks berikut untuk menjawab soal nomor 19 s.d.
E. Proses memelihara dan membudidayakan hewan. 21!
(1) Salah satu alasan para orang tua tidak memberikan
Bacalah teks berikut untuk menjawab soal nomor 16 s.d. pendidikan prasekolah bagi anaknya adalah karena tidak
18! ingin anaknya kehilangan masa kecil. (2) Telah umum
(1) Pemerintah sebaiknya meninjau kebijakan Ujian dikatakan bahwa masa kecil adalah masa permainan. (3)
Nasional. (2) Ujian Nasional yang memiliki standar soal dan Padahal, pendidikan prasekolah anak bukanlah pendidikan
penilaian sama untuk semua siswa di mana pun memiliki yang "menyeramkan" seperti halnya sekolah yang menuntut
lebih banyak mudarat daripada manfaatnya sehingga timbul anak untuk ini dan itu. (4) Pendidikan anak usia dini tetap
hal-hal yang justru tidak sesuai dengan tujuan pendidikan, memberikan kebebasan kepada anak untuk bermain dalam
seperti menanamkan nilai-nilai kejujuran pada siswa. (3) konteks bermain sambil belajar. (5) Lebih daripada itu, pen-
Kemendikbud menyebutkan sejumlah alasan mengapa Ujian didikan anak usia dini penting karena dapat membentuk
Nasional perlu dimoratorium. (4) Salah satu alasannya kesiapan diri anak dalam menghadapi masa sekolah. (6)
adalah Ujian Nasional kurang mendorong berkem-bangnya Kecerdasan kognitif, afektif, dan psikomotor anak akan
kemampuan siswa secara utuh. (5) Alasan lain adalah terbentuk dengan baik lewat pendidikan ini.
sulitnya memperoleh Ujian Nasional yang kredibel dan
bebas kecurangan karena cakupan ujian yang terlalu luas. 19. Kalimat yang menggunakan kata nonformal dalam
(6) Selain itu, untuk menggelar Ujian Nasional, dibutuhkan paragraf di atas adalah kalimat ....
sumber daya yang sangat besar. A. (1)
(Diadaptasi dari http://koran.tempo.co/konten/2016/11/29/ B. (3)
tak-perlu-ujian-nasional) C. (4)
D. (5)
16. Apakah inti kalimat (2)? E. (6)
A. Ujian Nasional memiliki lebih banyak mudarat
daripada manfaatnya. 20. Kata yang tidak tepat dalam teks di atas adalah....
B. Ujian Nasional seharusnya menanamkan nilai-nilai A. memberikan dalam kalimat (1)
kejujuran pada siswa. B. permainan dalam kalimat (2)
C. Ujian Nasional memiliki standar soal dan penilaian C. menyeramkan dalam kalimat (3)
yang sama untuk semua siswa. D. kesiapan dalam kalimat (5)
D. Ujian Nasional menimbulkan hal-hal yang tidak E. terbentuk dalam kalimat (6)
sesuai dengan tujuan pendidikan.
E. Ujian Nasional memiliki standar soal yang sama 21. Kalimat manakah yang seharusnya mengakhiri paragraf
untuk semua siswa. di atas?
A. Oleh karena itu, orang tua harus memperhatikan
17.Pernyataan mana yang tidak sesuai dengan isi teks kompetensi anak usia dini.
tersebut? B. Jadi, pendidikan anak usia dini telah memberikan
A. Ujian Nasional telah melenceng dari tujuan kompetensi yang lengkap dalam mempersiapkan
pelaksanaan yang sebenarnya. masa sekolah.
B. Pemerintah berencana memoratorium karena C. Untuk itu, seorang anak dapat mengembangkan
banyaknya hal negatif dalam Ujian Nasional. potensi secara mandiri di sekolah dengan bantuan
C. Kemendikbud merencanakan moratorium Ujian guru.
Nasional karena besarnya sumber daya. D. Dengan demikian, anak perlu didaftarkan pada
D. Pelaksanaan Ujian Nasional selama ini belum program pendidikan usia dini yang sesuai.
bebas dari praktik ketidakjujuran. E. Berdasarkan hal itu, sebelum memasuki masa usia
E. Ujian Nasional tidak jadi dimoratorium karena sekolah dasar, seorang anak perlu memperoleh
tidak disetujui oleh berbagai pihak. pendidikan usia dini.

18.Apa pesan tersirat di balik teks yang ingin disampaikan Bacalah teks berikut untuk menjawab soal nomor 22 s.d.
penulis? 26!
(1) Flu singapura pada dasarnya adalah flu yang menyerang B. Penularan virus flu singapura melalui kontak
kekebalan tubuh, terutama anak-anak, yang dilakukan oleh langsung.
virus RNA dari jenis Reterovirus. (2) Virus ini pertama kali C. Singapura menjadi tempat penyebaran flu.
ditemukan di Singapura dan itu pula yang kemudian D. Flu singapura menyerang kulit dan teng-gorokan.
menyebabkan flu ini dinamakan flu singapura. (3) Dalam E. Flu singapura bisa ditularkan melalui udara.
bahasa medis internasional flu ini dinamakan HMFD atau
Hand Mouth and Foot Dis-ease. (4) Penamaan ini berdasar Bacalah teks berikut untuk menjawab soal nomor 27 s.d.
pusat serangan yang terfokus pada luka di kaki, tangan, dan 30!
mu-lut. (1) Globalisasi menjadi tantangan untuk semua aspek
(5) Bila kebanyakan virus flu menyerang area pernafasan kehidupan termasuk kebudayaan. (2) Era global menuntut
dan area lendir, maka serangan virus ini cenderung lebih kesiapan kita untuk siap berubah menyesuaikan perubahan
muncul pada kulit. (6) Kadang gejala pada kulit akan zaman dan mampu mengambil setiap kesempatan. (3)
muncul pertama kali dari gejala demam dan tanda-tanda flu Budaya tradisional di Indonesia sebenarnya lebih kreatif dan
lain, seperti munculnya sariawan berat dan peradangan di tidak bersifat meniru, namun yang menjadi masalah adalah
area tenggorokan depan. (7) Proses penularannya cukup bagaimana mempertahankan jati diri bangsa. (4) Sebagai
mudah, tetapi kebanyakan penularan berawal dan interaksi contoh sederhana, budaya gotong royong di Indonesia saat
langsung kulit dengan media yang sudah terpapar virus ini hampir terkikis habis, yang digantikan oleh sikap
RNA. (8) Penularan itu bisa melalui proses sentuhan kulit individual dan tidak peduli kepada orang lain. (5) Perlu
dengan pengidap flu singapura atau tersentuh benda yang dipikirkan agar kebudayaan kita tetap dapat mencerminkan
sudah terkontaminasi flu singapura. (9) Aktivitas makan dan kepribadian bangsa.
minum bercampur dengan pengidap flu singapura juga bisa (6) Dalam era globalisasi, kebudayaan tradisional mulai
menyebabkan penularan. mengalami erosi. (7) Semua orang, terutama anak muda,
(Diadaptasi dari http://deherba.com/apa-obat-herbal-flu- lebih senang menghabiskan waktunya mengakses internet
singapura-terbaik.html/) daripada mempelajari tarian dari kebudayaan sendiri. (8)
Orang akan merasa bangga ketika dapat meniru gaya
22. Kata yang penulisannya tidak mengikuti kaidah ejaan berpakaian orang Barat dan menganggap budayanya
terdapat pada kalimat .... ketinggalan zaman. (9) Globalisasi akan selalu memberikan
A. (1) perubahan. (10) Oleh karena itu, harus meneliti apakah
B. (2) berbagai budaya yang masuk tersebut bersifat positif atau
C. (4) negatif.
D. (5) (Diadaptasi dari
E. (9) http://www.budayatradisionalindonesia.blogspot.co.id/)

23. Penggunaan konjungsi yang tidak tepat terdapat dalam 27. Kalimat yang tidak efektif dalam teks di atas adalah
kalimat .... kalimat ....
A. (1) A. (1)
B. (2) B. (2)
C. (5) C. (5)
D. (6) D. (8)
E. (7) E. (10)

24. Kata terkontaminasi pada kalimat (8) bermakna .... 28. Kalimat manakah yang merupakan simpulan dan isi
A. tersentuh teks di atas?
B. terkotori A. Budaya tradisional Indonesia orisinal dan kreatif.
C. terkena B. Kebudayaan tradisional perlu dipertahankan.
D. terpengaruh C. Globalisasi menggerus kebudayaan tradisional.
E. tercampuri D. Budaya gotong royong di Indonesia terkikis habis.
E. Generasi muda mengidolakan budaya Barat.
25. Ide pokok paragraf kedua adalah ....
A. penularan flu singapura 29. Gagasan pokok sebelum kedua paragraf di atas
B. media penularan flu singapura adalah…
C. gejala flu singapura A. unsur kebudayaan
D. penyebab flu singapura B. konsep globalisasi
E. akibat flu singapura C. keragaman budaya
D. kebudayaan tradisional
26. Pernyataan manakah yang sesuai dengan isi teks di E. dampak globalisasi
atas?
A. Orang dewasa cenderung tidak terserang flu 30. Mengapa kebudayaan tradisional mengalami erosi di
singapura. era globalisasi?
A. Globalisasi mengakibatkan perubahan kebudayaan.
B. Kebudayaan asing lebih baik daripada kebudayaan 2. Pada kalimat nomor berapa terdapat kesalahan
tradisional. penggunaan tanda baca koma?
C. Kebudayaan tradisional dinilai ketinggalan zaman. A. 1
D. Kebudayaan tradisional tidak disukai generasi B. 3
muda. C. 6
E. Globalisasi dianggap mampu menjawab tantangan D. 7
zaman E. 9

3. Pernyataan mana yang TIDAK sesuai dengan isi teks?


A. Permainan tradisional sangat baik untuk kesehatan
anak.
B. Permainan tradisional perlu mendapat perhatian
dari orang tua.
C. Permainan tradisional tidak mudah dimainkan di
kota-kota besar.
D. Permainan tradisional sangat bermanfaat untuk
kegiatan sosial anak.
E. Permainan tradisional tidak dipahami dengan baik
oleh orang tua.

4. Apa yang dirujuk oleh kata itu pada kalimat (9)?


A. Kendala orang tua
B. Kendala budaya
C. Kendala arena
D. Kendala alat
Teks 1 digunakan untuk menjawab soal nomor 01 sampai E. Kendala anak
dengan 05!
Teks 1 5. Apa kelemahan yang ada di dalam paragraf pertama?
(1) Permainan tradisional dilakukan dengan banyak gerakan A. Penggunaan kata dia pada kalimat (2)
oleh anak-anak, misalnya permainan kasti, gasing, dan B. Penggunaan kata mereka pada kalimat (3)
kelereng. (2) Dengan demikian, dia akan terhindar dan C. Penggunaan kata juga pada kalimat (4)
obesitas, (3) Sosialisasi dan komunikasi mereka tercapai, D. Penggunaan kata akan pada kalimat (5)
karena dalam permainan tradisional paling sedikit E. Penggunaan kata bagi pada kalimat (6)
dimainkan oleh dua anak. (4) Permainan tradisional juga
dapat menentukan strategi dalam bermain. (5) Mereka juga Teks digunakan untuk menjawab soal nomor 06 sampai
akan bekerja sama dengan anggota tim. (6) Memang, dengan 10.
permainan tradisional bagi anak sangat baik karena banyak Teks 2
nilai positifnya. (1) Sebuah studi menunjukkan bahwa anak yang dibiasakan
(7) Permainan tradisional saat ini tidak mudah dilakukan, mendengarkan cerita sejak dini dan dikenalkan dengan
terutama di kota-kota besar. (8) Permainan tradisional pada kebiasaan membaca memiliki perkembangan jaringan otak
umumnya memerlukan arena luas, umpamanya bermain yang lebih awal. (2) Sebaliknya, anak yang tidak dikenalkan
kelereng, bermain gasing, petak umpet, dan lain-lain. (9) dengan kebiasaan membaca memiliki perkembangan yang
Selain itu, banyak orang tua melarang anaknya bermain kurang pada jaringan tersebut. (3) Anak-anak balita dengan
permainan tradisional karena takut kotor atau takut kulit orang tua yang rutin membacakan buku untuk mereka
anaknya terbakar panas matahari. (10) Orang tua banyak mengalami perbedaan perilaku dan prestasi akademik
memberikan kepada anaknya permainan elektronik, dengan anak-anak dengan orang tua yang cenderung pasif
misalnya video game dan mobil-mobilan. (11) Permainan- dalam membacakan buku. (4) Menurut sebuah studi baru
permainan tersebut dimainkan di dalam rumah saja. (12) yang diterbitkan dalam jurnal Pediatrics menemukan
Akibatnya, anak kurang bersosialisasi dengan temannya dan perbedaan yang juga terjadi pada aktivitas otak anak.
kurang bergerak. (5) Peneliti mengamati perubahan aktivitas otak anak-anak
usia 3 sampai dengan 5 tahun yang mendengarkan orang tua
1. Apa gagasan utama paragraf ke-2? mereka membacakan buku melalui scanner otak yang
A. Permainan tradisional yang penting bagi anak disebut functional magnetic resonance imaging (FMRI). (6)
B. Tempat memainkan permainan tradisional Orang tua menjawab pertanyaan tentang berapa banyak
C. Penyebab punahnya permainan tradisional mereka membacakan cerita untuk anak-anak serta seberapa
D. Kendala memainkan permainan tradisional sering melakukan komunikasi. (7) Para peneliti melihat
E. Jenis-jenis permainan tradisional di kota bahwa ketika anak-anak sedang mendengarkan orang tua
bercerita, sejumlah daerah di bagian kiri otak menjadi lebih
aktif. (8) Ini adalah daerah yang terlibat dalam memahami C. Faktor-faktor yang dapat mendorong anak gemar
arti kata, konsep, dan memori. (9) Wilayah otak ini juga membaca
menjadi aktif ketika anak-anak bercerita atau membaca. (10) D. Manfaat kebiasaan membaca dan mendengarkan
Pada studi ini menunjukkan bahwa perkembangan daerah ini cerita pada anak
dimulai pada usia yang sangat muda. (11) Yang lebih E. Kebiasaan membaca dan mendengarkan cerita
menarik adalah bagaimana aktivitas otak di wilayah ini lebih pada anak
sibuk pada anak-anak yang orang tuanya gemar membaca.
(12) Membacakan buku untuk anak membantu pertumbuhan Teks 3A dan 3B digunakan untuk menjawab soal nomor 11
neuron di daerah ini yang akan menguntungkan anak di sampai dengan 15.
masa depan dalam hal kebiasaan membaca. Teks 3A
(Diadaptasi dari http:health.kompas.com/ (1) Bioteknologi merupakan teknologi dengan pemanfaatan
read/2016/02/07/135500623/ mikroorganisme, tanaman, atau hewan melalui modifikasi
membacakan.Buku.Meningkatkan.Kinerja.Otak.Balita) proses seluler untuk menghasilkan produk yang bermanfaat.
(2) Banyak negara, khususnya negara-negara maju,
6. Apa judul yang tepat untuk Teks 2 tersebut? menjadikan bioteknologi sebagai penahaaan terdepan
A. Pengenalan Kebiasaan Membaca sejak Dini ketahanan pangan. (3) Penel itian bioteknologi mencakup
B. Balita dan Kebiasaan Mendengarkan Cerita berbagai bidang, yaitu pertanian, peternakan,
C. Pembiasaan Anak dalam Mendengarkan Cerita farmakoseutika, kimia, pemrosesan makanan, dan
D. Peningkatan Kinerja Otak melalui Membaca fermentasi. (4) Di Indonesia sumber daya manusia yang
E. Peran Orang Tua dalam Membacakan Cerita berkompetensi di bidang bioteknologi masih sedikit dan
terbatas. (5) Padahal, perkembangan bioteknologi global dan
7. Kalimat manakah yang TIDAK efektif dalam Teks 2? bisnis yang terkait sangat menjanjikan. (6) Tenaga terampil
A. 1 dan 7 dan ahli yang kompeten
B. 2 dan 8 di bidang bioteknologi sangat diperlukan. (7) Pendidikan
C. 3 dan 9 dan pengembangan SDM di bidang bioteknologi harus
D. 4 dan 10 mendapat prioritas dan dukungan, baik dari pemerintah,
E. 5 dan 12 universitas, lembaga penelitian, maupun perusahaan swasta
terkait. (8) Semua pihak harus mendukung agar SDM
8. Bagaimana hubungan isi antarparagraf dalam Teks 2? bidang bioteknologi semakin banyak.
A. Paragraf ke-2 memaparkan simpulan penelitian Teks 3B
yang dibahas pada paragraf ke -1. (9) Dewasa ini perkembangan bioteknologi tidak hanya
B. Paragraf ke-2 memerinci temuan penelitian yang didasari oleh biologi semata, tetapi juga oleh ilmu-ilmu
dipaparkan pada paragraf ke-1. terapan dan murni lain, seperti biokima, computer, biologi
C. Paragraf ke-2 memaparkan perbedaan perilaku molecular, mikrobiologi, genetika, kimia, matematika, dan
yang dibahas pada paragraf ke- 1. lain sebagainya. (10) Dengan kata lain, bioteknologi adalah
D. Paragraf ke-1 memaparkan penelitian terhadap ilmu terapan yang mengabungkan berbagai cabang ilmu
balita yang diuraikan pada paragraf ke-2. dalam proses produksi barang dan jasa. (11) Banyak negara
E. Paragraf ke-1 memaparkan hasil penelitian yang menjadikan bioteknologi sebagai pertahanan terdepan
diuraikan pada paragraf ke-2. ketahanan panganannya, khususnya di negara-negara maju.
(12) Akan telapi, tidak semua pihak dapat menerima
9. Apa simpulan teks tersebut? bioteknologi karena dianggap bertentangan dengan kodrat
A. Anak-anak yang belajar membaca pada usia 3-5 alam. (13) Bioteknologi memunculkan kontroversi,
tahun akan mempercepat perkembangan otaknya. misalnya bayi tabung, pengklonan manusia, dan
B. Semakin awal kebiasaan membaca buku transplantasi organ. (14) Kemajuan di bidang bioteknologi
diperkenalkan, semakin aktif otak anak bekerja. tidak terlepas dari berbagai kontroversi yang melingkupi
C. Anak-anak yang mulai belajar membaca sejak perkembangan teknologinya.
balita akan menjadi anak-anak yang otak kirinya (Diadapatasi dari beberapa sumber)
lebih aktif.
D. Kebiasaan mendengarkan cerita dan membaca 11. Apa makna kata kompeten pada kalimat (6) Teks 3A?
sejak usia balita berdampak positif untuk A. Ahli
perkembangan otak. B. Pintar
E. Otak kiri bertanggung jawab terhadap proses C. Pakar
pemahaman seperti kata dan konsep. D. Hebat
E. Cakap
10. Apa gagasan utama yang tepat untuk paragraf
selanjutnya dari teks tersebut? 12. Berdasarkan isi Teks 3A, kepada siapa penulis
A. Tujuan membiasakan anak membaca cerita sejak berpihak?
usia dini A. Tenaga terampil bidang bioteknologi
B. Strategi penerapan hasil penelitian untuk B. Peneliti bidang bioteknologi
pendidikan anak C. Lembaga penelitian bioteknologi
D. Pangusaha bidang bioteknologi pembelajaran adalah pembelajaran seumur hidup, bukan
E. Sumber daya manusia bioteknologi demi ujian semata. (9) Guru tidak perlu khawaiir jika siswa
lupa tanggal peristiwa penting dalam sejarah, karena mereka
13. Apa perbedaan tujuan penulisan Teks 3A dengan Teks dapat mencarinya melalui buku dan web. (10) Guru perlu
3B? mengajari mereka cara belajar yang baik dan mendorong
A. Teks 3A menjelaskan definisi bioteknologi; Teks mereka untuk gemar membaca dan menulis. (11) Jadi, yang
3B memaparkan ilmu-ilmu yang mendasari terpenting bukan hanya tentang apa yang diketahui ketika
bioteknologi. mereka lulus, melainkan juga untuk mencintai pembelajaran
B. Teks 3A menjelaskan pentingnya pengembangan seumur hidup.
SDM bioteknologi; Teks 3B menjelaskan (Di adaptasi dari http: //koran.tempo.co/konten)
penolakan pemanfaatan bioteknologi.
C. Teks 3A menguraikan peran bioteknologi di 16. Kalimat topik paragraf pertama adalah kalimat....
bidang pangan; Teks 3B memaparkan peran A. (1)
bioteknologi di bidang kesehatan manusia. B. (2)
D. Teks 3A memaparkan keunggulan bioteknologi: C. (3)
Teks 3B memaparkan kelemahan bioteknologi. D. (4)
E. Teks 3A menjelaskan kendala pengembangan E. (6)
SDM bioteknologi; Teks 3B menjelaskan
kemajuan bidang bioteknologi. 17. Kesalahan penggunaan tanda baca koma terdapat pada
kalimat ....
14. Informasi apa yang ada di dalam Teks 3B, tetapi A. (3)
TIDAK dimuat dalam Teks 3A? B. (5)
A. Definisi bioteknologi C. (7)
B. Pengembangan SDM bioteknologi
D. (8)
C. Penolakan terhadap bioteknologi
E. (11)
D. Cakupan bidang bioteknologi
18. Pertanyaan manakah yang jawabannya tidak ditemukan
E. Komitmen pemerintah di bidang bioteknologi
dalam teks tersebut?
A. Apa keuntungan teknologi digital bagi generasi
15. Apa kelemahan isi teks?
sekarang?
A. Teks 3A tidak memuat secara terperinci contoh
bidang bioteknologi. B. Mengapa generasi sekarang berbeda dengan
B. Teks 3B tidak memuat manfaat bioteknologi bagi generasi sebelumnya?
kehidupan. C. Di manakah generasi sekarang biasa
C. Teks 3B tidak menjelaskan latar tentang produksi menggunakan teknologi digital?
barang dan jasa. D. Bagaimana pendapat pakar mengenai model
D. Teks 3A tidak memuat alasan pentingnya pembelajaran yang ideal?
pengembangan SDM bioteknologi. E. Apa tujuan jangka panjang pembelajaran bagi
E. Teks 3B tidak memuat komitmen negara maju siswa?
mendukung bioteknologi.
19. Kata ganti mereka pada kalimat (5) merujuk kepada ....
Teks berikut digunakan untuk menjawab soal nomor 16- A. generasi acuh tak-acuh
20. B. generasi ini
(1) Generasi hari ini berbeda dengan generasi sebelumnya C. generasi
karena generasi hari ini lahir di tengah kecanggihan D. remaja
teknologi digital sehingga mereka dimanjakan game online E. siswa
dan me-dia sosial. (2) Sejatinya, smart phone mendukung
proses belajar-mengajar sehingga proses transfer of 20. Kelemahan paragraf kedua teks tersebut adalah ....
knowledge dan pembinaan karakter dan keterampilan A. data tidak dipaparkan dengan jelas dan
berjalan lancar. (3) Namun, kita juga sering menjumpai menyeluruh
remaja yang berada dalam sebuah forum tanpa B. hal yang harus dilakukan guru dalam mengajar
berkomunikasi satu dengan yang lain, karena asyik dengan tidak dikemukakan
dunianya sendiri. (4) Meminjam bahasa Don Tapscott C. pentingnya pembelajaran seumur hidup tidak di
(2013), generasi ini adalah generasi acuh tak-acuh. (5) minat jelaskan
mereka hanya mengenai budaya populer, para pesohor, dan
D. gaya belajar sequential, sensing, dan visual tidak
teman-teman mereka. (6) Hal itu menunjukkan bahwa
dijabarkan
teknologi digital membawa sejumlah dampak positif dan
E. manfaat web dalam pembelajaran tidak dijelaskan
negatif.
(7) Menurut Felder dan Soloman (1993), “Pembelajar di
zaman informasi ini memunyai kecenderungan gaya belajar Berikut digunakan untuk menjawab soal nomor 21 s.d.
aktif, sequential, sensing, dan visual.” (8) Fokus 30.
(1) Puncak kejayaan maritim nusantara terjadi pada masa B. Indonesia saat ini tidak sejaya Majapahit
Kerajaan Majapahit. (2) Majapahit berhasil menguasai dan C. kecintaan rakyat Indonesia pada laut makin rendah
mempersatukan nusantara. (3) Mempunyai pengaruh sampai D. negara tidak berpihak kepada kemaritiman
ke negara-negara asing, seperti Thailand, Kamboja, India, E. fokus pembangunan saat ini masih berada di darat
Filipina, dan China. (4) Kejayaan ini disebabkan karena
kemampuan membaca potensi yang dimiliki. (5) Ketajaman 22. Diksi yang tidak tepat terdapat pada kalimat....
visi dan kesadaran terhadap posisi strategis nusantara telah A. (15)
mem-bawa negara ini disegani oleh negara-negara lain. B. (17)
(6) Namun sayang kini kejayaan itu tidak lagi banyak C. (19)
dikenang. (7) Kejayaan tersebut seakan tertutup oleh potret D. (21)
kemiskinan yang melanda rakyat Indonesia. (8) Kecintaan E. (22)
kita pada laut juga makin dangkal. (9) Rasa keterpihakan
negara terhadap dunia maritim pun masih lemah. (10) 23. Kesalahan penggunaan tanda baca ditemukan pada
Meskipun Kementerian Kelautan dan Perikanan sudah kalimat ....
dibentuk, namun fokus pembangunan negara ini masih A. (1)
berfokus di sektor darat. (11) Masalah utamanya adalah B. (3)
masalah paradigma. (12) Paradigma darat/agraris masih kuat C. (4)
melekat pada kebanyakan masyarakat Indonesia. (13) D. (6)
Bangsa Indonesia masih mengidap kerancuan identitas. (14) E. (10)
Di satu pihak bangsa Indonesia memunyai persepsi
kewilayahan tanah air, tetapi di pihak lain memosisikan diri 24. Mengapa paradigma menjadi masalah utama
secara kultural sebagai bangsa agraris dengan puluhan juta pembangunan Indonesia?
petani miskin yang tidak sanggup kita sejahterakan, A. Indonesia belum dapat mengembalikan kejayaan
sedangkan kegiatan industri modern sulit berkompetisi seperti zaman Majapahit.
dengan bangsa lain. B. Paradigma agraris masih kuat melekat pada
(15) Akibat dari hal ini adalah pembangunan perekonomian kebanyakan masyarakat Indonesia.
maritim dan pembangunan sumber daya manusia tidak C. Indonesia merupakan negara maritim, tetapi
pernah dijadikan arus utama pembangunan nasional, yang masyarakatnya berpikir agraris.
didominasi oleh persepsi dan kepentingan daratan semata. D. Kejayaan Indonesia masih tertutup oleh potret
(16) Dari paparan tersebut, kita mendapati kenyataan bahwa kemiskinan rakyat Indonesia.
Indonesia sebagai negara kepulauan terbesar di dunia belum E. Kecintaan pada laut dan keberpihakan pada
mampu member-dayakan potensi ekonomi kelautan. (17) maritim masih lemah.
Negeri ini belum mampu mentransformasikan sumber
kekayaan laut menjadi sumber kemajuan dan kemakmuran 25. Kalimat yang tidak efektif adalah kalimat....
bangsa. (18) Hal ini ditunjukkan pada tabel berikut. A. (3) dan (4)
Tabel Produk Perikanan Indonesia B. (5) dan (6)
C. (7) dan (8)
D. (9) dan (10)
E. (13) dan (14)

26. Apa simpulan isi teks tersebut?


A. Indonesia sebenarnya memiliki potensi maritim
yang besar.
B. Keuntungan sebagai negara maritim tidak di
manfaatkan.
C. Indonesia merupakan negara maritim yang salah
urus.
(19) Agar dapat menjadi bangsa yang kuat dan disegani di D. Indonesia belum mampu memberdayakan potensi
mata internasional, maka Indonesia harus kembali kelautan.
berwawasan maritim. (20) Permasalahannya adalah apakah E. bangsa Indonesia mengidap kerancuan identitas.
masih bisa membangkitkan kembali kejayaan masa lalu
tersebut di tengah krisis multidimensi yang belum usai 27. Tujuan penulisan teks tersebut adalah ....
menerpa bangsa kita. (21) Mengembalikan visi kemaritiman A. memaparkan bahwa Indonesia pernah disegani
bukan sesuatu hal yang mudah. (22) Selain dibutuhkan bangsa lain melalui kejayaan Majapahit
kemauan yang tinggi untuk merombak sistem yang ada, B. menggambarkan perkembangan maritim Indonesia
masalah penyediaan infrastruktur menjadi masalah sejak Majapahit hingga kini
tersendiri. C. menunjukkan bahwa Indonesia dahulu merupakan
negara maritim yang kuat
21. Gagasan utama paragraf kedua teks tersebut adalah .... D. membuktikan adanya potensi kekayaan laut
A. kini kejayaan itu tak lagi dikenang Indonesia yang melimpah
E. menyadarkan bangsa indonesia untuk kembali
berwawasan maritim

28. Apa judul yang tepat untuk teks tersebut?


A. Negara Maritim Versus Negara Agraris
B. Keracunan Identitas Bangsa Indonesia
C. Pemberdayaan Potensi Kelautan Indonesia
D. Pengembalian Kejayaan Indonesia Melalui
Maritim
E. Pembangunan Maritim Untuk mengatasi
Kemiskinan

29. Apa simpulan keseluruhan isi tabel dalam teks tersebut?


A. Seluruh jenis produksi ikan mengalami kenaikan
yang seimbang.
B. Ada kecenderungan produksi ikan meningkat pada
tahun berikutnya.
C. Produksi perikanan jenis budi daya Iebih baik
daripada perikanan tangkap.
D. Kenaikan produksi ikan tangkap Iebih tajam
dibandingkan budi daya tawar.
E. Kenaikan produksi ikan tangkap Iebih tajam
daripada perikanan budi daya.

30. Apa kelemahan teks tersebut?


A. Tidak ada hubungan isi paragraf pertama dan
kedua.
B. Data dalam tabel tidak mendukung keseluruhan isi
teks.
C. Tabel seharusnya diletakkan setelah paragraf
terakhir.
D. Tidak ada uraian mengenai definisi kerancuan
paradigma.
E. Tidak dijelaskan program kerja kementerian
kelautan.
level global yang telah diperbincangkan sejak abad ke-17.
Dalam penelitiannya yang berjudul “Death in Childbed
from the Eighteent Century to 1935,” Loudon
menjelaskan bahwa catatan-catatan terkait kasus kematian
ibu mulai muncul pada awal abad ke-17, seiring dengan
berkembangnya praktik kebidanan di masyarakat Inggris
(Loudon, 1986). Akan tetapi, komitmen masyarakat
global terkait penanganan kasus kematian ibu agaknya
baru hadir di akhir abad ke-20. Pada tahun 1987,
kekhawatiran terkait dampak dan tingginya kasus
kematian ibu mendorong WHO dan organisasi-organisasi
internasional lain untuk melahirkan The Safe Motherhood
Initiative (Women & Children First, 2015).

Bacalah teks berikut untuk menjawab soal No 1 s.d. 7!


World Health Organization (WHO) memiliki
beberapa istilah berbeda terkait dengan AKI. Istilah
pertama adalah maternal death — atau kematian ibu,
yang didefinisikan sebagai “kematian yang terjadi saat
kehamilan, atau selama 42 hari sejak terminasi
kehamilan, tanpa memperhitungkan durasi dan tempat
kehamilan, yang disebabkan atau diperparah oleh
kehamilan atau pengelolaan kehamilan tersebut, tetapi
bukan disebabkan oleh kecelakaan atau kebetulan”
(WHO, 2004). Konsep maternal death ini berbeda
dengan konsep maternal mortality ratio, atau yang lebih
dikenal sebagai Angka Kematian Ibu (AKI), jika
mengacu pada definisi Badan Pusat Statistik (BPS). Baik
BPS maupun WHO mendefinisikan maternal mortality
ratio/AKI sebagai angka kematian ibu per 100.000 1. Berdasarkan paragraf 1, manakah di bawah ini
kelahiran hidup (WHO, 2004; BPS, 2012). pernyataan yang BENAR?
Menurut laporan dari WHO, kematian ibu umumnya A. Penyebab Maternal Death adalah kematian
terjadi akibat komplikasi saat, dan pasca kehamilan. karena kecelakan atau karena kebetulan.
Adapun jenis-jenis komplikasi yang menyebabkan B. Maternal death adalah angka kematian ibu per
mayoritas kasus kematian ibu — sekitar 75% dari total 100.000 kelahiran hidup.
kasus kematian ibu — adalah pendarahan, infeksi, C. Maternal mortality ratio kematian yang terjadi
tekanan darah tinggi saat kehamilan, komplikasi saat kehamilan, atau selama 42 hari sejak
persalinan, dan aborsi yang tidak aman (WHO, 2014). terminasi kehamilan, tanpa memperhitungkan
Untuk kasus Indonesia sendiri, berdasarkan data dari durasi dan tempat kehamilan, yang disebabkan
Pusat Kesehatan dan Informasi Kemenkes (2014) atau diperparah oleh kehamilan atau pengelolaan
penyebab utama kematian ibu dari tahun 2010–2013 kehamilan tersebut, tetapi bukan disebabkan
adalah pendarahan (30,3% pada tahun 2013) dan oleh kecelakaan atau kebetulan.
hipertensi (27,1% pada tahun 2013). Hal ini sangat ironis, D. Angka Kematian Ibu adalah angka kematian ibu
mengingat berbagai penyebab kematian ibu di atas per 100.000 kelahiran hidup.
sebenarnya dapat dicegah, jika sang ibu mendapatkan E. World Health Organization (WHO) memiliki
perawatan medis yang tepat. beberapa istilah yang sama terkait dengan AKI.
Tingginya angka kasus kematian ibu sebenarnya
bukanlah masalah yang terbilang baru. Upaya 2. Berdasarkan paragraf 2, jenis-jenis komplikasi yang
penanganan kasus kematian ibu merupakan diskursus menyebabkan mayoritas kasus kematian ibu adalah
sebagai berikut KECUALI?
A. Pendarahan Bacalah teks berikut untuk menjawab soal No. 8 s.d. 15!
B. Infeksi. (1) Puncak kejayaan maritim nusantara terjadi pada
C. Tekanan darah rendah saat kehamilan. masa Kerajaan Majapahit. (2) Majapahit berhasil
D. Komplikasi persalinan. menguasai dan mempersatukan nusantara. (3) Mempunyai
E. Aborsi yang tidak aman pengaruh sampai ke negara- negara asing, seperti
Thailand, Kamboja, India, Filipina, dan China. (4)
3. Simpulan yang tepat sesuai isi paragraf kedua adalah.... Kejayaan ini disebabkan karena kemampuan membaca
A. Menurut laporan dari WHO, kematian ibu potensi yang dimiliki. (5) Ketajaman visi dan kesadaran
umumnya terjadi akibat komplikasi saat, dan terhadap posisi strategis nusantara telah membawa negara
pasca kehamilan. ini disegani oleh negara-negara lain.
B. Penyebab kematian ibu hamil dapat dicegah jika
mendapatkan perawatan medis yang tepat. (6) Namun sayang kini kejayaan itu tidak lagi banyak
C. Di Indonesia penyebab utama kematian ibu hamil dikenang. (7) Kejayaan tersebut seakan tertutup oleh
adalah pendarahan dan hipertensi. potret kemiskinan yang melanda rakyat Indonesia. (8)
D. Di Dunia penyebab utama kematian ibu hamil Kecintaan kita pada laut juga makin dangkal. (9) Rasa
adalah komplikasi saat, dan pasca kehamilan. keterpihakan negara terhadap dunia maritim pun masih
E. Ada berbagai jenis penyebab kematian ibu hamil. lemah. (10) Meskipun Kementerian Kelautan dan
Perikanan sudah dibentuk, namun fokus pembangunan
4. Berdasarkan paragraf 3, manakah pernyataan di bawah negara ini masih berfokus di sektor darat. (11) Masalah
ini yang PALING MUNGKIN benar mengenai kasus utamanya adalah masalah paradigma. (12) Paradigma
kematian ibu darat/agraris masih kuat melekat pada kebanyakan
A. Upaya penanganan kasus kematian ibu masyarakat Indonesia. (13) Bangsa Indonesia masih
merupakan diskursus level global yang telah
mengidap kerancuan identitas. (14) Di satu pihak bangsa
diperbincangkan sejak tahun 1700-an
B. Pada awal abad-17 telah tercatat kasus-kasus Indonesia memunyai persepsi kewilayahan tanah air,
kematian ibu seiring dengan berkembangnya tetapi di pihak lain memposisikan diri secara kultural
praktik kebidanan di masyarakat Inggris sebagai bangsa agraris dengan puluhan juta petani miskin
C. Komitmen masyarakat global terkait penanganan yang tidak sanggup kita sejahterakan, sedangkan kegiatan
kasus kematian ibu baru hadir di awal abad ke- industri modern sulit berkompetisi dengan bangsa lain.
20.
D. Pada abad ke-17, Loudon membuat penelitian (15) Akibat dari hal ini adalah pembangunan
terkait dengan kasus-kasus kematian ibu. perekonomian maritim dan pembangunan sumber daya
E. Pada akhir abad ke-20, kekhawatiran tunggal manusia tidak pernah dijadikan arus utama pembangunan
terhadap tingginya kasus kematian ibu nasional, yang didominasi oleh persepsi dan kepentingan
mendorong WHO merumuskan The Safe
daratan semata. (16) Dari paparan tersebut, kita
Motherhood Initiative.
mendapati kenyataan bahwa Indonesia sebagai negara
5. Di Indonesia, perbandingan penyebab utama kematian kepulauan terbesar di dunia belum mampu
ibu akibat pendarahan dan hipertensi dan tahun 2010- memberdayakan potensi ekonomi kelautan. (17) Negeri
2013 berkisar.... ini belum mampu mentransformasikan sumber kekayaan
A. 10 : 9 D. 2 : 3 laut menjadi sumber kemajuan dan kemakmuran bangsa.
B. 6 : 5 E. 2 : 1 (18) Hal ini ditunjukkan pada tabel berikut.
C. 3 : 4
(19) Agar dapat menjadi bangsa yang kuat dan
6. Berdasarkan gambar 1, persentase penurunan AKI disegani di mata internasional, maka Indonesia harus
tahun 2000 adalah.... kembali berwawasan maritim. (20) Permasalahannya
A. 5,0% D. 7,6%
adalah apakah masih bisa membangkitkan kembali
B. 6,3% E. 8,1%
C. 7,2% kejayaan masa lalu tersebut di tengah krisis multidimensi
yang belum usai menerpa bangsa kita. (21)
7. Berdasarkan gambar 1, jika gradien penurunan AKI Mengembalikan visi kemaritiman bukan sesuatu hal
tahun 2015-2019 sebanding dengan gradien penurunan yang mudah. (22) Selain dibutuhkan kemauan yang
AKI tahun 1995-2000, maka nilai AKI pada tahun
2019 berkisar....
A. 97 D. 110
B. 105 E. 112
C. 108
tinggi untuk merombak sistem yang ada, masalah B. Keracunan Identitas Bangsa Indonesia
penyediaan infrastruktur menjadi masalah tersendiri. C. Pemberdayaan Potensi Kelautan Indonesia
D. Pengembalian Kejayaan Indonesia Melalui Maritim
8. Gagasan utama paragraf kedua teks tersebut adalah.. E. Pembangunan Maritim Untuk mengatasi
A. Kini kejayaan itu tak lagi dikenang Kemiskinan
B. Indonesia saat ini tidak sejaya Majapahit
C. Kecintaan rakyat Indonesia pada laut makin rendah 15. Apa simpulan keseluruhan isi tabel dalam teks
tersebut?
D. negara tidak berpihak kepada kemaritiman
A. Seluruh jenis produksi ikan mengalami kenaikan
E. fokus pembangunan saat ini masih berada di darat yang seimbang.
B. Ada kecenderungan produksi ikan meningkat pada
9. Kesalahan penggunaan tanda baca ditemukan pada tahun berikutnya.
kalimat.... C. Produksi perikanan jenis budi daya lebih baik
A. 1 D. 6 daripada perikanan tangkap.
B. 3 E. 10 D. Kenaikan produksi ikan tangkap lebih tajam
C. 4 dibandingkan budi daya tawar.
E. Kenaikan produksi ikan tangkap lebih tajam
10. Mengapa paradigma menjadi masalah utama daripada perikanan budi daya.
pembangunan Indonesia?
A. Indonesia belum dapat mengembalikan kejayaan
seperti zaman Majapahit. Bacalah teks berikut untuk menjawab soal No 16 s.d. 18!
B. Paradigma agraris masih kuat melekat pada Sampai saat ini narkoba masih mengancam
kebanyakan masyarakat Indonesia. masyarakat Indonesia meski Indonesia telah
C. Indonesia merupakan negara maritim, tetapi berkomitmen untuk bebas dari narkoba dan HIV AIDS
masyarakatnya berpikir agraris pada tahun 2015. Hal ini dapat dilihat dari jumlah
D. Kejayaan Indonesia masih tertutup oleh potret pengguna narkoba yang terus meningkat setiap tahunnya.
kemiskinan rakyat Indonesia Pada tahun 1970 diperkirakan hanya 130.000 orang yang
E. Kecintaan pada laut dan keberpihakan pada maritim menggunakan narkoba dan pada tahun 2009 terdeteksi
masih lemah. 2% penduduk Indonesia pernah bersentuhan dengan
narkoba atau meningkat 0,5% dibandingkan tahun
11. Kalimat yang tidak efektif adalah kalimat.... sebelumnya. Hal tersebut sangat mengkhawatirkan semua
A. (3) dan (4) D. (9) dan (10) pihak, khususnya Badan Narkotika Nasionial. Dari 2%
B. (5) dan (6) E. (13) dan (14) penduduk yang pernah bersentuhan narkoba tersebut,
C. (7) dan (8) 60% berusia produktif dan 40% pelajar.

12. Apa simpulan isi teks tersebut? Awalnya, pengguna narkoba adalah orang dewasa,
A. Indonesia sebenarnya memiliki potensi maritim berusia sekitar 25 tahun dan dari kalangan ekonomi kelas
yang besar. menengah ke atas. Dalam perkembangannya, pengguna
B. Keuntungan sebagai negara maritim tidak di narkoba sudah merambah para remaja dan masyarakat
manfaatkan. kelas menengah ke bawah. Bahkan, gelandangan pun ada
C. Indonesia merupakan negara maritim yang salah yang kecanduan narkoba. Keadaan tersebut sungguh
urus. sangat ironis. Kondisi pengguna narkoba di Indonesia
D. Indonesia belum mampu memberdayakan potensi pada tahun 2005–2007 dipaparkan sebagai berikut.
kelautan.
E. Bangsa Indonesia mengidap kerancuan identitas.

13. Tujuan penulisan teks tersebut adalah....


A. Memaparkan bahwa Indonesia pernah disegani
bangsa lain melalui kejayaan Majapahit
B. Menggambarkan perkembangan maritim Indonesia
sejak Majapahit hingga kini
C. Menunjukkan bahwa Indonesia dahulu merupakan
negara maritim yang kuat
D. Membuktikan adanya potensi kekayaan laut
Indonesia yang melimpah
E. Menyadarkan bangsa Indonesia untuk kembali 16. Pernyataan manakah yang paling tidak sesuai dengan
berwawasan maritime isi tabel di atas?
A. Semakin rendah kelompok usia sesorang, semakin
14. Apa judul yang tepat untuk teks tersebut? kecil juga tingkat penggunaan narkoba.
A. Negara Maritim Versus Negara Agraris B. Pengguna narkoba pada usia 25–29 selalu lebih
banyak daripada kelompok-kelompok usia lain Seperti yang kita lihat sekarang, taman kanak-kanak
yang berusia < 24. bahkan meminta calon muridnya sudah dapat membaca
C. Dari tahun ke tahun jumlah pengguna narkoba atau berhitung karena pendidikan dianggap pabrik.
setiap kelompok usia terus meningkat. Perguruan Tinggi di Indonesia juga menganggap dirinya
D. Semakin tinggi kelompok usia sesorang semakin sebuah industri produk, bukan industri barang.
besar tingkat penggunaan narkobanya. Akibatnya, mutu Perguruan Tinggi diukur dari
E. Kelompok usia selalu menentukan jumlah atau lulusannya, seperti gaji lulusannya. Seperti industri
banyaknya pengguna narkoba. otomotif, mutunya tentu diukur dari mobil yang
dihasilkan. Oleh karena itu, cara pandang pendidikan
17. Kata tersebut pada kalimat ke-4 paragraf ke-1 di atas sebagai industri produk ini menyebabkan institusi
merujuk pada.... pendidikan tak tertarik mendidik anak “bodoh” atau
A. Narkoba masih mengancam “termarginalkan”. Mereka berpikir bahwa sebagai sebuah
B. Komitmen Indonesia pabrik, tentunya yang ideal itu dengan sedikit berupaya,
C. Jumlah pengguna narkoba menghasilkan lulusan sebanyak-banyaknya, secepat-
D. Persentase pengguna narkoba cepatnya, dan sekaya-kayanya.
E. Peningkatan penggunaan narkoba
Dampak anggapan pendidikan sebagai industri
produk ini pula yang menghasilkan pikiran untuk
18. Paragraf manakah yang paling tepat melengkapi teks di
menghasilkan “lulusan terstandarkan” yang
atas?
kecakapannya seragam. Seperti pabrik kompor,
A. Tabel tersebut menunjukkan bahwa pengguna
pendidikan sebagai industri produk mengangankan
narkoba semakin meningkat. Untuk mengatasinya
keluarannya (baca: lulusannya) memiliki ukuran dan
diperlukan upaya sinergis dan semua pihak,
mutu yang sama. Pendidikan sebagai industri produk
khususnya Badan Narkotika Nasional dengan
tentu mengangankan lulusannya benkecakapan sama,
masyarakat. Tanpa sinergi tersebut tidak mungkin
andal, dan patuh agar dapat diterima sebagai pekerja.
bahaya narkoba dapat diatasi.
Namun, tentu ini cara pandang tak beres. Lulusan
B. Kondisi di atas menunjukkan bahwa pengguna
pendidikan haruslah manusia-manusia yang
narkoba selalu meningkat. Pengguna narkoba sudah
berkemampuan optimum sesuai dengan talentanya.
merambah pada remaja berusia muda. Pengguna
narkoba juga merambah pada masyarakat dari 19. Apa hubungan isi antar paragraf teks tersebut?
kalangan ekonomi kelas menengah ke bawah.
A. Paragraf ke-1 merupakan akibat dari paragraf
C. Meskipun narkoba membahayakan, para penguna
ke-2
pada umunya tidak menyadarinya. Upaya
B. Paragraf ke-2 merupakan pembanding paragraf
menyadarkan pengguna narkoba harus dimulai dari
ke-1
pihak yang paling dekat dengan pengguna. Kalau
C. Paragraf ke-3 merupakan akibat dari paragraf
tidak, mustahil upaya tesebut dapat berhasil
ke-2
D. Dari tabel terebut tampak bahwa jumlah pengguna
D. Paragraf ke 2 merupakan rincian dari paragraf
narkoba selalu meningkat. Pada tahun 1970
ke-1
diperkirakan hanya 130.000 orang yang
E. Paragraf ke-3 merupakan penyebab dari
menggunakan narkoba, tetapi pada tahun 2009
paragraf ke-2
terdeteksi 2% penduduk pernah bersentuhan
narkoba.
20. Apa pertanyaan yang jawabaannya terdapat pada
E. Dari tabel tersebut tampak bahwa jumlah pengguna
paragraf ke-3?
narkoba mulai bervariasi. Mulai dari remaja sampai
A. Apa yang harus dilakukan agar lulusan bisa
orang tua banyak yang menjadi pengguna narkoba.
berkemampuan optimum?
Masyarakat kelas menengah ke bawah dan
B. Bagaimana cara membuat lulusan menjadi
gelandangan pun ada yang kecanduan narkoba.
seorang pekerja yang baik?
C. Siapa yang berperan dalam pendidikan sebagia
Bacalah teks berikut untuk menjawab soal No 19 s.d. 20! industri ini?
D. Apa dampak dari anggapan pendidikan sebagai
Gambaran pendidikan sebagai pabrik ini yang
industri?
disindir Pink Floyd dalam “The Wall”. Pada video “The
E. Apa yang terjadi jika pendidikan tidak
Wall”, masukannya adalah anak-anak, kemudian
beranggapan sebagai industri?
diproses, diseragamkan, sehingga keluarannya “orang-
orang” yang patuh, terampil, mampu kerja. Jika
Bacalah teks berikut untuk menjawab soal No. 21 s.d
pendidikan dianggap industri produk atau pabrik, tentu
24!
gambaran itu memang pas. Dampaknya, kita ingin
(1) Upaya kesehatan dalam Sistem Kesehatan
masukannya seseragam dan sebaik mungkin. Akibatnya,
Nasional (SKN) pada Perpres No. 72 tahun 2012 telah
persekolahan mensyaratkan calon siswanya yang paling
mengatur bahwa upaya kesehatan dibagi dua, yaitu
terampil, paling dekat dengan gambaran lulusannya yang
upaya kesehatan perorangan (UKP) dan upaya
diangankan.
kesehatan masyarakat (UKM). (2) Secara teoritis, pihak merasa pesimistis, bahkan pada tataran tertentu
UKP adalah kegiatan untuk mengobati kesehatan bersikap apatis terhadap penyelenggaraan MTQ yang
perorangan melalui pelayanan kesehatan perorangan dianggap tak ubahnya pesta pora dan pemborosan uang
(perawat, klinik, dokter, dokter spesialis, rumah sakit, negara.
dll.). (3) SKN membagi upaya kesehatan perorangan
(5) Munculnya reaksi pejoratif dari beberapa
atas tiga jenjang, UKP Primer, UKP Sekunder, dan
kalangan tersebut, menunjukkan dua hal. (6) Pertama,
UKP Tersier.
bahwa tidak semua komponen bangsa memahami secara
(4) UKP Primer adalah upaya kesehatan di tingkat bijak terhadap substansi dan makna strategis di balik
kecamatan atau desa ketika individu mendapatkan penyelenggaraan MTQ sehingga melupakan arti penting
pengobatan untuk pertama kalinya. (5) Fasilitas pelayanan dan mahalnya ongkos untuk sebuah syiar keagamaan. (7)
kesehatan primer, misalnya, adalah puskesmas, klinik, atau Kedua, perjalanan waktu telah menempatkan perhelatan
dokter praktik pribadi di tingkat kecamatan. (6) Sedangkan MTQ tidak sekadar menjadi wahana pembinaan
UKP Sekunder adalah upaya kesehatan rujukan pertama di masyarakat dalam bidang kegamaan. (8) Kini, tidak
tingkat Kabupaten/Kota di mana fasilitas pelayanan jarang momentum MTQ menjadi ajang mempertaruhkan
kesehatan mampu menangani masalah rujukan dasar, prestise yang berimplikasi serius secara politis. (9)
misalnya RS Umum Daerah (RSUD) kelas C di tingkat Akibatnya, dalam kasus-kasus tertentu, politisasi MTQ
Kabupaten/Kota. menjadi tak terelakkan. (10) Tindakan melanggar hukum
untuk sekadar memperoleh kejuaraan menjadi hal yang
21. Kalimat yang tidak efektif ditandai dengan nomor....
lumrah atau bahkan menjadi tradisi yang dilakukan secara
A. 1 dan 3 D. 5 dan 6
sadar dan sistematis. (11) Kondisi semacam inilah yang
B. 2 dan 6 E. 2 dan 3
melahirkan kegamangan dan menurunnya muruah MTQ.
C. 3 dan 4
25. Apa gagasan utama paragraf pertama?
22. Bentukan kata yang tidak tepat pada paragraf pertama
A. MTQ mendapat banyak kritik.
adalah...
B. MTQ menjadi perhelatan yang populer.
A. Perpres (kalimat 1)
C. Banyak orang yang pesimis dengan MTQ.
B. teoritis (kalimat 2)
D. MTQ membawa nilai-nilai spiritualitas.
C. spesialis (kalimat 2)
E. Ada yang menganggap bahwa MTQ hanya
D. dll. (kalimat 2)
pemborosan.
E. dll. (kalimat 2)
26. Pada kalimat berapa terdapat kesalahan tanda koma?
23. Hubungan antara kedua paragraf tersebut adalah....
A. 1 D. 4
A. paragraf kedua merupakan penjelasan dari
B. 2 E. 5
paragraf pertama
C. 3
B. paragraf pertama merupakan pembanding
paragraf kedua
27. Pernyataan yang TIDAK sesuai dengan teks di atas
C. paragraf pertama adalah penyebab dari paragraf
adalah?
kedua
A. Terjadi politisasi di bidang MTQ.
D. paragraf kedua adalah akibat dari paragraf pertam
B. MTQ mendapat pandangan negatif.
E. paragraf pertama merupakan penjelasan dari
C. MTQ dianggap menjadi ajang pemborosan.
paragraf kedua
D. MTQ menjadikan masyarakat religius dan
beradab.
24. Kata rujukan pada kalimat (6) bermakna....
E. Tindakan melanggar hukum terjadi di ajang MTQ.
A. Tujuan D. bersatu
B. kembali E. prioritas
28. Kata inilah pada kalimat (11) merujuk pada?
C. acuan
A. Penyelenggaraan MTQ
B. MTQ Nasional
Bacalah teks berikut untuk menjawab soal No. 25 s.d. 30!
C. Tindakan melanggar hokum
(1) Dalam potret budaya masyarakat Indonesia, MTQ
D. Kecurangan MTQ
telah menjadi perhelatan keagamaan yang populer dan
E. Kritikan
fenomenal. (2) Bahkan, kegiatan yang dilakukan secara
berjenjang dan berkala ini telah sukses menciptakan suatu
29. Diksi yang tidak tepat pada paragraf ke-2 adalah...
pola atau paradigma baru keberagamaan umat Islam
A. Pejoratif (5)
Indonesia yang khas. MTQ tidak sekadar
B. Substansi (6)
mempertontonkan eksklusivitas spiritual, tetapi ia juga
C. Sekadar (7)
membawa nilai-nilai pluralitas yang tecermin dari nuansa
D. Momentum (8)
tradisi yang mengemuka dalam hiruk-pikuk hajatan
E. Politisasi (9)
tahunan tersebut. (3) Namun, hiruk-pikuk dan kemegahan
MTQ—dengan segala manfaat dan kegunaan yang
30. Kata berimplikasi pada kalimat (8) bermakna....
ditimbulkannya—tidak berarti sepi dari kritik. (4) Banyak
A. Mempunyai hubungan keterlibatan C. Dampak kebarakan hutan
B. Berdampak D. Tumpulnya penegakan hutan terhadap kasus
C. Memiliki akibat kebakaran hutan
D. Memiliki sebab E. Diliburkannya sekolah karena asap kebakaran
E. Mengakibatkan hutan.

2. Pernyataan yang sesuai dengan paragraf kedua


adalah, kecuali....
A. Pada tahun 2015, 1,7 juta hektare dibakar
menurut CIFOR.
B. Jumlah titik api terbanyak terdapat di wilayah
Sumatra Selatan dan Kalimantan Tengah.
C. Terdapat 54.805 titik api selama sembilan bulan
di tahun 2015.
D. Kerugian ekonomi karena kebakaran hutan
mencapai Rp200 triliun.
E. Terdapat 222.984 warga Kalimantan yang
menderita inspeksi saluran pernapasan akut.

3. Tanda baca koma yang tidak tepat terdapat pada


Bacalah teks berikut untuk menjawab soal No. 1 s,d, 4! kalimat...
A. 2 D. 13
(1) Kita tentu kecewa dengan keputusan Polda Riau
B. 4 E. 15
memberhentikan penyidikan 15 perusahaan yang diduga
C. 7
membakar lahan dan hutan pada 2015. (2) Apalagi,
argumen Polda Riau di balik pemberian surat perintah
4. Kelemahan paragraf pertama adalah...
penghentian penyidikan (SP3) itu adalah tidak ada bukti
A. Kesalahan konjungsi sementara pada kalimat (7)
perusahaan terkait melanggar hukum atau membakar
B. Penggunaan kata memberhentikan pada kalimat
lahan/hutan. (3) Sekali lagi kita melihat tumpulnya
(1) tidak tepat.
penegakan hukum terhadap korporasi yang diduga
C. Kesalahan konjungsi padahal pada kalimat (4).
melakukan pembakaran lahan hutan. (4) Padahal, baru
D. Kesalahan penggunaan tanda baca garis miring (/)
delapan bulan lalu kebakaran hutan hebat melanda
pada kalimat (2).
sebagian Sumatra dan Kalimantan. (5) Asap kebakaran
E. Kesalahan penggunaan tanda baca koma pada
hutan saat itu sampai membuat gelap Thailand dan
kalimat (2).
Malaysia. (6) Malaysia bahkan sempat menutup 7.000
sekolahnya di Sabah karena terganggu asap. (7)
Bacalah teks berikut untuk menjawab soal No. 5 s.d. 7!
Sementara, Thailand kelimpungan karena daerah
(1) Melihat hasil reshuffle Kabinet Kerja yang
wisatanya juga terimbas asap hitam.
disampaikan Presiden Jokowi pada 27 Juli 2016, sepertinya
(8) Jumlah titik api yang direkam oleh satelit cuaca cukup memberi optimisme. (2) Dari sembilan wajah baru
sepanjang 2015 pun mencetak rekor baru. (9) Ada 54.805 yang ditampilkan, hanya empat yang berlatar belakang
titik api sepanjang Januari–September 2015. (10) politisi. (3) Kita tentu saja berharap bahwa komposisi yang
Jumlahnya meningkat 10 ribu titik api dari tahun ada saat ini bisa bekerja lebih baik dan memberikan hasil
sebelumnya. (11) Titik api terbanyak ada di Sumatra memuaskan bagi rakyat.
Selatan dan Kalimantan Tengah. (12) Kebakaran hutan
(4) Sebelumnya, isu perombakan Kabinet Kerja
hebat tahun lalu juga memakan korban jiwa dan sakit
sudah beberapa kali muncul, tapi tak kunjung
yang tidak sedikit. (13) Dinas kesehatan sejumlah
terlaksana. (5) Maraknya isu ini didorong oleh dua
provinsi di Sumatra dan Kalimantan mencatat, ada
hal yang cukup fundamental, yaitu: perubahan
222.984 warga yang menderita inspeksi saluran
konstelasi politik dan masalah kinerja kabinet.
pernapasan akut. (14) Korban tewas, mulai dari bayi baru
lahir hingga dewasa yang terkena asma. (15) Lembaga (6) Dalam beberapa bulan terakhir, terjadi
peneliti kehutanan CIFOR, mengestimasi kerugian perubahan konstelasi politik dalam tubuh pemerintahan
ekonomi akibat kebakaran hutan/lahan 2015 mencapai menyusul bergabungnya Partai Golkar dan PAN ke
Rp200 triliun. (16) Dari lembaga yang sama juga keluar dalam gerbong partai pendukung pemerintah. (7)
data soal sebesar 1,7 juta hektare lahan sudah dibakar Kehadiran mereka jelas membutuhkan ruang sebagai
pada tahun itu. kompensasi. (8) Meski kursi menteri merupakan hak
prerogatif Presiden, tetapi sangat sulit untuk
1. Apa ide pokok paragraph pertama?
memisahkannya dengan tarik- menarik kepentingan
A. Pemberhentian penyidikan kebakaran hutan.
politik
B. Kekecewaan terhadap pemberhentian penyidikan
kasus kebakaran hutan. 5. Kalimat yang tidak baku terdapat pada nomor...
A. 1 dan 3 D. 5 dan Indonesia. (3) Sementara itu, Pulau Jawa hanya mempunyai
B. 2 dan 4 E. 7 dan 8 4 persen, tetapi dihuni hampir 60 persen penduduk
C. 5 dan 8 Indonesia. (4) Hal tersebut menyiratkan adanya potensi
kelangkaan air.
6. Kata reshuffle pada kalimat (1) berarti... (5) Kekeringan dan kelangkaan air adalah dua hal
A. Perombakan yang berbeda. (6) Kekurangan air secara alami akibat iklim
B. Pemulihan dinamakan kekeringan. (7) Kekurangan air akibat
C. Pengacakan pengurangan air oleh manusia dinamakan kelangkaan air.
D. Perubahan susunan (8) Ketersediaan air per kapita di Jawa adalah
E. Pencalonan ulang 1.168 meter kubik/orang/tahun, padahal batas ambang ideal
adalah 1.700 meter kubik/orang/tahun. (9) Karena itu, perlu
7. Tujuan penulis dari teks di atas adalah.... dilakukan strategi penanggulangan kekeringan dengan
A. Menginformasikan bahwa perubahan kabinet mitigasi kekeringan, manajemen kebutuhan air, dan
terjadi karena adanya perubahan konstelasi politik manajemen alokasi air. (10) Pengelolaan sumber daya air
B. Menjelaskan pentingnya unsur nonpolitisi dalam dengan pendekatan Integrated Water Resources
Kabinet Kerja Management atau IWRM. (11) Selain itu, dilakukan prediksi
C. Memberitahukan bahwa Presiden memiliki hak kekeringan hidrologi, prediksi kelangkaan air, dan
prerogatif untuk mengangkat menteri perkembangan iptek untuk mengatasi kelangkaan air.
D. Meyakinkan pembaca bahwa keadaan politik (12) Pemerintah juga berencana membangun 49
akan menjadi lebih baik bendungan di seluruh Indonesia untuk membantu memenuhi
E. Menginformasikan bahwa Partai Golkar dan PAN kebutuhan air masyarakat. (13) Selain sejumlah strategi
bergabung ke dalam pemerintahan. yang disebutkan di atas, pemerintah, melalui Kementrerian
Pekerjaan Umum dan Perumahan Rakyat, akan bekerja sama
Tulisan berikut diikuti oleh dua butir pertanyaan. dengan Kementerian Lingkungan Hidup dan Kehutanan,
Pertimbangkan apakah kata atau kalimat pada setiap Kementerian Perindustrian, pemda setempat, masyarakat,
nomor bercetak tebal TIDAK PERLU DIPERBAIKI serta lembaga swadaya masyarakat.
(A) atau diganti dengan pilihan lain yang tersedia (B,
C, D, atau E) 10. Gagasan pokok seluruh bacaan di atas adalah..
A. potensi sumber daya air dan kelangkaan air di
Indonesia.
Semua mata pelajaran yang diajarkan di sekolah selalu
B. perbedaan kekeringan dan kelangkaan air di
memiliki tantangan masing-masing. Beberapa jenis
Indonesia.
mata pelajaran membutuhkan kemampuan analisa,
C. cara-cara penanggulangan kelangkaan air di
sementara beberapa yang lain sangat 8teoritis. Indonesia.
Namun satu hal yang pasti, yaitu siswa sering kesulitan D. rencana pemerintah dalam penanggulangan
mengikuti pelajaran-pelajaran tersebut, apapun jenisnya. kekeringan.
Hal tersebut dikhwatirkan menjadi sebab banyaknya E. kerja sama antarinstansi dalam rangka mengatasi
kekeringan.
siswa yang menyontek 9pekerjaan rumah yang
diberikan oleh guru mereka di sekolah. 11. Kata potensi dalam kalimat (4) bermakna...
A. kecenderungan
B. kemampuan
8. A. TIDAK PERLU DIPERBAIKI C. perbedaan
B. teoretis D. keberadaan
C. teori E. permasalahan
D. teoritas
E. “teoritis” 12. Kalimat perbaikan dari kalimat (10) adalah...
A. pengelolaan sumber daya air melalui pendekatan
9. A. TIDAK PERLU DIPERBAIKI Integrated Water Reources Management atau
B. pekerjaanrumah IWRM.
C. pekerjaan-rumah B. pengelolaan sumber daya air dilakukan dengan
D. pekerjaan rumah pendekatan Integrated Water Resources
E. “pekerjaan rumah” Management atau IWRM
C. pengelolaan sumber daya air melakukan strategi
Bacalah teks berikut untuk menjawab soal No. 10 s.d. Integrated Water Resources Management atau
14! IWRM
(1)Potensi sumber daya air Indonesia sebenarnya D. strategi pengelolaan sumber daya air dengan
cukup besar, yaitu 3.960 miliar meter kubik. (2) Kalimantan, melakukan Integrated Water Resources
Papua, dan Sumatra, merupakan area dengan potensi air Management atau IWRM
permukaan sebesar 82 persen dari seluruh air permukaan di E. strategi pengelolaan sumber daya air melakukan
cara Integrated Water Resources Management atau Setelah itu, kulit batang diblender hingga halus. (14)
IWRM Langkah selanjutnya melakukan ekstraksi dengan
mencampurkan tepung kulit batang duku dengan parutan
13. Kata tersebut pada kalimat (4) merujuk pada.... methanol. (15) Hasil pencampuran tersebut disimpan di
A. persentase penduduk yang menghuni Pulau Jawa suhu kamar dalam jangka waktu tiga sampai lima hari.
B. perbandingan keadaan di Pulau Jawa dan pulau- (16) Dengan cara ini, kandungan konsentrat flavanoid dan
pulau lain di Indonesia saponin akan keluar dari kulit tersebut. (17) Kemudian
C. perbandingan potensi air permukaan dengan dilakukan penyaringan untuk memisahkan residu dengan
penduduk di Pulau Jawa cairan zat flavonoid dan saponin. (18) Obat ini ditemukan
D. potensi air permukaan yang menunjukkan oleh Okta Saputra. (19) Obat ini tidak berefek samping
perbedaan yang mencolok karena dibuat dari bahan alami.
E. potensi air permukaan yang menyebabkan
15. Apa makna kata diekstraksi pada kalimat (6)?
kelangkaan air di Pulau Jawa
A. dicampur
14. Simpulan bacaan di atas adalah.... B. diperas
A. Penanggulangan kekeringan dan kelangkaan air di C. disarikan
Indonesia perlu dilakukan sesegera mungkin D. dicairkan
B. Penanggulangan kekeringan dan kelangkaan air E. dikeluarkan
akan melibatkan pemerintah dan banyak pihak.
C. Penanggulangan kekeringan dan kelangkaan air 16. Berdasarkan isinya, kepada siapa Teks A berpihak?
merupakan dua hal yang penting yang bentuknya A. penemu obat
berbeda. B. penjual obat
D. Penanggulangan kekeringan dan kelangkaan air C. produsen obat
merupakan bantuan dari pemerintah dan D. pemakai obat
masyarakat. E. pembasmi nyamuk
E. Penaggulangan kekeringan dan kelangkaan air
merupakan dua hal yang sangat penting dan 17. Apa perbedaan tujuan penulisan Teks A dan Teks B?
mendesak. A. Teks 3A menjelaskan, sedangkan Teks 3B
bertujuan memengaruhi
Bacalah teks berikut untuk menjawab soal No. 15-19! B. Teks 3A bertujuan meyakinkan, sedangkan Teks
3B bertujuan menjelaskan
Teks A C. Teks 3A untuk memaparkan, sedangkan Teks 3B
(1) Okta Saputra adalah penemu obat pembasmi untuk meyakinkan
nyamuk dari kulit batang duku. (2) Ternyata, asap yang D. Teks 3A untuk memengaruhi, sedangkan Teks 3B
timbul dari pembakaran kulit batang duku itu ampuh untuk menjelaskan
mengusir nyamuk. (3) Agar produk lebih praktis, Okta E. Teks 3A untuk meyakinkan, sedangkan Teks 3B
merancangnya seperti parfum. (4) Adapun proses untuk membandingkan
pembuatan obat ini dimulai dengan mengeringkan kulit
batang hingga tingkat kadar airnya berkurang. (5) 18. Informasi apa yang ada di dalam Teks B tetapi tidak
Kemudian dipanggang dalam oven dengan suhu 100 termuat dalam Teks A?
derajat Celcius dalam jangka waktu dua jam. (6) Setelah A. bahan baku obat nyamuk
itu, diblender hingga halus dan selanjutnya diekstraksi B. konsentrat pada kulit batang duku
dengan cara mencampur dengan tepung kulit batang duku C. proses pemisahan residu
dengan parutan methanol. (7) Hasil pencampuran tersebut D. manfaat kulit batang duku
disimpan di suhu kamar dalam jangka waktu tiga sampai E. pengeringan kulit batang duku
lima hari hingga kandungan konsentrat flavanoid dan
saponin keluar dari kulitnya. (Diadaptasi dari harian 19. Apa keunggulan Teks B dibandingkan Teks A?
Kompas edisi 22 Januari 2016) A. Teks 3B mengemukakan bahan obat pembasmi
nyamuk dari kulit batang duku lebih rinci
B. Teks 3B memaparkan langkah pembuatan obat
Teks B pembasmi nyamuk dari kulit batang duku lebih
(8) Kulit batang duku ternyata bisa dijadikan obat rinci
pembasmi nyamuk alami yang ampuh. (9) Kulit batang C. Teks 3B memuat manfaat obat pembasmi nyamuk
duku mengandung flavonoid dan saponin yang ampuh dari kulit batang duku lebih lengkap
untuk mengusir nyamuk. (10) Untuk mengekstraksi D. Teks 3B memuat informasi penemu obat pembasmi
kandungan itu, ada beberapa cara yang harus dilakukan. nyamuk dari kulit batang duku lebih lengkap
(11) Proses dimulai dengan pengeringan kulit batang E. Teks 3B memerinci larutan methanol sebagai bahan
hingga tingkat kadar airnya berkurang. (12) Lalu, kulit pengekstrak kulit batang duku
batang kering itu dipanggang dalam oven dengan suhu
100 derajat Celcius dalam jangka waktu dua jam. (13) Bacalah teks berikut untuk menjawab soal No 20 s.d. 22!
Selama tahun 2014, setidaknya ada dua contoh hasil C. teori migrasi manusia purba Out of Africa sudah
penelitian menonjol yang dihasilkan para peneliti lokal: disepakati para arkeolog sedunia
pertama, tentang terkuaknya hunian sekaligus kuburan D. teknologi yang dibawa manusia purba ke Indonesia
massal Homo sapiens yang usianya lebih dari 14.825 tahun pada periode migrasi kedua makin canggih
di Gua Harimau, Ogan Komering Ulu, Sumatra Selatan, dan E. dari temuan arkeologis di Gua Harimau patut
kedua, tentang penemuan data migrasi baru manusia purba diduga adanya kontak antara ras Australomelanesid
di Situs Sangiran, Jawa Tengah, yang selama ini belum dan ras Mongoloid di Indonesia
masuk dalam literatur. Hingga bulan Mei 2014, Tim Pusat
Arkeologi Nasional berhasil mengekskavasi 78 kerangka 22. Hal yang dapat disimpulkan dari bacaan di atas
Homo sapiens di Gua Harimau. Yang menarik, dari 78 adalah....
kerangka itu, 4 individu merupakan ras Australomelanesid A. ada banyak temuan baru dalam bidang arkeologi di
dan 74 individu lain adalah ras Mongoloid. Dengan Indonesia
demikian, dapat diduga bahwa pernah ada masa ketika B. kemajuan dalam bidang arkeologi berhasil
kedua ras ini bertemu dan berinteraksi. mengungkap cakrawala masa lampau manusia
Sementara itu, dalam tiga tahun terakhir, di Sangiran, Indonesia
terpantau ada data baru migrasi manusia purba beserta C. kemampuan para arkeolog Indonesia makin tinggi
budayanya setelah migrasi tertua, yaitu pada masa 800.000- sehingga banyak fakta baru masa lampau manusia
an tahun lalu. Hal ini terdeteksi dari keberadaan kapak- Indonesia yang berhasil diungkap
kapak genggam dan pembelah yang merupakan ciri khas D. dua temuan penting di Gua Harimau dan situs
peralatan di Afrika. Arkeolog prasejarah, Prof. Harry Sangiran sedikit demi sedikit memverifikasi
Truman Simanjuntak, mengatakan bahwa migrasi ini beberapa teori arkeologis selama ini
disebut-sebut sebagai Out of Africa kedua yang melengkapi E. masih banyak yang harus diungkap dari kedua
teori migrasi tertua pertama Out of Africa saat Homo erectus temuan penting pada tahun 2014 itu
keluar dari Afrika 1,8 juta tahun lalu. Menurut arkeolog ini,
kapan Out of Africa kedua keluar dari Afrika belum 23. Kalimat manakah yang salah satu katanya ditulis
diketahui secara pasti, entah itu 1,1 juta tahun lalu atau 1,2 secara tidak tepat?
juta tahun lalu. Yang jelas, mereka sampai di Indonesia A. Uji cobakan gagasanmu di depan dewan juri paling
bersamaan dengan Tiongkok sekitar 800.000 tahun lalu. lama 30 menit!
Budaya migrasi kedua manusia purba ini terus B. Propinsi Sulawesi Selatan mendapatkan beberapa
berkembang pada periode selanjutnya walaupun tidak penghargaan nasional.
mampu menggantikan budaya yang tertua. Dari sisi C. warga nonmuslim di daerahku juga menghadiri
teknologi, model teknologi generasi kedua memang lebih acara halalbihalal.
canggih dengan teknik pangkasan kapak yang lebih baik dan D. Dalam bersepak bola kerja sama antar- pemain
bentuk simetris yang menarik. Akan tetapi, rupanya, alat- sangat dibutuhkan.
alat seperti ini tidak terlalu dibutuhkan di kawasan E. Orang tua itu kesana kemari mencari kebutuhan
Indonesia. Hal itu disebabkan bahan peralatan tersedia hidup keluarganya.
melimpah di mana-mana sehingga manusia purba saat itu
tidak perlu membawa alat ke mana-mana. 24. Kalimat manakah yang di dalamnya terdapat pilihan
kata yang tidak tepat?
20. Yang menjadi pokok masalah dalam teks di atas A. Pada kesempatan berbahagia ini saya mengucapkan
adalah.... terima kasih kepada hadirin.
A. ditemukannya teknologi manusia purba Afrika di B. Kepribadian siswa yang satu tidak boleh
situs Sangiran dibandingkan dengan kepribadian siswa lain.
B. temuan arkeologis yang menarik perhatian ilmuwan C. Kita harus berusaha secara maksimal untuk
pada tahun 2014 mencapai hasil gemilang.
C. pendapat arkeolog prasejarah mengenai temuan di D. Akan tetapi, akhirnya pemerintah tetap memperoleh
Gua Harimau hak kepemilikan lahan tersebut.
D. ditemukannya data baru mengenai migrasi manusia E. Tenggat pengumpulan tugas kelompok belajar
purba Afrika kelas XI belum diumumkan guru kami.
E. kontak antara manusia purba ras Australomelanesid
dan ras Mongoloid di Gua Harimau pada suatu 25. Kalimat manakah yang mengandung bentukan kata
masa yang tidak tepat?
A. Komponis itu merubah irama lagu asli ketika
21. Pernyataan yang tidak relevan dengan isi teks di atas mengaransemen ulang lagu itu.
adalah.... B. Christiano Ronaldo memosisikan dirinya sebagai
A. teori migrasi manusia purba Out of Africa sudah pemain sepak bola professional.
disepakati para arkeolog sedunia C. Peserta standup comedy memarodikan kejadian-
B. sepanjang sejarah manusia terjadi dua kali migrasi kejadian actual.
manusia purba dari Afrika yang mencapai D. Mahasiswa tingkat akhir mempraktikkan salah satu
Indonesia teori untuk menganalisis data.
E. Pemerintah batal menerapkan peraturan baru di adalah…
bidang energi kelistrikan . A. Bagaimana rasa nyeri yang disebabkan oleh
penggunaan gadget?
26. Manakah kalimat yang tidak efektif? B. Apa akibat dari rasa nyeri yang tidak diobati?
A. Masyarakat adat mengkhawatirkan hak mereka atas C. Mengapa banyak anak dan remaja menderita nyeri
tanah, wilayah, dan sumber daya alam. pada tulang belakang?
B. Dengan demikian, terdapat 5.000 kelompok D. Bagaimana kebiasaan remaja memainkan gadget?
masyarakat adat yang berada di 90 negara di dunia. E. Kapan para anak dan remaja biasanya
C. Dalam musyawarah masyarakat adat tahun lalu menggunakan gadget?
telah dibicarakan hak dan kewajiban mereka.
D. Perubahan iklim menjadi persoalan bukan hanya 30. Kata sambung yang salah adalah ....
bagi manusia, melainkan juga bagi alam. A. sambil pada kalimat (4)
E. Bentuk kegiatan ini adalah menyusun desain, B. sehingga pada kalimat (5)
mengembangkannya, dan pemasaran produknya C. apabila pada kalimat (7)
D. seperti pada kalimat (8)
Bacalahteks berikut untuk menjawab soal No. 27 s.d. 30! E. tetapi pada kalimat (11)
(1)Dalam beberapa tahun terakhir, mulai banyak pasien
anak dan remaja yang memiliki keluhan pada tulang
belakang. (2) Padahal, rasa sakit seperti ini biasanya dialami
orang yang berusia 40 tahun ke atas. (3) Anak dan remaja ini
ternyata gemar menggunakan gadget, seperti telepon
pintar, komputer, dan tablet. (4) Banyak dari mereka yang
bermain gadget sambil tengkurap, membungkuk, atau
lehernya ke bawah untuk menatap layar monitor, sehingga
kepala membebani leher. (5) Ada pula yang badannya
bersandar di kursi dengan posisi layar lebih tinggi dari pada
mata. (6) Di samping itu, juga ada yang memiringkan kepala
ke satu sisi untuk menjepit gadget di antara telinga dan
pundak ketika menelepon. (7) Apabila ini dilakukan terus
menerus, tulang belakang akan “protes” dengan
mengirimkan sinyal nyeri.
(8) Ada berbagai macam sensasi nyeri pada tulang
belakang, seperti ditusuk-tusuk, kese- mutan, tersetrum,
dan nyeri cenat cenut seperti sakit gigi. (9) Pada nyeri yang
diakibatkan gadget, biasanya sensasinya seperti tersetrum.
(10) Nyeri ini memang tak berbahaya. (11) Tetapi, jika
dibiarkan terus menerus, nyeri ini bisa merusak postur
tulang. (12) Ada tiga tahapan nyeri pada tulang belakang,
yaitu rasa nyeri yang dialami otot, kemudian menjalar ke
sendi, dan terakhir mengenai tulang.

27. Gagasan utama pada paragraf pertama adalah…


A. Penyakit tulang belakang meningkat beberapa
tahun terakhir.
B. Keluhan pada tulang belakang biasanya dialami
orang tua.
C. Jumlah anak dan remaja yang mengeluhkan sakit
tulang belakang meningkat.
D. Anak dan remaja gemar menggunakan gadget
dengan cara yang tidak benar.
E. Posisi menggunakan gadget menyebabkan nyeri
tulang belakang.

28. Kesalahan penggunaan tanda baca ditemukan pada


kalimat ....
A. 2 D. 6
B. 3 E. 8
C. 4

29. Pertanyaan yang jawabannya tidak terdapat dalam teks


(A) Mengapa pendidikan karakter meru-pakan
pendidikan budi pekerti?
(B) Mengapa kecerdasan emosi penting dalam
mempersiapkan masa depan?
(C) Bagaimana pendidikan karakter yang sistematis dan
berkelanjutan?
(D) Bagaimana pendapat Lickona tentang aspek
pendidikan, karakter?
(E) Siapa yang mengemukakan Sembilan pilar karakter
yang berasal dari nilai luhur universal?

03. Apa perbedaan gagasan antarparagraf dalam teks


tersebut?
(A) Paragraf pertama memaparkan pendidikan karakter
secara umum, sedangkan paragraf kedua
memaparkan pendidikan karakter secara khusus.
(B) Paragraf pertama memaparkan konsep pendidikan
karakter, sedangkan paragraf kedua memaparkan
unsur pendidikan karakter.
(C) paragraf pertama memaparkan pendidikan secara
umum, sedangkan paragraf kedua memaparkan
Teks berikut digunakan untuk menjawab soal nomor 01- tujuan pendidikan secara khusus.
04! (D) Paragraf pertama memaparkan konsep pendidikan
(1) Semua orang pasti mengenal pendidikan. (2) secara umum, sedangkan paragraf kedua
Pendidikan adalah proses internalisasi budaya ke dalam diri memaparkan konsep pendidikan karakter.
seseorang dan masyarakat sehingga membuat orang dan (E) Paragraf pertama memaparkan pendidikan secara
masyarakat jadi beradab. (3) Pendidikan bukan hanya umum, sedangkan paragraf kedua memaparkan
merupakan sarana transfer ilmu pengetahuan, tetapi lebih konsep pendidikan karakter
luas lagi, yakni sebagai sarana pembudayaan dan penyaluran
nilai (enkulturisasi dan sosialisasi). (4) Anak harus 04. Apa kelemahan isi paragraf ke-1?
mendapatkan pendidikan itu mencakup sekurang-kurangnya (A) Tidak ada hubungan antara penjelasan kecerdasan
tiga hal penting mendasar. emosi (kalimat 9) dengan Sembilan pilar karakter
(6) Pendidikan karakter adalah pendidi-kan budi (kalimat 10)
pekerti plus, yaitu yang melibatkan aspek pengetahuan (B) Plus pada pendidikan budi pekerti (kalimat 6) tidak
pengetahuan, perasaan, dan tindakan. (7) efektif. (8) Dengan dijelaskan secara rinci pada bagian berikutnya.
pendidikan karakter yang diterapkan secara sistematis dan (C) Penjelasan tentang pendidikan sebagai sarana
berkelanjutan, seorang anak akan manjadi cerdas emosinya. transfer ilmu (kalimat 3) bertentangan dengan
(9) Kecerdasan emosi ini adalah bekal penting dalam konsep pendidikan (kalimat 2).
mempersiapkan anak menyongsong masa depan. (10) (D) Tidak ada hubungan antara ketiga aspek yang
Terdapat Sembilan pilar karakter yang berasal dari nilai- dimaksud Lickona (kalimat 7) dengan nilai plus
nilai luhur universal, yaitu karakter cinta Tuhan dan segenap pada pendidikan budi pekerti (kalimat 6).
ciptaan-Nya: kemandirian dan tanggung jawab: (E) Penjelasan tentang kecerdasan emosi (kalimat 8)
kejujuran/amanah dan diplomatis: hormat dan santun: tidak berkaitan dengan pen-didikan karakter
dermawan, suka menolong, dan gotong royong/kerja sama; (kalimat 7).
percaya diri dan pekerja keras; kepemimpinan dan keadilan;
baik dab rendah hati; serta toleran dan cinta damai. Teks berikut digunakan untuk menjawab soal nomor 05
(Dikutip dengan pengubahandan blogdetik.com) – 09.
(1) Motivasi berasal dari motove atau movere yang berarti
01. Manakah gagasan utama paragraf ke-1? "mengerahkan". (2) Sepentang dikatakan Liang Gie (dalarn
(A) Pendidikan dikenal setiap orang Martoyo, 2000), motif atau dorongan adalah suatu dorongan
(B) Pendidikan adalah internalisasi budaya. yang menjadt pangkal seseorang melakukan sesuatu atau
(C) Pendidikan bukan sarana transfer ilmu. bekerja. (3) Orang yang Sangat termotivasi adalah orang
(D) Pendidikan merupakan sarana pembudayaan yang melaksanakan upaya substansial, guna menunjang
tujuan-tujuan produksi kesatuan kerjanya, dan organisasi
(E) Pendidikan harus berdimensi kemanusiaan
dirnana ia bekerja. (4) Seseorang yang tidak termotivasi,
hanya memberikan upaya minim ketika bekerja.
02. Manakah pernyataan yang jawabannya terdapat pada
(5) Konsep motivasi, merupakan sebuah konsep penting
paragraf ke - 2
dalam studi tentang kinerja individual. (6) Dengan demikian
motivasi berarti pemberian motif atau hal yang
menirnbulkan dorongan atau keadaan yang rnenimbulkan (B) Tujuan organisasi belum menjadi tujuan bersama
dorongan. (7) Dapat juga dikatakan bahwa motivasi adalah karyawan.
faktor yang mendorong orang untuk bertindak dengan cara (C) Imbalan karyawan belum sepadan dengan beban
tertentu. kerjanya.
(8) Menurut Martoyo (2000) manusia akan rnemiliki (D) Organisasi belum memerhatikan finansial dan
semangat untuk mengerjakan suatu hal jika dapat nonfinansialnya.
menghasilkan sesuatu yang dianggapnya sangat berharga. (E) Motivasi yang dilakukan organisasi kepada
yang berdampak pada kelangsungan hidupnya. karyawan tidak tepat.
menimbulkan rasa tentram, rasa aman dan seba-gainya. (9)
Motivasi atau dorongan kepada pegawai untuk bersedia Teks berikut digunakan untuk menjawab soal nomor
bekerja sama demi tercapainya tujuan bersama dapat (1) Pembentukan Masyarakat Ekonomi ASEAN 2015
diklasifikasi rnenjadi dua macam, yaitu motivasi finansial membuka peluang sekaligus tantangan bagi Indonesia dalam
dan nonfinansial. meningkatkan kesejahteraan bangsa melalui integrasi
(10) Motivasi finansial adalah dorongan yang dilakukan ekonomi dan keuangan di kawasan. (2) Luasnya wilayah
dengan memberikan imbalan finansial kepada pegawai. (11) Indonesia dengan komoditas unggulan ekspor relatif
Imbalan tersebut sering tersebut dengan istilah intensif. (12) bervariasi antarprovinsi membuka peluang peningkatan
Motivasi nonfinansial adalah dorongan yang tidak diversifikasi ekspor Indonesia, baik dalam rangka
diwujudkan dalam bentuk finansial atau uang. melainkan meningkatkan ekspor ke kawasan ASEAN, Asia lainnya
diwujudkan dalam bentuk pujian, penghargaan. pendekatan maupun dunia. (3) Secara spesifik, ekspor Indonesia
antar manusia dan lain sebagainya. (13) Dari penjelasan ini, ditentukan oleh kinerja ekonomi dab perdagangan
dapat disimpulkan bahwa motivasi sangat penting bagi suatu provinsi.(4) Perdagangan internasional di suatu negara dapat
perusahaan atau instansi karena dapat mendorong karyawan dibangun oleh perdagangan dari setiap provinsi.
untuk bertindak secara ikhlas dalam mencapai tujuan (5) Unggulan ekspor yang relative bervariasi antar provinsi
perusahaan secara efisien. (Dikutip dengan pengubahan dari dapat didayagunakan untuk memacu pertumbuhan
rids-wanrzzbic.word press.com) ekspornya. (6) Hingga saat ini, provinsi yang paling
dominan melakukan kegiatan ekspor adalah Jawa Barat
05. Penggunaan kata yang tidak tepat terdapat pada kata …. giikuti Riau, Jawa Timur, dan Jakarta. (7) Sebanyak 26,2%
(A) minim dalam kalimat 4 ekspor Jabar ditunjukan ke ASEAN dengan negara tujuan
(B) motif dalam kalimat 6 utama Malaysia, Filipina, Thailand, dan Vietnam. (8)
(C) substansial dalam kalimat 3 Provinsi Riau melakukan kegiatan ekspor/terutama ke
(D) individual dalam kalimat 5 kawasan Singapura.
(E) efisien dalam kalimat 13 (9) Jika dilihat secara lebih detail, maka setiap wilayah di
Indonesia memiliki kek-hususan komoditas utama yang
06. Kata ini pada kalimat 13 merujuk pada …. diekspor, misalnya Sumatera : minyak sawit dan karet
(A) motivasi finansial dan nonfinansial mentah ; Kalimantan : batu bara ; Sulawesi : coklat, minyak
(B) dampak motivasi sawit dan metalliferous : Jawa dan Bali : tekstil, garmen,
(C) imbalan finansial kertas, furniture dan produk kayu; Nusa Tenggara : metallif-
(D) pemberian motif trous ; serta Irian : metalliferous. (10) Dari data
(E) konsep dan jenis motivasi pertumbuhan investasi dan Kinerja Ekonomi Makro
Indonesia diketahui sebagian besar provinsi di Indonesia
07. Kesalahan penggunaan ejaan ditemukan pada kalimat hanya memiliki komoditas ekspor utama terbatas pada satu
…. hingga dua komoditas. (11) Hanya beberapa provinsi saja di
(A) 1 dan 6 pulau Jawa yang memiliki komoditas ekspor unggulan yang
(B) 2 dan 12 lebih terdiverifikasi.
(C) 3 dan 9
(D) 5 dan 7
(E) 8 dan 10 Provinsi Tabel Kontribusi Provinsi terhadap Ekspor

08. Bagaimana hubungan isi paragraf dalam teks tersebut?


Ekspor
(A) Paragraf ke-1 menjadi penyebab paragraf ke-2 Provinsi
(B) Paragraf ke-2 menjadi penyebab paragraf ke-3 2005 2006 2007
(C) Paragraf ke-2 menjadi pembanding paragraf ke-1 Jawa Barat 23,2 21,4 14,4
(D) Paragraf ke-3 memaparkan contoh paragraf ke-1
Riau 13,6 13 13,5
(E) Paragraf ke-4 menjelaskan paragraf ke-3
Jawa Timur 9,9 10,4 11,5
Jakarta 8,1 7,9 7,8
09. Mengapa di sebuah organisasi ada karyawan yang tidak
Sumut 6,6 6,4 6,5
maksimal dalam bekerja?
(A) Finansial yang diterima karyawan tidak sepadan Kaltim 5,9 6,2 5,9
dengan tugasnya. Banten 7,7 7,6 6,6
Jawa Tengah 4,2 3,9 3,8
Irian 3,8 3,9 4,8
Kalsel 3,1 3,7 3,8 14. Apa kelemahan isi teks tersebut?
(A) Pengintegrasian ekonomi dan keuangan tidak
10. Pernyataan manakah yang paling sesuai dengan isi tabel diikuti penjelasan tentang wujud pengintegrasian
tersebut? tersebut.
(A) Hanya Provinsi Jawa Timur yang menunjukan (B) Pernyataan bahwa produk ekspor antar provinsi
perkembangan positif ekspor nonmigas. bervariasi tidak didukung oleh komoditas ekspor
(B) Hanya Provinsi Jawa Barat yang menunjukan tiap provinsi.
persentase ekspor menurun dari tahun 2005 hingga (C) Dominasi ekspor Jawa Barat tidak disertai
tahun 2007. penjelasan tentang seberapa besar persentase
(C) Provinsi yang paling dominan melakukan kegiatan ekspor tersebut.
ekspor adalah provinsi-provinsi di Pulau Jawa. (D) Isian tabel tidak menjelaskan perkembangan
(D) Provinsi Riau, Sumut, dan Kaltim mengalami kontribusi ekspor provinsi dan perbandingannya
penurunan ekspor pada tahun 2006 dan bangkit antarprovinsi.
kembali pada tahun 2007. (E) Tantangan untuk meningkatkan ekspor Indonesia
(E) Kontribusi ekspor nonmigas Provinsi Riau tidak diikuti penjelasan tentang faktor penentu
melebihi akumulasi provinsi-provinsi di pulau ekspor tersebut.
Kalimantan.
15. Joko Widodo mendiskusikan tentang penanggulangan
bahaya banjir.
11. Kalimat yang tidak efektif terdapat pada … Kalimat di atas tidak dapat dipakai untuk karya ilmiah
(A) 1 dan 3 karena adanya penggunaan kata...
(B) 2 dan 6 (A) Joko Widodo
(C) 4 dan 9 (B) Mendiskusikan
(D) 5 dan 10 (C) Tentang
(E) 7 dan 11 (D) Penanggulangan
(E) Banjir
12. Simpulan manakah yang paling tepat untuk teks
tersebut? 16. Bacalah teks berikut!
(A) Komoditas ekspor Indonesia yang sangat bervariasi Berita yang dilansir Antara bulan lalu dengan tajuk “Bahasa
memacu peningkatan disversifikasi ekspor ke Indonesia Berpeluang Jadi Bahasa Kedua ASEAN”,
dunia. memberikan satu harapan besar bagi Indonesia untuk
(B) Setiap wilayah di Indonesia memiliki kekhususan berbicara di kancah Internasional. Sekarang ada kesadaran
komoditas utama yang diekspor ke kawasan di kalangan warga Philipina teristimewa di kawasan selatan
ASEAN. Negara ini bahwa mereka perlu belajar bahasa Indonesia
(C) Pembentukan masyarakat ekonomi ASEAN 2015 karena bahasa itu dapat dimengerti di empat negara anggota
memacu pertumbuhan ekspor Indonesia. ASEAN lainnya: Brunei, Malaysia, Singapura, dan Thailand
(D) Sebagian besar provinsi di Indonesia hanya (Selatan). Selain keempat Negara ASEAN itu, sebagian
memiliki komoditas ekspor utama yang terbatas warga suku Champ di Kamboja, Laos, dan Vietnam
(E) Peningkatan kesejahteraan bangsa dapat dilakukan mengerti bahasa Melayu yang menjadi induk bahasa
melalui pengintegrasian ekonomi dan keuangan Indonesia.
kawasnan. Manakah kalimat yang tepat untuk menutup paragraf di
atas?
13. Penulis teks tersebut bertujuan agar pembaca… (A) Untuk itu, kita perlu menggalakkan penyebaran
(A) meyakini bahwa komoditas ekspor nonmigas di kamus bahasa Indonesia agar memudahkan orang
Indonesia penting untuk diperhatikan lebih asing.
seksama kedepannya. (B) Dengan demikian, jelaslah bahwa bahasa Indonesia
(B) memiliki gambaran bahwa potensi ekspor menjadi bahasa yang diidamkan di kawasan
nonmigas Indonesia lebih besar daripada ekspor ASEAN.
nonmigas. (C) Oleh karena itu, bangsa Indonesia harus dapat
(C) menyimpulkan bahwa beberapa provinsi Jawa menggunakan bahasa Indonesia dengan baik dan
memiliki komoditas ekspor unggulan yang lebih benar.
terdiverifikasi. (D) Untuk itu, perlu diupayakan agar bahasa Indonesia
(D) menyadari bahwa kevariasian unggulan produk menjadi bahasa utama ASEAN.
provinsi harus didayagunakan untuk meningkatkan (E) Sehubungan dengan hal itu, menjunjung tinggi
pertumbuhan ekspor. bahasa Indonesia menjadi kewajiban seluruh
(E) menyadari bahwa pada tahun 2015 anggota bangsa Indonesia.
ASEAN siap menghadapi kerja sama ekonomi
antaranggota.
17. Seseorang akan mengembangkan karangan yang Sampai saat ini narkoba masih mengancam masyarakat
bertema upaya meningkatkan kemampuan Indonesia meski Indonesia telah berkomitmen untuk bebas
berwirausaha bagi remaja dengan kerangka berikut : dari narkoba dan HIV AIDS pada tahun 2015. Hal ini dapat
(1) Saran bagi remaja berwirausaha dilihat dari jumlah pengguna narkoba yang terus meningkat
(2) Pentingnya meningkatkan kemampuan setiap tahunnya. Pada tahun 1970 diperkirakan hanya
berwirausaha 130.000 orang yang menggunakan narkoba dan pada tahun
(3) Langkah meningkatkan kemampuan ber-wirausaha 2009 terdeteksi 2% penduduk Indone-sia pernah
(4) Komponen yang mendukung kemampu-an bersentuhan dengan narkoba atau meningkat 0,5%
berwirausaha dibandingkan tahun sebelumnya. Hal tersebut sangat
(5) Ketidakmampuan remaja berwirausaha mengkhawatirkan semua pihak, khususnya Badan Narkotika
Urutan kerangka yang paling tepat untuk tema karangan Nasional. Dari 2% penduduk yang pernah bersentuhan
di atas adalah ... narkoba tersebut, 60% berusia produktif dan 40% pelajar.
(A) 5,1,2,4,3 Awalnya, pengguna narkoba adalah orang dewasa, berusia
(B) 2,4,5,1,3 sekitar 25 tahun dan dari kalangan ekonomi kelas menengah
(C) 2,3,4,5,1 ke atas. Dalam perkembangannya, pengguna narkoba sudah
(D) 4,5,2,1,3 merambah para remaja dan masyarakat kelas menengah ke
(E) 5,2,4,3,1 bawah. Bahkan, gelandangan pun ada yang kecanduan
narkoba. Keadaan tersebut sungguh sangat ironis. Kondisi
pengguna narkoba di Indonesia pada tahun 2005--2007
dipaparkan sebagai berikut.
Tabel Pengguna Narkoba di Indonesia Tahun 2005--2007
18. Bacalah kedua teks berikut!
Teks 1 Usia 2005 2006 2007
Obat telah menjadi bagian yang tidak terpisahkan <16 127 175 1`80
darikehidupan manusia masa kini. Bahkan, karena begitu 16-19 1.668 2.447 2.617
akrabnya, orang jsutru semakin tergantung pada obat,
semakin biasa menggunakan obat. Penggunaan obat-obatan 20-24 5.503 8.383 8.275
kini justru terlihat sebagai gaya hidup modern. Mereka perlu
tampil seksi, bukan sehat, perlu obat kuat, bukan hidup 25-29 6.442 8.105 9.278
harmonis, dan lain-lain. Dan faktanya zat-zat yang
terkandung dalam obat dan efeknya bagi tubuh kita sering >29 9.040 12.525 15.889
tidak kita perhatikan karena menganggap obat itu
menyembuhkan tanpa memperhatikan apa yang sebenarnya 19. Pernyataan manakah yang paling tidak sesuai dengan isi
terkandung dalam obat tersebut. Dalam pemilihan obat, tabel di atas?
harus diperhatikan adanya kandungan bahan-bahan kimia (A) Semakin rendah kelompok usia sesorang, semakin
yang justru menimbulkan dampak buruk terhadap tubuh kecil juga tingkat penggunaan narkoba.
kita. (B) Pengguna narkoba pada usia 25-29 selalu lebih
Teks 2 banyak daripada kelompok-kelompok usia lain
Pada umumnya, orang memandang obat sebagai sesuatu yang berusia <24.
yang ajaib dapat menyembuhkan segala penyakit dengan (C) Dari tahun ke tahun jumlah pengguna narkoba
cepat tanpa efek samping. Semua obat adalah racun, kecuali setiap kelompok usia terus meningkat.
diminum sesuai dengan dosis yang diizinkan. Hal itu (D) Semakin tinggi kelompok usia sesorang semakin
menyadarkan kita bahwa dengan mengkonsumsi obat, kita besar tingkat penggunaan narkobanya.
harus memperhatikan dosis yang tepat untuk kesembuhan (E) Kelompok usia selalu menentukan jumlah atau
yang diharapkan. Masyarakat menaruh harapan besar akan banyaknya pengguna narko-ba.
obat, namun hal tersebut tidak sebanding dengan
pengetahuan masyarakat akan obat. Informasi tentang 20. Kata tersebut pada kalimat ke-4 paragraf ke-1 di atas
pentingnya obat, penggunaan, dan bahayanya yang sangat merujuk pada …
penting bagi masyarakat. (A) Narkoba masih mengancam
(B) Jumlah pengguna narkoba
Kedua teks di atas menjelaskan tentang … (C) Peningkatan penggunaan narkoba
(A) Kebutuhan manusia akan obat. (D) Komitmen Indonesia
(B) Cara konsumsi obat. (E) Persentase pengguna narkoba
(C) Anggapan masyarakat akan obat.
(D) Pengetahuan masyarakat akan obat. 21. Paragraf manakah yang paling tepat melengkapi teks di
(E) Harapan manusia akan obat. atas ?
(A) Tabel tersebut menunjukkan bahwa pengguna
Bacalah teks berikut untuk menjawab soal nomor 19 narkoba semakin meningkat. Untuk mengatasinya
sampai dengan 21! diperlukan upaya sinergis dari semua pihak,
khususnya Badan Narkotika Nasional dengan TOKI meraih 2 medali perak dan 1 medali perunggu. (13)
masyarakat. Tanpa sinergi tersebut tidak mungkin Kini, TOKI mengirimkan empat siswa dengan harapan
bahaya narkoba dapat diatasi. kembali dapat memenangkan medali. (14) Seluruh bangsa
(B) Kondisi di atas menunjukkan bahwa pengguna Indonesia turut serta mendoakan agar para siswa yang
narkoba selalu meningkat. Pengguna narkoba sudah pahlawan itu berhasil berkompetisi di ajang internasional.
merambah pada remaja berusia muda. Pengguna (15) Keberhasilan itu kian mendongkrak harga diri bangsa
narkoba juga merambah pada masyara-kat dari yang ternyata tidak kalah berprestasi dari bangsa-bangsa lain
kalangan ekonomi kelas menengah ke bawah. di dunia.
(C) Meskipun narkoba membahayakan, para penguna
pada umunya tidak menyadarinya. Upaya 22. Hubungkan isi antarparagraf dalam teks di atas yang
menyadarkan pengguna narkoba harus dimulai dari paling tepat adalah...
pihak yang paling dekat dengan pengguna. Kalau (A) Paragraf ke-2 menjadi akibat paragraf ke -1
tidak, mustahil upaya tesebut dapat berhasil. (B) Paragraf ke-3 merupakan penyebab paragraf ke-2
(D) Dari tabel terebut tampak bahwa jumlah pengguna (C) Paragraf ke-2 memaparkan rincian paragraf ke-1
narkoba selalu meningkat. Pada tahun 1970 (D) Paragraf ke-3 merupakan pembanding paragraf ke-
diperkirakan hanya 130.000 orang yang 1
menggunakan narkoba , tetapi pada tahun 2009 (E) Paragraf ke-3 merupakan contoh paragraf ke-2
terdeteksi 2% penduduk pernah bersentuhan
narkoba. 23. Penulis teks di atas berpihak kepada...
(E) Dari tabel tersebut tampak bahwa jumlah pengguna (A) Para siswa SMA
narkoba mulai bervariasi. Mulai dari remaja sampai (B) Pemerintah Indonesia
orang tua banyak yang menjadi pengguna narkoba. (C) Bangsa Indonesia
Masyarakat kelas menengah ke bawah dan (D) Tim Olimpiade Sains Indonesia
gelandangan pun ada yang kecanduan narkoba. (E) Penyelenggara Olimpiade Sains

Bacalah teks berikut untuk menjawab soal nomor 22 24. Kesalahan penggunaan ejaan ditemukan pada kalimat ...
sampai dengan 26! (A) 6 dan 9
(1) Indonesia kembali mengirimkan para siswa SMA ke (B) 5 dan 10
Olimpiade sains internasional tahun 2011 untuk bidang (C) 4 dan 11
fisika, matematika, biologi, kimia dan informatika ke (D) 3 dan 12
sejumlah negara penyelenggara. (2) ketika mengirim tim-tim (E) 2 dan 15
tersebut, Fasli Jalal menyampaikan bahwa para peserta
Olimpiade diseleksi dengan ketat mulai dari sekolah 25. Kalimat yang tidak efektif ditemukan pada...
masing-masing sampai ke tingkat regional. (A) 2 dan 6
(3) Jadi, mereka yang terpilih benar-benar memiliki (B) 3 dan 7
kemampuan yang dapat diandalkan dan berpeluang untuk (C) 5 dan 9
meraih medali. (4) cakupan asal daerah peserta semakin (D) 8 dan 14
merata, mulai dari kawasan barat sampai timur nusantara, (E) 10 dan 15
yang menunjukkan semakin meratanya prestasi peserta didik
di Indonesia. (5) “Indonesia tidak henti-hentinya 26. Rangkuman manakah yang paling tepat untuk teks di
menunjukkan kebolehan siswa di berbagai forum atas ?
terhormat”, katanya. (6) Pada bidang fisika Tim Olimpiade (A) Tim Olimpiade Sains Indonesia yang meliputi
Fisika Indonesia (TOFI) akan berlaga di olimpiade fisika empat bidang akan kembali mengikuti olimpiade
internasional atau International Physics Olympaid (IPHO) dengan mengirimkan siswa-siswa SMA ke
ke-42 di Bangkok. (7) Pada bidang matematika Indonesia berbagai negara di dunia.
mengirimkan enam siswa ke International Mathematical (B) Tim Olimpiade Sains Indonesia akan mengirimkan
Olympaid (IMO) ke-52 yang diselenggarakan di para siswa SMA mengikuti olipiade dalam empat
Amsterdam. (8) Tim-tim ini ditargetkan meraih 2 medali bidang ke berbagai negara dengan harapan dapat
perak dan 4 medali perunggu, sedang tahun-tahun lalu tim memenangkan medali.
IMO Indonesia berada di ranking ke-30 dari 96 negara dan (C) Ada empat bidang dalam Tim Olimpiade sains
berhasil menyabet 1 medali perak, 4 medali perunggu, dan 1 Indonesia yang akan dikirimkan ke berbagai negara
honorable mention. (9) pada bidang Biologi, tim akan untuk mengikuti olimpade yaitu mencakup fisika,
berlaga di arena kompetisi International Biology Olympaid matematika, biologi, dan komputer.
(IBO) ke-22 di Taipei, Taiwan. (10) Indonesia kini (D) Tim Olimpiade Sains Indonesia yang dikirim ke
mengirimkan 4 siswa yang ditargetkan meraih medali, dan berbagai negara untuk mengikuti olimpiade empat
pada ajang tahun lalu, tim IBO meraih 2 medali emas dan 2 bidang adalah mencakup bidang fisika, mematika,
perunggu. (11) Pada bidang informatika Tim Olimpiade bioligi, dan komputer.
Komputer Indonesia (TOKI) akan berkompetisi di (E) Pengiriman Tim Olimpiade Sains Indonesia ke
International Olympaid in Informatics (IOI) ke-23 di berbagai negara untuk mengikuti olimpiade
Pattaya.(12) Pada penyelenggaraan IOI ke-22 di Kanada,
diharapkan mempu memenangkan banyak medali (E) Dampak kelestarian sungai adalah air bersih tetap
agar dapat mendongkrak harga diri bangsa. tersedia dan kesehatan masyarakat tetap terjaga.

Bacalah teks berikut untuk menjawab soal nomor 27 29. Manakah simpulan yang paling tepat untuk teks di atas?
sampai dengan nomor 30! (A) Sumber pencemaran sungai adalah segala kegiatan
Tidak diragukan lagi bahwa yang merupakan sumber hidup manusia yang berkaitan erat dengan proses
pencemaran sungai adalah segala kegiatan hidup manusia produksi dan reproduksi.
yang berkaitan erat dengan proses produksi dan reproduksi. (B) Sumber pencemaran sungai berkaitan erat dengan
Peningkatan jumlah penduduk serta turunnya standar hidup kegiatan manusia, peningkatan jumlah penduduk,
merupakan parameter yang juga ikut menentukan laju serta turunnya standar hidup.
pencemaran sungai. Jika diperhatikan, sungai-sungai yang (C) Peningkatan jumlah penduduk dan turunnya
mengalir di berbagai tempat, kendati kondisinya sangat standar hidup merupakan parameter yang
menjijikkan, sering dipakai untuk mandi, mencuci pakaian, menentukan pencemaran sungai.
mencuci piring, bahkan menggosok gigi. Sementara itu, (D) Kepadatan penduduk dengan standar hidup yang
sungai tersebut secara terus-menerus dijejali dengan memperihatinkan memberikan dampak negatif
timbunan sampah, air limbah, dan tinja penduduk. Makin terhadap lingkungan sungai.
hilir, warna sungai makin hitam dan pekat. (E) Perilaku hidup tidak sehat masyarakat
Apabila sungai tercemar, hal itu akan memberikan dipengaruhi oleh kebiasaan negatif mereka
keleluasaan bagi bibit penyakit untuk berkembang biak.
dalam memanfaatkan air sungai.
Akhirnya, kesehatan masyarakat akan terganggu. Gangguan
kesehatan yang banyak muncul tipus, dan maag.
Untuk menghindari penumpukan sampah yang sulit 30. Ragam kalimat baku terdapat dalam kalimat...
melapuk, perlu dibuat tempat penimbunan khusus di luar (A) Kenaikan harga barang-barang karena
kota. Hal yang baik adalah apabila sampah rumah tangga kenaikan biaya produksi.
sejak dari rumah sudah dilakukan pemisahan antara sampah
(B) Dalam rapat itu membicarakan kenaikan
yang mudah lapuk dan sulit lapuk. Sudah waktunya,sampah
tidak dibuang ke sungai. Selain itu, pembuatan sepetic tank SPP.
di rumah-rumah akan sangat membantu menanggulangi (C) Meninggalnya orang itu sebab kecelakaan.
pencemaran sungai. Khusus air buangan industri, seharusnya (D) Bagi saya soal itu sangat mudah.
sebelum dibuang ke sungai terlebih dahulu diolah melalui (E) Dengan cara ini akan memudahkan
instalasi penjernihan. Hal itu dimaksudkan untuk mencegah
adanya zat-zat yang bersifat racun.
komunikasi antaranggota.
Usaha pencegahan harus ditunjang oleh adanya kesadaran
masyarakat akan pentingnya menjaga kelestarian sungai.
Kesadaran masyarakat modal dasar yang positif bagi
peningkatan perilaku yangg baik dalam memperlakukan
sungai. Sungai yang lestari mencermikan keserasian
lingkungan hidup manusia. Dampaknya adalah ekologi
lingkungan aka menjadi lebih baik, air bersih tetap tesedia,
dan kesehjahteraan masyarakat tetap terjaga.
27. Kelemahan paragraf ke-3 di atas adalah ...
(A) Gagasan utamanya terletak di tengah
(B) Tidak ada kalimat utamanya
(C) Terdapat kalimat sumbang ditengah
(D) Istilah asing tidak dijelaskan artinya
(E) Gagasan utamanya lebih dari satu.

28. Apakah gagasan utama paragraf ke-4 di atas?


(A) Kesadaran masyarakat untuk mencegah
pencemaran sungai akan membantu kelestarian
sungai.
(B) Kesadaran menjadi modal dasar bagi peningkatan
perilaku yang baik dalam memperlakukan sungai.
(C) Perilaku tidak membuang sampah dan tidak
membangun rumah di bantaran sungai membantu
kelestarian sungai.
(D) Kesadaran menciptakan sungai yang lestari
mencerminkan keserasian lingkungan hidup
manusia.
tetapi musim panas adalah saat lebah seharusnya
berkembang, terbang dari bunga ke bunga dengan makanan
berlimpah dan cuaca indah. Tetapi data juga menunjukkan
bahwa, meskipun ada upaya baru-baru ini untuk mengurangi
penggunaan pestisida tertentu - dan dorongan White House
untuk mengurangi kerugian koloni musim dingin hingga
15% selama sepuluh tahun ke depan - kerugian keseluruhan
tampaknya sama buruknya dengan yang biasanya terjadi,
atau bahkan lebih buruk (set data 2015-2016 belum
lengkap).
Di Washington Post, Chelsea Harvey menulis bahwa
ada semakin banyak pemahaman bahwa kehilangan koloni
lebah didorong oleh berbagai faktor, terutama penggunaan
pestisida, penyakit, dan masalah lingkungan. Kerugian yang
begitu tinggi di musim panas menunjukkan bahwa apa pun
yang membunuh lebah dapat menyebabkan masalah serius
bahkan di saat seharusnya lebih sedikit masalah bagi
penyerbuk. Masih terlalu dini untuk mengusir tren tentang
apa yang terjadi pada populasi lebah dari data ini, menurut
Steinhauer. Tetapi menurut USDA, kerugian koloni tahunan
ini perlu dikurangi menjadi sekitar 18,7% jika dapat
dikelola. Jika tidak, kita mungkin perlu mengucapkan
selamat tinggal pada banyak makanan lezat dan sebagian
besar ekonomi dunia.

Bacalah teks berikut untuk menjawab soal No. 1 s.d. 7!


Lebah madu hampir sama pentingnya dengan tanaman
dalam sistem pangan kita. Lebah sebagai polinator
dibutuhkan untuk memproduksi satu pertiga makanan yang
kita makan. Lebah juga menyumbang 17 M dolar US untuk
industri pertanian Amerika Utara setiap tahunnya. Akan
sangat sulit dan mahal untuk mendapatkan kopi tanpa lebah
madu, membuat mereka adalah kunci dari ekonomi global
Dari survei terbaru, ditemukan bahwa lebah madu
dalam masalah serius. Lebah telah berjuang secara umum
selama bertahun-tahun, menghadapi masalah yang
mendatangkan malapetaka pada populasi, seperti gangguan
keruntuhan koloni. Tetapi laporan baru dari Bee Informed
Partnership (BIP), sekelompok peneliti terkemuka yang
didanai oleh United States Department of Agriculture
(USDA) dan National Institute of Food and Agriculture,
sangat memprihatinkan.
"Ketika kami memulai survei, kami fokus pada
kerugian musim dingin karena musim dingin umumnya
dipandang sebagai waktu paling penuh tekanan bagi lebah,"
Nathalie Steinhauer, koordinator survei dan Ph.D. kandidat
dalam entomologi di University of Maryland. Tetapi mereka
mulai melacak hal-hal lebih dekat untuk menilai efek dari
praktik perlebahan yang berbeda dan menjadi sadar akan
beberapa informasi yang mengganggu sebagai hasilnya. 1. Manakah pernyataan yang sesuai dengan
"Tingkat kehilangan musim panas yang tinggi membuat paragraf 2?
kami terkejut," tulis Steinhauer. Tren ini adalah masalah A. Semua survei mengenai lebah madu menghasilkan
besar karena beberapa alasan. Satu, kerugian musim panas data yang memprihatinkan.
sekarang sama buruknya dengan kerugian musim dingin, B. Sebagian peneliti lebah madu didanai oleh USDA
dan National Institute of Food and Agriculture. makanan jika lebah madu sudah tidak ada.
C. Semua survei menyatakan lebah madu dalam E. Seharusnya musim panas merupakan musim untuk
masalah serius. lebah untuk berkembang.
D. Semua peneliti lebah madu didanai oleh USDA dan
National Institute of Food and Agriculture. 6. Pada periode mana terdapat peningkatan tertinggi
E. Sebagian lebah tidak merasakan masalah populasi kematian lebah madu di Amerika Serikat?
yang serius. A. 2009-2010
B. 2010-2011
2. Manakah simpulan paragraf 4? C. 2011-2012
A. Kehilangan koloni dapat ditangani meskipun tidak D. 2012-2013
memperbaiki masalah lingkungan. E. 2013-2014
B. Jika dapat dikelola, kerugian koloni tahunan pada
2017 perlu dikurangi 25% dari tahun 2016. 7. Berapa persen penurunan tertinggi kehilangan koloni
C. Meskipun penggunaan pestisida, penyakit, dan lebah pada musim panas?
masalah lingkungan sudah ditangani, faktor yang A. 6%
menurunkan koloni lebah akan tetap tinggi. B. 10%
D. Masalah lebah seharusnya tetap sama pada musim C. 15%
panas dan musim dingin. D. 19%
E. Jika penurunan koloni lebah madu tidak dapat E. 25%
dikurangi maka ekonomi dunia akan terganggu.
Bacalah teks berikut untuk menjawab soal No. 8 s.d. 14!
3. Manakah simpulan yang sesuai dengan paragraf 3? Pro-kontra vaksinasi di Indonesia kembali merebak
A. Upaya yang sudah dilakukan adalah pengurangan setelah anak dari seorang pesohor terkena campak karena
penggunaan pestisida dan memperbanyak bunga diduga tidak mengikuti program vaksinasi. Silang pendapat
untuk lebah mencari makan. antar kelompok pro vaksin dan antivaksin di dunia maya
B. Musim panas bukanlah waktu yang tepat untuk pun tak terhindarkan. Masing- masing memaparkan
lebah berkembang. argumennya, mulai dari isu keselamatan, efektivitas, agama,
C. Musim dingin dan musim panas adalah waktu bahkan teori konspirasi.
penuh tekanan bagi lebah madu. Vaksinasi dan imunisasi sudah terbukti secara ilmiah
D. Pelacakan dilakukan untuk menilai efek dari sebagai metode yang efektif dalam mencegah penyakit
praktik perlebahan yang berbeda dan mendapat menular khususnya penyakit yang dapat dicegah dengan
informasi yang mengganggu sebagai hasilnya. imunisasi (PD3I). Vaksinasi dan imunisasi diperkirakan
E. White house memerintahkan untuk mengurangi mencegah tiga juta kematian tiap tahunnya. Cara kerja
kerugian koloni musim dingin dan panas. vaksinasi adalah dengan memberikan antigen bakteri atau
virus tertentu yang sudah dilemahkan atau dimatikan dengan
tujuan merangsang imun tubuh untuk membentuk antibodi.
4. Manakah di bawah ini yang mendukung pernyataan Dengan terbentuknya antibodi, maka anak akan memiliki
Nathalie Steinhauer? kekebalan terhadap penyakit tertentu, walau tidak 100%,
A. Musim dingin adalah waktu paling penuh tekanan namun jauh lebih baik dibandingkan anak tanpa vaksinasi.
bagi lebah Dilihat dari peraturan perundang-undangan, vaksin
B. Musim panas sama dengan musim dingin yaitu dan imunisasi wajib diberikan oleh pemerintah dan
memiliki tekanan yang berat bagi lebah. sepatutnya didukung oleh seluruh pihak. Hal ini sesuai
C. Kehilangan koloni lebah yang cukup tinggi sudah dengan UU nomor 36 tahun 2009 tentang kesehatan pasal
diprediksikan sejak 2013. 130 yang berbunyi "pemerintah wajib memberikan
D. Kerugian akibat kehilangan koloni lebah sudah imunisasi lengkap kepada setiap bayi dan anak". Sebaliknya,
diatasi dengan penurunan penggunaan pestisida. pelarangan pemberian vaksin kepada anak dapat disamakan
E. Beberapa faktor dapat mendorong hilangnya koloni dengan bentuk penelantaran dan pelanggaran akan hak anak.
lebah, yaitu penggunaan pestisida, penyakit, dan Adapun pelanggaran ini harusnya dikenakan sanksi pidana
masalah lingkungan. sebagaimana yang tercantum dalam UU nomor 23 tahun
2003 tentang perlindungan anak yang berbunyi
5. Manakah yang tidak sesuai dengan teks atau grafik? "penelantaran terhadap anak yang mengakibatkan anak
A. Kehilangan koloni lebah pada musim panas baru mengalami sakit atau penderitaan baik fisik, mental, maupun
mulai diteliti tahun 2010. sosial" "dipidana dengan pidana penjara paling lama lima
B. Pada tahun 2014, kehilangan lebah pada musim tahun dan/atau denda paling banyak Rp 100.000.000".
panas lebih tinggi dari musim dingin. Diperlukan upaya komprehensif dan lintas sektor guna
C. Pengurangan penggunaan pestisida cukup untuk mendorong program imunisasi dan vaksinasi di masyarakat.
mengurangi kerugian akibat kehilangan koloni Upaya dilakukan dengan mengedukasi masyarakat dan
lebah. membuka ruang dialog mengenai hal- hal berkaitan dengan
D. Sulit untuk mendapatkan kopi dan beberapa vaksinasi. Hal ini dilakukan utamanya oleh tenaga kesehatan
dan kader. Kerja sama lintas sektor dengan tokoh agama D. Kerja sama tenaga kesehatan dengan pemuka
juga diperlukan guna memberikan keamanan dan agama dan perbaikan akses imunisasi merupakan
ketenangan kepada masyarakat yang ingin melakukan upaya untuk mendorong program imunisasi.
vaksinasi. Selain itu akses untuk vaksinasi serta cakupan E. Perbaikan akses vaksin adalah kunci dari
produk vaksinasi juga secara perlahan perlu ditingkatkan berjalannya program imunisasi.
demi menjamin kesehatan masyarakat Indonesia secara
umum dan anak Indonesia secara khusus. 11. Manakah pernyataan yang mendukung kalimat
pertama paragraf keempat?
A. Vaksin sangat penting karena dapat meningkatkan
kekebalan anak terhadap penyakit menular.
B. Vaksinasi anak merupakan kewajiban pemerintah
saja.
C. Cakupan imunisasi Indonesia pada taun 2013 dan
2018 masih jauh dari target.
D. Kerja sama dengan tokoh agama tidak terlalu
diperlukan karena tidak akan mengubah persepsi
masyarakat.
E. Cakupan imunisasi dasar lengkap untuk anak 12-
23 bulan di Indonesia sudah cukup.

12. Manakah pernyataan yang SALAH di bawah ini?


8. Berdasarkan paragraf 2, manakah pernyataan di bawah
A. Isu keselamatan dan efektivitas merupakan hal
yang BENAR?
yang banyak dipertanyakan oleh masyarakat.
A. Tidak ada penyakit menular yang dapat dicegah
B. Vaksin merupakan pemberian antigen dari bakteri
dengan vaksin dan imunisasi.
dan virus kepada anak sehingga tubuh dapat
B. Semua penyakit menular dapat dicegah dengan
memproduksi antibodi.
vaksin dan imunisasi.
C. Bagi orang tua yang melarang anaknya
C. Sebagian penyakit menular tidak dapat dicegah
mendapatkan imunisasi bisa terkena hukuman
dengan vaksin maupun imunisasi.
pidana penjara sampai lima tahun dan/atau denda
D. Tidak ada penyakit menular yang tidak dapat
paling banyak Rp100.000.000,00.
dicegah dengan vaksin dan imunisasi.
D. Upaya yang dilakukan masih belum berhasil
E. Semua penyakit menular tidak dapat dicegah dengan
meningkatkan cakupan vaksinasi di Indonesia.
vaksin maupun imunisasi.
E. Semua provinsi di Indonesia masih memerlukan
upaya untuk meningkatkan cakupan vaksinasi.
9. Simpulan apa yang dapat ditarik dari paragraf 3?
A. Jika orang tua menolak memberikan vaksin dan
13. Berapakah selisih antara target cakupan imunisasi
imunisasi maka akan terkena sanksi pidana penjara 5
pada 2019 dan rata-rata cakupan imunisasi Indonesia
tahun dan/atau denda Rp100.000.000,00.
pada 2018?
B. Jika vaksin dan imunisasi tidak diberikan maka sama
A. 32,6%
saja dengan penelantaran dan pelanggaran akan hak
B. 57,9%
anak.
C. 59,2%
C. Jika orang tua memberikan vaksin kepada anaknya
D. 35,1%
maka dia dapat dihukum dengan UU no. 23 tahun
E. 46,7%
2003.
D. Jika pemerintah tidak membuat undang- undang
14. Manakah provinsi dengan peningkatan cakupan
mengenai kewajiban vaksinasi dan imunisasi anak
imunisasi paling tinggi dari tahun 2013 ke tahun
maka akan semakin banyak kelompok antivaksin.
2018?
E. Jika vaksin bukanlah hal penting maka pemerintah
A. Papua Barat
tidak akan mengatur mengenai vaksinasi dan
B. Aceh
imunisasi.
C. Riau
D. Bali
10. Berdasarkan paragraf 4, manakah simpulan di bawah
E. Kalimantan Selatan
ini yang PALING MUNGKIN benar?
A. Upaya yang harus dilakukan adalah edukasi oleh
Bacalah teks berikut untuk menjawab soal No. 15 s.d. 21!
kader atau memperbaiki akses vaksin.
(1) Pada Sabtu, 30 Maret 2019, Kementerian Pertanian
B. Hanya kader yang mengemban tugas mengedukasi
(Kementan) menggelar pemusnahan benih induk jagung
masyarakat mengenai pentingnya vaksinasi.
hibrida dan padi yang mengandung hama penyakit bakteri
C. Edukasi oleh kader dan pemuka agama diperlukan
pseudomonas syringae di Balai Besar Karantina Soekarno
guma memberikan keamanan dan ketenangan bagi
Hatta Kementerian Pertanian, Tangerang. (2) Sebanyak 6
masyarakat.
ton benih jagung dan 2 kilogram benih padi dimusnahkan.
(3) Benih jagung tersebut berasal dari India dan diimpor E. Pemusnahan merupakan satu-satunya jalan
oleh PT Metahelix Lifescience sedangkan benih padi
dibawa dari Jepang. (4) Kepala Badan Karantina 20. Kalimat manakah yang bukan merupakan kalimat
Kementerian Pertanian mengatakan, benih padi dari Jepang efektif?
tersebut selain mengandung organisme perusak juga A. (2)
terbukti tidak memiliki syarat administratif yang lengkap. B. (3)
(5) Direktur Jenderal Tanaman Pangan Kementerian C. (5)
Pertanian, Sumardjo Gatot Irianto, mengatakan benih D. (7)
jagung dan padi tersebut telah terbukti mengandung E. (9)
Organisme Penganggu Tumbuhan Karantina (OPTK)
golongan A1. (6) OPTK golongan A1 merupakan golongan 21. Kalimat perbaikan dari kalimat (13) adalah…
OPTK yang belum terdapat di Indonesia. (7) Benih jagung A. Gatot menambahkan bahwa apabila lolos dari
dan padi tersebut merupakan benih golongan F1 atau pengawasan, masuknya benih jagung dan padi
hibrida (benih turunan pertama). (8) Dalam penggunaannya, tersebut akan menyebabkan kerugian yang besar.
benih tersebut digunakan sebagai benih induk untuk B. Gatot menambahkan bahwa apabila benih jagung
ditanam kembali. (9) Untuk itu, Gatot menyebut bahwa dan padi tersebut berhasil lolos pengawasan, akan
pemusnahan merupakan satu-satunya jalan sebab menyebabkan kerugian yang besar.
pengendaliannya akan sulit dilakukan. (10) Di Indonesia, C. Gatot menambahkan bahwa jika benih jagung dan
organisme berbahaya ini belum ada. (12) Upaya padi tersebut berhasil lolos pengawasan, maka
pengendaliannya pun belum ada. (13) Gatot menambahkan akan menyebabkan kerugian yang besar.
bahwa apabila benih jagung dan padi tersebut berhasil lolos D. Gatot menambahkan bahwa apabila berhasil lolos
pengawasan, maka akan menyebabkan kerugian yang besar. pengawasan, maka benih jagung dan padi tersebut
(14) Namun, ia tak memerinci perkiraan jumlah akan menyebabkan kerugian yang besar.
kerugiannya. E. Gatot menambahkan bahwa apabila benih jagung
dan padi tersebut berhasil lolos dari pengawasan,
15. Kata yang penulisannya tidak mengikuti kaidah ejaan maka akan menyebabkan kerugian yang besar.
terdapat pada kalimat… Bacalah teks berikut untuk menjawab soal 22 s.d 28!
A. (5) D. (2) (1) Balai Penyelidikan dan Pengembangan Teknologi
B. (4) E. (1) Kebencanaan Geologi (BPPTKG) menyebutkan bahwa
C. (3) Gunung Merapi di perbatasan Daerah Istimewa Yogyakarta
dan Jawa Tengah dua kali meluncurkan awan panas
16. Pada kalimat nomor berapa kesalahan tanda baca guguran pada Senin pagi. (2) Melalui akun Twitter
ditemukan? resminya, BPPTKG menyatakan luncuran awan panas
A. (2) D. (8) guguran Merapi teramati pada pukul 06.17 WIB dengan
B. (4) E. (9) jarak luncur 1.000 meter
C. (6) selama 104 detik ke arah Kali Gendol. (3) Sedangkan awan
panas guguran kedua teramati keluar dari Gunung Merapi
17. Ide pokok tulisan di atas adalah… pada pukul 08.58 WIB dengan jarak luncur 900 meter ke
A. Pemusnahan benih jagung dan padi dari India. arah hulu Kali Gendol selama 90 detik. (4) Berdasarkan
B. Hama padi mengancam kesehatan manusia. pengamatan BPPTKG sejak pukul 00:00 WIB hingga 06:00
C. Benih jagung mengandung organisme WIB, selain awan panas guguran juga teramati satu kali
pengganggu tumbuhan. guguran lava pijar ke hulu Kali Gendol dengan jarak luncur
D. Pemusnahan benih jagung dan padi yang 450 meter.
mengandung hama penyakit. (5) Pada periode itu, asap kawah tidak teramati. (6)
E. Pengendalian benih jagung dan padi yang Cuaca di gunung itu berawan, mendung, dan hujan. (7)
mengandung hama penyakit. Angin bertiup lemah ke arah timur dengan suhu udara 16—
22.1 derajat celsius, kelembaban udara 63—89 persen, dan
18. Kata administratif pada kalimat (4) bermakna… tekanan udara 628—708.3 mmHg. (8) Selain itu, terekam
A. dokumen pendukung juga satu kali gempa awan panas dengan amplitudo 60 mm
B. surat kontrak selama 108,1 detik, tujuh kali gempa guguran dengan
C. tata kelola amplitudo 5—27 mm selama 27,7—65,9 detik, satu kali
D. seleksi gempa hembusan dengan amplitudo 20 mm selama 28 detik,
E. tata laksana dua kali gempa frekuensi rendah dengan amplitudo 5—15
mm selama 19,3—28,5 detik, dan satu kali gempa fase
19. Gagasan utama kalimat (9) dalam paragraf di atas banyak dengan amplitudo 8 mm selama 9,2 detik.
adalah… (9) Hingga saat ini, BPPTKG mempertahankan status
A. Gatot menyebut Gunung Merapi pada Level II atau Waspada. (10) Untuk
B. Gatot menyebut pemusnahan sementara, BPPTKG tidak merekomendasikan kegiatan
C. Gatot menyebut pemusnahan satu-satunya jalan pendakian kecuali untuk kepentingan penyelidikan dan
D. Pemusnahan satu-satunya jalan penelitian yang berkaitan dengan mitigasi bencana.
(11) BPPTKG mengimbau warga tidak melakukan E. rentang waktu
aktivitas dalam radius tiga Km dari puncak Gunung Merapi.
(12) Sehubungan dengan kejadian guguran awan panas yang Tulisan berikut diikuti oleh enam butir pertanyaan.
jarak luncurnya semakin jauh, BPPTKG mengimbau warga Pertimbangkan apakah kata atau kalimat pada setiap nomor
yang tinggal di kawasan alur Kali Gendol meningkatkan bercetak tebal TIDAK PERLU DIPERBAIKI (A) atau
kewaspadaan. (13) Masyarakat juga diminta tidak diganti dengan pilihan lain yang tersedia (B, C, D, atau E).
terpancing isu-isu mengenai erupsi Gunung Merapi yang
tidak jelas sumbernya dan tetap mengikuti arahan aparat Sekretaris Dinas Pariwisata dan Ekonomi Kreatif
pemerintah daerah atau menanyakan langsung ke Pos (Disparekraf), Provinsi Nusa Tenggara Timur (NTT), Beny
Pengamatan Gunung Merapi, media sosial BPPTKG, atau Wahon mengatakan akan berupaya maksimal
ke kantor BPPTKG. mempertahankan28 Manggarai Barat sebagai satu- satunya
habitat29 komodo di dunia. "Kasus perdagangan komodo ini
22. Pikiran utama teks di atas adalah… memang bisa mengganggu, tetapi pemerintah akan
A. Guguran awan panas Merapi mengambil langkah-langkah yang diperlukan untuk tetap
B. Gempa kawah Merapi mempertahankan Manggarai barat sebagai satu-satunya
C. Aktivitas Gunung Merapi habitat komodo di dunia," ujar Beny.
D. Isu-isu erupsi Gunung Merapi Ia menjelaskan langkah-langkah yang perlu diambil.
E. Kegiatan pendakian Gunung Merapi Misalnya dari aspek pengelolaan, perlu ada kolaborasi yang
sinergis antara Kementerian Kehutanan dengan Lingkungan
23. Judul untuk teks di atas yang paling tepat adalah… Hidup, Pemerintah Provinsi NTT, dan Pemerintah
A. Status Gunung Merapi Ditingkatkan Kabupaten Manggarai Barat. Sinergi ini, terutama dalam
B. Bahaya Pendakian Gunung Merapi ketika Awan konteks pengawasan terhadap lingkungan di sekitar kawasan
Panas Datang Taman Nasional Komodo (TNK) bertujuan agar kasus
C. Erupsi Gunung Merapi pencurian komodo tidak terulang lagi. Langkah lain,
D. Awan Panas Guguran Meluncur dari Merapi adalah meningkatkan promosi yang lebih inten terkait
E. Imbauan BPPKG Mengenai Gunung Merapi keunikan Komodo sebagai binatang purba yang hanya
24. Hal-hal berikut dikemukakan dalam teks di atas, punya habitat khusus di Pulau Komodo dan bukan di
kecuali… wilayah lainnya di dunia.30 Promosi perlu dikemas secara
A. cuaca di Gunung Merapi berawan, mendung, dan profesional.
hujan ketika periode guguran awan panas terjadi Para pengunjung bukan hanya datang melihat fisik
B. status Gunung Merapi ditingkatkan menjadi Level komodo, tapi memahami bagaimana kehidupan komodo
II (Waspada) oleh BPPTKG sebagai binatang purba dengan segala aktivitas hidupnya.
C. BPPTKG meminta masyarakat untuk tidak "Jadi pengunjung yang datang mendapat suguhan yang utuh
terpancing isu-isu erupsi gunung tentang komodo dan tentunya bisa memperoleh kepuasan
D. guguran awan panas dan luncuran lava pijar tersendiri" katanya.
mengarah ke Kali Cendol
E. warga di sekitar Kali Cendol diimbau 28. Mempertahankan
meningkatkan kewaspadaan A. TIDAK PERLU DIPERBAIKI
B. Memertahankan
25. Penggunaan konjungsi yang salah terdapat pada C. Memertahanan
kalimat… D. Pertahankan
A. (1) E. Menyelamatkan
B. (3)
C. (6) 29. Habitat
D. (8) A. TIDAK PERLU DIPERBAIKI
E. (10) B. Lingkungan
C. Ekologi
26. Penggunaan tanda baca yang tidak sesuai PUEBI D. Habitasi
terdapat dalam kalimat… E. Teritori
A. (2)
B. (3) 30. Langkah lain, adalah meningkatkan promosi yang lebih
C. (4) inten terkait keunikan Komodo sebagai binatang purba
D. (7) yang hanya punya habitat khusus di Pulau Komodo dan
E. (8) bukan di wilayah lainnya di dunia.
A. TIDAK PERLU DIPERBAIKI
27. Kata periode pada kalimat (5) bermakna… B. Langkah lain adalah meningkatkan promosi yang
A. Durasi lebih intens terkait keunikan Komodo sebagai
B. Termin binatang purba yang hanya mempunyai habitat
C. Era khusus di Pulau Komodo dan bukan di wilayah
D. Siklus lainnya di dunia.
C. Langkah lain, adalah meningkatkan promosi yang B. Kemenangan Liverpool atas Tottenham Hotspur
lebih inten terkait keunikan Komodo sebagai membuktikan keangkeran Stadion Anfield.
binatang purba yang hanya punya habitat khusus di C. Prestasi anak-anak asuh Juergen Klopp.
pulau Komodo. D. Rekor gemilang pasukan Juergen Klopp.
D. Langkah lain adalah meningkatkan promosi yang E. Dominasi Liverpool atas Tottenham Hotspur.
lebih inten terkait keunikan Komodo sebagai
binatang purba yang hanya punya habitat khusus di 2. Berikut ini yang merupakan kata tidak baku pada teks di
Pulau Komodo. atas adalah…
E. Langkah lain adalah meningkatkan promosi yang A. markas (kalimat 1)
lebih intens terkait keunikan Komodo sebagai B. gemilang (kalimat 3)
binatang purba yang hanya mempunyai habitat C. menelan (kalimat 4)
khusus di Pulau Komodo. D. kasta (kalimat 5)
E. dari pada (kalimat 7)

3. Penggunaan tanda baca yang tidak sesuai PUEBI


terdapat pada kalimat…
A. (5) D. (8)
B. (6) E. (9)
C. (7)

4. Kalimat pada teks di atas yang salah satu penulisan


katanya tidak sesuai dengan PUEBI adalah…
A. (6) D. (3)
B. (5) E. (2)
C. (4)
5. Penulisan bentukan kata yang tidak tepat terdapat pada
kalimat…
A. (2) D. (8)
B. (4) E. (10)
C. (6)
Bacalah teks berikut untuk menjawab soal No. 1 s.d. 5!
(1) Kemenangan atas Tottenham Hotspur pada laga
pekan ke-32 Liga Inggris menjadi bukti keangkeran markas Bacalah teks berikut untuk menjawab soal No. 6 s.d. 9!
Liverpool, Stadion Anfield. (2) The Reds, julukan (1) Walt Disney Co akan melarang pengunjung yang
Liverpool, meraih kemenangan dengan skor 2-1 atas merokok, baik dengan rokok biasa maupun elektronik, di
Tottenham pada Minggu, 23 Maret 2019. (3) Selain area taman bermain Disney World California dan Florida
mengantarkan Liverpool ke puncak klasemen sementara mulai 1 Mei mendatang. (2) Membawa kereta bayi
Premier League, kemenangan atas Tottenham juga berukuran besar di taman main tersebut juga dilarang
menambah rekor gemilang pasukan Juergen Klopp itu. (4) pengunjung.
Liverpool kini tercatat tidak pernah menelan kekalahan (3) Pembatasan ini dirancang untuk menangani
dalam 31 laga Premier League di Anfield selama dua tahun kerumunan pengunjung yang berbondong-bondong masuk
terakhir. (5) Dalam 31 pertandingan kandang di kompetisi ke dalam Disney World. (4) Mereka diperkirakan akan
kasta teratas Inggris itu, The Reds menorehkan 27 masuk ke atraksi baru “Star Wars" di Walt Disney World
kemenangan dan hanya 10 kali imbang. (6) Kali terakhir Orlando, Florida dan di Disneyland di Anaheim, California
Liverpool gagal meraih satu pun angka di markas sendiri pada akhir tahun ini.
adalah ketika dikalahkan 1-2 oleh Crystal Palace pada 23
April 2017. (5) Disney mengungkapkan bahwa mereka akan
(7) Prestasi anak-anak asuh Juergen Klopp tersebut menghapus area merokok di Walt Disney World,
hanya kalah dari pada Chelsea. (8) Chelsea mencatatkan 86 Disneyland, taman air, ESPN Wide World of Sports
laga kandang tanpa kalah di Liga Inggris antara 2004-2008. Complex di Florida, dan distrik perbelanjaan Downtown
(9) Sementara itu, kemenangan ini semakin mengukuhkan Disney di California. (6) Area merokok akan tersedia di luar
dominasi Liverpool atas Tottenham di Premier League. (10) pintu masuk taman dan di hotel- hotel Disney. (7) Selain itu,
Dimulai dari musim 1992—1993, The Reds sudah ukuran kereta dorong akan terbatas pada ukuran lebar 31
mengumpulkan 25 kemenangan berbanding Spurs yang inchi dan panjang 52 inchi. (8) Kereta dorong bayi
hanya 14 kali menang dari 54 pertemuan. (11) Adapun 15 berbentuk seperti gerobak tidak diizinkan. (9) Pembaruan ini
laga sisanya berakhir imbang untuk kedua tim. dimaksudkan untuk memberi pengalaman yang lebih
menyenangkan bagi pengunjung antara lain, mengurangi
1. Gagasan utama teks di atas adalah… arus tamu dan mengurangi kemacetan.
A. Liverpool gagal meraih satu angka di markas sendiri. 6. Gagasan utama kalimat pertama adalah…
A. Walt Disney Co akan melarang pengunjung yang
merokok. sebagai binatang purba dengan segala aktivitas hidupnya.
B. Walt Disney Co melarang pengunjung merokok di "Jadi pengunjung yang datang mendapat suguhan yang utuh
area taman bermain. tentang komodo dan tentunya bisa memperoleh kepuasan
C. Walt Disney Co melarang pengunjung. tersendiri" katanya.
D. Walt Disney Co melarang pengunjung merokok
elektronik.
E. Larangan merokok mulai Mei mendatang. 10. Manggarai barat
A. TIDAK PERLU DIPERBAIKI
7. Yang merupakan kalimat tidak efektif pada teks di atas B. manggarai barat
adalah… C. barat Manggarai
A. (5) D. Manggarai Barat
B. (4) E. Manggarai bagian Barat
C. (3)
D. (2) 11. dengan
E. (1) A. TIDAK PERLU DIPERBAIKI
B. Dari
8. Pengunaan kata yang tidak tepat pada teks di atas C. Dan
adalah… D. Serta
A. kerumunan pada kalimat (3) E. Sekaligus
B. atraksi pada kalimat (4)
C. area pada kalimat (5) 12. Bukan hanya
D. inchi pada kalimat (7) A. TIDAK PERLU DIPERBAIKI
E. pembaruan pada kalimat (8) B. Bukan
9. Penggunaan tanda koma yang tidak tepat terdapat pada C. Tidak
kalimat… D. tidak hanya
A. (9) D. (3) E. entah
B. (7) E. (1)
C. (5) 13. tapi
A. TIDAK PERLU DIPERBAIKI
Tulisan berikut diikuti oleh enam butir pertanyaan. B. Tetapi
Pertimbangkan apakah kata atau kalimat pada setiap C. tetapi juga
nomor bercetak tebal TIDAK PERLU DIPERBAIKI (A) D. melainkan
atau diganti dengan pilihan lain yang tersedia (B, C, D, E. melainkan juga
atau E).
Bacalah teks berikut untuk menjawab soal No. 14 s.d.20!
Sekretaris Dinas Pariwisata dan Ekonomi Kreatif Seleksi Bersama Masuk Perguruan Tinggi Negeri
(Disparekraf), Provinsi Nusa Tenggara Timur (NTT), Beny (SBMPTN) merupakan salah satu dari dua jalur tes masuk
Wahon mengatakan akan berupaya maksimal perguruan tinggi negeri yang diselenggarakan Kementerian
mempertahankan Manggarai Barat sebagai satu- satunya Riset, Teknologi, dan Pendidikan Tinggi (Kemenristekdikti)
habitat komodo di dunia. "Kasus perdagangan komodo ini melalui Lembaga Tes Masuk Perguruan Tinggi (LTMPT).
memang bisa mengganggu, tetapi pemerintah akan Jalur tes lainnya adalah Seleksi Nasional Masuk Perguruan
mengambil langkah-langkah yang diperlukan untuk tetap Tinggi Negeri (SNMPTN). Pada jalur SNMPTN, para
mempertahankan Manggarai barat10 sebagai satu-satunya peserta diseleksi melalui penelusuran prestasi akademik
habitat komodo di dunia," ujar Beny. dengan menggunakan rapor dan portofolio akademik. Rapor
Ia menjelaskan langkah-langkah yang perlu diambil. yang diukur yaitu dari semester satu hingga semester lima
Misalnya dari aspek pengelolaan, perlu ada kolaborasi yang bagi siswa SMA/SMK/MA dengan masa belajar tiga tahun.
sinergis antara Kementerian Kehutanan dengan11 Berbeda dengan SNMPTN, para peserta di SBMPTN
Lingkungan Hidup, Pemerintah Provinsi NTT, dan diseleksi melalui ujian tulis. Sejak 2016, SBMPTN
Pemerintah Kabupaten Manggarai Barat. Sinergi ini, memperkenalkan Ujian Tulis Berbasis Komputer (UTBK)
terutama dalam konteks pengawasan terhadap lingkungan di dalam proses seleksi. PTN yang termasuk dalam SBMPTN
sekitar kawasan Taman Nasional Komodo (TNK) bertujuan menyediakan kuota minimal 40 persen dan 20 persen di
agar kasus pencurian komodo tidak terulang lagi. Langkah SNMPTN dari total kuota masing-masing PTN. Meskipun
lain, adalah meningkatkan promosi yang lebih inten terkait menyediakan kuota yang lebih besar daripada SNMPTN,
keunikan Komodo sebagai binatang purba yang hanya Kemenristekdikti menilai SBMPTN dinilai lebih bergengsi
punya habitat khusus di Pulau Komodo dan bukan di ketimbang SNMPTN. ―Seleksi melalui jalur rapor atau
wilayah lainnya di dunia. Promosi perlu dikemas secara SNMPTN memang kurang menarik buat siswa karena
profesional. sebagian berpendapat kalau SBMPTN lebih
Para pengunjung bukan hanya12 datang melihat fisik mempertaruhkan kemampuan dan bergengsi bagi siswa
komodo, tapi13 memahami bagaimana kehidupan komodo untuk seleksi masuk PTN,‖ ujar Direktur Kemahasiswaan
Ditjen Pembelajaran dan Kemahasiswaan Kemenristekdikti
Didin Wahidin kepada Antara di Jakarta, Kamis (8/3/2018). C. Terdapat satu jalur tes masuk perguruan tinggi
Namun demikian, pertanyaannya adalah berapa negeri yaitu Seleksi Nasional Masuk Perguruan
banyak pendaftar melalui SBMPTN tahun-tahun Tinggi Negeri (SNMPTN).
sebelumnya, dan program studi apa saja yang paling D. Terdapat satu jalur tes masuk perguruan tinggi
diminati oleh para peserta? Pada SBMPTN 2014, peserta negeri yaitu Seleksi Bersama Masuk Perguruan
tercatat sebanyak 664.509 yang terdiri atas 583.590 Tinggi Negeri (SBMPTN).
peserta reguler dan 80.919 peserta bidik misi. Para peserta E. Terdapat dua jalur tes masuk perguruan tinggi
tersebut memperebutkan tempat di 64 PTN yang turut negeri yang salah satunya adalah Seleksi Bersama
serta dengan daya tampung sebesar 91.294 mahasiswa Masuk Perguruan Tinggi Negeri (SBMPTN).
baru. Jumlah tersebut meningkat pada 2015 menjadi
764.185 peserta yang memperebutkan 115.788 kursi 15. Berdasarkan paragraf 2, maka simpulan yang BENAR
mahasiswa baru di 74 PTN. Para peserta terdiri atas 763.499 adalah…
peserta reguler dan 90.686 peserta bidikmisi. Pada 2017 dan A. SBMPTN dinilai lebih bergengsi meskipun
2018, meskipun daya tampung yang disediakan tidak kuotanya sama besar dibandingkan SNMPTN.
berubah sebanyak 128.085 kursi mahasiswa baru, tetapi B. SBMPTN dinilai tidak bergengsi meskipun
peserta pada SBMPTN 2018 meningkat 7.90 persen menjadi kuotanya lebih kecil dibandingkan SNMPTN.
860.001 peserta dibandingkan SBMPTN 2017 sebesar C. SBMPTN dinilai tidak bergengsi meskipun
kuotanya lebih besar dibandingkan SNMPTN.
D. SBMPTN dinilai lebih bergengsi meskipun
kuotanya lebih besar dibandingkan SNMPTN.
E. SBMPTN dinilai lebih bergengsi meskipun
kuotanya lebih kecil dibandingkan SNMPTN.

16. Berdasarkan paragraf 3, maka simpulan yang PALING


MUNGKIN BENAR adalah…
A. Peserta SBMPTN 2018 mengalami peningkatan
daya tampung ketimbang dengan jumlahnya.
B. Jumlah peserta SBMPTN 2018 mengalami
penurunan walaupun daya tampungnya naik.
C. Jumlah peserta SBMPTN 2018 mengalami
penurunan walaupun daya tampungnya tidak naik.
D. Jumlah peserta SBMPTN 2018 mengalami
peningkatan walaupun daya tampungnya tidak naik.
E. Jumlah peserta SBMPTN 2018 mengalami
peningkatan walaupun daya tampungnya naik.

17. Berdasarkan grafik, pada tahun berapakah terjadi


penurunan jumlah peserta SBMPTN dibandingkan
dengan tahun 2015…
A. 2015
B. 2016
C. 2017
D. 2018
E. 2019

18. Pernyataan yang sesuai dengan wacana dan tabel


berikut, kecuali…
797.023 peserta. A. Jika dibandingkan dengan tahun sebelumnya, daya
tampung SBMPTN tahun 2018 dalam situasi
14. Berdasarkan paragraf 1, maka simpulan yang BENAR stagnan.
adalah… B. Pada tahun 2016, terjadi penurunan jumlah peserta
A. Tidak ada jalur tes masuk perguruan tinggi negeri dan daya tampung SBMPTN.
yang salah satunya Seleksi Bersama Masuk C. Daya tampung SBMPTN selalu naik kecuali pada
Perguruan Tinggi Negeri (SBMPTN). tahun 2018.
B. Terdapat banyak jalur tes masuk perguruan tinggi D. Jumlah peserta SBMPTN setiap tahunnya
selain Seleksi Nasional Masuk Perguruan Tinggi mengalami kenaikan kecuali pada tahun 2016.
Negeri (SNMPTN) dan Seleksi Bersama Masuk E. Jumlah peserta SBMPTN tahun 2015 merupakan
Perguruan Tinggi Negeri (SBMPTN). ketiga tertinggi dalam rentan 2014-2018.
19. Pada tahun 2018, perbandingan antara daya tampung lain pihak, negara tetangga justru yang lebih sigap,‖ katanya
dan jumlah peserta SBMPTN sekitar berapa persen? kepada Tirto.
A. 25%
B. 20%
C. 15%
D. 10%
E. 5%

20. Kenaikan jumlah terbesar peserta SBMPTN


dibandingkan dengan tahun sebelumnya terjadi pada
tahun…
A. 2014
B. 2015
C. 2016
D. 2017
E. 2018

Bacalah teks berikut untuk menjawab soal No. 21 s.d.27!


Jokowi mengatakan investasi sangat penting demi
membuka lapangan kerja baru. Untuk itu, segala hal yang
menghambat investasi harus dipangkas, mulai dari perizinan
yang lambat, berbelit-belit hingga persoalan pungli. ―Hati- 21. Berdasarkan paragraf 1, maka simpulan yang BENAR
hati, ke depan saya pastikan akan saya kejar, akan saya adalah…
kontrol, akan saya cek, dan akan saya hajar kalau A. Semua menteri tidak ikut dalam rapat terbatas
diperlukan.‖ Pasalnya, pada tahun sebelumnya, Jokowi yang diadakan oleh Jokowi untuk membahas
menunjukkan kekecewaannya terkait investasi saat masalah investasi.
menggelar rapat terbatas di Kantor Presiden pada 19 Juni B. Semua menteri ikut dalam rapat terbatas yang
2019. Dalam rapat terbatas itu, presiden ditemani sejumlah diadakan oleh Jokowi untuk membahas masalah
menteri dan pimpinan lembaga. Terdapat beberapa menteri investasi.
yang hadir termasuk Menko Bidang Perekonomian Darmin C. Sebagian menteri ikut dalam rapat terbatas yang
Nasution, Menteri Keuangan Sri Mulyani, Menteri diadakan oleh Jokowi untuk membahas masalah
Perindustrian Airlangga Hartanto, Menteri Perdagangan investasi.
Enggartiasto Lukita, Menteri Perhubungan Budi Karya D. Sebagian menteri yang tidak ikut dalam rapat
Sumadi. terbatas yang diadakan oleh Jokowi diberikan
Kemarahan Jokowi sesungguhnya beralasan. Hal ini sanksi.
dikarenakan investasi asing atau biasa disebut dengan E. Sebagian menteri yang ikut dalam rapat terbatas
Penanaman Modal Asing (PMA) yang masuk ke Indonesia yang diadakan oleh Jokowi diberikan sanksi akibat
selama periode pertama pemerintahannya tidak begitu masalah investasi.
bagus. Tren pertumbuhan PMA bisa dikatakan stagnan.
Rata-rata pertumbuhan nilai PMA di era Jokowi hanya naik 22. Berdasarkan paragraf 3, maka simpulan yang BENAR
1 persen per tahun. Capaian mantan Gubernur DKI Jakarta adalah…
ini jauh lebih rendah saat Indonesia dipimpin Susilo A. Rasio nilai investasi di Indonesia akan meningkat
Bambang Yudhoyono (SBY) yang rata-rata apabila SBY menjabat lagi.
pertumbuhannya sekitar 18 persen per tahun. B. Birokrasi yang berbelit-belit, kebijakan yang tidak
Pada awal 2010 di mana masa itu merupakan awal konsisten dapat diselesaikan apabila pemerintah
periode kedua kepemimpinan SBY, realisasi PMA mencapai mampu menaikkan rasio nilai investasi di
16,21 miliar dolar AS. Di akhir jabatan SBY pada 2014, Indonesia.
realisasi PMA tercatat mencapai 28,53 miliar dolar AS, atau C. Rasio nilai investasi di Indonesia akan menurun
naik 76 persen. Sementara itu, Jokowi pada tahun apabila pemerintah mampu menyelesaikan
pertamanya sebagai presiden mencatatkan angka realisasi masalah birokrasi yang berbelit-belit, kebijakan
PMA sebesar 29,27 miliar dolar AS pada 2015. Kemudian yang tidak konsisten dan lain sebagainya.
pada 2018, realisasi PMA mencapai 29,30 miliar dolar AS D. Rasio nilai investasi di Indonesia akan meningkat
atau naik tipis sebesar 0.1 persen. Menurut Akademisi apabila pemerintah mampu menyelesaikan
Fakultas Ekonomi Universitas Indonesia (UI) Fithra Faisal masalah birokrasi yang berbelit-belit, kebijakan
Hastiadi, jika pemerintah dalam hal ini Jokowi mampu yang tidak konsisten dan lain sebagainya.
menyelesaikan persoalan klasik di Indonesia, maka E. Rasio nilai investasi di Indonesia akan meningkat
sangatlah memungkinkan apabila nilai investasi di Indonesia apabila masalah yang dihadapi tidak klasik.
bertambah dengan pesat seperti pada jaman Presiden SBY.
―Persoalannya masih itu-itu saja, birokrasi yang berbelit-
belit, kebijakan yang tidak konsisten dan lain sebagainya. Di 23. Berdasarkan paragraf 2, maka simpulan yang PALING
MUNGKIN benar adalah… B. Para mahasiswa yang akan mengikuti tes
A. Kemarahan Jokowi diakibatkan investasi asing kemampuan bahasa Jepang secara gratis
yang masuk ke Indonesia selama periodenya diharapkan mendaftarkan diri di ruang kantor.
stagnan. C. Para ahli matematika merasa kagum, ketika
B. Kemarahan Jokowi diakibatkan PMA yang masuk mengetahui perhitungan lebah yang sangat cermat.
ke Indonesia selama periode pertamanya tidak D. Seorang pedagang di pasar mengatakan, sambil
mengalami stagnasi. melayani pelanggannya, bahwa kenaikan harga
C. Kemarahan Jokowi karena stagnasi yang dilakukan cabai dan bawang merah sudah berlangsung
oleh para menteri yang menghadiri rapat terbatas. selama tiga tahun.
D. Rata-rata pertumbuhan nilai PMA di era Jokowi E. Para penyelenggara negara yang dalam batas
hanya naik 1 persen per tahun yang diakibatkan waktu tertentu tidak melaporkan harta
oleh stagnasi. kekayaannya ke KPK, akan dikenakan sanksi yang
E. Stagnasi yang di era Jokowi diakibatkan rata- rata berupa denda atau hukuman.
pertumbuhan nilai PMA-nya hanya naik 1 persen
per tahun. 29. Pemakaian kata berimbuhan yang tidak tepat terdapat
dalam kalimat ....
24. Berdasarkan tabel, pada tahun berapakah terjadi A. Kekeliruan penyimpanan memori di otak bisa
kenaikan jumlah proyek terbesar dibandingkan dengan terjadi setelah mendengar atau membaca banyak
tahun sebelumnya selama periode 2009- 2018? informasi.
A. 2012 B. Pemecahan ombak untuk mengurangi abrasi dapat
B. 2013 dilakukan dengan beberapa cara.
C. 2014 C. Sebuah organisasi memberikan pemahaman yang
D. 2015 bagus tentang bakau kepada Kelompok Cinta
E. 2016 Lingkungan.
D. Penanaman pohon jati dilakukan di lahan seluas
25. Pernyataan yang sesuai dengan wacana dan tabel dua hectare.
tersebut, kecuali… E. Penambahan jumlah warga yang dengan sadar ikut
A. Pada tahun 2010, terjadi kenaikan sebesar 540.434 menjaga ekosistem cukup besar
tenaga kerja dari tahun sebelumnya.
B. Terjadi penurunan tenaga kerja penanaman modal 30. Gabungan kata bercetak miring yang bukan kata
asing sebesar 116.565 pada tahun 2014 majemuk terdapat dalam kalimat…
C. Pada tahun 2013, terjadi kenaikan persentase A. Kurikulum terbaru mengamanatkan agar setiap
tenaga kerja sebesar 47 persen. mata pelajaran di jenjang pendidikan dasar sampai
D. Pada awal 2010, nilai PMA di Indonesia mencapai sekolah tingkat atas mengandung pendidikan
16.21 miliar dolar AS. karakter.
E. Terjadi kenaikan persentase PMA pada tahun 2010 B. Kapal-kapal modern juga didorong mematuhi
sampai 2014 sebesar 76 persen. aturan sandar, tidak melewati garis batas zona
kapal tradisional.
26. Berdasarkan tabel, NILAI penanaman modal asing C. Pameran pendidikan dan peralatan sekolah paling
YANG PALING MUNGKIN mengalami bergengsi di Indonesia diselenggarakan oleh
kecenderungan stagnasi terjadi pada tahun… pemerintah bersama universitas terkenal.
A. 2010-2011 D. Pembangunan rumah susun model linear
B. 2011-2012 menunjukkan kepedulian pemerintah terhadap
C. 2012-2013 kebutuhan hunian warga ibu kota.
D. 2013-2014 E. Duta besar luar biasa dan berkuasa penuh adalah
E. 2014-2015 pejabat diplomatik yang ditugaskan ke negara lain
untuk mewakili negara.
27. Kenaikan persentase proyek penanaman modal asing
pada tahun 2013 dibandingkan 2010 adalah
A. 87%
B. 136%
C. 144%
D. 212%
E. 231%

28. Penggunaan tanda baca koma (,) yang tepat terdapat


dalam kalimat ....
A. Beberapa mahasiswa yang lolos tes dan memenuhi
persyaratan, akan mendapat beasiswa dari
universitas terkenal di Tokyo.
(D) Diperlebarkan
(E) Mengenyampingkan

05. Kalimat berikut merupakan contoh kalimat baku,


kecuali...
(A) Kita harus dapat membuktikan bahwa kita mampu
melaksanakan tugas ini.
(B) Marilah kita memulai pertemuan ini.

(A) Dalam pertemuan ini kita akan membicarakan


rencana perjalanan wisata ke Bandung.
(B) Kita akan bicarakan lagi masalah ini.
(C) Akan kita diskusikan lagi masalah ini besok.

06. Bagi yang menemukan dompet supaya diserahkan


kepda polisi. Kalimat tersebut dapat dijadikan efektif
dengan...
(A) membuang kata bagi
(B) mengubah kata supaya menjadi harus
(A) mengubah kata diserahkan menjadi menyerahkan
(B) mengubah kalimat tersebut menjadi yang
menemukan dompet harus menyerahkannya kepada
polisi.
(C) mengubah kalimat tersebut menjadi kalimat aktif.
01. Padanan kata efektif adalah...
(A) Mankus
07. Kalimat efektif terdapat pada pilihan...
(B) Sankil
(A) Dilarang masuk, kecuali karyawan.
(C) Tepat
(A) Yang sudah membayar SPP boleh mengikuti ujian.
(D) Akibat
(B) Dalam tata bahasa mengenal ilmu tentang bunyi
(E) Hasil
bahasa.
(C) Untuk pelaksanaan pembangunan ini memerlukan
02. Bagi yang tidak bersepatu dilarang memasuki ruangan
karyawan yang terampil.
ini. Kalimat di atas tidak baku karena...
(D) Saya akan baca surat itu.
(A) pemilihan kata-katanya tidak tepat
(B) mengandung bentuk pasif yang tidak tepat
08. Kalimat yang menggunakan kata baku adalah...
(C) kalimat tidak bervariasi
(A) Dia cuma mengatakan bahwa dia tidak mengetahui
(D) fungsi subjek yang tidak jelas
hal itu.
(E) diawali preposisi
(B) Sekalipun risiko yang dihadapinya berat, ia tetap
03. Penulisan kata yang salah terdapat dalam kalimat... tegar.
(A) Kita tidak boleh terpancing oleh hasutan-hasutan (C) Akhirnya mereka menyadari hakekat belajar
yang dilakukan pihak-pihak yang tidak berbagai bahasa.
proorganisasi. (D) Fikirannya risau sejak adiknya berangkat ke luar
(B)Jabatan Ketua KPK tidak lama lagi akan negeri.
dinonaktifkan. (E) Kau katakan kepada dia kenapa engkau tidak jadi
(C) Pesta olahraga SEA GAMES akan mempererat berangkat.
persahabatan antarbangsa di Asia Tenggara.
Bacalah teks berikut ini!
(D) Perbedaan warga negara seperti pribumi dan
(1) Trend jumlah penduduk miskin di Indonesia dari waktu
nonpribumi sudah tidak relevan lagi dengan
ke waktu lebih banyak tinggal di wilayah pedesaan. Tahun
perkembangan zaman.
1970 sampai dengan 1978, penduduk miskin lebih banyak
(E) Aktifitas para siswa akhir-akhir ini adalah
tinggal di pedesaan, antara tahun 1980 sampai dengan 1990
mengadakan aksi turun ke jalan menentang lebih banyak tinggal di kawasan perkotaan,dan 1993 hingga
kekerasan. 2000 jumlah penduduk miskin kembali lebih banyak di
pedesaan. Bahkan tahun 1998, selisih jumlah penduduk
04. Bentuk-bentuk kerancuan afiks terapat pada kata-kata di miskin yang tinggal di pedesaan cukup menyolok lebih
bawah ini, kecuali... besar daripada mereka yang tinggal di kota.
(A) Memperbesarkan (2) Beberapa karakteristik penduduk miskin dapat dilihat
(B) Memperbaiki dari jumlah anggota rumah tangga,pendidikan kepala rumah
(C) Mempelajarkan
tangga,dan sumber pendapatan. Menurut BPS,tahun 1993 Berbagai tantangan telah dihadapi selama pelaksanaan
rata-rata jumlah anggota keluarga penduduk miskin sebesar konstruksi, mulai dari tahap pemancangan, pengecoran,
5,9 orang,sedangkan jumlah keluarga rumah tangga yang sampai pemasangan girder dan bangunan atas. Kondisi
hidup di atas garis kemiskinan lebih kecil dari itu. Hal ini geologis dan kondisi geografis linkungan proyek yang berda
menunjukkan bahwa kepala rumah tangga miskin di atas laut menjaditantangan untuk dapat menyelesaikan
mempunyai beban yang lebih berat daripada yang tidak pekerjaan ini. Belum lagi cuaca yang menjadi kendala.
miskin. Perkiraan biaya pembangunan jembatan ini adalah Rp 4,5
(3)Penduduk miskin di pedesaan 72,00 persen dipimpin oleh triliyun. Besarnya biaya untuk pembangunan jembatan ini
kepala rumah tangga yang tidak tamat SD dan 24,30 persen tentunya menjadi kendala tersendiri bagi keberlanjutan
dipimpin oleh kepala rumah tangga berpendidikan tamat penyelesaian jembatan ini. karena itulah ketika krisis
SD.Penduduk miskin di perkotaan 57,00 persen dipimpin ekonomi melanda Indonesia , sempat menunda
oleh kepala rumah tangga tidak tamat SD dan 31,80 persen keberlanjutan pembanguna jembatan Suramadu. (disadur
dikepalai oleh seorang tamatan SD. dari berbagai sumber-TPA Center BPPK)
(4) Karakteristik lain yang berkaitan dengan pendidikan dan
distribusi penduduk adalah sumber pendapatan.Sumber 11. Yang dimaksud konsorsium dalam bacaan di atas
pendapatan utama penduduk miskin dari sektor pertanian adalah …..
sebesar 62,00 persen,sektor perdagangan 10,40 persen,sector (A) himpunan beberapa pengusaha yang mengadakan
industri sebesar 7,40 persen, sektor jasa 6,50 persen dan usaha bersama
selebihnya dari selain sektor yang disebutkan tersebut. (B) seminar tentang proses kontruksi dan proses
Dengan demikian,sektor pertanian merupakan kantong pembuatan jembatan
kemiskinan penduduk. (sumber:Gunawan (C) penelitian dan pengembangan
Sumodiningrat,2001). (D) jajak pendapat tentang pembangunan jembatan
09. Bacaan di atas lebih tepat jika diberi judul… (E) pertemuan untuk memcahkan masalah
(A) Trend jumlah penduduk miskin
(B) Karakteristik penduduk miskin 12. Pernyataan berikut yang sesuai dengan bacaan 2 adalah
(C) Distribusi penduduk miskin …
(D) Trend dan karakteristik penduduk miskin (A) setiap kota yang memiliki ikon tertentu merupakan
(E) Trend dan distribusi penduduk miskin kota maju
(F) (B) tidak ada jembatan lain yang mengalahi jembatan
10. Kata trend pada paragraf pertama dapat diganti kata… Suramadu
(A) Pada umumnya (C) pengiriman SBG memerlukan waktu 120 hari
(B) Dominan (D) sebelum dilakukan pembangunan jembatan
(C) Faktor determinan Suramadu dilakukan konsorsium terlebih dahulu
(D) Terkonsentrasi (E) kendala tersendiri bagi keberlanjutan pembangunan
(E) Kecenderungan jembatan Malang.

Bacalah teks berikut ini! 13. Cahaya matahari berlomba menyusup lekukan kaku
Kuala Lumpur punya ikon menara kembar dinding-dinding beton kelam di jantung kota Berlin.
Petronas, Jakarta punya ikon Tugu Monas, maka Surabaya Pola kalimat yang sama dengan kalimat tersebut
dan Madura punya ikon baru, Jembatan Suramadu. adalah…
Jembatan ini menghubungkan pulau Jawa (di Surabaya) dan (A) Bangunan itu dikelilingi dinding sepanjang 100
pulau Madura (di Bangkalan). Jembatan Suramadu akan meter yang terpahatkan nama kota dan desa tempat
menjadi jembatan terpanjang di Indonesia saat ini. Jembatan peristiwa itu berlangsung
Suramadu dimulai pembangunannya pada tanggal 20 (B) Terobosan manajer televisi itu dilakukan untuk
Agustus 2003. Dari total panjang jembatan sejauh 5.438 m menggaet iklan agar melimpah
terdiri dari causeway sisi Surabaya 1.458 m, causeway sisi (C) Holocaust Memorial berada di tengah megahnya
Madura 1.818 m. Bentang tengah panjang keseluruhan Brandenburger Tor, lambang bersatunya dua
mencapai 2.162 m terdisi dari dua Approach Bridge masing- Jerman
masing 672 m dan Main Bridge sepanjang 818 m. Panjang (D) Matanya memandang jauh menyusuri lekuk-lekuk
jalan pendekat dari sisi Surabaya mencapai 4,35 km dan sisi pekuburan Yahudi tertua di Eropa itu
Madura 11,50 km. (E) Bangungan-bangunan megah nan anggun itu jelas
Selain konsorsium nasional, maka yang juga kontras dengan ribuan batu nisan yang menyelimuti
memiliki peranan penting dalam pembangunan jembatan areal seluas dua kali lapangan sepak bola.
Suramadu adalah perusahaan konstruksi dari China. Itulah
sebabnya alas jembatan yang disebut dengan Steel Box 14. Di antara kalimat berikut ini yang menggunakan ragam
Girder (SBG) pembuatannya dilakukan di Tiongkok. kalimat baku adalah…
Ternyata selain pembuatannya harus dilakukan di Tiongkok, (A) Garis keturunan dari pihak ibu masih sulit
pengirimannya memakan waktu cukup lama. diakomodasi dalam RUU Kewarganegaraan
Pengiriman dari Tiongkok dilakukan empat kali (B) Menginjak tahun 1970-an kemajuan ilmu dan
selama 30 hari. Itupun masih harus dirakit lagi di Gresik. tekonologi di segala bidang maju pesat dengan
ditemukannya microchip di mana merupakan (D) Komisi pemilihan umum berencana mengatur
komponen dasar computer penetapan calon terpilih yang berpihak pada calon
(C) Dari dialog itu tampak banyak sekali kelemahan perempuan, jika partai politik meraih tiga kursi
daripada Undang-undang Nomor 62 itu. DPR/DPRD, salah satunya mesti diberikan kepada
(D) Ketidakadilan dalam RUU itu terlihat terutama calon perempuan yang perolehan suaranya
dalam ketentuan yang mana mengatur perkawinan terbanyak.
antar-bangsa (E) Berdasarkan pada data di Dinas Bina Marga
(E) Banyak pengusaha nakal yang melarikan diri ke Pemkot Surabaya, maka titik-titik genangan itu
luar negeri dengan membawa asset dari negara menyebar hampir di seluruh wilayah Surabaya
secara melanggar hokum. mulai sisi paling barat, timur, hingga ke selatan.

15. Krisis ekonomi, yang turut berperan dalam penambahan 19. Adat Badui yang ketat tercermin dalam pepatah "Yang
jumlah pengangguran di Indonesia, juga ditambah panjang tidak boleh dipotong, yang pendek tidak boleh
berbelitnya birokrasi di negara ini, yang menjadikan ditambah". Hal ini sengaja ditekankan agar tradisi lama
para investor asing enggan untuk menanamkan yang sesungguhnya tetap dapat dipertahankan. Namun,
modalnya ke Indonesia menyebabkan pemerintah harus tidak berarti masyarakat Badui tidak menerima sesuatu
berjuang memperbaiki struktur ekonominya. yang baru, atau sama sekali menutup sesuatu dari dunia
Kalimat tersebut akan menjadi baku bila diperbaiki luar. Di sans sini teriihat prang Badui membuat pelbagai
dengan cara… percobaan dengan mengubah kombinasi atau variasi
(A) menghilangkan tanda koma (,) sesudah kata unsur-unsur adat walaupun tetap dalam batas-batas
ekonomi yang ditetapkan oleh tetua adat.
(B) mengganti juga dengan kata yang. Inti paragraf di atas adalah…
(C) menghilangkan tanda koma (,) sebelum kata juga. (A) Adat Badui menekankan kemurnian tradisi lama
(D) menghilangkan untuk. sehingga sedapat mungkin mereka menolak
(E) menambah tanda koma (,) di depan kata berkontak dengan dunia luar.
menyebabkan. (B) Adat Badui berkembang dengan inovasi unsur-
16. Pembentukan kata serapan yang betul terdapat dalam unsur adat dan membuka diri terhadap campur
kalimat… tangan dari dunia luar.
(A) Di dunia olahraga yang di pentingkan adalah (C) Adat Badui mempertahankan tradisi dan
sportivitas. mengembangkannya dengan berbagai percobaan
(B) Sikap yang kaku dari seorang pemimpin atas variasi unsur-unsur adat.
menimbulkan watak arogan dan otoritair. (D) Adat Badui telah mengubah tradisi lama dengan
(C) Majelis juga mengusulkan, agar kalangan eksekutip tradisi baru sehingga menjadi lebih maju dan
dan legislatip peka terhadap aspirasi rakyat. modern.
(D) Mereka juga memprotes kwalitas beras yang (E) Adat Badui mengikuti batas-batas yang ditetapkan
mereka terima. oleh tetua adat yang menekankan kesederhanaan.
(E) Rapat itu gagal karena yang hadir tidak memenuhi
korum. 20. Pada saat ini, kerisauan temyata bukan hanya milik para
17. Kalimat yang baku adalah… petani yang kesulitan mendapatkan pupuk. Bambang
(1) Walaupun banyak mendapat protes, namun Riyanto, seorang perajin jenang dan tempe keripik asal
pemerintah tetap akan menaikkan harga BBM. Purwokerto, bahkan harus pontangpanting mencari
(2) Walaupun banyak mendapat protes, pemerintah minyak tanah agar usahanya itu tetap berlangsung.
tetap akan menaikkan harga BBM. Kerisauan jelas membayang ketika akan memasuki
(3) Walaupun banyak mendapat protes, tetapi musim liburan akhir tahun lalu. "Musim panen"bagi
pemerintah tetap akan menaikkan harga BBM. industri kecil makanan oleh-oleh ini tentu tidak akan
(4) Pemerintah tetap akan menaikkan harga BBM, dibiarkan lewat begitu saja. Untuk mendapatkan minyak
walaupun banyak mendapat protes. tanah yang cukup untuk produksi, ia harus
mengerahkan seluruh anggota keluarganya, bahkan para
18. Kalimat berikut tidak baku, kecuali… tetangganya, untuk ikut antre membeli minyak tanah.
(A) Dengan teknik tersebut para astronom dapat Pikiran pokok paragraf di atas adalah…
memperkirakan ukuran planet meskipun belum (A) kerisauan para petani dan perajin makanan oleh-
diketahui massanya, namun diperkirakan antara 5,7 oleh di Purwokerto karena sulitnya pupuk dan
hingga 11 massa bumi. minyak tanah
(B) Landasan pacu Bandara Ahmad Yani Semarang (B) "musim panen" bagi industri kecil makanan oleh-
tergenang sepanjang 500 meter dan harus oleh.
menggunakan 23 pompa air untuk menyurutkan (C) kerisauan Bambang Riyanto, seorang perajin
genangan. jenang dan tempe keripik asal Purwokerto.
(C) Bandara menunda semua penerbangan sejak (D) kelangkaan minyak tanah di Purwokerto.
Minggu pagi hingga pukul 21.00 karena kondisi (E) suka duka perajin jenang dan tempe keripik asal
landasan pacu masih buruk. Purwokerto
(E) Masyarakat Pegunungan Meratus belum menikmati
Bacalah bacaan berikut dengan cermat, kemudian kebutuhan sekunder dan tersier.
jawablah pertanyaan nomor 21 sampai dengan 23.
Begitu matahari terbit di Batu Kambar, Desa Hinas 24. Depresi dari penggunaan sosial media tersebut dalam
Kiri, Kecamatan Batang Alai Timer, Kabupaten Hulu bentuk perasaan tidak bahagia, rasa iri, setelah
Sungai Tengah, Kalimantan Selatan, ratusan warga ke luar menggunakan media sosial.
rumah. Ada yang langsung pergi ke pahumaan (ladang), ada
Kalimat di atas menjadi kalimat baku jika perbaikannya
yang pergi ke pasar, ada pula yang berangkat ke sekolah.
Warga di pedalaman Pegunungan Meratus memang terbiasa sebagai berikut …
bangun pagi karena mereka jugs sudah terbiasa tidur sore (A) Menghilangkan tanda baca koma setelah kata iri.
akibat tidak adanya aliran IWO. Karena itu, aktivitas pagi di (B) Menghilangkan kata dari.
Pegunungan Meratus betul-betul menggeliat dan dinamis. (C) Menghilangkan kata tersebut.
Hari itu kebetulan hari pasar tradisional yang hanya ada (D) Menambahkan kata timbul setelah kata tersebut.
sekali dalam sepekan. Masyarakat dari berbagai balai adat (E) Menambahkan kata menangis setelah kata rasa iri.
Dayak Meratus, yang jarak tempuhnya hingga 18 jam
dengan jalan kaki, berduyun-duyun ke pasar untuk membeli
25. Aktifitas anak bergizi buruk biasanya rendah; berat
keperluan hidup "modem", mulai dari bahan pangan,
sandang, hingga papan. badannya dibawah normal yang menunjukan kwalitas
Tradisionalisme, mulai dari cara makan, cara asupan makanannya tidak baik.
mandi,cara berinteraksi, hingga pandangan hidup, betul- Kalimat di atas menjadi kalimat baku jika perbaikannya
betul masih melekat dan menjadi identitas warga pedalaman sebagai berikut kecuali…
Pegunungan Meratus. Lambat laun keterisolasian pedalaman (A) Mengubah penulisan aktifitas menjadi aktivitas
memang bisa ditembus, tetapi hingga kini kehidupan alami (B) Memisahkan penulisan dibawah menjadi di bawah
masih melekat. Baik di hulu Sungai Selatan maupun di hulu
(C) Mengubah penulisan menunjukan menjadi
Sungai Tengah yang mempunyai komunitas adat terbesar di
Kalimantan Selatan suasananya sama-sama masih menunjukkan
tradisional. Tradisionalisme di satu sisi menjadi kebanggaan, (D) Mengubah penulisan kwalitas menjadi kualitas
tetapi di sisi lain menyisakan pertanyaan akan capaian (E) Mengubah penulisan asupan menjadi masukan.
sebuah kesejahteraan masyarakat.
26. Kesalahan penggunaan kata di mana terdapat pada
21. Judul yang tepat untuk bacaan di atas adalah… kalimat…
(A) Kisah Kehidupan dari Batu Kambar, Desa Hinas
(A) Di mana tempat tinggalmu sekarang?
Kiri
(B) Kehidupan Tradisional Masyarakat Pegunungan (B) Anda membaca berita itu di mana?
Meratus. (C) Anda bebas memilih tempat makan di mana saja.
(C) Identitas Warga Pedalaman Pegunungan Meratus. (D) Di mana ada dia, di situ ada saya.
(D) Tradisionalisme Menjadi Kebanggaan bagi (E) Pemerintah hanya akan memberikan bantuan di
Masyarakat Pegunungan Meratus. mana rakyat membutuhkan.
(E) Selama 18 jam Berjalan Menuju Pasar untuk
Keperluan Hidup "Modern".
27. Dilihat berdasarkan usia, terdapat pembagian usia yaitu
22. Inti kalimat keenam dalam paragraf pertama bacaan di di bawah usia dua puluh tahun, 21-30 tahun, 31-40
atas adalah… tahun, 41-50 tahun dan di atas 51 tahun.
(A) Masyarakat berjalan ke pasar selama 18 jam. Perbaikan penulisan EYD kalimat di atas adalah…
(B) Masyarakat membeli keperluan hidup "modern". (A) Dilihat berdasarkan usia, terdapat pembagian usia
(C) Masyarakat berduyun-duyun ke pasar. yaitu di bawah usia 20 tahun, 21-30 tahun, 31-40
(D) Masyarakat Dayak Meratus terdiri atas berbagai
tahun, 41-50 tahun, dan di atas 51 tahun.
balai adat.
(E) Masyarakat Dayak Meratus berjalan kaki ke pasar. (B) Dilihat berdasarkan usia, terdapat pembagian usia
(F) yaitu di bawah usia 20 tahun, 21/30 tahun, 31/40
23. Dalam bacaan di atas terdapat pernyataan implisit tahun, 41/50 tahun, dan di atas 51 tahun.
berikut, kecuali… (C) Dilihat berdasarkan usia, terdapat pembagian usia
(A) Warga Pedalaman Pegunungan Meratus tidak yaitu di bawah usia 20 tahun, 21-30 tahun, 31-40
mengenal kendaraan bermotor. tahun, 41-50 tahun dan di atas 51 tahun.
(B) Warga Pedalaman Pegunungan Meratus belum
(D) Dilihat berdasarkan usia terdapat pembagian usia
menggunakan listrik.
(C) Tradisionalisme di satu sisi adalah kesedihan atas yaitu di bawah usia 20 tahun, 21-30 tahun, 31-40
keterbelakangan. tahun, 41-50 tahun, dan di atas 51 tahun.
(D) Pasar tradisional hanya ada satu kali dalam
seminggu.
(E) Dilihat berdasarkan usia, terdapat pembagian usia
yaitu: di bawah usia 20 tahun, 21-30 tahun, 31-40
tahun, 41-50 tahun dan di atas 51 tahun.

28. Bagi semua mahasiswa yang telah lulus diwajibkan


melakukan registrasi ulang pada tanggal yang telah
ditentukan oleh universitas.
Kalimat di atas tidak baku karena penggunaan…
(A) bagi
(B) semua
(C) melakukan
(D) pada
(E) oleh

29. Kapolda Aceh Dirjen Pol Husein Hamidi memberikan


apresiasi kepada Perwira Muda Komandan Kapal Dit
Pol Air Baharkam Polri yang berhasil memimpin
penangkpann kapal dan ABK-nya.
Makna kata apresiasi pada kalimata di atas adalah…
(A) Nilai tinggi
(B) Penghargaan
(C) Perjuangan
(D) Hak kerja
(E) Hak istimewa.

30. Penulisan tanda baca yang salah terdapat pada


kalimat…
(1) abad ke21 merupakan tantangan bagi umat
manusia.
(2) PT Merdeka Jaya memenangkan tender.
(3) Ia membeli formulir ujian itu seharga
Rp.350.000,00.
(4) Mereka memerlukan uang pecahan 5000-an.
(D) Bank itu dinyatakan pailit awal tahun ini; padahal
kinerjanya dianggap baik tahun lalu.
(E) Karena modal di bank terbatas, tidak semua
pengusaha lemah memperoleh kredit.

04. Pasar kerja di negeri kita cukup banyak. Tentu saja


yang dimaksud di sini adalah penelitian terapan yang
dapat digunakan untuk meningkatkan mutu keadaan
atau hasil suatu keadaan dalam masyarakat. Penelitian
eksploratoris atau jenis penelitian lain yang bersifat
teoretis juga memiliki pasar kerja, tetapi biasanya
sangat terbatas. Ada juga dana yang disediakan oleh
yayasan yang besar, seperti Toyota Foundation dan
Ford Foundation. Bahkan, tersedia dana yang besar
yang ditawarkan oleh pemerintah, seperti sejumlah
proyek yang didukung oleh Kementerian Pendidikan
Nasional. Belum lagi perusahaan atau lembaga lain
01. Penggunaan tanda baca titik dua (:) yang benar terdapat yang berkepentingan terhadap penelitian tertentu. Pada
pada… dasarnya, pasar kerja penelitian di Indonesia terbuka
(1) Para karyawan mempunyai tuntutan yang keras: lebar.
direktur mundur atau dimundurkan. Gagasan pokok paragraf di atas terdapat pada…
(2) Mereka ingin memiliki kebebasan dasar seperti: (A) Awal kalimat
bebas dari rasa takut dan bebas untuk menentukan (B) Kalimat kedua
jalan hidup. (C) Awal dan akhir kalimat
(3) (Esron, 2000: 12-15) (D) Akhir kalimat
(4) Para pejabat Pemda Kalteng mengatakan: relokasi (E) Semua kalimat
itu sulit dilakukan dalam waktu dekat.
05. Payung umumnya digunakan untuk melindungi diri
02. NTT, menurut Frans, memiliki potensi itu sehingga dari hujan atau sengatan matahari. Fungsi ini tidak
pemerintah dan masyarakat sangat mendukung promosi berbeda jauh dari maknanya yang dalam bahasa Inggris
dan ajakan itu untuk mendatangkan 1.500 penyelam disebut umbrella. Umbrella berasal dari bahasa Latin,
yang ditargetkan ke berbagai daerah di Tanah Air. umbra, yang berarti ‘melindungi’. Payung juga
Gagasan utama kalimat tersebut adalah… mempunyai sinonim lain, yaitu parasol. Hanya saja,
(A) NTT mendatangkan penyelam. parasol biasanya digunakan untuk melindungi diri dari
(B) Pemerintah dan masyarakat mendatangkan sengatan matahari.
penyelam. Topik paragraf di atas adalah..
(C) Pemerintah dan masyarakat mendukung promosi. (A) Fungsi payung sebagai alat untuk melindungi diri
(D) NTT memiliki potensi. dari hujan dan matahari.
(E) Pemerintah mendatangkan penyelam untuk (B) Kata umbrella berasal dari bahasa Latin, umbra.
berbagai daerah. (C) Kata umbrella dan parasol bersinonim.
(D) Makna parasol berbeda makna dengan umbrella.
03. Tanda titik koma digunakan secara tepat dalam (E) Asal-usul payung.
kalimat…
(A) Suntikan obligasi dapat mendatangkan pendapatan 06. Fernando Alonso dari spanyol mengangkat tangan
yang pasti; penghilangan kredit menjadikan neraca setelah berhasil meraih posisi pole pada kualifikasi
perbankan bersih. Sabtu, 25 Desember 2010, mengungguli Sebastian
(B) Kehadiran obligasi rekap memberikan tiga Vettel dari Jerman pada balap mobil formula satu di
keajaiban; pendapatan yang pasti, kolektibiltas Singapura.
yang lancar, dan pencadangan yang kecil. Gagasan utama kalimat di atas adalah…
(C) Pertama, bank memasyarakatkan produk layanan (A) Fernando Alonso dari spanyol mengangkat tangan.
jasa kepada masyarakat; menyalurkan kredit ke (B) Fernando Alonso mengangkat tangan.
sektor perdagangan, jasa, transportasi, dan industri. (C) Fernando Alonso berhasil meraih posisi pole.
(D) Fernando Alonso meraih posisi pole. (C) Sedikit pun dalam menentukan pilihannya dia tidak
(E) Fernando Alonso mengungguli Sebastian Vettel. ragu.
(D) Sudah saya antar anak itu ke sekolah.
(E) Saya sangat berharap regu kitalah yang menjadi
07. “saya tahu ada kemungkinan saya melahirkan lebih dari
juaranya.
satu bayi karena dokter menaruh empat embrio di rahim
saya agar kemungkinan saya mendapatkan seorang bayi 10. Semua kalimat di bawah ini termasuk kalimat aktif
lebih besar” kata Sharon. intransitif, kecuali…
Penulisan kalimat di atas yang sesuai dengan EYD (A) Gadis itu menyapa saya dengan ramah.
adalah… (B) Donal kehujanan tadi pagi.
(A) “saya tahu ada kemungkinan saya melahirkan lebih (C) Tas baru itu telah dia bawa.
(D) Adik berteriak-teriak kegirangan.
dari satu bayi karena dokter menaruh empat embrio
(E) Kubaca buku itu sampai tuntas.
di rahim saya agar kemungkinan saya mendapatkan
seorang bayi lebih besar,” kata Sharon. 11. Bentuk pasif dari kalimat Ayah membelikan adik buku
(B) “Saya tahu ada kemungkinan saya melahirkan lebih baru adalah…
dari satu bayi karena dokter menaruh empat embrio (A) Ayah dibelikan adik buku baru.
di rahim saya agar kemungkinan saya mendapatkan (B) Buku baru dibelikan ayah untuk adik.
seorang bayi lebih besar” kata Sharon. (C) Adik dibelikan ayah buku baru.
(D) Adik beli ayah bukubaru.
(C) “Saya tahu ada kemungkinan saya melahirkan lebih
(E) Buku baru ayah dibelikan adik.
dari satu bayi karena dokter menaruh empat embrio
di rahim saya agar kemungkinan saya mendapatkan 12. Penghilangan unsurkalimat terdapat pada…
seorang bayi lebih besar,” kata Sharon. (A) Bagaimana keadaan ibumu?
(D) “Saya tahu ada kemungkinan saya melahirkan lebih (B) Saya menonton televisi kemarin.
dari satu bayi karena dokter menaruh empat embrio (C) Kemarin saya menonton televisi.
di rahim saya agar kemungkinan saya mendapatkan (D) Saya menonton televisi dan adik juga.
(E) Dia bunuh diri.
seorang bayi lebih besar.” kata Sharon.
(E) “Saya tahu ada kemungkinan saya melahirkan lebih 13. Nenek…masa lalunya yang indah di tepi danau itu.
dari satu bayi karena dokter menaruh empat embrio Kata yang tepat untuk mengisi bagian yang rumpang
di rahim saya agar kemungkinan saya mendapatkan adalah..
seorang bayi lebih besar”. kata Sharon. (A) Bercerita
(B) Menceritakan tentang
08. Adapun Jokowi pada 22-24 Maret lalu melakukan (C) Bercerita tentang
(D) Bercerita dengan
kunjungan kerja di Kalimantan Barat, Kalimantan
(E)Menceritakan mengenai
Timur, dan Kalimantan Utara. Pada kunjungan kerjanya
itu, Jokowi mersemikan Jembatan Tayan, di Kecamatan 14. Adik saya belajar matematika dengan serius.
Tayan, Kabupaten Sanggau. Jembatan Tayan adalah Pola kalimat di atas tidak sama dengan pola kalimat di
bagian dari jalan yang menghubungkan Kalimantan bawah ini, kecuali…
Barat dengan Kalimantan Tengah. Jokowi juga (1) Ibu bernyanyi dengan merdu.
(2) Mereka mempelajari fisika selama dua tahun.
meninjau Pos Batas Lintas Negara (PBLN) di
(3) Ketika Rudi datang, Ali pergi ke Jakarta
Kecamatan Entikong, Kalimantan Barat. (4) Ia kejatuhan mangga di teras.
Gagasan utama kalimat di atas adalah…
(A) Jokowi melakukan kunjungan kerja. 15. Untuk mengamankan jalur pantura, polisi menurunkan
(B) Jokowi meresmikan Jembatan Tayan. 2.000 anggota pasukan penembak jitu. Berdasarkan
(C) Jokowi meresmikan Pos Batas Lintas Negara konteks tersebut, tahapan proses pembentukan
(PBLN). kelopmok kata anggota pasukan penembak jitu
adalah…
(D) Jokowi berada di Kalimantan Barat, Kalimantan
(1) Pasukan-anggota pasukan- anggota pasukan
Timur, dan Kalimantan Utar. penembak – anggota pasukan penembak jitu.
(E) Tinjauan kerja di Kalimantan. (2) Penembak – penembak jitu – pasukan penembak
jitu – anggota pasukan penembak jitu.
09. Kalimat yang mempunyai pola O2-P-S-K terdapat pada (3) Penembak – pasukan penembak – pasukan
kalimat… penembak jitu – anggota pasukan penembak jitu.
(A) Olahraga renang masih dianaktirikan. (4) Pasukan – pasukan penembak – pasukan penembak
(B) Kejuaraan cabang tenis meja dimenangkan oleh jitu – anggota pasukan penembak jitu.
atlet Jawa Barat.
16. Pola dasar kalimat bahasa Indonesia yang terdiri dari (D) Orang muda harus bekerja keras untuk
kata benda dan kata benda terdapat pada kalimat… mengembangkan kariernya.
(A) Bukuku hilang di kantor. (E) Pemimpin itu memiliki karisma yang tinggi.
(B) Saudaraku belajar fisika.
(C) Ini makanan ayam. 22. Penulisan huruf miring yang benar terdapat dalam
(D) Ambilkan the sebotol. kalimat ....
(E) Uangnya dibelikan makanan. (A) Saya pernah membaca Senja di Pelabuhan Kecil
sebuah sajak karangan Khairil Anwar.
17. Dokter muda yang memakai baju putih itu sedang (B) “Bola Lampu” adalah sebuah cerpen yang dimuat
memeriksa pasiennya tadi siang. dalam Dari Suatu Masa dari Suatu Tempat.
Kalimat di atas termasuk kalimat, kecuali… (C) Para elite politik harus dapat memahami pikiran
(A) Kalimat nominal rakyat.
(B) Kalimat aktif (D) Jika kita akan berkemah, jangan lupa membawa
(C) Kalimat sempurna Kompas.
(D) Kalimat subordinatif (E) Silakan baca Sahabat Saya Cordiaz sebuah cerpen
(E) Kalimat deklaratif. karya Asrul Sani.

18. Kalimat yang tidak merupakan pola dasar kalimat 23. Penulisan kata dicetak tebal yang tepat adalah....
bahasa Indonesia adalah… (A) Penghancur leburan negara Afganistan oleh tentara
(A) Ayah membaca surat kabar. Amerika Serikat merupakan pelanggaran hak asasi
(B) Kakak ke pasar. manusia.
(C) Ateng pelawak. (B) Pelajaran ektra kurikuler harus lebih digalakan
(D) Kamar rumahnya empat. lagi di sekolah.
(E) Ia memang cantik. (C) Marilah kita bersyukur kepada tuhan yang
Mahapengasih.
(D) Tank-tank Israil membabibuta menghancurkan
19. Penggunaan tanda baca yang benar terdapat pada .... permukiman penduduk.
(1) Kepala Humas PT KA Daerah Operasi I Jakarta, (E) Setelah menjadi syah bandara di Pelabuhan
Sugeng Prijona, mengemukakan bahwa jumlah Tanjung Priok, dia menjadi kaya raya.
penumpang kereta api sudah normal.
(2) Di Terminal Pula Gadung, Jakarta Timur, tercatat 24. Penulisan gabungan kata dalam kalimat berikut ini
18.510 penumpang yang tiba pada hari Minggu. yang benar adalah ....
(3) Sejak Sabtu malam, sistem buka tutup di Simpang (A) Gempa bumi yang telah menghancur-leburkan
Jomin tidak diberlakukan lagi karena arus lalu daerah i tu pada Mei 2006 yang lalu tidak
lintas sudah sepi. menyurutkan semangat warga untuk membangun
(4) “Selama arus balik tahun ini tidak terjadi kembali rumah – rumah mereka.
kemacetan karena kami memberlakukan pola satu (B) Sampai dengan enam bulan pasca gempa masih
arah,” kata Kapolres Banyumas, Komisaris Besar banyak warga yang terpaksa harus berteduh di
R.Z. Panca Putra. bawah tenda-tenda darurat
(C) Kerukunan antarwarga tercermin dari semangat
20. Penulisan yang salah terdapat pada .... bergotong royong mereka dalam membangun
(A) Mengajukan pertanyaan kepada narasumber kembali tempat tinggalnya.
diperlukan untuk mendapat informasi. (D) Bahkan, lebih dari dua per tiga bangunan rumah
(B) Keterampilan mengajukan pertanyaan diperlukan di desa-desa itu roboh dan tidak dapat dihuni lagi.
sepanjang hidup manusia. (E) Meskipun demikian mereka tetap tampak tabah
(C) Dengan bertanya, kita dapat memahami berbagai menerima cobaan yang menjungkir balikan
hal yang belum kita pahami. kehidupan mereka itu.
(D) Dalam bertanya, intonasi, jeda, serta lafal harus
jelas. 23. Kalimat di bawah ini yang dapat dipakai dalam ragam
(E) Pada bagian ini, kamu akan berlatih menyampaikan resmi adalah ....
informasi kepada orang lain. (A) Mereka masih memikirkan tentang hal itu.
(B) Kita patut bersyukur atas perlindungan yang
21. Kata bercetak miring dalam kalimat berikut ini diberikan oleh Tuhan Yang Maha Esa
termasuk kata baku, kecuali ... (C) Penjambretan itu telah mengakui akan
(A) Hari ini akan dilakukan gladi resik upacara kesalahannya.
wisuda. (D) Semua pekerjaan itu telah dia selesaikan dengan
(B) Mereka menghadapi masalah yang sangat baik.
kompleks. (E) Apa yang telah mereka kerjakan selama ini.
(C) Dalam menghadapi masalah kelompok, tidak boleh
terlalu ekstrim.
24. Bagi perusahaan yang melakukan penyesuaian dengan (C) Setelah menyelesaikan program strata satu di
menekan anggaran pengeluaran, rasionalisasi karyawan Universitas Udayana, Dimas Purnama pergi ke
menjadi salah satu kebijakan perusahaan. Dengan Tokyo untuk mengambil program pascasarjana.
melakukan pemutusan hubungan kerja (PHK), (D) Fotokopi ijazah yang telah dilegalisir oleh kepala
terutama atas buruh, maka perusahaan dapat melakukan sekolah harus disertakan pada formulir pendaftaran
penghematan yang akan mengurangi biaya produksi untuk ke universitas.
dialihkan untuk menambah biaya listrik. Biaya lain (E) Hanya akibat dari persoalan yang sangat sederhana,
yang ditekan untuk melakukan penghematan adalah pertikaian__bahkan perkelahian__antar wilayah
overtime yang ditiadakan. Namun, untuk memenuhi seringkali terjadi pada tahun-tahun terakhir.
permintaan produksi dan pesanan tanpa adanya lembur
perusahaan mengadakan kebijakan untuk
mengoptimalisasikan jam kerja dengan pengawasan 28. (a) Dewasa ini telekomunikasi merupakan media
ketat atas kinerja karyawan. komunikasi yang sangat penting peranannya. (b)
Paragraf tersebut akan menjadi paragraf yang baik jika Kebutuhan telekomunikasi itu dirasakan oleh seluruh
diperbaiki dengan cara berikut, kecuali .... lapisan masyarakat, baik kalangan bisnis, pemerintahan,
(A) Menghilangkan kata maka maupun individu. (c) Kebutuhan itu dirasakan pula oleh
(B) Menghilangkan tanda koma (,) sebelum kata seluruh tingkatan masyarakat, mulai dari tingkat
terutama ekonomi mampan sampai tingkat bawah. (d) Dalam
(C) Mengubah tulisan overtime menjadi over time pemakaian sarana telekomunikasi itu, masih diperlukan
(D) Menambahkan tanda koma (,) setelah kata namun banyak pertimbangan mengingat tingginya biaya tarif
(E) Mengganti kata mengoptimalisasikan dengan telepon yang berlaku di Indonesia. (e) Tarif telepon
mengoptimalkan. yang berlaku di Indonesia, tergolong mahal bila
dibandingkan tarif telepon yang berlaku di negara lain.
25. Pukul 08.00 di Tomok, Pulau Samosir, matahari baru Dalam alinea tersebut terdapat kalimat yang
saja muncul di Danau Toba. Udara dingin dan kabut menggunakan tanda (,) yang tidak sesuai dengan kaidah
belum beranjak. Tetapi, sekitar dua ratus warga desa EYD. Kalimat yang dimaksud adalah …
telah bergegas ke rumah peninggalan Raja Sidabutar (A) kalimat (a)
dengan menggunakan pakaian adat Batak, ialah kain (B) kalimat (b)
ulor dan ikat kepala tiga warna, merah, putih, dan (C) kalimat (c)
hitam. (D) kalimat (d)
Kalimat di atas akan menjadi kalimat baku jika (E) kalimat (e)
diperbaiki dengan cara berikut, kecuali ...
(A) mengganti kata muncul dengan terbit 29. Undangan Direktur Utama P.T. Mutiara Sakti DR.
(B) mengganti kata tetapi dengan namun Dimas Purnama, SH. mengejutkan para pemilik saham
(C) mengganti kata ialah dengan yaitu karena sebelumnya tidak ada rencana pertemuan dalam
(D) mengganti kata menggunakan dengan mengenakan waktu dekat ini.
(E) mengganti dua ratus dengan 200 Kalimat di atas termasuk kalimat yang tidak baku,
berikut ini beberapa saran perbaikan, kecuali …
(A) P.T. seharusnya tanpa tanda titik.
26. Kalimat berikut ini yang ditulis sesuai dengan aturan (B) DR. Dimas Purnama, SH seharusnya ditulis Dr.
EYD adalah … Dimas Purnama, S.H.
(A) Saya tidak akan membeli mobil mahal, karena tidak (C) kata Sakti diikuti tanda koma (,)
punya uang. (D) kata Direktur Utama seharusnya ditulis dengan
(B) Oleh karena itu saya harus rajin menabung. huruf kecil semua
(C) Atas bantuan saudara saya mengucapkan terima (E) kata mengejutkan didahului koma (,)
kasih.
(D) Malam makin larut; pekerjaannya belum selesai 30. Penulisan kata bilangan yang sesuai dengan EYD
juga. terdapat pada kalimat berikut:
(E) Semua siswa baik yang laki-laki maupun (A) Usulnya baru dibicarakan pada tahap ke-10.
perempuan, mengikuti ujian. (B) Anaknya dapat mencapai juara ke tiga.
(C) Pada hari yang ke-tiga anak diperiksa lagi
27. Terdapat penulisan kata yang salah dalam kalimat- darahnya.
kalimat berikut, kecuali (D) Asosiasi para lulusan SMA I hari ini mengadakan
(A) Seseorang dapat dipastikan menderita suatu peringatan ulang tahunnya yang ke duabelas.
penyakit tertentu setelah melalui hasil pemeriksaan (E) Mahasiswa yang masa studinya pada saat ini
laboratium. berada pada semester ke IX mendapat peringatan.
(B) Setiap perusahaan akan selalu berusaha
meningkatkan produktifitas para pegawainya agar
perusahaan itu dapat meningkatkan layanan kepada
konsumen secara maksimum.
2. Kalimat yang perlu diperbaiki adalah…
A. Kalimat 2
B. Kalimat 3
C. Kalimat 5
D. Kalimat 6
E. Kalimat 7

3. Kata yang penulisannya tidak mengikuti kaidah ejaan


adalah…
A. Kebun Raya Bogor (Kalimat 1)
B. reorganisasi (Kalimat 4)
C. obyek (Kalimat 4)
D. unit pelaksana teknis (kalimat 6)
E. degradasi (Kalimat 11)

4. Penggunaan konjungsi yang tidak tepat terdapat pada


kalimat…
A. adapun (kalimat 5)
B. serta (kalimat 7)
Bacalah teks berikut untuk menjawab soal No. 1 s.d. 8! C. maka (kalimat 7)
(1)Lembaga Ilmu Pengetahuan Indonesia (LIPI) D. jika (kalimat 9)
semestinya membatalkan keputusan menurunkan status E. padahal (kalimat 10)
pengelolaan Kebun Raya Bogor. (2)Perubahan dari pusat
konservasi tumbuhan menjadi unit pelaksana teknis balai 5. Kata yang tepat untuk mengisi bagian rumpang pada
konservasi tumbuhan akan mendegradasi peran kebun kalimat (3) adalah…
berusia 200 tahun lebih itu menjadi sekadar kawasan A. Mengeluarkan
wisata. B. Mengesahkan
(3)Perubahan itu berlaku sejak LIPI … Peraturan C. Penerbitan
Nomor 1 Tahun 2019 pada Januari lalu. (4)Dalam D. Membuat
reorganisasi ini, pengelola balai hanya bertugas mengatur E. Perumusan
koleksi vegetasi, menyelenggarakan pendidikan
lingkungan, dan menjadikan Kebun Raya sebagai obyek 6. Kata ini pada kalimat (4) merujuk pada…
wisata. (5)Adapun fungsi penelitian dan pengembangan A. LIPI
yang erat kaitannya dengan konservasi diserahkan kepada B. pengelolaan Kebun Raya Bogor
lembaga terpisah. C. Peraturan Nomor 1 Tahun 2019
(6)Status unit pelaksana teknis itu membuat D. perubahan aturan
pengelola akan lebih berkonsentrasi meningkatkan E. kawasan wisata
pendapatan. (7)Dengan koleksinya berupa 13.061 pohon
yang terdiri atas 213 suku, 1.200 marga, dan 3.151 spesies 7. Inti kalimat (9) adalah…
serta lokasi yang strategis di tengah kota, maka target itu A. Fungsi Kebun Raya bisa terabaikan
mudah dicapai. (8)Tahun lalu, pengunjung kawasan ini B. Fungsi Kebun Raya
mencapai 1,6 juta orang, meningkat hampir dua kali lipat C. Pengelola mengejar pendapatan
dari dua tahun sebelumnya. D. Kebun Raya sebagai kawasan konservasi
(9)Problemnya, jika pengelola hanya mengejar E. Kawasan terabaikan
pendapatan, fungsi Kebun Raya sebagai kawasan
konservasi di luar habitat aslinya bisa saja terabaikan. 8. Kata yang bukan merupakan padanan kata konservasi
(10)Padahal hal itu merupakan alasan utama pendirian (kalimat 5) adalah…
Kebun Raya. (11)Secara spesifik Peraturan Presiden Nomor A. pelestarian
93 Tahun 2011 tentang Kebun Raya, pada bagian B. pengamanan
pertimbangan, huruf a, menyatakan peran utama Kebun C. pemertahanan
Raya adalah untuk mengurangi laju degradasi D. pemeliharaan
keanekaragaman tumbuhan. E. pelindungan

1. Judul yang tepat untuk karangan di atas adalah… Bacalah teks berikut untuk menjawab soal No. 9 s.d. 14!
A. Perubahan Status Kelola Kebun Raya (1)Kementerian Pertanian (Kementan) resmi
B. Pembatalan Status Kelola Kebun Raya membuka Museum Pertanian pertama yang ada di
C. Tata Kelola Kebun Raya Indonesia, di pusat Kota Bogor, Jawa Barat, Senin (22/4).
D. Fungsi Kebun Raya untuk Konversi (2) Museum tersebut … sebagai museum pertanian terbesar
E. Peraturan Baru LIPI yang ada di wilayah Asia Tenggara.
(3) Sekretaris Jenderal Kementan—Syukur Iwantoro
—mengatakan berdirinya Museum Pertanian tersebut atau diganti dengan pilihan lain yang tersedia (B, C, D,
diharapkan dapat menjadi jendela informasi bahwa atau E).
Indonesia merupakan negara agraris. (4) Bahkan, predikat Kandungan mikroplastik dalam air botol kemasan
sebagai negara agraris tersebut telah berlaku sejak masa seharusnya menjadi catatan khusus dalam peringatan 15hari
kolonial Hindia Belanda di mana Jalur Rempah menjadi Air Sedunia. 16Partikel kecil plastik itu yang telah
merubah perekonomian dunia saat itu. (5) Dia berharap, menteror masyarakat dunia, termasuk Indonesia.
hadirnya museum tersebut dapat membuka cakrawala Celakanya, tidak 17satu pun produsen air kemasan di sini
semua orang, terlebih generasi milenial. (6) Alasannya, yang bebas dari 18mikroplastik. Padahal, hampir seluruh
sektor pertanian yang selama ini kerap dipandang sebelah masyarakat kota telah menyandarkan kesehatan air
mata dan kurang diminati. (7) Untuk itu, dia mengajak minumnya pada pabrik-pabrik modern 19supaya rela
generasi milenial untuk melirik sektor bisnis tersebut menebus biaya 0,6 liter air yang terkadang lebih mahal
dengan menggali informasi sebanyak-banyaknya tentang daripada 1 liter bensin. Mahal ini untuk mengganti biaya
pertanian. (8) Salah satunya, menurut dia, adalah dengan botol plastik. Sayangnya, harga yang tinggi itu tidak
berkunjung ke Museum Pertanian. sebanding dengan keamanan plastiknya. Botol, atau diduga
tutup botol tersebut, mudah melepaskan mikroplastik.
9. Penggunaan tanda baca yang salah terdapat pada Mikroplastik merupakan suatu partikel dengan ukuran
kalimat… kurang dari 5 mm. Mikroplastik awalnya banyak
A. 1 D. 5 mencemari pantai, tetapi hari ini sudah mencemari tubuh
B. 3 E. 8 manusia melalui makanan dan minuman kemasan plastik.
C. 4 Hal tersebut menjadi ancaman kesehatan manusia karena
sulit terurai dan dapat menumpuk dalam jaringan dan sel
10. Pemilihan diksi yang salah terdapat pada kalimat… manusia. 20Apabila belum ada laporan yang valid mengenai
A. 1 D. 5 dampaknya pada manusia, 21tertumpuknya terus-menerus
B. 3 E. 6 dalam sel berpotensi mengubah kromosom yang dapat
C. 4 mengakibatkan kanker dan kesehatan reproduksi.
D. 5
15. hari Air Sedunia
11. Kata yang tepat untuk mengisi bagian titik-titik di A. TIDAK PERLU DIPERBAIKI
kalimat (2) adalah… B. Hari Air Sedunia
A. Diklaim C. Hari air se-Dunia
B. Ditaksir D. ―Hari Air Sedunia‖
C. Diterka E. “Hari Air Sedunia”
D. Diduga
E. Diperkirakan 16. Partikel kecil plastik itu yang telah menteror
masyarakat dunia termasuk Indonesia
12. Kalimat yang tidak efektif pada teks di atas adalah… A. TIDAK PERLU DIPERBAIKI
A. Kalimat (1) B. Partikel kecil plastik itu telah meneror masyarakat
B. Kalimat (3) dunia termasuk Indonesia
C. Kalimat (4) C. Partikel kecil plastik itu yang telah menteror
D. Kalimat (6) masyarakat dunia, termasuk Indonesia
E. Kalimat (7) D. Partikel kecil plastik itu yang telah meneror
masyarakat dunia, termasuk Indonesia
13. Imbuhan - nya pada kalimat (8) merujuk pada… E. Partikel kecil plastik itu telah meneror masyarakat
A. pencarian informasi dunia, termasuk Indonesia
B. pelirikan sektor pertanian
C. bentuk ajakan kepada milineal 17. satu pun
D. peminatan sektor pertanian A. TIDAK PERLU DIPERBAIKI
E. bisnis sektor pertanian B. Satupun
C. satu-pun
14. Perbaikan kata merubah (kalimat 4) yang tepat adalah… D. satu pula
A. Mengubah E. satu hal
B. Pengubahan
C. Perubahan
18. mikroplastik
D. Pengubah
A. TIDAK PERLU DIPERBAIKI
E. Berubah
B. Mikroplastik
C. mikro-plastik
Tulisan berikut diikuti oleh beberapa butir pertanyaan. D. mikro plastic
Pertimbangkan apakah kata atau kalimat pada setiap E. mikro-palstik
nomor bercetak tebal TIDAK PERLU DIPERBAIKI (A)
19. supaya B. Perdagangan satwa liar di
A. TIDAK PERLU DIPERBAIKI Indonesia meningkat
B. Sehingga C. Polisi mengatakan kegiatan perdagangan satwa liar
C. Dan illegal
D. Lalu D. Perdagangan satwa liar sebanding kajahatan
E. Kemudian narkoba
20. Tertumpuknya E. Indonesia memiliki keragaman yang dapat
A. TIDAK PERLU DIPERBAIKI diperjualbelikan
B. Penumpukan
C. Menumpuknya 24. Simpulan keseluruhan teks di atas adalah…
D. Ditumpuknya A. Perdagangan satwa liar memiliki jalur transaksi
E. tertumpuk yang kompleks; di Indonesia angka kejahatan
tersebut terus meningkat
Bacalah teks berikut untuk menjawab soal No. 21 s.d. B. Satwa liar diperjualbelikan dan nilainya sebanding
27! dengan kejahatan narkotik, perdagangan manusia,
(1) Secara global, Asia merupakan pusat perdagangan dan senjata api
satwa liar yang dilindungi. (2) Kawasan ini menjadi C. Perdagangan satwa liar di Indonesia dari tahun ke
sumber, jalur transit, sekaligus pasar satwa langka. (3) tahun terus meningkat
Sebagian satwa yang diperjualbelikan termasuk jenis D. Asia, termasuk Indonesia, menjadi jalur penjualan
satwa terancam punah dan bernilai jual tinggi, misalnya satwa liar yang terancam punah
harimau, gajah, badak, hiu, dan orang utan. E. Kejahatan perdagangan satwa liar di Indonesia
(4) Kejahatan ini membesar seiring dengan menempati urutan ketiga dan terus meningkat
perkembangan waktu. (5) Menurut International
Enforcement Agency, bahwa nilai perdagangan global 25. Kata yang penulisannya tidak sesuai ejaan pada teks di
satwa liar setara dengan nilai perdagangan manusia, atas adalah..
narkotik, dan senjata.(6) Kegiatan ini menggunakan jalur A. narkotik (kalimat 5 dan 9)
transaksi yang rumit dan melalui perniagaan elektronik, B. Wildlife Conservation Society (kalimat 10)
marketplace, dan media sosial. C. marketplace (kalimat 6)
(7) Indonesia—dengan berbagai ragam kekayaan D. terorisme (kaliamt 9)
hayati—menjadi sumber terpenting perdagangan gelap E. Rp13 triliun (kalimat 10)
satwa liar yang dilindungi . (8) Kepolisian menyebutkan,
kegiatan ilegal ini menduduki urutan ketiga kejahatan di 26. Kalimat yang tidak efektif pada teks di atas adalah
Tanah Air. (9) Jumlahnya hanya kalah apabila A. Kalimat (1)
dibandingkan dengan perdagangan narkotik dan terorisme. B. Kalimat (4)
(10) Lembaga yang bergerak dalam pelindungan satwa, C. Kalimat (5)
Wildlife Conservation Society Indonesia Program, mencatat D. Kalimat (7)
bahwa perdagangan gelap satwa liar di Indonesia pada 2016 E. Kalimat (10)
meningkat empat kali lipat sejak 2010 dengan nilai
perdagangan mencapai Rp13 triliun per tahun. 27. Pengunaan tanda baca yang salah pada teks di atas
terdapat pada kalimat…
21. Simpulan paragraf pertama teks di atas adalah… A. Kalimat (1)
A. Asia sebagai pusat transit perdagangan hewan B. Kalimat (2)
B. satwa-satwa liar bernilai jual tinggi ada di Asia C. Kalimat (5)
C. Asia sebagai pusat perdagangan D. Kalimat (7)
hewan terancam punah E. Kalimat (8)
D. perdagangan satwa liar yang terancam punah
E. satwa liar yang terancam punah
diperjualbelikan 28. Dalam Konferensi Internasional Kebudayaan Papua itu
tidak diputuskan tempat penyelenggaraan konferensi
22. Simpulan paragraf kedua teks di atas adalah…. berikutnya.
A. perdagangan satwa memiliki jalur transaksi yang Kalimat yang memiliki pola yang sama dengan kalimat
kompleks di atas adalah…
B. kejahatan perdagangan satwa A. Untung Subagyo, pejabat yang terpilih itu,
C. perkembangan kejahatan perdagangan satwa mengawali kariernya dari bawah.
D. perdagangan satwa liar sebanding penjualan B. Seperti telah kita ketahui, perekonomian Amerika
senjata Serikat sangat berpengaruh.
E. kejahatan perdagangan satwa terus membesar C. Telah kami bicarakan dalam rapat terdahulu bahwa
kenaikan gaji pegawai baru bisa dilaksanakan tahun
23. Simpulan paragraf ketiga teks di atas adalah… depan.
A. Indonesia sumber perdagangan satwa liar D. Kita sadari bahwa bencana alam dapat datang
kapan saja tanpa kita duga.
E. Berdasarkan pengarahan pimpinan, pembangunan
kampus baru itu dapat dilakukan secara bertahap

29. Akibat vertical speed yang terlalu tinggi, pesawat


menjadi sulit dikontrol saat roda hendak menyentuh
landasan sehingga benturan keras terjadi dan pesawat
pun bisa terpental ke luar landasan.
Kalimat inti dari kalimat di atas adalah ....
A. vertical speed terlalu tinggi
B. pesawat sulit dikontrol
C. roda pesawat menyentuh landasan
D. terjadi benturan keras
E. pesawat terpental ke luar landasan

30. Kalimat yang polanya tidak sama dengan kalimat yang


lain adalah ....
A. Banyak buku dan tulisan mampu mengubah nasib
sebuah bangsa.
B. Kegiatan membaca akan dapat membangun
karakter masyarakat.
C. Setiap pelajar diharuskan menghasilkan sebuah
tulisan pendek.
D. Membaca tidak cukup dijadikan sebagai ajakan
atau imbauan.
E. Mereka pasti akan menemukan buku yang tepat.
BIMBINGAN BELAJAR
“STAR SCIENCE EDUCATION SIDIKALANG” FOKUS LULUS UTBK SBMPTN
Be a Star With The Star

angkot, juga semakin menyusut karena dijual ke luar daerah


atau tidak diperpanjang izin trayeknya akibat semakin
sepinya penumpang.

Bacalah teks berikut untuk menjawab soal No 1 s.d. 4! 5. Pernyataan yang tidak berhubungan dengan isi bacaan di
atas adalah ....
(1)Penggunaan antibiotik secara tidak rasional dan masif
A. angkutan perkotaan ternyata menambah beban
pada manusia serta hewan ternak memicu kekebalan bakteri
konsumen
terhadap antibiotik. (2) Bahkan, beberapa di antaranya
B. penumpang dari desa harus naik angkutan lebih dari
mengembangkan multi-resistensi terhadap antibiotik. (3)
satu kali untuk menuju pusat pemerintahan
Kini sudah muncul superbug yang kebal terhadap hampir
C. biaya perpanjangan trayek yang mahal membuat
semua antibiotik yang ada di pasaran. (4) Kasus ini agar
pemilik angkutan tidak memperpanjang izin
mendorong pakar mikrobiologi Indonesia—Windi Indra
trayeknya
Muziasari— meneliti gen resisten antibiotik pada bakteri
D. terjadi perubahan jumlah kendaraan di Gunungkidul
di Helsinki University, Finlandia.
E. penumpang semakin berkurang sehingga banyak
1. Penulisan kata yang tidak mengikuti ejaan pada teks di angkutan yang tidak beroperasi
atas adalah…
6. Ide pokok bacaan di atas adalah ....
A. Antibiotik
A. hasil kajian terhadap penataan angkutan di
B. multi-resistensi
Gunungkidul
C. superbug
B. penyatuan angkot dan angkudes
D. kebal
C. alasan keberatan penumpang angkutan
E. resisten
D. menyusutnya jumlah angkutan di Gunungkidul
E. sepinya penumpang angkutan di Gunungkidul
2. Apa inti kalimat (3)?
A. kebal terhadap abiotic
Teks ini untuk menjawab nomor 7 s.d 13!.
B. superbag yang kebal
Kondisi tanah dan iklim di Indonesia memang
C. sudah muncul superbag
sesuai untuk perkebunan kelapa sawit. Sumbangan ekspor
D. superbag kebal terhadap antibiotic
sawit juga merupakan salah satu yang tertinggi di antara
E. muncul superbag yang kebal
lima komoditas lainnya. Hasil riset Perkumpulan Prakarsa
\
menyebutkan, minyak sawit menjadi komoditas
3. Konjungsi yang tidak tepat pada teks di atas adalah…
penyumbang ekspor terbesar di Indonesia kurun 1989-2017,
A. dan (kalimat 1)
dengan rata-rata pertumbuhan nilai ekspor minyak sawit per
B. serta (kalimat 1)
tahun mencapai 2.782 persen. Produktivitas kelapa sawit
C. bahkan (kalimat 2)
lebih tinggi dibanding jenis tanaman lain dalam
D. yang (kalimat 3)
menghasilkan minyak nabati. Tingkat produksi minyak
E. agar (kalimat 4)
sawit mencapai 3,6 ton per hektar per tahun untuk periode
1970-2017. Saat ini, produksi CPO Indonesia sebesar 44
4. Padanan kata pakar (kalimat 4) pada teks di atas
juta ton sampai dengan 46 juta ton per tahun, dengan luas
adalah…
lahan sebesar 14 juta hektare.
A. Peneliti
Badan Pengkajian dan Penerapan Teknologi
B. Akademisi
(BPPT) memperkirakan produksi sawit akan mencapai 51,7
C. Eksper
juta ton pada 2025. Jika ini terus berlanjut, kondisi
D. Intelektual
oversupply CPO bisa menimpa Indonesia pada 2030
E. doktor
mendatang. Kondisi ini menurut Kepala BPPT Hammam
Riza, bagaikan pedang bermata dua. Jika kelebihan produksi
Bacaan ini dipergunakan untuk menjawab soal nomor 5 sawit bisa diserap oleh pasar, maka hal tersebut sangat baik
sampai 6. untuk industri termasuk kesejahteraan petani. Di sisi lain,
Dinas Perhubungan Gunungkidul telah melakukan kajian oversupply produksi sawit ini juga bisa mengakibatkan
terhadap penataan angkutan di daerah tersebut. Armada kelebihan pasokan yang pada akhirnya membuat harga jual
angkutan perkotaan yang ada ternyata menambah beban produk ini menjadi lebih rendah.
konsumen, terutama yang berasal dari daerah-daerah Sebab, sebagaimana hukum ekonomi yang berlaku,
sehingga angkutan perkotaan (angkot) akan dilebur menjadi "Apabila volume produksi melebihi kebutuhan pasar, maka
satu dalam angkutan pedesaan (angkudes). Para penumpang kelebihan pasokan akan membuat harga jual menjadi jatuh.
yang berasal dari desa terpaksa berganti angkutan untuk Tentunya hal itu merugikan petani maupun pengusaha
sampai ke kota, baik pusat pemerintahan kabupaten, pasar, perkebunan sawit," jelas Hammam kepada Tirto. Sebagai
maupun tempat lainnya sehingga menambah biaya. Di catatan, data dari Gabungan Pengusaha Kelapa Sawit
samping itu, jumlah kendaraan, khususnya angkudes dan Indonesia (GAPKI) menunjukkan bahwa ekspor minyak
147
SSE SIDIKALANG : Jl. A. Yani no 56 & 58 Bimbel Sse Sidikalang bimbelsse
BIMBINGAN BELAJAR
“STAR SCIENCE EDUCATION SIDIKALANG” FOKUS LULUS UTBK SBMPTN
Be a Star With The Star

sawit (Crude Palm Oil/CPO dan produk turunannya, A. Oleochemical dan Biodiesel merupakan produk
Biodiesel dan Oleochemical) secara keseluruhan mencapai turunan dari kelapa sawit yang sedang banyak
34,71 juta ton di 2018. Angka ini meningkat 8 persen dari digunakan oleh masyarakat.
32,18 juta ton di 2017. Sementara itu, di dalam negeri, B. Oleochemical digemari oleh masyarakat,
penyerapan biodiesel melalui program B20 mencapai 3,8 sedangkan Biodiesel tidak digemari oleh
juta ton pada 2018, naik 72 persen dibandingkan 2017. masyarakat.
C. Oleochemical dan Biodiesel bukan merupakan
produk turunan dari kelapa sawit.
D. Oleochemical dan Biodiesel tidak digemari oleh
masyarakat.
E. Oleochemical merupakan produk turunan dari
kelapa sawit, sedangkan Biodiesel bukan
merupakan produk turunan dari kelapa sawit.

10. Manakah pernyataan dari teks diatas yang BENAR?


A. Kondisi oversupply mengakibatkan harga jual
produk meningkat.
B. Kelapa sawit memiliki produktivitas yang lebih
rendah dalam menghasilkan minyak nabati
dibanding tanaman lain.
C. Minyak sawit merupakan komoditas penyumbang
ekspor terbesar di Indonesia.
D. Penyerapan biodiesel dalam negeri naik 72 persen
dibandingkan tahun 2016.
E. Kesejahteraan petani menurun jika kelebihan
produksi sawit mudah diterima oleh pasar.
F.
7. Berdasarkan paragraf 1 dan 2, manakah simpulan 11. Pernyataan yang sesuai dengan wacana diatas,
yang tepat? kecuali…
A. Semua komoditas Indonesia memiliki nilai A. Produksi CPO Indonesia berkisar 44-46 juta ton
pertumbuhan ekspor yang tinggi dan mengalami per tahun.
oversupply. B. Kondisi oversupply mungkin dapat terjadi di
B. Sebagian komoditas Indonesia memiliki nilai Indonesia pada tahun 2030.
pertumbuhan ekspor yang tinggi dan mengalami C. Kelebihan pasokan kelapa sawit dapat
oversupply. mengakibatkan turunnya harga jual.
C. Semua komoditas Indonesia memiliki nilai D. Biodiesel adalah salah satu produk turunan kelapa
pertumbuhan ekspor yang rendah dan mengalami sawit.
oversupply. E. Tingkat penyerapan biodiesel menurun jika
D. Sebagian komoditas Indonesia memiliki nilai dibandingkan dengan tahun lalu.
pertumbuhan ekspor yang tinggi dan mengalami
undersupply. 12. Berdasarkan tabel, manakah simpulan yang BENAR?
E. Semua komoditas Indonesia memiliki nilai A. Perkebunan negara mengalami tren peningkatan
pertumbuhan ekspor yang rendah dan mengalami pada tahun 2014-2016.
undersupply. B. Perkebunan swasta mengalami tren penurunan
pada tahun 2016-2018.
8. Simpulan apa yang dapat ditarik dari paragraf 2? C. Perkebunan rakyat mengalami tren penurunan
A. Kelebihan produksi sawit sebanding dengan pada tahun 2014-2018.
permintaan pasar. D. Total ketiga jenis perkebunan mengalami tren
B. Kelebihan produksi sawit mudah diterima oleh peningkatan pada tahun 2014-2018.
pasar. E. Total ketiga jenis perkebunan mengalami tren
C. Kelebihan produksi sawit berbanding lurus dengan penurunan pada tahun 2016-2018.
permintaan pasar.
D. Kelebihan produksi sawit berbanding terbalik 13. Berdasarkan tabel, manakah simpulan yang BENAR?
dengan permintaan pasar. A. Perkebunan negara merupakan penyumbang
E. Kelebihan produksi sawit sesuai dengan ekspektasi minyak sawit terbesar kedua pada tahun 2018.
pasar. B. Perkebunan rakyat memproduksi minyak sawit
9. Berdasarkan paragraf 3, manakah simpulan yang yang lebih banyak dibandingkan dengan
paling tepat? perkebunan swasta pada tahun 2014.
148
SSE SIDIKALANG : Jl. A. Yani no 56 & 58 Bimbel Sse Sidikalang bimbelsse
BIMBINGAN BELAJAR
“STAR SCIENCE EDUCATION SIDIKALANG” FOKUS LULUS UTBK SBMPTN
Be a Star With The Star

C. Perkebunan swasta memproduksi minyak sawit harus berdasarkan kondisi seseorang yang
sekitar 10 kali lipat daripada perkebunan negara mengonsumsinya. (23) Seseorang yang memiliki riwayat
pada tahun 2018. gangguan pencernaan, seperti mag atau tukak lambung,
D. Perkebunan rakyat memproduksi minyak sawit tidak disarankan untuk mengonsumsi ibuprofen. (24) Hal
setengah dari total ketiga perkebunan sawit pada ini dikarenakan ibuprofen dapat melukai lambung.(25)
tahun 2015. Namun, untuk seseorang yang memiliki riwayat gangguan
E. Jumlah minyak sawit terbesar yang diproduksi pada liver atau hati, tidak disarankan mengonsumsi
oleh perkebunan swasta berada pada tahun 2017. parasetamol. (26) Karena diketahui proses metabolisme
parasetamol adalah di liver, mengonsumsi parasetamol
Bacalah teks berikut untuk menjawab soal No. 14 s.d. 20! dapat meningkatkan beban kerja liver. (27) Oleh karena itu,
(1)Demam dan sakit kepala adalah tanda paling umum sebelum melakukan pemilihan obat antara ibuprofen dan
yang muncul ketika ada gangguan pada tubuh. (2) parasetamol, berkonsultasilah dengan dokter atau apoteker
Walaupun tidak selalu menandakan terjadinya penyakit agar bisa mencapai efek terapi sesuai dengan yang
yang parah, tetapi kedua gejala tersebut tetap dibutuhkan.
mengakibatkan kecemasan, terutama jika terjadi pada anak. 14. Judul yang tepat untuk teks di atas adalah…
(3) Oleh karena itu, tetap perlu dilakukan upaya untuk A. Obat yang Tepat untuk Mengatasi Demam dan
meredakannya. (4) Obat untuk meredakan demam dan sakit Sakit Kepal
kepala dijual secara bebas, di apotek ataupun di B. Khasiat Ibuprofen dan Parasetamol
supermarket. (5) Parasetamol dan ibuprofen merupakan dua C. Perbedaan Ibuprofen dan Parasetamol
bahan aktif obat yang umumnya dikonsumsi oleh D. Fungsi dan Cara Kerja Ibuprofen dan Parasetamol
masyarakat. (6) Keduanya diketahui ampuh dapat E. Pilih Ibuprofen atau Parasetamol?
meredakan demam pada anak atau orang dewasa. (7) 15. Kalimat yang perlu diperbaiki dari teks di atas
Namun, saat memilih obat, sering kali terjadi kebingungan adalah…
terkait obat yang lebih baik, parasetamol atau ibuprofen. A. kalimat 2
(8) Ibuprofen adalah golongan obat yang masuk dalam B. kalimat 3
obat antiinflamasi nonsteroid atau biasa disingkat dengan C. kalimat 7
NSAID. (9) Ibuprofen bekerja dengan mencegah tubuh D. kalimat 14
membuat senyawa yang menyebabkan peradangan. (10) E. kalimat 16
Oleh karena itu, dapat membantu mengurangi bengkak, rasa
sakit, atau demam.(11) Ibuprofen mulai memiliki efek
sekitar 30—60 menit setelah dikonsumsi. (12) Pada orang 16. Gagasan utama paragraf pertama teks di atas adalah…
dewasa, ibuprofen memiliki kadar tertinggi dalam tubuh, A. Demam dan sakit kepala adalah gejala utama
yaitu, 2 jam setelah dikonsumsi untuk tablet, 62 menit gangguan pada tubuh.
untuk tablet kunyah dan 47 menit untuk ibuprofen bentuk B. Diperlukan upaya untuk meredakan demam dan
sirup. (13) Pada anak-anak, untuk mencapai kadar tertinggi sakit kepala.
dalam tubuh, ibuprofen membutuhkan waktu sekitar 86 C. Masyarakat sering kali bingung memilih ibuprofen
menit setelah dikonsumsi untuk tablet kunyah dan 58 menit atau parasetamol.
untuk bentuk sirup. D. Ibuprofen dan parasetamol ampuh meredakan
(14) Sementara itu, parasetamol adalah obat golongan demam.
analgesik, pereda nyeri dan antipiretik, atau pereda demam. E. Ibuprofen dan parasetamol umum dikonsumsi
(15) Parasetamol bekerja pada hipotalamus agar terbentuk masyarakat untuk meredakan demam dan sakit
senyawa untuk meredakan demam. (16) Parasetamol juga kepala.
dapat meredakan nyeri dengan mencegah pembentukan
impuls nyeri atau dengan menghambat pembentukan 17. Perbaikan yang tepat dari kalimat (10) pada teks di atas
senyawa yang dapat menyebabkan timbulnya nyeri. (17) adalah…
Parasetamol membutuhkan waktu 1 jam setelah dikonsumsi A. mengganti kata oleh karena itu menjadi jadi
untuk memberikan efek kerjanya dan memiliki kadar B. mengganti kata atau menjadi dan
tertinggi setelah 6 jam dikonsumsi. (18) Namun, seiring C. mengganti kata mengurangi menjadi meringankan
berkembangnya zaman, terdapat beberapa produk tablet D. menambahkan subjek sebelum kata dapat
parasetamol yang bekerja secara cepat sehingga untuk membantu
mencapai kadar tertinggi, hanya dibutuhkan waktu 10—60 E. menghilangkan kata oleh karena itu
menit.
(19) Parasetamol dapat meredakan nyeri, tetapi dalam 18. Kalimat yang tidak perlu diperbaiki pada teks di atas
skala yang ringan. (20) Jika nyeri yang dialami karena adalah…
terdapat peradangan, sebaiknya kita menggunakan A. kalimat 18
ibuprofen. (21) Ibuprofen merupakan obat yang juga dapat B. kalimat 20
mengatasi peradangan. (22) Selain berdasarkan fungsi atau C. kalimat 24
kecepatan obat memberikan hasil, pemilihan obat juga D. kalimat 26
149
SSE SIDIKALANG : Jl. A. Yani no 56 & 58 Bimbel Sse Sidikalang bimbelsse
BIMBINGAN BELAJAR
“STAR SCIENCE EDUCATION SIDIKALANG” FOKUS LULUS UTBK SBMPTN
Be a Star With The Star

E. kalimat 27 karakteristik dari transisi sehat menuju kedewasaan dan


bagian penting menemukan rasa diri. (15) Dukungan orang
19. Kesalahan penggunaan tanda koma pada teks di atas tua sangat penting untuk memastikan remaja berkembang
terdapat pada kalimat… menjadi orang dewasa yang berpengetahuan luas dengan
A. kalimat 2 harga diri yang kuat pula untuk tetap mandiri dan mampu
B. kalimat 7 mengendalikan hidup. (16) Dorongan dan penguatan adalah
C. kalimat 12 dua faktor penting yang akan memastikan anak remaja ada
D. kalimat 23 bersama orang tua dan tidak menjauh.
E. kalimat 26
21. Berdasarkan teks di atas, simpulan yang tepat dari
keseluruhan isi teks yang tepat adalah…
20. Simpulan teks di atas yang tepat adalah… A. Anak mengalami perubahan pada masa remaja,
A. Ibuprofen dan parasetamol sama-sama ampuh baik perubahan secara fisik maupun secara
dalam meredakan demam dan nyeri. emosional.
B. Ibuprofen memberikan efek 30—60 menit setelah B. Masa remaja adalah proses pencarian jati diri dan
dikonsumsi, sedangkan parasetamol memberikan identitas.
efek 1 jam setelah dikonsumsi. C. Orang tua harus memahami perubahan
C. Ibuprofen termasuk obat antiinflsmasi nonsteroid, karakteristik anak remajanya.
sedangkan parasetamol termasuk obat analgesik, D. Orang tua harus memberikan dukungan dan
pereda nyeri dan antipiretik, atau pereda demam. penguatan pada masa transisi anak remajanya agar
D. Ibuprofen berfungsi mengobati nyeri disertai hubungan mereka tidak menjauh.
peradangan dan tidak disarankan dikonsumsi oleh E. Orang tua merasa anak remajanya semakin jauh
penderita mag atau tukak lambung, sedangkan dari mereka.
parasetamol berfungsi mengobati nyeri ringan dan
tidak disarankan dikonsumsi oleh penderita 22. Berdasarkan teks di atas, kesalahan PUEBI terdapat
gangguan liver dalam kalimat…
E. Pemilihan ibuprofen dan parasetamol harus A. 1 dan 4
berdasarkan fungsi, kecepatan memberikan hasil, B. 4 dan 5
dan kondisi seseorang yang mengonsumsinya C. 4 dan 8
dibutuhkan. D. 8 dan 11
E. 11 dan 14
23. Berdasarkan teks di atas, penggunaan konjungsi yang
Teks berikut untuk menjawab soal No. 21 s.d. 25! tidak tepat terdapat pada…
(1)Sebagian orang tua mengeluhkan anak remajanya A. kalimat 4
yang menuntut lebih banyak privasi. (2) Mereka merasa B. kalimat 7
sulit untuk menyesuaikan dengan situasi itu. (3).... (4) C. kalimat 8
Alhasil, orang tuapun merasakan anak remajanya semakin D. kalimat 10
jauh dari mereka. E. kalimat 15
(5) Menurut psikolog anak dari Flinto R&D Center,
Divya Palaniappan, kekhawatiran orang tua akan kasus 24. Berdasarkan teks di atas, kalimat yang tepat untuk
seperti itu masuk akal. (6) Masa remaja adalah masa yang mengisi bagian rumpang pada kalimat (3) adalah…
ditandai oleh perubahan. (7) Anak remaja mengalami A. Remaja tidak jarang melancarkan protes ketika
perubahan drastis, tidak hanya dalam penampilan fisik, ayah atau ibunya mencoba untuk campur tangan
melainkan juga secara emosional. dalam kehidupannya.
(8)“Ini adalah fase transisi menuju kedewasaan dan B. Remaja selalu merasa lebih nyaman berada di luar
ditandai oleh karakteristik keingintahuan seksual”, katanya, rumah daripada berada di dalam rumah.
dilansir laman Times of India. C. Remaja selalu merasa lebih nyaman bercerita
(9)Masa remaja juga adalah periode pencarian jati diri tentang masalahnya kepada orang lain daripada
dan identitas. (10) Jadi, secara alami masa itu keluarganya.
memunculkan masalah independen, pertanggung jawaban, D. Orang tua menjadi merasa tidak dipedulikan oleh
dan pengambilan keputusan yang berkaitan dengan anak remajanya.
identitas diri. (11) Perilaku remaja sering kali tidak terduga E. Remaja selalu menyalahkan orang tua yang banyak
dan impulsif. (12) Mereka mungkin menikmati banyak bertanya.
perilaku berisiko dan mengembangkan minat selama fase
ini. (13) Sebenarnya, mencari privasi adalah salah satu 25. Berdasarkan teks di atas, kalimat yang tidak efektif
karakteristik kunci masa remaja. terdapat pada…
(14) Menurut Divya, sebaiknya, sebagai orang tua, A. kalimat 9
harus memahami bahwa semua perubahan di atas adalah B. kalimat 11
150
SSE SIDIKALANG : Jl. A. Yani no 56 & 58 Bimbel Sse Sidikalang bimbelsse
BIMBINGAN BELAJAR
“STAR SCIENCE EDUCATION SIDIKALANG” FOKUS LULUS UTBK SBMPTN
Be a Star With The Star

C. kalimat 12
D. kalimat 14 29. (1)Indonesia mau mengadopsi model yang
E. kalimat 15 dikembangkan oleh Amerika Serikat dalam ekonomi
digital. (2) Model tersebut bernama plug and play,
26. Jadi, secara alami masa itu memunculkan masalah yakni perusahaan yang menyiapkan perkembangan
independen, pertanggung jawaban, dan pengambilan perusahaan rintisan. (3) Pemerintah secara serius
keputusan yang berkaitan dengan identitas diri. menyiapkan inkubator dan permodalan. (4) Hal
Kata yang bercetak tebal dalam kalimat di atas dapat tersebut dapat memudahkan pelaku usaha rintisan. (5)
diperbaiki menjadi… Dengan berbagai upaya itu, ekosistem ekonomi digital
A. independen dan pertanggungjawaban yang kompetitif akan terbentuk.
B. independensi dan pertanggungjawaban Kata beragam nonformal terdapat di dalam kalimat ….
C. independensi dan mempertanggungjawabkan A. (1)
D. independen dan tanggung jawab B. (2)
E. independensi dan tanggung jawab C. (3)
D. (4)
27. (1)Hobbit ditemukan tahun 2003 oleh tim arkeolog E. (5)
yang berafiliasi dengan Arkenas. (2) Tinggi jenis
manusia itu hanya 106 cm karena disebut manusia 30. Kalimat yang memenuhi kaidah kalimat efektif
kerdil. (3) Penampakannya mirip dengan manusia yang adalah…
hidup di Asia dan Afrika 1-3 juta tahun lalu. (4) A. Pelangi merupakan cahaya aneka warna dengan
Kontroversi pun kemudian muncul. (5) Sejumlah garis sejajar yang tampak dilangit.
ilmuwan menyatakan bahwa Hobbit merupakan jenis B. Pelangi muncul karena butiran-butiran air hujan
manusia tersendiri, sedangkan beberapa ilmuwan lain bertebaran di atmosfer pada saat sebelum atau
menganggapnya bagian dari Homo erectus atau bahkan setelah hujan,terkena sinar matahari.
Homo sapiens yang mengalami kecacatan. C. Sinar matahari ini akan dipatahkan oleh butiran-
Kata sambung yang tidak tepat pada paragraf di atas butiran air hujan.
adalah… D. Pada saat titik air hujan membiaskan cahaya dan
A. yang pada kalimat (1) menghasilkan deretan warna yang berbeda-beda.
B. karena pada kalimat (2) E. Deretan warna berbeda-beda antara satu sama lain
C. yang pada kalimat (3) yaitu spektrum.
D. sedangkan pada kalimat (5)
E. bahkan pada kalimat (5)

28. (1)Hampir satu setengah abad lalu, Dewi Sartika dan


R.A. Kartini memperjuangkan emansipasi. (2)
Perjuang mereka terbatas pada hak perempuan
terhadap akses pendidikan, bukan lainnya. (3) Buah
perjuangan mereka dapat kita rasakan, saat ini. (4)
Kaum perempuan sekarang mempunyai akses yang
luas terhadap pendidikan; dari pendidikan dasar hingga
universitas. (5) Benarkah semangat perjuangan kedua
perempuan tersebut tetap berada pada jalurnya? (6)
Kali ini, dalam rangka memperingati hari lahir R.A.
Kartini, kami akan membicarakan kiprah sejumlah
wanita Indonesia.
Dua tanda baca yang salah dalam paragraf di atas
adalah ….
A. tanda koma sesudah kata lalu kalimat (1) dan tanda
koma sesudah kata pendidikan pada kalimat (2)
B. tanda koma sesudah kata rasakan pada kalimat (3)
dan tanda titik koma sesudah kata pendidikan pada
kalimat (4)
C. tanda titik pada kata R.A. Kartini pada kalimat (1)
dan tanda titik pada kalimat (3)
D. tanda koma sesudah kata ini dan tanda koma
sesudah kata Kartini pada kalimat (6)
E. tanda titik pada kalimat (2) dan tanda tanya pada
kalimat (5)
151
SSE SIDIKALANG : Jl. A. Yani no 56 & 58 Bimbel Sse Sidikalang bimbelsse
BIMBINGAN BELAJAR
“STAR SCIENCE EDUCATION SIDIKALANG” FOKUS LULUS UTBK SBMPTN
Be a Star With The Star

(D) Target pencapaian program pemberian ASI


eksklusif sebaiknya tidak terlalu tinggi.
(E) Pemberian ASI pada bayi sangat bermanfaat bagi si
ibu dan masyarakat pada umumnya.

Bacalah teks berikut untuk menjawab soal nomor 01 Bacalah teks berikut untuk menjawab soal nomor 03
sampai nomor 02. sampai nomor 04.
Berbagai penelitian telah mengkaji manfaat Di antara kearifan lokal yang dihasilkan dari pengalaman
pemberian air susu ibu (ASI). ASI eksklusif menurunkan adaptasi masyarakat dengan lingkungannya adalah adanya
mortalitas bayi dan morbiditas bayi, mengoptimalkan konsep hutan larangan. Konsep tersebut merupakan
pertumbuhan bayi, membantu perkembangan kecerdasan pandangan yang bersumber pada pengetahuan masyarakat
anak, dan memperpanjang jarak kehamilan ibu. Di dalam upaya pengelolaan lingkungan secara tradisional.
Indonesia, Kementerian Kesehatan Republik Indonesia Melalui konsep hutan larangan, masyarakat menerapkan
melalui program perbaikan gizi masyarakat menargetkan norma pengendali sikap dan perilaku hidup dalam
cakupan ASI eksklusif 6 bulan sebesar 80%. Namun, angka pengelolaan hutan. Hutan dianggap sebagai habitat warisan
ini sulit dicapai, bahkan tren prevalensi ASI eksklusif dari yang perlu dipertahankan. Oleh karena itu, pada komunitas
tahun ke tahun terus menurun. Data Survei Demografi dan masyarakat tertentu berkembang kearifan lokalyang
Kesehatan Indonesia 1997—2007 memperlihatkan ditujukan untuk menjaga kelestarian fungsi lingkungan
penurunan prevalensi ASI eksklusif dari 40,2% pada tahun hidup.
1997 menjadi 39,5%
dan 32% pada tahun 2003 dan 2007. 03. Pernyataan yang paling tepat dengan isi bacaan di atas
Alasan kegagalan praktik ASI eksklusif bermacam- adalah ...
macam, antara lain budaya pemberian makanan pralaktal, (A) Hutan dianggap sebagai warisan yang perlu
keharusan pemberian tambahan susu formula karena ASI dilestarikan.
tidak keluar, penghentian pemberian ASI karena bayi atau (B) Pelestarian hutan dapat dilakukan melalui konsep
ibu sakit, ibu harus bekerja, dan ibu yang ingin mencoba hutan larangan.
susu formula. Studi kualitatif Fikawati Syafiq melaporkan (C) Konsep hutan larangan merupakan bentuk kearifan
bahwa faktor kegagalan ASI eksklusif terjadi karena ibu lokal.
kurang mempunyai pengetahuan dan pengalaman dan (D) Konsep hutan larangan dilakukan secara tradisional
karena ibu tidak difasilitasi melakukan inisiasi menyusu dini oleh masyarakat.
(IMD). (E) Konsep hutan larangan merupakan pemikiran untuk
Bayi yang lahir normal dan diletakkan di perut ibu menjaga habitat hutan.
segera setelah lahir dengan kulit ibu melekat pada kulit
bayiselama setidaknya 1 jam dalam 50 menit akan berhasil 04. Makna kearifan lokal yang sesuai dengan bacaan diatas
menyusu, sedangkan bayi lahir normal yang dipisahkan dari adalah ...
ibunya 50% tidak bisa menyusu sendiri. Berbagai studi juga (A) Kebijakan yang bersumber dari pengetahuan
melaporkan bahwa IMD terbukti meningkatkan keberhasilan masyarakat tertentu.
ASI eksklusif. (B) Cara pandang yang khas yang dimiliki oleh suatu
01. Bacaan di atas membahas ... masyarakat tertentu.
(A) inisiasi menyusui dini. (C) Pengetahuan yang hanya dimiliki masyarakat
(B) pemberian ASI pada bayi yang lahir normal. tertentu dalam menghadapi masalah dalam
(C) manfaat program pemberian ASI eksklusif selama lingkungannya.
6 bulan. (D) Pemahaman yang hanya dimiliki oleh masyarakat
(D) kesuksesan program pemberian ASI eksklusif tertentu yang tidak dimiliki masyarakat lain.
selama 6 bulan. (E) Sikap dan perilaku masyarakat tertentu dalam
(E) kendala pelaksanaan program pemberian ASI menghadapi masalah dalam lingkungannya.
eksklusif selama 6 bulan.
05. Dari perspektif sosiologi-politik, transisi demokrasi
02. Kesimpulan dari bacaan di atas adalah ... yang ditengarai oleh dominannya peran institusi partai
(A) Pencapaian target program pemberian ASI politik sebagai mesin demokrasi, dalam praktiknya akan
eksklusif selama 6 bulan dari tahun ke tahun terus menimbulkan berbagai distorsi yang berimplikasi
menurun. terhadap proses demokratisasi.
(B) Bayi yang lahir normal sebaiknya langsung Kata tengara dalam kata ditengarai dalam kalimat
dilekatkan dengan perut ibunya supaya dia dapat diatas bermakna ...
menyusu sendiri. (A) mata angin.
(C) Faktor utama kegagalan program pemberianASI (B) firasat.
eksklusif terletak pada kegiatan Inisiasi Menyusu (C) bentuk.
Dini. (D) tanda.
152
SSE SIDIKALANG : Jl. A. Yani no 56 & 58 Bimbel Sse Sidikalang bimbelsse
BIMBINGAN BELAJAR
“STAR SCIENCE EDUCATION SIDIKALANG” FOKUS LULUS UTBK SBMPTN
Be a Star With The Star

(E) lambang bagian dari hutan ketika, misalnya, mereka akan


membuka lahan ladang baru. Perlengkapan kapongo
06. Digitalisasi saat ini telah menjadi bagian kehidupan terdiri atas sirih, pinang, kapur, dan tembakau, yang
masyarakat dunia. Ancaman terhadap dunia digital diletakkan pada sebuah "rumah" yang tingginya sekitar
muncul. Untuk mencegah masalah tersebut, diperlukan 40–50 cm dari permukaan tanah.
proteksi terhadap kehidupan digital. Proteksi awal Kalimat topik pada paragraf ketiga dalam bacaan di atas
dalam mencegah masalah ituadalah menghapus surat adalah ...
elektronik yang meragukan, mengetik langsung alamat (A) Perlengkapan kapongo terdiri atas sirih,
situs internet ketika mengunjungi sebuah situs untuk pinang,kapur, dan tembakau.
pertama kali, menyimpan catatan transaksi online, dan (B) Suku Wana mencegah kemurkaan pemelihara
mengubah kata sandi secara berkala. Selainitu, lingkungan dengan memberi persembahan atau
perubahan kebiasaan buruk juga perlu dilakukan, seperti sesajen (kapongo).
menggunakan sandi yang sama saat login online dan (C) Suku Wana sangat mempercayai adanya ruh
tidak mengubahnya secara berkala. Proteksi juga dapat (spirit).
dilakukan dengan menggunakan sistem operasi yang (D) Setiap kerusakan lingkungan ataupun perubahan
asli.... siklus alam yang tidak berjalan sebagaimana
Kalimat yang dapat menjadi kalimat penutup pada teks biasanya, dipercayai sebagai tanda murkanya sang
di atas adalah ... penjaga.
(A) Dengan demikian, pengguna dapat memanfaatkan (E) Kehidupan orang Wana sangat tergantung pada
sistem digital dengan baik. kearifan lingkungan alam sekeliling mereka.
(B) Akhirnya, proteksi terhadap kehidupan digital
dapat tercipta. 08. Simpulan yang sesuai untuk bacaan di atas adalah....
(C) Sistem operasi asli saat ini sudah tersedia dengan (A) Suku Wana adalah suku terasing.
harga murah. (B) Hutan merupakan sumber kehidupan suku Wana.
(D) Dengan menggunakan sistem operasi yang (C) Suku Wana berpegang pada sistem adat.
asli,perangkat elektronik akan terhindar dari (D) SukuWana tinggal di Hutan Tropis Cagar Alam
virus,seperti trojan dan worm. Morowali.
(E) Dengan demikian, kehidupan digital dapat (E) Suku Wana adalah salah satu suku yang hidup
berlangsung tanpa masalah. bersama dengan alam.

07. (1) Suku Wana adalah satu suku terasing yang 09. Pernyataan yang sesuai dengan isi bacaan adalah ...
mendiami hutan belantara di wilayah Sulawesi Tengah. (A) Suku Wana terisolasi dari peradaban modern
Mereka disebut sebagai suku terasing karena wilayah karena tinggal di atas pohon.
mereka yang "terisolasi" dan keadaan ketidakberdayaan (B) Suku Wana sangat memedulikan kebersamaan
dari suku ini.Mereka membuat rumah dan tinggal di mereka dengan alam.
atas pohon. Suku ini pun enggan bertemu dengan orang (C) Setiap hari suku Wana mengerjakan lahan kebun
luar. Mereka telah lama terisolasi dari peradaban dan menuai tanaman.
modern. (D) Kapongo hanya disajikan ketika suku Wana
(2) Pola hidup suku itu dipengaruhi oleh sistem adat membuka lahan baru.
yang dianut oleh mereka. Mereka menyandarkan hidup (E) Suku Wana tidak sengaja mengisolasi keberadaan
mereka pada hutan adat. Penyikapan mereka terhadap mereka terhadap peradaban modern.
hutan yang dianggap sebagai petunjuk dalam menjalani
hidup memberi arah hidup orangWana sehari-hari. Di 10. Kepada anak-anak, orang berusia lanjut, serta orang
luar hari-hari mengerjakan lahan kebun dan masa dewasa yang mempunyai penyakit kronis, seperti
menuai tanaman, mereka menghabiskan hari-hari penyakit paru obstruktif menahun, diabetes melitus,
mereka di hutan tropis Cagar Alam Morowali. Bagi gagal ginjal kronik, sirosis hati, dan kanker sistem
suku Wana, hutan menyediakanapa saja yang darah, dianjurkan imunisasi pneumokok. Juga bagi
dibutuhkan mereka dalam hidup. mereka yang pernah mengalami operasi limpa
(3) Kehidupan orangWana sangat bergantung pada dianjurkan untuk menjalani imunisasi pneumokok. Pada
kearifan lingkungan alam sekeliling mereka. Oleh anak, imunisasi ini perlu diberikan beberapa kali
karena itu,mereka sangat memercayai adanya ruh bergantung pada umur anak. Namun, pada orang
(spirit) yang menjaga atau memelihara setiap jengkal dewasa cukup diberikan satu kali.
tanah dan hutan. Setiap kerusakan lingkungan ataupun Terdapat dua macam vaksin pneumokok, yaitu vaksin
perubahan siklus alam yang tidak berjalan sebagaimana polisakarida dan konjugat. Vaksin polisakarida dapat
biasanya dipercayai sebagai tanda murkanya sang diberikan kepada anak yang berumur dua tahun,
penjaga. Untuk mencegah kemurkaan para penjaga sedangkan jenis yang kedua dapat diberikan kepada
yang memelihara lingkungan, mereka memberi anak yang usianya lebih muda. Vaksin ini menurunkan
persembahan atau sesajen (kapongo) di setiap bukit atau risiko penularan kuman Streptococcus pneumonia.
153
SSE SIDIKALANG : Jl. A. Yani no 56 & 58 Bimbel Sse Sidikalang bimbelsse
BIMBINGAN BELAJAR
“STAR SCIENCE EDUCATION SIDIKALANG” FOKUS LULUS UTBK SBMPTN
Be a Star With The Star

Kuman ini, selain menimbulkan penyakit yang relatif terabaikan atau tidak berjalan karena adanya kebijakan
ringan, yaitu otitis media (infeksi pada telinga tengah) instan yang memberikan akses masuknya para pemodal,
dan sinusitis, dapat juga menimbulkan penyakit yang kerabat elite politik, dan kerabat birokrat tanpa melalui
berat, seperti pneumonia, meningitis (radang otak), dan seleksi yang terprogram.
bakteriemia (kuman ditemukandalam darah). Sebagian Gagasan utama kalimat di atas adalah ...
orang mengandung kuman ini di tenggorokan dan dapat (A) orientasi kekuasaan dan politik transisional di
menularkan kuman ini kepada orang lain melalui batuk kalangan partai politik dalam nuansa demokrasi
dan bersin. kapitalistik tersebut menguat
Penyakit meningitis dan bakteriemia disebut sebagai (B) menguatnya orientasi kekuasaan dan politik
penyakit pneumonia invasif. Meskipun pneumonia tidak transisional menjadikan fungsi kaderisasi politik
tergolong penyakit invasif, angka kematiannya tinggi, terabaikan
terutama pada anak dan orang berusia lanjut. Sekitar 52 (C) kebijakan instan memberikan akses masuknya para
persen orang yang berusia lanjut yang dirawat di pemodal, kerabat elite politik, dan kerabat birokrat
Rumah Sakit Cipto Mangunkusumo menderita (D) para pemodal, kerabat elite politik, dan kerabat
pneumonia. birokrat masuk tanpa melalui seleksi yang
terprogram
11. Topik bacaan di atas adalah ... (E) fungsi kaderisasi politik terabaikan atau tidak
(A) pneumonia. Berjalan
(B) imunisasi pneumokok.
(C) vaksin pneumokok. 15. Aurora adalah fenomena alam yang menyerupai
(D) penyakit yang disebabkan kuman pancaran cahaya yang menyala-nyala pada lapisan
Streptococcuspneumonia. ionosfer dari sebuah planet sebagai akibat adanya
(E) vaksin pneumokok bagi anak-anak dan dewasa. interaksi antara medan magnetik yang dimiliki planet
tersebut dengan partikel bermuatan yang dipancarkan
12. Pernyataan yang tidak terdapat dalam bacaan di atas oleh matahari.
adalah ... Gagasan utama dari kalimat di atas adalah ...
(A) Penyakit radang otak termasuk dalam jenis (A) Aurora adalah fenomena alam.
penyakit pneumonia. (B) Aurora menyerupai pancaran cahaya.
(B) Anak berusia satu tahun dapat divaksin asi dengan (C) Aurora menyala-nyala pada lapisan ionosfer.
vaksin konjugat. (D) Aurora muncul sebagai akibat adanya interaksi
(C) Jumlah pemberian vaksin pada anak-anak dan antara medan magnetik planet.
dewasa berbeda. (E) Aurora terjadi karena interaksi medan magnetik
(D) Vaksinasi pneumokok membasmi kuman planet dan partikel matahari.
Streptococcus pneumonia.
(E) Kuman Streptococcus pneumonia dapat menyebar 16. Dengan panjang saluran primer 280 km, 3.000 km
di dalam dan ke luar tubuh. saluran sekunder, 16 bendungan, dan sekitar 4.000 pintu
pembagi, daerah irigasi Jatiluhur, meliputi Kabupaten
13. Untuk kinerja implementasi fungsi anggaran, Bekasi, Karawang, Subang, dan sebagian Indramayu,
tampaknya juga terjadi interaksi oknum internal menjadi daerah terbesar di Indonesia.
anggota dewan dengan instansi sektoral atau dinas Kalimat tersebut bergagasan utama ....
tertentu yang beraroma kolusi dalam rangka (A) Daerah irigasi Jatiluhur meliputi Kabupaten Bekasi,
membesarkan anggaran sektoral dengan harapan Karawang, Subang, dan sebagian Indramayu.
mendapatkan imbalan atau komisi. (B) Daerah irigasi Jatiluhur mempunyai panjang
Gagasan utama kalimat di atas adalah ... saluran primer 280 km, 3.000 km saluran sekunder,
(A) kinerja implementasi fungsi anggaran tampaknya 16 bendungan, dan sekitar 4.000 pintu pembagi.
terjadi interaksi. (C) Terdapat dua jenis saluran, bendungan, dan pintu
(B) terjadi interaksi oknum internal anggota dewan pembagi di Daerah Irigasi Jatiluhur.
dengan instansi sektoral atau dinas tertentu. (D) Daerah irigasi Jatiluhur menjadi daerah terbesar.
(C) dalam rangka membesarkan anggaran sektoral (E) Keluasan Daerah Irigasi Jatiluhur.
dengan harapan mendapatkan imbalan atau komisi.
(D) tampaknya terjadi interaksi internal dengan 17. Film itu menyodorkan fakta seputar peristiwa penting
sektoral. yang luput dari perhatian masyarakat.
(E) tampaknya juga terjadi interaksi antara oknum Kata yang bercetak miring secara tepat bermakna ...
internal dan instansi sektoral (A) menyuguhkan-hilang.
(B) menggambarkan-lepas.
14. Menguatnya orientasi kekuasaan dan politik transisional (C) menyajikan-terlepas.
di kalangan partai politik dalam nuansa demokrasi (D) memberikan-meleset.
kapitalistik menjadikan fungsi kaderisasi politik (E) menjelaskan-terhindar.
154
SSE SIDIKALANG : Jl. A. Yani no 56 & 58 Bimbel Sse Sidikalang bimbelsse
BIMBINGAN BELAJAR
“STAR SCIENCE EDUCATION SIDIKALANG” FOKUS LULUS UTBK SBMPTN
Be a Star With The Star

(E) Kita dapat menempatkan segala sesuatu dengan


18. Sebagai perbandingan, dengan biaya penggalian rata- baik
rata mencapai sekitar Rp10 juta, metode eksplorasi
tradisional sangatlah mahal. 22. Masalah eutanasia kini sedang ramai diperdebatkan
Kata tradisional dalam kalimat di atas bersinonim orang. Beberapa saat lalu kita tercengang kembali atas
dengan kata ... upaya pengesahan eutanasia dalam hukum Belanda.
(A) sehari-hari. Seseorang yang menderita penyakit yang tak terobati
(B) lama. berhak memilih mati untuk mengahiri penderitaanya
(C) lampau. dan UU yang menjamin hak tersebut perlu disahkan.
(D) biasa. Angket yang disebarkan pun menunjukan paling sedikit
(E) adat. tiga perempat penduduk Belanda cenderung memilih
hak individu untuk mati. Persentase ini mengecewakan
19. Tampilnya para calon legislatif yang bernuansa kaum rohaniawan di sana yang selama ini berusaha
kekerabatan politik dengan basis rekam jejak dengan kukuh dan sengit memerangi upaya bunuh diri.
latarbelakang yang masih diragukan kompetensi dan Pertanyaan yang sesuai dengan kutipan di atas adalah
militansinya sebagai politisi dan negarawan karena ….
tanpa melalui proses seleksi dan kaderisasi menurunkan (A) Sebagian besar penduduk di belanda menyetujui
kualitas lembaga legislatif dalam melaksanakan fungsi eutanasia
legislasi, pengawasan, dan anggaran. (B) Sebagian besar penduduk di Belanda kecewa
Bagian kalimat yang tidak perlu digunakan dalam terhadap pelaksanaan eutanasia
kalimat tersebut adalah ... (C) Sebagian besar penduduk di Belanda mentoleril
(A) tampilnya para calon legislatif yang bernuansa eutanasia
kekerabatan politik. (D) Sebagian besar penduduk di Belanda
(B) yang bernuansa kekerabatan politik dengan basis memperdebatkan eutanasia
rekam jejak latar belakang. (E) Sebagian besar penduduk di belanda menerapkan
(C) masih diragukan kompetensi dan militansinya. eutanasia
(D) sebagai politisi dan negarawan karena tanpa
melalui proses seleksi dan kaderisasi. 23. Aktivitas anak Gunung Krakatau di selat Sunda
(E) dalam melaksanakan fungsi legislasi, pengawasan, menunjukan peningkatan yang cukup tajam dalam dua
dan anggaran. hari terakhir ini. Setidaknya, selasa, kemarin terjadi
gempa vulkanik dalam dan dangkal di bawah gunung
20. Kejadian ini mencoreng …. pemerintah dalam itu sebanyak 32 kali, lebih tinggi dibandingkan
pelaksanaan ibadah haji. biasanya yang hanya 2-9 kali. Peningkatan aktivitas
Kata yang tepat untuk mengisi rumpang pada kalimat di tersebut terjadi sejak selasa, pukul 00.00 WIB. Akibat
atas adalah …. peningkatan aktivitas ini, Direktorat Vulkanologi dan
(A) prioritas Mitigasi Bencana Geologi meningkatkan statusnya
(B) stabilitas menjadi waspada ( level II ) dari ststus aktif normal
(C) kreativitas ( level I ).
(D) kredibilitas Kutipan di atas dapat digunakan dengan kalimat ….
(E) fasilitas (A) Aktivitas anak Gunung Krakatau di Selat Sunda
meningkat dalam dua hari terakhir sehingga
21. Salah satu hal penting dalam manajemen diri adalah ststusnya menjadi level II.
keterampilan mengukur diri. Artinya, kita harus mampu (B) Akhirnya anak Gunung Krakatau di selat sunda
melihat diri kita dengan jujur. Contoh kesalahan diakibatkan terjadinya gempa vulkanik dalam dan
mengukur diri adalah ketika kita merasa bisa diangkat sebanyak 32 kali.
melakukan sesuatu, padahal tidak, sehingga akhirnya (C) Aktivitas anak Gunung Krakatau meningkat
tugas yang seharusnya diselesaikan ternyata tidak sehingga Direktorat Vulkanologi dan Mitigasi
terselesaikan dengan baik. Oleh karena itu, jangan meningkatkan statusnya
heran, jika akhirnya kita menjadi stress. Sebaiknya, kita (D) Aktivitas anak Gunung Krakatau meningkat
harus terampil mengukur diri dan membaca situasi sehingga statusnya menjadi waspada.
secara tepat sehingga kita bisa menepatkan segala (E) Aktivitas anak Gunung Krakatau meningkat
sesuatu dengan baik sehingga Direktorat Vulkanologi dan Mitigasi
Gagasan pokok paragraf di atas adalah …. meningkatkan status.
(A) Kita harus jujur
(B) Kita harus bisa mengukur diri 24. Fenomena pekerja anak di Indonesia semula lebih
(C) Pentingnya manajemen diri dan pengukuran diri berkaitan dengan tradisi atau budaya membantu orang
(D) Membaca situasi dengan baik tua. Sebagian besar orang tua beranggapan bahwa
memberi pekerjaan kepada anak-anak merupakan upaya
155
SSE SIDIKALANG : Jl. A. Yani no 56 & 58 Bimbel Sse Sidikalang bimbelsse
BIMBINGAN BELAJAR
“STAR SCIENCE EDUCATION SIDIKALANG” FOKUS LULUS UTBK SBMPTN
Be a Star With The Star

proses belajar, yaitu belajar menghargai pekerjaan, dan (D) Dengan demikian, milikilah kamus dan biasakan
belajar bertanggung jawab terhadap pekerjaan. Selain membaca kamus dengan cermat.
dapat melatih dan memperkenalkan anak kepada dunia (E) Oleh sebab itu, biasakan membaca kamus secara
kerja, mereka juga berharap dapat mengurangi beban tepat dan cepat.
kerja keluarga. Namun sejalan dengan perkembangan
waktu, fenomena anak yang bekerja banyak berkaitan 27. (1) Bagi seorang manusia primitif perbuatan mencari
erat dengan alasan ekonomi keluarga dan kesempatan buah-buahan untuk makan pada saat lapar merupakan
memperoleh pendidikan. Orang tua tidak mampu lagi sesuatu yang datang dari nalurinya. (2) Ia berubah
menutupi kebutuhan hidup keluarga. Di pihak lain, maju setingkat apabila ia mencari batu untuk melempar
biaya pendidikan di Indonesia yang relatif tinggi telah hewan yang hendak dimakannya. (3) Akan tetapi,
ikut memperkecil kesempatan mereka untuk langkah ke arah sivilisasi baru terwujud apabila ia
mendapatkan pendidikan. memasukkan sebutir jagung kedalam tanah,
Intisari kutipan diatas adalah …. memelihara anak pohon jagung, serta menunggu musim
(A) Selain berkaitan dengan tradisi atau budaya untuk panen. (4) Jadi, kemajuan nyata dalam
membantu orang tua, anak-anak menjadi pekerja peradaban manusia berpangkal pada usaha mengelola
karena alasan ekonomi keluarga. sumber daya alam, yaitu tanah , lahan, dan air yang
(B) Orangtua mempekerjakan anak lebih karena tersedia.
masalah ekonomi. Gagasan pokok paragraf di atas adalah ….
(C) Masalah pekerja anak berkaitan dengan masalah (A) Bagi seorang manusia primitif perbuatan mencari
ekonomi dan pendidikan buah-buahan untuk makan pada saat lapar
(D) Pekerjaan anak merupakan fenomena yang merupakan sesuatu yang datang dari nalurinya
berkaitan dengan upaya proses belajar, yaitu belajar (B) Manusia primitif berubah maju setingkat apabila ia
menghargai pekerjaan, dan belajar bertanggung mencari batu untuk melempar hewan yang hendak
jawab terhadap pekerjaan. dimakannya
(E) Orangtua yang anaknya bekerja tidak mampu lagi (C) Seorang manusia primitif akan melangkah ke arah
menutupi kebutuhan hidup keluarga. silvilisasi baru apa bila ia memasukan sebutir
jagung ke dalam tanah, memelihara anak pohon
25. Penggantian Jaksa Agung yang sangat mendadak jagung, serta menunggu musim untuk panen
mengundang keingintahuan masyarakat. Pertanyaan, (D) Langkah ke arah silvilisasi baru terwujud apabila ia
dugaan, spekulasi, dan desas-desus menyelimuti memasukkan sebutir jagung ke dalam tanah,
peristiwa itu. Ada yang menduga Jaksa Agung bukan memelihara anak pohon jagung, serta menunggu
diganti, tetapi diberhentikan. Kalau betul musim untuk panen
diberhentikan, mengapa? (E) Kemajuan nyata dalam peradaban manusia
Pesan yang diungkapkan dalam kutipan di atas adalah berpangkal pada usaha mengelola sumber daya
…. alam, yaitu tanah, lahan, dan air yang tersedia
(A) Masyarakat heran akan penggantian Jaksa Agung.
(B) Penggantian Jaksa Agung sangat mendadak.
28. Primordialisme merupakan faktor penting untuk
(C) Masyrakat ingin tahu alasan penggantian Jaksa
memperkuat ikatan kelompok kebudayaan ketika
Agung.
ada ancaman dari luar terhadap kelompok
(D) Masyarakat menduga Jaksa Agung diberhentikan.
kebudayaan tersebut. Namun primordialisme
(E) Masyarakat mempertanyakan penggantian Jaksa
dipandang sangat negatif karena mengganggu
Agung.
kelangsungan hidup suatu bangsa. Primordialisme
sering dianggap bersifat primitif regresif dan
26. Keterampilan membaca kamus merupakan
merusak. Primordialisme dianggap, dianggap akan
keterampilan yang penting dimiliki. Keterampilan itu
menghambat modernisasi, proses pembangunan,
sangat membantu Anda dalam mencari dan menemukan dan merusak integritas nasional.
arti kata secara tepat. Jika hal itu tidak dimiliki, bisa
Kalimat yang dapat ditambahkan sesudah kalimat
jadi Anda akan menghabiskan waktu setengah jam atau terakhir kutipan di atas adalah ….
lebih hanya untuk mencari arti sebuah kata.
(A) Walaupun dianggap negatif, primordialisme
Kalimat berikut yang tepat mengakhiri paragraf tersebut mempunyai nilai positif juga.
adalah ….
(B) Jadi, primordialisme sebenarnya bukanlah sesuatu
(A) Oleh karena itu, beli dan milikilah kamus agar
yang perlu dipikirkan terlalu serius.
mudah menemukan arti kata.
(B) Dengan demikian, keberadaan kamus di (C) Orang yang mempertahankan primordialisme
perpustakaan sekolah sangat penting. merupakan salah satu perusak kelangsungan hidup
(C) Jadi, Anda perlu mengoleksi berbagai kamus agar suatu bangsa.
mudah mencari arti kata. (D) Kekuatan suatu bangsa tidak ditentukan oleh
primordialisme.
156
SSE SIDIKALANG : Jl. A. Yani no 56 & 58 Bimbel Sse Sidikalang bimbelsse
BIMBINGAN BELAJAR
“STAR SCIENCE EDUCATION SIDIKALANG” FOKUS LULUS UTBK SBMPTN
Be a Star With The Star

(E) Kekuatan primordalisme akan dapat memicu


potensi konflik antara kebudayaan suku-suku
bangsa yang ada.

29. Kehidupan masyarakat Jawa selalu diwarnai oleh


kehidupan simbolis. Unsur simbolis itu sangat berperan
dalam kehidupan sehari-hari. Dalam menjalani
kehidupan, masyarakat jawa mengungkapkan perasaan
dan perilaku dengan mengkaitkannya pada hal-hal
yang bersifat simbolis. Kebiasaan yang dilakukannya
sering dituangkan dalam bentuk upacara. Tak pelak
lagi, dalam upacara tersebut unsur simbolis sangat
berperan di dalamnya. unsur simbolis itu sangat
berkaitan dengan pandangan hidup masyarakatnya.
Oleh karenanya, Unsur simbolis itu haruslah dihayati
dan dipahami sehingga ungkapan serta keinginan
masyarakat dapat terkuak dan menjadi pedoman
hidupnya.
Inti kutipan di atas adalah ….
(A) Kehidupan masyarakat Jawa selalu diwarnai
kehidupan simbolis.
(B) Kehidupan masyarakat Jawa selalu diwarnai oleh
kehidupan simbolis yang harus selalu dihayati dan
dipahami.
(C) Kehidupan masyarakat Jawa yang bersifat simbolis
selalu dituangkan dalam upacara.
(D) Unsur Sinbolis dalam upacara yang dilakukan
masyarakat Jawa sangat berkaitan dengan
pandangan hidupnya.
(E) Masyarakat Jawa menjalani kehidupan dengan
mengkaitkannya pada hal-hal yang bersifat
simbolis.

30. (1) Sebenarnya, eutanasia telah lama di tolerir di


Belanda. (2) Sejak tahun 1970-an, penerapannya telah
diperbincangkan secara terbuka dan hukum yang
berlaku cenderung merupakan suatu pengendalian,
alih-alih larangan mutlak. (3) Hal inilah yang masih
menjadi hambatan psikologis bagi para dokter yang
melakukan praktik eutanasia ini. (4) Perdaban selama
bertahun-tahun yang kemudian menghasilkan
kompromi pembuatan garis-garis paduan sementara
tetap tidak mengesahkan penerapannya secara resmi.
Kalimat keempat pada kutipan di atas tidak menyatakan
hal sebagai berikut ….
(A) Eutanasia sudah lama dibicarakan.
(B) Pendebatan eutanasia menghasilkan penyusunan
garis-garis panduan pelaksanaan eutanasia.
(C) Garis-garis panduan penting pelaksanaan eutanasia
sementara sudah disusun.
(D) Pelaksanaan eutanasia secara sah belum dapat
dilakukan.
(E) Penerapan panduan pelaksanaan eutanasia
dikompromikan.

157
SSE SIDIKALANG : Jl. A. Yani no 56 & 58 Bimbel Sse Sidikalang bimbelsse
BIMBINGAN BELAJAR
“STAR SCIENCE EDUCATION SIDIKALANG” FOKUS LULUS UTBK SBMPTN
Be a Star With The Star

(C) Gaya hidup vegetarian semakin diminati orang


dewasa dan anak-anak dengan berbagai alasan.
(D) Sebagian orang menjadi vegetarian karena berbagai
faktor.
(E) Gaya hidup vegetarian dilakukan agar pola
hidupnya lebih sehat.
01. Penggunaan narkotika dapat berakibat negatif bagi
yang memakaiannya. Apabila digunakan secara 03. Keterampilan membaca kamus merupakan
berlebihan, narkotika dapat membahayakan kesehatan keterampilan yang penting. Keterampilan itu akan
tubuh dan jiwa pemakainya.tubuh orang yang menemukan arti kata secara cepat, jika hal itu tidak
menggunakan narkotik biasanya lemah dan kurus dimiliki, bisa jadi Anda akan menghabiskan waktu
karena sering sakit-sakitan. Jiwa orang tersebut juga setengah jam atau lebih hanya untuk mencari arti
terganggu karena penggunaan narkotik yang sebuah kata.
berlebihan dapat mempengaruhi fungsi otak dan Kalimat berikut yang tepat mengakhiri paragraf tersebut
kejiwaan seseorang. Penggunaan narkotik yang adalah ….
berlebihan juga dapat menimbulkan kecanduan. Orang (A) Oleh karena itu, beli dan milikilah kamus agar
yang sudah terbiasa menggunakannya akan merasa mudah menentukan arti kata.
membutuhkan narkotik sebagai obat untuk (B) Dengan demikian, keberadaan kamus di
menenangkan jiwanya. Akibat negatif yang terakhir, perpustakaan sekolah sangat penting.
narkotik dapat membuat orang menjadi malas. Karena (C) Jadi, Anda perlu mengoleksi berbagai kamus agar
keadaan orang yang kecanduan tersebut biasanya mudah mencari arti kata.
selalu membutuhkan obat penenang. Sering kali dia
(D) Dengan demikian, milikilah kamus dan biasakan
malas untuk melakukan kegiatan di dalam hidupnya.
membaca kamus dengan cermat.
Ikhtisar kutipan di atas adalah ….
(E) Oleh sebab itu, biasakanlah membaca kamus secara
(A) Jiwa seseorang dapat terganggu jika menggunakan
tepat dan cepat.
narkotik secara berlebihan.
(B) Seseorang yang pernah menggunakan narkotik
membutuhkan narkotik untuk menenangkan jiwa 04. Vietnam gagal mencetak diri sebagai tim pertama yang
pemakainya. lolos ke babak perempat final setelah pada pertandingan
kedua dipaksa bermain imbang 1-1 oleh Qatar di
(C) Pemakaian narkotik dapat berakibat negatif karena
Stadion My Dinh Hanoi. Vietnam yang memiliki nilai
dapat mempengaruhi fungsi otak.
sempurna setelah mengalahkan Uni Emirat Arab 2-0 di
(D) Orang yang kecanduan narkotik akan selalu partai perdana harus tertekan oleh Qatar sejak 10 menit
membutuhkan narkotik itu sebagai penenang paruh pertama. Pada menit ke-20, penyerang
(E) Jiwa narkotika akan tertanam ketika dipakai. naturalisasi Qatar. Sebastian Andres Quitana mencetak
gol pembuka. Namun sayangnya, tendangan itu masih
02. Menjadi vegetarian dewasa ini semakin populer dapat di baca dengan baik oleh Duong Hong Son.
sebagai gaya hidup sehat. Bukan hanya orang dewasa Delapan menit kemudian. Quitana kembali menjadi
yang beralih menghindari berbagai produk pangan momok diserang, justru Vietnam yang mampu
yang mengandung unsur hewani (sapi, kambing, babi, membuka keunggulan lebih dulu. Mereka mencetak gol
angsa, ikan, dan sebagainya), anak-anak pun tak 10 menit kemudian.
ketinggalan. Semakin banyak orang yang menjadi Pernyataan yang sesuai dengan bacaan tersebut
vegetarian. Alasan untuk menjadi veggie (sebutan bagi adalah….
kaum vegetarian) pun beragam. Sebagian orang (A) Vietnam dikalahkan Uni Emirat Arab 20 sehingga
menjadi vegetarian karena faktor agama, sebagian lagi harus mengalahkan Qatar
karena adanya penyakit atau kelainan fungsi organ (B) Pertandingan antara Vietnam dan Qatar berakhir
tubuh, selera, ekonomi, moral, dan yang terpenting seri
karena sebagian orang ingin menjalani pola hidup
(C) Vietnam terus menyerang Qatar dalam
yang lebih sehat agar tubuh menjadi lebih bugar.
pertandingan tersebut
Kesimpulan yang dapat diambil dari kutipan di atas
(D) Qatar berhasil mencetal gol lebih dahulu dari pada
adalah ….
Vietnam
(A) Gaya hidup menjadi seorang vegetarian dilakukan
oleh orang dewasa untuk menghindari produk (E) Qatar lebih banyak menyerang Vietnam
pangan yang mengandung unsur hewani.
(B) Vegetarian adalah gaya hidup dengan tidak 05. Besarnya jumlah siswa/mahasiswa Indonesia di
mengkonsumsi produk pangan yang mengandung Singapura dan Malaysia tentu tidak lepas dari berbagai
unsur hewani. faktor. Pertama siswa/ mahasiswa Indonesia tidak akan
mengalami kejutan budaya bila belajar di Singapura
atau Malaysia. Dari segi geografis jarak tempuh dari
158
SSE SIDIKALANG : Jl. A. Yani no 56 & 58 Bimbel Sse Sidikalang bimbelsse
BIMBINGAN BELAJAR
“STAR SCIENCE EDUCATION SIDIKALANG” FOKUS LULUS UTBK SBMPTN
Be a Star With The Star

Indonesia ke Singapura dan Malaysia tidak terlalu jauh. ibu-ibu rumah tangga kembali ke pola lama, yaitu
Ketiga cuaca di Singapura, Malaysia, dan Indonesia membawa keranjang belanja ke pasar. (3) Dengan
sama. Keempat, makanan yang ada di Malaysia, demikian, jumlah kantong plastik yang dibawa ke
Singapura, dan Indonesia tidak jauh berbeda. rumah akan berkurang (tereduksi). (4) Selain itu, bila
Kalimat yang tepat untuk bagian awal paragraf adalah setiap orang lebih memilih menggunakan saputangan
… daripada tisu, terjadi penghematan terhadap bahan baku
(A) Negeri favorit mahasiswa Indonesia untuk studi untuk tisu. (5) Jika setiap orang melakukan hal tersebut,
adalah Singapura dan Malaysia beberapa ton sampah yang akan tereduksi per bulan dan
(B) Observasi menunjukkan bahwa jumlah beberapa hasil hutan dapat diselamatkan.
siswa/mahasiswa Indonesia yang studi di Singapura Kalimat topik pada paragraf tersebut adalah …
dan Malaysia banyak (A) kalimat 5
(C) Banyak mahasiswa Indonesia bercita-cita untuk (B) kalimat 4
belajar di Singapura dan Malaysia (C) kalimat 3
(D) Kenyataan membuktikan bahwa singapura dan (D) kalimat 2
Malaysia adalah dua Negara di minati untuk studi (E) kalimat 1
(E) Jumlah siswa/mahasiswa Indonesia di Singapura
dan Malaysia menarik untuk diminati 09. Duvurger dalam bukunya berhasil mem-perjelas
konflik-konflik teoretis antara Marxisme dengan
06. Kiper nomor satu Qatar dapat menangkap bola dari Demokrasi Liberal. Ia berhasil menyoroti sesuatu yang
penyerang Vietnam. Akan tetapi, tangkapannya lemah selama berabad-abad ini menjadi isu intelektual dan
sehingga bola pun menyusur masuk ke dalam gawang praktis yang paling penting dalam bidang politik. Isu ini
sendiri. Pada penghujung babak pertama, sebenarnya masih relevan dengan situasi politik yang ada pada
Qatar memiliki peluang untuk menyamakan sebagian negara di dunia.
kedudukan melalui kaki Adel. Namun, tendangannya Arti ungkapan yang paling sesuai untuk kata relevan
masih lemah. Tendangan itu sempat mengenai bek dalam paragraf di atas adalah …
Vietnam sehingga hanya menghasilkan tendangan (A) sesuai
penjuru. Hingga babak pertama berakhir, kedudukan (B) cocok
tetap tidak berubah. Usai turun minum, tempo (C) tepat
permainan berjalan kian menarik. Penyerang Qatar (D) sepadan
kembali mengancam gawang Vietnam. Tendangan (E) bergayut
bebas dari Waleed Abdulla sangat terukur ke arah
gawang, namun Duong lebih cepat dalam 10. Ruly, seorang TKW di Taiwan asal Ponorogo,
mengantipasi gerakan bola. memberikan keterangannya lewat SMS bahwa gaji
Gagasan utama pada bacaan di atas adalah… bersih yang diterimanya setiap bulan sebagai pekerja
(A) Penyerang sering mengancam gawang Vietnam rumah tangga dengan bekal ijazah SMA adalah 10.000
Tendangan bebas penyerang Qatar mampu dolar Taiwan (TWD) atau jika dirupiahkan senilai 2,5
diantisipasi Duong juta rupiah.
(B) Tendangan bebas Walled Abdulla sangat terukur Gagasan utama kalimat di atas adalah …
(C) Tempi permainan berjalan kian menarik (A) Ruly seorang TKW di Taiwan asal Ponorogo
(D) Duong lebih cepat mengantisipasi gerakan bola (B) Seorang TKW di Taiwan asal Ponorogo
(E) Duong lebih cepat mengantisipasi gerakan bola (C) Ruly memberikan keterangannya
(D) Gaji bersih yang di terima setiap bulan
07. Michael Phelps, yang hiperaktif, selalu meminta (E) Gaji bersih pekerja rumah tangga adalah10.000
perhatian, tidak bisa fokus, dan bermasalah sejak kecil, dolar Taiwan (TWD)
menjadi atlet terbesar Olimpiade sepanjang masa.
Gagasan utama kalimat tersebut adalah … 11. Ruang dalam (ndalem) merupakan pusat dari sebuah
(A) Michael Phelps bermasalah, namun menjadi atlet rumah. Bagian belakang rumah, yang disebut njero
terbesar Olimpiade omah, dibagi menjadi tiga ruangan penting, yaitu
(B) Michael Phelps hiperaktif, meminta perhatian tidak senthong lengen (kanan), senthong tengah (tengah), dan
bisa fokus, dan bermasalah senthong kiwa (kiri). Senthong tengen biasanva
(C) Michael Phelps menjadi atlet terbesar Olimpiade digunakan untuk bersemedi, bermediasi, atau
(D) Michael selalu meminta perhatian dan tidak bisa mendekatkan diri kepada Tuhan. Senthong tengah
fokus khusus untuk pasangan pengantin atau pasangan ayah
(E) Michael phelps yang hiperaktif dan bermasalah dan ibu. Senthong kiwa biasanya berfungsi sebagai
tempat menyimpan padi atau beras dan simbol-simbol
08. (1) Reduce berarti mengurangi atau mereduksi sampah Dewi Sri lainya. Paragraf di atas dikembangkan dengan
yang akan terbentuk. (2) Hal ini dapat dilakukan apabila cara….
(A) proses
159
SSE SIDIKALANG : Jl. A. Yani no 56 & 58 Bimbel Sse Sidikalang bimbelsse
BIMBINGAN BELAJAR
“STAR SCIENCE EDUCATION SIDIKALANG” FOKUS LULUS UTBK SBMPTN
Be a Star With The Star

(B) umum khusus (D) Sebagian warga Dago mengubah rumah mereka
(C) penjelasan menjadi tempat usaha.
(D) rincian (E) Saat ini Jalan Dago dipenuhi hotel, butik, restoran,
(E) penambahan bank dan tempat-tempat lain-nya.

12. Pulau Penyengat yang berpenduduk 1.000 jiwa itu 15. Sejak pengurusan izin usaha, investor memasuki labirin
memiliki nama lain, yaitu Maskawin, karena pulau dengan lama waktu pengurusan izin hingga 151 hari
tersebut merupakan mas kawin Sultan Mahamudsyah, dan melewati belasan tahapan atau pintu. Labirin dalam
raja kedelapan Kesultanan Riau Lingga ketika ia kalimat tersebut mengandung makna ....
mempersunting Engku Putri Raja Hamidah. (A) sesuatu yang memakan waktu
Gagasan utama kalimat tersebut adalah.... (B) sesuatu yang berbelit-belit
(A) Pulau Penyengat berpenduduk 1.000 jiwa. (C) urusan yang bertahap
(B) Pulau Penyengat merupakan mas kawin Sultan (D) sistem banyak pintu
Mahmudsyah. (E) jalan yang panjang
(C) Sultan Mahmudsyah adalah raja kedelapan
Kesultanan Riau Lingga. 16. Sejalan dengan arah pelaksanaan politik luar negeri RI
(D) Pulau Penyengat memiliki nama lain. serta sekaligus dalam upaya menyerap masukan dari
(E) Sultan Mahmudsyah mempersunting Engku Putri berbagai pemangku kepentingan, Departemen Luar
Raja Hamidah. Negeri dalam tahun 2010 telah melakukan berbagai
kegiatan, seperti seminar, sosialisasi , dan pertemuan
13. Membuat karangan dan membuat ringkasan merupakan kelompok ahli, baik di Jakarta maupun di berbagai
dua sejawat yang berhubungan erat. Menyusun daerah.
karangan melibatkan analisis dan sintesis. analisis dan Gagasan pokok kalimat di atas adalah....
sintesis dengan sendirinya bergayutan. Setelah (A) Arah pelaksanaan politik luar negeri RI..
menganalisis, orang akan membuat sintesis. Untuk (B) Arah pelaksanaan politik luar negeri Departemen
meringkas bacaan, kita harus menafsirkan isinya, Luar Negeri dan upaya menyerap masukan dari
menguraikan unsur-unsurnya, memisahkan pikiran- pemangku kepentingan.
pikiran utamanya dan penjelasnya (analisis), kemudian (C) Kegiatan Departemen Luar Negeri sejalan dengan
membuat rekonstruksi kerangka bacaan, dan menyusun arah pelaksanaan politik luar negeri.
ringkasarinya (Sintesis). (D) Departemen Luar Negeri melakukan kegiatan.
Berdasarkan teks di atas, pernyataan yang paling tepat (E) Pertemuan kelompok ahli baik di Jakarta maupun
adalah……. di berbagai daerah
(A) Meringkas dan mengarang merupakan kegiatan 17. Pasar kerja penelitian di negeri kita cukup banyak.
yang saling berkaitan. Tentu saja yang di maksud di sini adalah penelitian
(B) Mengarang melibatkan analisis dan sintesis. terapan yang dapat digunakan untuk meningkatkan
(C) Meringkas melibatkan analisis dan sintesis. mutu keadaan atau hasil suatu keadaan dalam
(D) Meringkas dan mengarang merupakan kegiatan masyarakat. Penelitian eksploratoris atau jenis
yang melibatkan analisis dan sintesis. penelitian lain yang bersifat teoretis juga memiliki pasar
(E) Mengarang merupakan kegiatan awal sebelum kerja, tetapi biasanya sangat terbatas. Ada Juga dana
melakukan sintesis. yang disediakan oleh yayasan yang besar. seperti
Toyota Foundation dan Ford Foundation. Bahkan,
14. Pada masa itu, Dago adalah daerah elite tempat tinggal tersedia dana yang be-sar yang ditawarkan oleh
orang Belanda, Cina, dan orang asing lainnya. Namun. pemerintah, seperti sejumlah proyek yang didukung
kini kawasan Dago Mengalami perubahan total dari oleh Ke-menterian Pendidikan Nasional. Belum lagi
kawasan hunian menjadi kawasan bisnis. Perubahan perusahaan atau lembaga lain yang berkepentingan
fungsi mi mengubah pola hidup warga Dago. Saat ini, terhadap penelitian tertentu. Pada dasarnya, pasar kerja
banyak warga Dago membuka usaha di sepanjang Jalan penelitian di Indonesia terbuka lebar.
Dago. Sepanjang jalan Dago dipenuhi oleh hotel, butik, Gagasan pokok paragraf di atas terdapat…
restoran, bank, dan tempat-tempat hiburan lainnya. (A) Awal kalimat
Gagasan pokok paragraf tersebut adalah..,, (B) Kalimat kedua
(A) Pada masa lalu, Dago adalah daerah elite tempat (C) Awal dan akhir kalimat
tinggal kaum non pribumi. (D) Akhir kalimat
(B) Perubahan fungsi mengubah pola hidup warga (E) Semua kalimat
Dago.
(C) Kawasan Dago mengalami perubahan total dari 18. Pada waktu dipakai untuk mengecet mobil, air dalam
kawasan hunian menjadi kawasan bisnis. cat menguap sehingga tinggallah butiran cat yang
berikatan melapisi permukaan yang dicat. Berbeda
dengan cat modem, pada cat konvesional, yang partikel
160
SSE SIDIKALANG : Jl. A. Yani no 56 & 58 Bimbel Sse Sidikalang bimbelsse
BIMBINGAN BELAJAR
“STAR SCIENCE EDUCATION SIDIKALANG” FOKUS LULUS UTBK SBMPTN
Be a Star With The Star

cat-nya berukuran mikrometer, permukaan ba-han berteriak, "Oh, Tuhan, jika Engkau selamatkan hidupku,
berpeluang dicat dengan tidak sempurna sehingga tidak aku akan mengabdi kepada-Mu!" Doanya terkabulkan.
tertutup partikel cat. Akibatnya, udara, kotoran, dan ….. Kalimat yang tepat pada Simpulan pada akhir
debu dapat leluasa mencapai permukaan yang tidak paragraf tersebut adalah...
tertutup partikel cat. Jika yang dicat adalah logam dan (A) Sejak saat itu Cat Stevens rajin beribadah di gereja.
tujuannya untuk mencegah karat, pengecatan hanya (B) Pengalaman itulah yang kemudian mempertebal
memperlambat karat, tetapi tidak mencegah sama keyakinan spiritualnya.
sekali. Di samping itu, debu atau kototan tertangkap (C) Kini, Cat Stevens beralih memadi penyanyi lagu
atau bersarang dalam struktur partikel cat yang rohani.
berukuran mikrometer tersebut. Lama-kelamaan cat (D) Karena peristawa itu, jalan hidup Cat Stevens
menjadi pudar warnanya dan logam mengalami proses berubah.
perkatatan. (E) Karena selamat, kemudian ia menjadi pemusik
Pernyataan yang tidak sesuai dengan isi bacaan di atas rohani.
adalah …
(A) Mobil akan berkarat jika dicat dengan cat 21. Pandangan kedokteran holistik menyatakan bahwa
konvensional. kesehatan badan berkaitan langsung dengan kesehatan
(B) Debu dan kotoran dapat menempel pada cat jiwa. Problem dipikiran dapat menyebabkan masalah
sehingga warna cat menjadi pudar. kesehatan badan. Sebaiknya, penyakit dibadan pun
(C) Cat konvensional meninggalkan permukaan tidak dapat menyebabkan masalah dipikiran. Pandangan ini
tertutup cat secara sempurna. ternyata juta berlaku dibidang kesehatan mulut dan gigi.
(D) Mobil yang tidak dicat dengan sempurna dapat Jika dokter ahli jiwa punya ungkapan "mindbady
menerima udara, kotoran, dan debu dengan leluasa. connection", dokter gigi juga punya ungkapan,
(E) Proses perkaratan logam terjadi karena proses “mouthbody connection". Problem dibadan dapat
pengecatan rang tidak sempurna. menyebabkan masalah gigi dan mulut. Demikian
sebaiknya, masalah gigi dan mulut pun dapat
19. Secara sosial, rokok menjadi sarana komunikasi, menyebabkan problem diorgan lain.
jembatan perkenalan dengan orang baru didalam Simpulan yang dapat dirumuskan dan teks di atas
perjalanan kereta api, perjamuan, atau rapat. Bagi yang adalah …
sudah saling mengenal, rokok menjadi tali peneguh sila- (A) Keadaan tertentu pada tubuh memengaruhi keadaan
turahmi dan solidaritas social. Secara spiri-tual, dalam bagian lain pada tubuh kita.
ritus religi, rokok menjadi bagian dan kelengkapan (B) Penyakit dalam tubuh membuat orang menjadi
sesaji. Kata pemanis “uang rokok” jugaa menandakan stress.
fungsi sosial rokok. Jadi, bagi sebagian orang, (C) Pandangan kedokteran gigi sejajar dengan
mengharamkan rokok merupakan keputusan absurd pandangan kedokteran jiwa.
karena dasar-dasar komunalitasnya rapuh dan dalam (D) Ungkapan "mindbody connection" ber-rmakna
praktik mustahil untuk diterapkan secara efektif. sama dengan “mouthbody connection”.
Berdasarkan teks di atas. penulisnva bermaksud ... (E) Penyakit di dalam tubuh berkaitan dengan keadaan
(A) Menspertahankan pendapatnya untuk tidak pikiran.
mengharamkan rokok karena rokok mempunyai
fungsi secara sosial. 22. Dokar membawa kami ke Kantor Polisi Surabaya. Aku
(B) Mengajukan pendapat bahwa mengharamkan rokok dipersilakan duduk menunggu di ruang tamu. Sudah
sulit diptaktikkan karena rokok berfungsi secara timbulkeinginanku untuk menanyakan persoalanku.
social. Tampaknya dalam udara pagi berhalimun orang tak
(C) Menjelaskan fungsi rokok secara sosial sehingga berada dalam suasanamemberi keterangan. Aku tak jadi
tidak mungkin mengharamkan rokok. bertanya. Dokar masih tetap menunggu di depan kantor.
(D) Mengungkapkan ketidaksetujuannya Agen itu malah meninggalkanaku seorang diri tanpa
mengharamkan rokok karena rokok mempunyai berpesan.
fungsi sosial Betapa lama Matahari tak juga terbit. Waktu terbit tidak
(E) Menyakinkan pembaca tentang fungsi rokok yang mampu mengusir halimun. Butir butir air yang kelabu
masih relavan sehingga tidak perlu mengharamkan itu merajaisegalanya, bahkan juga di pedalaman paru-
rokok. paruku. Lalu lintas di depan kantor mulai ramai: dokar,
andong, pejalan kaki,penjaja, pekerja. Aku masih juga
20. Seiak bergelut dengan penyakit tuberkolosis pada 1969 duduk seorang diri di ruang tamu.
yang nyaris merenggut nyawanya, Cat Stevens memutu
pengembaraan spiritualnva. Tahun 1976, di tengah Salah satu pernyataan berikut sesuai dengan teks.
kepopuleranya sebagai penyanyi lagu rakyat, Cat (A) Aku adalah seseorang yang bekerja di kantorpolisi.
Stevens nyaris tenggelam ketika sedang berenang dalam (B) Aku sedang duduk menunggu matahari terbit.
Pantai Malibu. Stevens, yang saat itu-berusia 28 tahun,
161
SSE SIDIKALANG : Jl. A. Yani no 56 & 58 Bimbel Sse Sidikalang bimbelsse
BIMBINGAN BELAJAR
“STAR SCIENCE EDUCATION SIDIKALANG” FOKUS LULUS UTBK SBMPTN
Be a Star With The Star

(C) Aku datang ke kantor polisi untuk memberikan Menurutnya, Mahkamah Agung perlu mendesak
keterangan. perubahan pasal-pasal yang melemahkan kewenangan
(D) Aku datang ke kantor polisi untuk menemui hakim agung. Dia mengeluhkan adanya pasal yang
seorang agen polisi. melarang hakim memberi putusan hukuman lebihberat
(E) Aku tiba di kantor polisi pada pagi hari. daripada putusan pengadilan tingkat lebih bawah. Dia
23. Maksud kalimat Waktu terbit tidak mampu mengusir mengatakan bahwa selama ini masih banyak putusan
halimun adalah ... . pengadilan yang salah.
(A) Saat matahari terbit, cuaca tetap berkabut.
(B) Hujan turun sesudah matahari terbit. Kalimat yang tepat sebagai gagasan pokok
(C) Kabut pada pagi hari itu sangat tebal. paragrafsekaligus sebagai kalimat penutup paragraf di
(D) Sinar matahari tidak mampu menembus kabut itu. atasadalah ... .
(E) Saat itu, cuaca mendung. (A) Oleh karena itu, ketiga lembaga penegak hokum
tersebut jangan saling tuding menghadapi Dewan
24. Di persimpangan itu, terlihat sederet rumah berlantai Rakyat dan Pemerintah.
dua yang berlanggam Cina yang diduga merupakan (B) Dengan demikian, sebaiknya ketiga lembaga
tinggalan sebuah perkumpulan cendekiawan sekitar penegak hukum itu bersama-sama berjuang
abad ke-18 yang kemudian dibeli sebagian oleh seorang menentang pembahasan KUHP dan KUHAP.
warga setempat untuk dilestarikan. (C) Jadi, ketiga lembaga itu merasa senasib dalam
Pokok kalimat di atas adalah ... . masalah pembahasan KUHP dan KUHAP oleh
(A) Di persimpangan itu terlihat. Dewan Rakyat dan Pemerintah
(B) Rumah berlantai dua yang berlanggam Cina diduga (D) Meskipun demikian, kedudukan ketiga lembaga
merupakan tinggalan sebuah perkumpulan penegak hukum itu tidak sama dalam KUHP
cendekiawan. danKUHAP.
(C) Terlihat sederetan rumah berlantai dua. (E) Untuk selanjutnya, ketiga lembaga tersebut harus
(D) Rumah itu dibeli sebagian oleh seorang warga selalu berpendapat sama dalam menghadapi
setempat. pembahasan KUHP dan KUHAP di Dewan Rakyat.
(E) Rumah tersebut dilestarikan.
27. Perbedaan kedua rasio terletak pada numerator. Pada
25. Kultur kepemimpinan dengan kedekatan pribadi bagian atas rasio tunai harus dikeluarkan nilai
(personal touch) tidak perlu selalu diartikan secara persediaan, termasuk aset yang dapat cepat ditukar
harfiah sebagai kepemimpinan yang mengedepankan dengan uang tunai serta tagihan bisnis. Adapun pada
tatap muka. Sujoko, sejak kepemimpinannya, dapat rasio lancar, semua aset berumur kurang dari satu tahun.
membuat orang merasa dekat dengan sosoknya. Rasio ini sama nilainya bila bisnis yang dijalankan tidak
Kehadirannya dan sikap terbuka untuk mendengardan mempunyai aset lancar selain uang tunai. Sementara itu,
berkomunikasi menjadi ciri yang membuat orang numerator dari rasio ini adalah utang yang berumur
merasa “dekat”. Adik kita kenal berusaha melayani kurang dari satu tahun. Aset lancar dapat
semua orang yang mengirimkan SMS. Meskipun ia dikelompokkan dari berbagai item yang termasuk dalam
mungkin dibantu asistennya, sikap responsif aset tersebut. Uang tunai merupakan yang paling utama
menunjukkan betapa ia “hadir” untuk dalam aset lancar. Uang tunai dapat secara langsung
melayani.Kedekatan pribadi dibangun dalam bentuk membayar kewajiban yang jatuh tempo segera. Artinya,
kultur. rasio tunai selalu lebih kecil daripada rasio lancar,
Kultur kepemimpinan dengan kedekatan pribadi itulah kemudian diikuti rekening surat berharga, yang
yang dibentuk oleh para pemimpin yang berfokus pada merupakan item yang dapat diubah ke tunai dan
manusia. langsung membayar utang yang jatuh tempo.

Paragraf di atas terdiri atas tujuh kalimat. Gagasan Pada paragraf kedua terdapat kalimat yang seharusnya
pokok paragraf terdapat pada kalimat... . terletak di paragraf pertama, yakni kalimat ...
(A) pertama (A) pertama
(B) keenam dan ketujuh (B) kedua
(C) ketujuh (C) ketiga
(D) pertama dan ketujuh (D) keempat
(E) pertama, keenam, dan ketujuh (E) kelima

26. Peneliti Pukat UGM mengatakan bahwa bukan hanya Untuk soal nomor 28 sampai nomor 30.
Lembaga Antikorupsi yang dilemahkan dalam KUHP Penggunaan plastik ternyata menimbulkan masalah-masalah
dan KUHAP yang sedang dibahas di Dewan Rakyat itu, tertentu. Padahal, penggunaan plastik telah meluas hampir
melainkan juga kepolisian dan kejaksaan. Seorang ke seluruh bidang kehidupan. Berbagai produk dan peralatan
mantan hakim dari Yogyakarta juga sependapat. dihasilkan dari bahan ini karena dinilai lebih ekonomis,
162
SSE SIDIKALANG : Jl. A. Yani no 56 & 58 Bimbel Sse Sidikalang bimbelsse
BIMBINGAN BELAJAR
“STAR SCIENCE EDUCATION SIDIKALANG” FOKUS LULUS UTBK SBMPTN
Be a Star With The Star

tidak mudah pecah, fleksibel, dan ringan. Salah satu contoh


produk berbahan plastik yang paling sering dipakai oleh
masyarakat adalah botol plastik. Peningkatan jumlah
pemakaian botol plastik menimbulkan dampak bagi
lingkungan ketika sudah tidak terpakai. Plastik adalah benda
yang sulit terurai. Proses terurai plastik dapat berlangsung
antara 450 sampai 1.000 tahun. Padahal, di seluruh dunia,
setiap tahun digunakan sekitar 50 miliar botol plastik
sehingga diprediksikan pada masa depan, lokasi
pembuangan sampah hampir tidak akan cukup untuk
menampung semuanya.
Upaya daur ulang dan penggunaan kembali botol plastik
tidak sepenuhnya mengatasi masalah lingkungan karena
tingkat penggunaan botol plastik sangat tinggi. Selain itu,
penggunaan kembali plastik yang didaur ulang
menimbulkan masalah baru, yaitu masalah kesehatan. Hal
ini disebabkan plastik yang terurai akan melepaskan zat
kimia yang disebut Bisphenol A (BPA) ke dalam air. Zat
BPA ini memiliki be-berapa efek negatif pada hormon tubuh
yang dapat menimbulkan gangguan kesehatan, seperti
gangguan perilaku, penurunan kekebalan, gangguan belajar,
diabetes, dan obesitas.
28. Kutipan di atas memiliki gagasan pokok ....
(A) upaya daur ulang dan penggunaan kembali botol
plastik tidak berguna
(B) botol plastik mengandung zat kimia Bisphenol
(C) penggunaan plastik dapat menimbulkan masalah
(D) botol plastik dapat menimbulkan efek negatifbagi
kesehatan tubuh
(E) dampak negatif penggunaan botol plastik

29. Pernyataan yang tidak sesuai dengan kutipan di atas


adalah ....
(A) Daur ulang dapat mengatasi masalah lingkungan
akibat penggunaan botol plastik.
(B) Plastik yang berasal dan produk daur ulang
berbahaya bagi kesehatan.
(C) Pada masa depan, lokasi pembuangan sampah
diprediksikan tidak mampu menampung sampah
botol plastik.
(D) Produk yang berasal dari plastik dinilai lebih
ekonomis dan praktis.
(E) Salah satu masalah kesehatan yang muncul dari
penggunaan produk daur ulang plastik adalah
penyakit gula.

30. Simpulan yang tepat untuk kutipan di atas adalah ....


(A) produk dari plastik lebih ekonomis dan praktis
(B) produk dari plastik dapat didaur ulang
(C) penggunaan botol plastik dapat menimbulkan
masalah lingkungan dan kesehatan
(D) masyarakat suka memakai botol plastik
(E) lokasi pembuangan sampah tidak dapat
menampung sampah botol plastik

163
SSE SIDIKALANG : Jl. A. Yani no 56 & 58 Bimbel Sse Sidikalang bimbelsse
BIMBINGAN BELAJAR
“STAR SCIENCE EDUCATION SIDIKALANG” FOKUS LULUS UTBK SBMPTN
Be a Star With The Star

1. Berdasarkan paragraf 3, manakah simpulan yang tepat?


A. Semua perusahaan tidak dapat melakukan pinjaman
dana dari bank dan tidak mampu membayar utang.
B. Sebagian perusahaan tidak dapat melakukan
Teks ini untuk menjawab nomor 1–7! pinjaman dana dari bank dan tidak mampu
Apakah Pos Indonesia memang terancam pailit? Tidak membayar utang
sedikit masyarakat Indonesia yang menganggap bahwa pailit C. Semua perusahaan dapat melakukan pinjaman dana
itu sama dengan bangkrut. Padahal, dua hal tersebut dari bank dan tidak mampu membayar utang
merupakan hal yang berbeda. Menurut UU No. 37/2004 D. Sebagian perusahaan dapat melakukan pinjaman
tentang Kepailitan dan Penundaan Kewajiban Pembayaran dana dari bank dan mampu membayar utang
Utang (PKPU), kepailitan adalah sita umum atas semua E. Semua perusahaan dapat melakukan pinjaman dana
kekayaan debitur pailit yang pengurusannya dilakukan oleh dari bank dan mampu membayar utang.
kurator di bawah pengawasan hakim pengawas. Artinya,
perusahaan dapat dinyatakan pailit dengan putusan 2. Simpulan manakah yang dapat ditarik dari paragraf 2?
pengadilan apabila perusahaan (debitur) yang mempunyai A. Perusahaan tidak dapat memberikan gaji karyawan
dua atau lebih kreditor tak membayar lunas sedikitnya satu tepat waktu.
utang yang telah jatuh waktu dan dapat ditagih. B. Perusahaan tidak mampu memberikan insentif ke
Sementara itu, kata 'bangkrut' tidak dikenal dalam karyawan.
undang-undang di Indonesia. Alhasil, tidak ada kepastian C. Perusahaan mengalami kesulitan dalam membayar
terkait definisi bangkrut. Meski begitu, dalam putusan utang
Mahkamah Konstitusi No.18/PPU-VI/2008, disebutkan D. Perusahaan tidak sedang memiliki utang.
bahwa secara umum, perusahaan bangkrut lantaran dua E. Perusahaan mampu membayar utang dengan tepat
faktor. Kedua faktor itu yaitu faktor eksternal di luar waktu.
kewenangan pelaku usaha dan missmanagement. Jika pailit
dikarenakan tidak mampu membayar utang, maka Posindo 3. Manakah pernyataan dari teks diatas yang BENAR?
sebetulnya tidak sedang pailit dan belum dapat pula A. Pailit dan bangkrut memiliki makna yang sama.
dikatakan terancam. Pasalnya, Posindo masih lancar B. Faktor-faktor yang dapat membuat perusahaan
membayar seluruh utang-utangnya sampai dengan saat ini. bangkrut adalah faktor eksternal dan miss
Hal ini ditegaskan oleh Benny Otoyo selaku management.
Sekretaris Perusahaan Posindo. Ia menyebutkan utang C. Perusahaan dapat dikatakan pailit meskipun dapat
Posindo masih lancar. Hal itu juga dibuktikan dengan rating membayar utangnya dengan tepat waktu.
untuk surat utang jangka menengah (medium term D. Posindo selalu menunggak pembayaran utang.
note/MTN) perseroan yang berada di level A-. Benny juga E. Gaji karyawan Posindo dibayar dari pinjaman bank.
membantah bahwa gaji karyawan Posindo disebut-sebut
dibayar dari pinjaman bank. Menurutnya, tidak akan ada
bank yang mau memberi pinjaman untuk tujuan bayar gaji, 4. Berdasarkan teks diatas, manakah simpulan yang
meskipun gaji termasuk dalam biaya operasi yang didanai PALING TEPAT?
dari pinjaman. Ia mengakui perseroan saat ini sedang A. Posindo tidak melakukan pinjaman dari bank.
bergumul dengan era disrupsi teknologi. Namun, bukan B. Posindo mampu membayar utang dengan tepat
berarti perseroan diam, akan tetapi Posindo justru sedang waktu.
melakukan transformasi bisnis dari semua aspek, mulai dari C. Posindo sedang bergumul di era disrupsi teknologi.
bisnis, sumber daya manusia, hingga produk. D. Posindo sedang melakukan transformasi bisnis.
E. Posindo tidak sedang maupun terancam pailit.

5. Pernyataan yang sesuai dengan wacana diatas, kecuali…


A. Definisi “bangkrut” tidak tertuang dalam undang-
undang di Indonesia.
B. Posindo sedang tidak terancam pailit.
C. Bank tidak akan memberikan pinjaman dana untuk
tujuan bayar gaji.
D. Posindo sedang melakukan transformasi bisnis pada
semua aspek.
164
SSE SIDIKALANG : Jl. A. Yani no 56 & 58 Bimbel Sse Sidikalang bimbelsse
BIMBINGAN BELAJAR
“STAR SCIENCE EDUCATION SIDIKALANG” FOKUS LULUS UTBK SBMPTN
Be a Star With The Star

E. Rating untuk surat utang jangka menengah berada di wisata Bantimurung atau Rammang-Rammang, wisatawan
level A+. lokal dan mancanegara akan singgah menyaksikan pesta
adat Katto Bokko ataupun Appalili. (12) Appalili adalah
6. Berdasarkan grafik, manakah simpulan yang BENAR? upacara turun sawah yang digelar sekali setahun secara
A. Pendapatan terendah Posindo adalah pada tahun besar-besaran.
2012. (13)Pemangku Adat Karaeng Marusu berharap
B. Pendapatan Posindo pada tahun 2016 lebih tinggi bahwa perhatian dari pemerintah ini akan menjadi wujud
daripada beban usahanya. kepedulian dalam melestarikan budaya. (14) Sinergi wisata
C. Pendapatan Posindo pada tahun 2014 lebih rendah alam dan budaya sekaligus merangsang generasi muda
daripada beban usahanya. untuk mengenal dan melesarikan budayanya. (15)
D. Laba tahun berjalan Posindo menempati titik Menurutnya, upacara adat yang dilakukan turun-temurun
terendah pada tahun 2011. sejak ratusan tahun silam pada prakemerdekaan tetap
E. Beban usaha Posindo mengalami tren peningkatan mereka jaga agar anak cucu mereka masih bisa melihat dan
dalam periode 2011-2017 merasakan wujud kesyukuran saat panen raya. (16) Selain
itu, wujud kebersamaan dan kesatuan antara pihak turunan
7. Berdasarkan grafik, pada periode tahun berapakah Laba raja dan masyarakat dapat terus dibina melalui pesta adat
Tahun Berjalan mengalami peningkatan paling drastis? Katto Bokko.
A. 2011-2012
B. 2012-2013 8. Kata tersebut pada kalimat (10) merujuk pada…
C. 2013-2014 A. objek wisata Kabupaten Maros
D. 2015-2016 B. objek wisata yang dikenal hingga ke mancanegara
E. 2016-2017 C. Taman Wisata Alam Bantimurung
D. Taman Wisata Alam Bantimurung, Taman Purbakala
Bacalah teks berikut uintuk menjawab soal No. 8 s.d. 13! Leang-Leang, dan Taman Wisata Pegunungan Karst
(1)Pemerintah Kabupaten Maros melalui Dinas Rammang-Rammang
Kebudayan dan Pariwisata siap memadukan objek wisata E. Beberapa objek wisata Kabupaten Maros yang
alam dan wisata budaya di bumi butta salewangang. (2) dikenal hingga ke mancanegara
Pemaduan dua objek ini dimaksudkan untuk mendukung
program pemerintah, Wonderful Indonesia. (3) Kepala 9. Gagasan utama paragraf pertama adalah…
Dinas Kebudayaan dan Pariwisata Kabupaten Maros, A. Pemerintah Kabupaten Maros siap memadukan
Muhammad Ferdiansyah, di Kabupaten Maros, mengatakan objek wisata alam dan wisata budaya di bumi butta
bahwa selama ini hanya lebih banyak fokus pada objek salewangang.
wisata dan kini wisata budaya akan diangkat untuk saling B. Perpaduan objek wisata alam dan wisata budaya
mendukung. Kabupaten Maros.
(4) Dia juga mengatakan bahwa salah satu wisata C. Kabupaten Maros selama ini hanya fokus pada objek
budaya yang masih tetap bertahan dan dipelihara turun- wisata alam.
temurun adalah pesta adat panen raya Katto Bokko. (5) D. Kabupaten Maros kini akan mengangkat wisata
Pesta adat panen ini dilakukan oleh Pemangku Adat budaya.
Kerajaan Marusu, Abd Waris Karaeng Sioja, bersama E. Pemaduan objek wisata alam dan wisata budaya
masyarakat setempat. (6) Menurut Ferdiansyah ke depan untuk mendukung program pemerintah Wonderful
pesta adat yang lahir dari nilai-nilai budaya ini akan bekerja Indonesia
sama dengan Asosiasi Pengusaha Travel Indonesia
(ASITA) dan Perhimpunan Pengusaha Hotel dan Restoran 10. Kata berimbuhan yang tidak tepat pada tulisan 1 di atas
Indonesia (PHRI). (7) Harapannya, kerja sama ini dapat adalah…
memperluas gaung Katto Bokko di Maros. (8) “Jadi, ke A. pemaduan (kalimat 2)
depan, pesta adat tersebut tidak hanya dapat disaksikan oleh B. diangkat (kalimat 3)
orang Maros, tetapi juga orang dari luar Sulawesi Selatan, C. bekerja sama (kalimat 6)
bahkan oleh turis mancanegara, karena akan dimasukkan D. memperluas (kalimat 7)
dalam kalender pariwisata mendukung program pemerintah E. dimasukkan (kalimat 8)
(Wonderful Indonesia),” katanya.
11. Yang bukan padanan makna kata merangsang pada
(9)Beberapa objek wisata Kabupaten Maros, seperti kalimat 14 adalah…
Taman Wisata Alam Bantimurung, Taman Purbakala A. Menyebabkan
Leang-Leang, dan Taman Wisata Pegunungan Karst B. Membangkitkan
Rammang-Rammang, sudah dikenal hingga ke C. Memicu
mancanegara. (10) Objek wisata tersebut akan disenergikan D. Mendorong
dengan wisata budaya di Kabupaten Maros. (11) Oleh E. Mengangkat
karena itu, menurut Kadisbudpar Maros, sebelum ke objek

165
SSE SIDIKALANG : Jl. A. Yani no 56 & 58 Bimbel Sse Sidikalang bimbelsse
BIMBINGAN BELAJAR
“STAR SCIENCE EDUCATION SIDIKALANG” FOKUS LULUS UTBK SBMPTN
Be a Star With The Star

12. Penggunaan PUEBI yang tidak tepat terdapat pada… menghambat pertumbuhan janin pada bulan-bulan
A. kalimat 2 terakhir sebelum kelahiran.
B. kalimat 3 Kata yang bukan padanan kata penelitian pada kalimat
C. kalimat 4 di atas adalah…
D. kalimat 6 A. Penyelidikan D. peninjauan
E. kalimat 11 B. Pemeriksaan E. pembuktian
13. Suntingan yang tepat untuk kalimat (7) adalah… C. Pengusutan
A. Harapannya adalah kerja sama ini dapat memperluas 15. Ide pokok paragraf pertama teks di atas adalah…
gaung Katto Bokko di Maros. A. Polusi udara menghambat pertumbuhan janin pada
B. Kerja sama ini diharapkan dapat memperluas gaung trimester terakhir kehamilan.
Katto Bokko di Maros. B. Penelitiann dari Universitas Aberdeen mengenai
C. Harapannya, yaitu kerja sama ini dapat memperluas polusi udara
gaung Katto Bokko di Maros. C. Gas yang ditemukan dalam polusi udara dan
D. Harapannya dapat memperluas gaung Katto Bokko di pengaruhnya terhadap janin
Maros. D. Bobot lahir bayi berkaitan dengan asap rokok dan
E. Harapannya Pemerintah Maros adalah kerja sama ini faktor lain.
dapat memperluas gaung Katto Bokko di Maros. E. Penelitian dari Universitas Aberdeen menunjukkan
bahwa polusi udara menghambat pertumbuhan
janin pada trimester terakhir kehamilan.
Bacalah teks berikut untuk menjawab soal No 14 s.d. 19!
(1)Sebuah penelitian dari Universitas Aberdeen
16. Pernyataan yang tidak sesuai dengan teks di atas
Skotlandia menyebutkan, polusi udara menghambat
adalah…
pertumbuhan janin pada bulan-bulan terakhir sebelum
A. Paparan nitrogen eksogen dari knalpot kendaraan
kelahiran. (2) Berkurangnya bobot lahir bayi dikaitkan
bermotor mengakibatkan ukuran kepala bayi lebih
dengan paparan asap rokok serta banyak faktor lain. (3)
kecil.
Tim peneliti yang dipimpin oleh Profesor Steve Turner
B. Polusi udara menghambat pertumbuhan janin pada
mengatakan bahwa pihaknya telah meninjau selama satu
trimester terakhir kehamilan.
dekade atau kurang lebih 10 tahun untuk meneliti masalah
C. Penelitian tentang polusi udara yang dikaitkan
tersebut hingga akhirnya ditemukan hubungan antara gas
dengan ukuran janin telah dilakukan berbagai
yang ditemukan dalam polusi udara dan pengaruhnya pada
negara di dunia.
janin.
D. Asap rokok memilik dampak buruk bagi janin usia
(5)Turner mengatakan bahwa paparan nitrogen
7 bulan atau 3 bulan terakhir kehamilan.
dioksida yang dipancarkan dari knalpot kendaraan
E. Tim Profesor Steve Turner telah melakukan
bermotor, pembuatan dan pembakaran gas dan batu bara,
penelitian selama satu dekade.
mengakibatkan ukuran kepala yang lebih kecil pada bayi.
(6)Meskipun tidak dianggap sebagai gas rumah kaca,
17. Kata yang tidak baku pada teks di atas adalah…
nitrogen dioksida adalah polutan yang berkontribusi
A. menyebutkan (kalimat 1)
terhadap kabut asap. (7) Selain itu, efeknya sangat jelas
B. meninjau (kalimat 3)
dalam tiga bulan terakhir kehamilan. (8) Sumber-sumber
C. butana (kalimat 8)
lain, termasuk asap rokok dan pemanas serta kompor
D. intervensi (kalimat 11)
butana dan minyak tanah, juga memiliki dampak buruk bagi
E. meminimkan (kalimat 12)
janin yang berusia 7 bulan atau 3 bulan terakhir kehamilan.
(9) Selain itu, Turner juga menjelaskan bahwa
18. Pemakaian tanda koma yang tidak tepat terdapat
berbagai penelitian tentang polusi udara yang dikaitkan
pada…
dengan ukuran janin telah dilakukan oleh berbagai wilayah
A. kalimat 1
geografis di dunia, termasuk Australia, AS, dan beberapa
B. kalimat 5
negara di Eropa. (10) Namun, dalam penelitian ini, Turner
C. kalimat 8
dan timnya menemukan bukti jelas bahwa pada trimester
D. kalimat 9
ketiga khususnya, paparan nitrogen dioksida mengurangi
E. kalimat 10
pertumbuhan janin. (11) Untuk itu, Turner menekankan
pentingnya intervensi potensial pada tahap awal kehamilan.
19. Judul yang tepat untuk teks atas adalah…
(12) Selain itu, temuan ini juga menunjukkan bahwa
A. Polusi Udara dan Dampaknya
langkah-langkah kesehatan masyarakat sangat diperlukan
B. Bahaya Nitrogen Dioksida
untuk meminimkan paparan ibu hamil terhadap nitrogen
C. Pertumbuhan Janin pada Trimester Terakhir
dioksida.
Kehamilan
D. Pengaruh Penelitian Turner terhadap Janin
14. Sebuah penelitian dari Universitas Aberdeen,
E. Polusi Udara Menghambat Pertumbuhan Janin
Skotlandia, menyebutkan bahwa polusi udara

166
SSE SIDIKALANG : Jl. A. Yani no 56 & 58 Bimbel Sse Sidikalang bimbelsse
BIMBINGAN BELAJAR
“STAR SCIENCE EDUCATION SIDIKALANG” FOKUS LULUS UTBK SBMPTN
Be a Star With The Star

Bacalah teks berikut ini untuk menjawab soal No. 20 & B. Menghapus kata sebagai
21! C. Menyatukan kedua kalimat
(1)Daging ayam merupakan makanan terpopuler D. Mengganti kata yakni dengan kata yaitu
saat ini. (2) Orang Amerika mencatat bahwa konsumsi E. Menambah kata sama setelah kata kerja
daging ayam lebih tinggi daripada daging lain. (3) Andrew
Lawler, jurnalis veteran dan penulis buku Why Did the 23. Kalimat yang memuat afiks yang salah adalah...
Chicken Cross the World: The Epic Saga of the Bird That A. Pemertahanan wilayah yang strategis perlu
Powers Civilization, menjelaskan bahwa ayam goreng dilakukan segera sebelum musuh merebut wilayah
sekarang ini sebenarnya berasal dari Afrika. (4) Ia itu.
mengklaim bahwa hewan yang paling penting bagi B. Daerah rebutan itu berhasil kami pertahankan
penyebaran peradaban seluruh dunia adalah ayam. (5) Dia meskipun korban dipihak kami berjatuhan.
juga mengisahkan sejarah domestikasi ayam: dari ayam C. Penelitian dapat dilakukan dalam bentuk
hutan di kawasan Asia Selatan sampai Asia Tenggara wawancara, pengamatan, kajian pustaka, dan lain-
sekitar 10.000 tahun lalu, sampai menjadi spesies fleksibel lain.
yang mampu "melayani kebutuhan" makanan manusia. (6) D. Tayangan foto akan dilaksanakan setelah semua
Menurutnya, ayam yang sekarang ini hidup di bumi pemakalah selesai mempresentasikan makalah.
merupakan keturunan ayam hutan merah yang amat pemalu E. Pengunjung sedang memerhatikan tampilan
sehingga saking pemalunya saat ditangkap, mereka bisa berupagambar-gambar yang menarik.
mati terkena serangan jantung.
(7) Namun, para ilmuwan juga yakin kalau ayam Tulisan berikut diikuti oleh beberapa butir pertanyaan.
pada mulanya tidak dibawa melintasi benua. (8) Dalam Pertimbangkan apakah kata atau kalimat pada setiap
masyarakat kuno, maupun dalam banyak warisan tradisi nomor bercetak tebal TIDAK PERLU DIPERBAIKI (A)
yang masih lestari dewasa ini, ayam memiliki peranan luar atau diganti dengan pilihan lain yang tersedia (B, C, D,
biasa: mempunyai fungsi sosial-spiritual seperti di ritual atau E).
keagamaan. (9) Menurut Lawyer, sebenarnya dalam seluruh
sejarah umat manusia, ayam sudah ada dan muncul di Menristekdikti: Guru Harus Manfaatkan Teknologi
berbagai kebudayaan berbeda. (10) Pada buku itu juga
dipaparkan mengenai terjalinnya relasi antarspesies, (1)Menurut Menteri Riset, Teknologi, dan Pendidikan
manusiadan ayam, yang digambarkan hubungan itu sukses, Tinggi (Menristekdikti), Mohamad Nasir, mengatakan
bersimbiosis mutualisme, sehingga evolusi ayam telah di era revolusi industri 4.0 guru perlu mengikuti segala
mendukung peradaban manusia. teknologi yang dapat membantunya mengajar24. (2) Hal
itu diungkapkan saat menghadiri Wisuda Sekolah Tinggi
20. Kalimat yang memuat fakta mengenai ayam dalam teks Keguruan dan Ilmu Pendidikan Persatuan Guru Republik
di atas dapat ditemukan pada... . Indonesia (STKIP PGRI) Jombang, pada Sabtu (20/4). (3)
A. Kalimat (1) paragraf pertama Menurut Nasir, di era sekarang ini, guru tidak boleh
B. Kalimat (3) paragraf pertama ketinggalan di dalam teknologi yang mendukung literasi
C. Kalimat (5) paragraf pertama karena kalau tidak mengetahui teknologi, seorang guru pasti
D. Kalimat (7) paragraf kedua akan kalah oleh murid.
E. Kalimat (9) paragraf kedua (4) Nasir mengingatkan bahwa ilmu yang diajarkan
kepada murid tidak berhenti setiap tahun, melainkan25 terus
21. Popularitas ayam dalam kebudayaan manusia, diperbarui. (5) Oleh sebab itu, guru tidak boleh berpuas diri
menurut teks di atas, tidak hanya terjadi sekarang dengan buku ajaran yang sudah lama terbit. (6) Menurutnya,
karena... dosen dan guru jangan sampai ketinggalan teknologi dengan
A. Ayam mudah diternakkan menggunakan buku yang dipakai cetakan tahun-tahun yang
B. Ayam mudah dijinakkan daripada ungags liar lain lalu, tahun 2005 atau 2010, padahal sekarang sudah tahun
C. Berperan penting dalam tradisi masyarakat 2019 dan harus updater.
D. Memiliki hubungan mutualistic dengan manusia (7) Lebih lanjut, Nasir menyatakan bahwa di era
E. ayam mendukung perkembangan peradaban Revolusi Industri 4.0 ini akan ada banyak mata pelajaran
manusia dan mata kuliah yang dapat diajarkan tanpa guru atau dosen
sama sekali. (8) Sehingga26, guru dan dosen perlu
22. Kegiatan ini sebagai upaya konservasi warisan budaya meningkatkan kemampuan mengajarnya juga dengan
bawah air merupakan hasil kerja beberapa pihak. bantuan teknologi. (9) Dengan demikian, para guru tetap
Yakni Direktorat Pelestarian Cagar Budaya dan dapat memberikan pengajaran yang maksimal. (10) “Kami
Permuseuman, Direktorat Jenderal Kebudayaan, datang ke Korea Selatan, Jepang, dan Cina, bahkan sampai
sertaKementerianPendidikandanKebudayaan. Agar dengan Kanada. (11) Ada suatu proses ajaran27 dengan
kalimat di atas menjadi lebih efektif, hal yang harus kondisi mahasiswa sudah tidak lagi bersentuhan dengan
dilakukan adalah sebagai berikut,kecuali… orang di tingkat pendidikan tinggi,” kata Nasir.
A. Memindahkan kata ini ke posisi setelah kata air
167
SSE SIDIKALANG : Jl. A. Yani no 56 & 58 Bimbel Sse Sidikalang bimbelsse
BIMBINGAN BELAJAR
“STAR SCIENCE EDUCATION SIDIKALANG” FOKUS LULUS UTBK SBMPTN
Be a Star With The Star

(12) Dalam kesempatan yang sama ketua umum E. Pelajaran


Persatuan Guru Republik Indonesia (PGRI) Pusat
Unifah Rosyidi mengatakan, guru di Indonesia akan 28. Dalam kesempatan yang sama ketua umum Persatuan
tetap dibutuhkan di era Revolusi Industri 4.0 28. (13) Guru Republik Indonesia (PGRI) Pusat Unifah Rosyidi
Namun, tentu saja para guru harus terus meningkatkan mengatakan, guru di Indonesia akan tetap dibutuhkan
profesionalismenya. (14) “Profesionalisme29 adalah kata di era Revolusi Industri 4.0.
kunci dan menjadi perhatian global30. (15) Oleh karena itu, A. TIDAK PERLU DIPERBAIKI
dengan perkembangan ilmu pengetahuan dan teknologi, kita B. Dalam kesempatan yang sama Ketua Umum
akan terus belajar dan profesionalisme itulah yang menjadi Persatuan Guru Republik Indonesia (PGRI) Pusat
kuncinya. (15) Dengan terus mengasah diri, insya Allah kita Unifah Rosyidi mengatakan, guru di Indonesia
akan bisa memosisikan diri, apa pun tantangannya di era akan tetap dibutuhkan di era Revolusi Industri 4.0.
4.0,” ujar Unifah. C. Dalam kesempatan yang sama ketua umum
Persatuan Guru Republik Indonesia (PGRI) Pusat,
24. Menurut Menteri Riset, Teknologi, dan Pendidikan Unifah Rosyidi, mengatakan, guru di Indonesia
Tinggi (Menristekdikti), Mohamad Nasir, mengatakan akan tetap dibutuhkan di era Revolusi Industri 4.0..
di era revolusi industri 4.0 guru perlu mengikuti segala D. Dalam kesempatan yang sama, Ketua Umum
teknologi yang dapat membantunya mengajar. Persatuan Guru Republik Indonesia (PGRI) Pusat,
A. TIDAK PERLU DIPERBAIKI Unifah Rosyidi, mengatakan bahwa guru di
B. Menurut Menteri Riset, Teknologi, dan Pendidikan Indonesia akan tetap dibutuhkan di era Revolusi
Tinggi (Menristekdikti), Mohamad Nasir, bahwa di Industri 4.0.
era Revolusi Industri 4.0 guru perlu mengikuti E. Dalam kesempatan yang sama, Ketua umum
segala teknologi yang dapat membantunya Persatuan Guru Republik Indonesia (PGRI) Pusat
mengajar. Unifah Rosyidi mengatakan bahwa guru di
C. Menurut Menteri Riset, Teknologi, dan Pendidikan Indonesia akan tetap dibutuhkan di era Revolusi
Tinggi (Menristekdikti) Mohamad Nasir Industri 4.0.
mengatakan di era Revolusi Industri 4.0 guru perlu
mengikuti segala teknologi yang dapat 29. Profesionalisme
membantunya mengajar. A. TIDAK PERLU DIPERBAIKI
D. Menteri Riset, Teknologi, dan Pendidikan Tinggi B. Profesionalisasi
(Menristekdikti), Mohamad Nasir, mengatakan di C. Professional
era Revolusi Industri 4.0 guru perlu mengikuti D. Profesionalitas
segala teknologi yang dapat membantunya E. Profesi
mengajar.
E. Menteri Riset, Teknologi, dan Pendidikan Tinggi 30. Global
(Menristekdikti), Mohamad Nasir, mengatakan A. TIDAK PERLU DIPERBAIKI
bahwa di era Revolusi Industri 4.0, guru perlu B. Globalisasi
mengikuti segala teknologi yang dapat C. Dunia
membantunya mengajar. D. seluruh negara
E. masyarakat
25. Melainkan
A. TIDAK PERLU DIPERBAIKI
B. Sedangkan
C. Bahkan
D. Padahal
E. Tetapi

26. Sehingga
A. TIDAK PERLU DIPERBAIKI
B. Akan tetapi
C. Oleh karena itu
D. Selain itu
E. Jadi

27. Ajaran
A. TIDAK PERLU DIPERBAIKI
B. Pembelajaran
C. Belajar
D. Mengajar
168
SSE SIDIKALANG : Jl. A. Yani no 56 & 58 Bimbel Sse Sidikalang bimbelsse
BIMBINGAN BELAJAR
“STAR SCIENCE EDUCATION SIDIKALANG” FOKUS LULUS UTBK SBMPTN
Be a Star With The Star

A. Jumlah korban meninggal dunia akibat banjir


bandang di Sentani tercatat hanya 79 orang.
B. Kepala Pusat Data Informasi dan Humas Badan
Nasional Penanggulangan Bencana (BNPB) Sutopo
Purwo Nugroho mengatakan ada sembilan belas
kelurahan terdampak akibat bencana.
C. Dari sembilan kelurahan yang terdampak akibat
bencana, yang paling parah hanya kelurahan Doyo
Baru.
D. Korban hilang dan tewas akibat bencana banjir
bandang Sentani paling banyak ditemukan di
Teks berikut ini untuk menjawab soal No.1 s.d. 7! Kelurahan Dobonsolo, Honekombe, dan Doyo Baru.
Jumlah korban meninggal dunia akibat banjir E. Terdapat 34 orang yang masih hilang dalam banjir
bandang di Sentani, Kabupaten Jayapura, Papua sudah bandang di Sentani.
mencapai 79 jiwa. Jumlah tersebut kemungkinan masih bias
bertambah karena saat ini tercatat 43 orang masih hilang. 2. Berdasarkan paragraf satu, manakah simpulan di bawah
Kepala Pusat Data Informasi dan Humas Badan Nasional ini yang benar?
Penanggulangan Bencana (BNPB) Sutopo Purwo Nugroho A. Data korban bandang di Sentani sementara 79 jiwa
mengatakan ada Sembilan kelurahan terdampak akibat meninggal dunia, 43 orang dinyatakan masih hilang,
bencana. Tiga kecamatan di antaranya terdampak paling dan dan data ini kemungkinan masih akan
parah. “Dari Sembilan kelurahan ini, yang paling parah ada bertambah.
di Kelurahan Dobonsolo, Doyo Baru, dan Hinekombe. Jadi B. Banjir bandang di Sentani menimbulkan banyak
korban yang paling banyak di tiga kelurahahn ini,” kata kerugian.
Sutopo dalam jumpa pers di Graha BNPB , Jakarta, Senin C. Banjir bandang di Sentani menimbulkan banyak
(18/3. korban jiwa dan korban harta.
Dalam musibah ini, tercatat ada 74 orang terluka, D. Banjir bandang di Sentani dinyatakan karena ulah
4.266 orang mengungsi, dan 11.725 keluarga yang manusia yang menebang phon sembarangan.
terdampak. Kerugian material pun tak kalah besar. E. Terdapat Sembilan daerah yang terdampak parah
Setidaknya ada 350 unit rumah rusak berat, 211 unit rumah akibat banjir bandang Sentani, empat di antaranya
terendam air, 8 unit sekolah rusak berat, 3 jembatan rusak kelurahan Dapapre, Dobonsolo, Honekombe, dan
berat, dan 1 unit pesawat hilang. Proses evakuasi dilakukan Doyo Bayu.
oleh tim SAR gabungan mulai dari BPBD, TNI, Pemda,
hingga relawan. “Kelada BNPB Doni Mordano disebut 3. Berdasarkan bacaan di atas, manakah pernyataan di
sudah tiba di lokasi musibah sejak pagi ini untuk meninjau bawah ini yang paling benar?
penanganan bencana. Kepala BNPB melakukan rapat A. Proses evakuasi dilakukan oleh tim SAR gabungan
koordinasi dengan bupati dan petugas yang ada di sana mulai dari BPDB, TNI, Pemda, Kepolisian Sentani,
untuk melakukan penanganan-penanganan,” imbuh Sutopo. hingga relawan.
Sementara itu, di lokasi yang berbeda, Kepala B. “Dari sembilan kelurahan ini, yang paling parah ada
Kepolisian Daerah Papua Irjen Pol Martuani Sormin di Kelurahan Dobonsolo, Doyo Baru, dan
mengatakan bahwa banjir bandang Sentani, Jayapura diduga Hinekombe. Jadi korban yang paling banyak di tiga
terjadi akibat pembalakn liar yang terjadi di kawasan kelurahan ini,” kata Sutopo dalam jumpa pers di
Pegunungan Cyloops. “Bahkan, akibat pepohonan sudah Graha BNPB, Jayapura, Senin (18/3).
ditebang oleh oknum yang tidak bertanggung jawab C. Setidaknya ada 350 unit rumah terendam air, 8 unit
menyebabkan berbagai material, seperti bebatuan dan sekolah rusak berat, 3 jembatan rusak berat, dan 1
batang kayu serta lumpur, ikut terseret,” kata dia di unit pewasat Twin Otter rusak.
Jayapura, Minggu (17/3). Banjir bandang yang terjadi Sabtu D. Tercatat 74 orang terluka, 4.266 orang mengungsi,
(16/3) malam itu, disebabkan berkurangnya pepohonan di dan 11.725 keluarga yang terdampak banjir bandang
kawasan gunung Cyloop meruoakan kawasan hutan lindung di Sentani, Jayapura.
yang seharusnya dijaga kelestariannya. Oleh karena itu, E. Kepala BNPB Doni Mornado disebut sudah tiba di
lindung yang tidak dijaga kelestariannya, saat curah hujan lokasi musibah sejak pagi, (17/3) untuk meninjau
tinggi menjadi penyebab bandang yang bahkan penanganan bencana.
mengakibatkan jatuhnya korban, termasuk harta benda. “Ke
depan diharapkan tidak lagi terjadi pembalakan karena 4. Berdasarkan paragraf dua, manakah pernyataan yang
dampak yang ditimbulkan dapat menyebabkan korban jiwa tidak benar?
dan harta benda.” kata Irjen Pol Sormin. A. Tercatat jumlah korban banjir bandang Sentani 79
jiwa meninggal dunia dan kemungkinan akan
1. Berdasarkan paragraf satu, manakah pernyataan di bertambah.
bawah ini yang benar?
169
SSE SIDIKALANG : Jl. A. Yani no 56 & 58 Bimbel Sse Sidikalang bimbelsse
BIMBINGAN BELAJAR
“STAR SCIENCE EDUCATION SIDIKALANG” FOKUS LULUS UTBK SBMPTN
Be a Star With The Star

B. Kepala BNPB Doni Mordano disebut sudah tiba di


lokasi musibah sejak pagi, (18/3) untuk meninjau Teks berikut digunakan untuk menjawab soal No. 1 s.d.
penanganan bencana. No. 7!
C. Satu unit pesawat Twin Onner digunakan untuk Kementrian Perhubungan (Kemenhub) mengaku
menyelamatkan korban banjir bandang di Sentani. tengah menyusun aturan baru terkait tiket pesawat terbang.
D. Proses evakuasi dan penyisiran masih berlangsung Aturan disusun untuk merespons keluhan masyarakat soal
karena 43 orang masih berstatus hilang. tarif tiket pesawat yang belakangan ini dianggap terlalu
E. Kepala BNPB melakukan rapat koordinasi dengan mahal. Kepala Biro Komunikasi dan Informasi Publik
Bupati dan petugas yang ada di sana untuk Kementrian Perhubungan Hengki mengatakan aturan
melakukan penanganan-penanganan. tersebut disusun dengan melibatkan pihak maskapai dan
tengah difinalisasi.
5. Berdasarkan paragraf tiga, manakah yang paling Masyarakat mulai melancarkan protes terkait
mungkin terjadi? mahalnya tiket pesawat awal tahun ini. Mereka
A. Pepohonan yang sudah ditebang oleh oknum yang menyampaikan seragkaian petisi di situs change.org. petisis
tidak bertanggung jawab menyebabkan berbagai tersebut direspons positif oleh Indonesia Nasional Air Carier
material, seperti bebatuan dan batang kayu serta Association (INACA) dengan kebijakan penurunan harga
lumpur, tidak ikut terseret. tiket. INACA menyebut rentang penurunan harga tiket
B. Banjir bandang mungkin terjadi jika pembalakan liar domestik berkisar 20% - 60%, bergantung kebijakan
tidak terjadi di kawasan Pegunungan Cyloop. masing-masing maskapai. Kebijakan ini berlaku untuk 34
C. Hutan lindung, jika tidak dijada kelestariannya, maskapai yang tergabung dalam INACA, termasuk di
banjir bandang tidak akan terjadi. dalamnya Gardu Indonesia., Citilink, Lion Air, Sriwijaya.
D. Pembalakan liar tidak terjadi, maka ke depan Presiden Joko Widodo bahkan juga turun tangan dengan
dampak yang ditimbulkan dapat menyebabkan mengimbau maskapai agar menurunkan tariff tiket pesawat.
korban jiwa dan harta benda. Meski demikikan, masih banyak masyarakat yang
E. Banjir bandang yang terjadi Sabtu (16/3) disebabkan hingga kini masih mengeluhkan mahalnya tarif tiket
bertambahnya pepohonan di kawasan hutan Cyloop pesawat. Salah satunya, Agus Yustiadi (27 tahun). Agus
sehingga saat hujan deras menguyur kawasan itu mengaku terpaksa mengurungkan niatnya pulang ke
menyebabkan banjir. kampong halaman karena harga tiket pesawat masih mahal.
Ia mengaku tak mampu merogoh kocek untuk membayar
6. Berdasarkan bacaan di atas, manakah pernyataan berikut tiket pulang pergi Jakarta – Pontianak seharga Rp 1,88 juta.
paling benar? Padahal, sebelumnya, ia hanya membayar Rp 1,2 juta untuk
A. Paragraf satu menjelaskan daerah-daerah yang rute yang sama. “Ini berarti 50% lebih mahal. Katanya
terdampak banjir. sufah diturunkan, tapi malah naiknya nggak kira-kira,”
B. Paragraf dua merincikan daftar jumlah korban jiwa ujarnya kepada CNN Indonesia.com. Selain soal kebijakan
dan kerusakan akibat banjir bandang di Sentani, tariff, sejumlah pihak juga meminta Kemenhub untuk
Jayapura. membuat pengaturan terkait tarif bagasi. Pengaturan tersebut
C. Paragraf ketiga merincikan bukti-bukti mengapa pun kini masih dirumuskan.
terjadi banjir bandang di Sentani, Jayapura.
D. Paragraf 1 dan 2 memaparkan rincian dari paragraf Daftar Harga Tiket beberapa maskapai penerbangan
3. pada 31 Maret 2019 untuk sekali terbang diakses dari
E. Paragraf 2 dan 3 merupakan akibat dari paragraf 1. tiket.com Sabtu, 30 Maret 2019 pukul 08.15 WIB
Masakapai Jumlah Waktu Harga Tiket
7. Berdasarkan bacaan di atas, manakah kalimat di bawah Penerbangan Penerbangan
ini yang termasuk opini? Air Asia 4 jam 30 menit Rp 1.748.000
A. Kepala BNPB Doni Mornado disebut sudah tiba di Lion Air 2 jam 20 menit Rp 1.907.000
lokasi musibah sejak pagi (18/3) untuk meninjau Batik Air 2 jam 20 menit Rp 2.101.000
penangan bencana. Sriwijaya Air 2 jam 15 menit Rp 2.176.000
B. Tercatat ada 74 orang terluka, 4.266 orang Garuda 2 jam 15 menit Rp 2.448.000
mengungsi, dan 11.725 keluarga yang terdampak. Indonesia
C. Pepohonan yang sudah ditebang oleh oknum yang Citilink 3 jam 20 menit Rp 1.981.000
tidak bertanggung jawab menyebakan berbagai
material, seperti bebatuan dan batang kayu serta 8. Berdasarkan paragraf tiga, manakah pernyataan di
lumpur ikut terseret. bawah ini yang benar?
D. Kepala Kepolisian Daerah Papua Irjen Pol Martuani A. Pengaturan terkait harga tiket pesawat dan tarif
Sormin memberikan keterangan penyebab bagasi sudah dirumuskan oleh pihak maskapai
terjadinya banjir bandang di Sentani, Jayapura. penerbangan dengan Kemenhub.
E. Banjir bandang di Sentani, Jayapura terjadi Sabtu
(16/3) malam.
170
SSE SIDIKALANG : Jl. A. Yani no 56 & 58 Bimbel Sse Sidikalang bimbelsse
BIMBINGAN BELAJAR
“STAR SCIENCE EDUCATION SIDIKALANG” FOKUS LULUS UTBK SBMPTN
Be a Star With The Star

B. Banyak masyarakat terpaksa mengurungkan niatnya B. INACA menyebut rentang penurunan harga tiket
pulang kampung menggunakan maskapai domestik berkisar 20% - 60%, bergantung
penerbangan karena mahalnya harga tiket pesawat. kebijakan masing-masing maskapai.
C. Harga tiket dari Rp 1,2 juta menjadi 1,88 juta C. Presiden Joko Widodo bahkan juga turun tangan
mengalamai kenaikan sebesar 100%. dengan mengimbau INACA agar menyesuaikan
D. Agus Yustiadi pulang ke kampung halamannya tarif tiket pesawat.
dengan menggunakan maskapai penerbangan rute D. INACA merupakan sebuah Asosiasi Perusahaan
Jakarta – Pontianak. Penerbangan Nasional Indonesia yang menaungi
E. Salah satu masyarakat berpendapat bahwa ia 34 maskapai penerbangan termasuk di dalamnya
mengurungkan niatnya pulang kampung karena Garuda Indonesia, Citilink, Lion Air, dan
mahalnya harga tiket pesawat. Sriwijaya.
E. INACA berhasil menurunkan harga tiket dan tarif
9. Berdasarkan paragraf tiga, jika pengaturan harga tiket bagasi 34 masakapai penerbangan yang tergabung
pesawat dan tarif bagasi berhasil dirumuskan, manakah di dalamnya.
simpulan di bawah ini yang benar?
A. Masyarakat mengeluhkan naiknya harga tiket 12. Berdasarkan tabel, maskapai manakah yang lebih
pesawat dan tariff bagasi. efisien jika dilihat dari harga tiket?
B. Agus Yustiadi pulang ke kampung halamannya A. Citilink dan Air Asia
menggunakan maskapai penerbangan. B. Lion Air dan Citilink
C. Harga tiket pesawat menjadi 50% lebih mahal dari C. Air Asia dan Lion Air
harga sebelumnya. D. Air Asia dan Batik Air
D. Harga tiket rute Jakrta – Pontianak menjadi Rp 1,2 E. Batik Air dan Lion Air
juta per penerbangan.
E. Maskapai penerbangan akan mendapat keuntungan 13. Berdasarkan tabel, manakah masakapai penerbangan
yang sangat besar. yang lebih diminati berdasarkan jumlah waktu
terbang?
10. Berdasarkan paragraf dua, apabila penurunan harga tiket A. Citilink dan Sriwijaya
domestik berkisar 20% - 60 % sudah diputuskan, B. Garuda Indonesia dan Batik Air
manakah simpulan di bawah ini yang PALING C. Batik Air dan Lion Air
MUNGKIN benar? D. Lion Air dan Sriwijaya
A. Tiga puluh empat maskapai yang bergabung dalam E. Garuda Indonesia dan Air Asia
INACA termasuk di dalamnya Lion Air, Garuda 14. Berdasarkan tabel, manakah yang PALING
Indonesia, Citilink, Sriwijaya akan menurunkan MUNGKIN terjadi?
harga tiket pesawat. A. Maskapai Lion Air akan lebih diminati karena jam
B. Tiga puluh empat maskapai yang tergabung dalam terbang yang efisien.
INACA, termasuk di dalamnya Lion Air, Garuda B. Masakapai Air Asia laku keras dibandingkan
Indonesia, Citilink, Sriwijaya akan menurunkan penerbangan lainnya karena tiketnya lebih murah.
harga tiket pesawat dan tarif bagasi. C. Maskapai Garuda Indonesia lebih diminati karena
C. Tiga puluh empat maskapai yang bergabung dalam aman.
INACA termasuk di dalamnya Lion Air, Garuda D. Maskapai Sriwijaya sepi penumpang.
Indonesia, Citilink, Sriwijaya akan menurunkan tarif E. Tiket maskapai penerbangan Batik Air habis
bagasi. diburu calon penumpang.
D. Tiga puluh empat maskapai yang bergabung dalam
INACA termasuk di dalamnya Lion Air, Garuda Bacalah teks berikut untuk menjawab soal No. 15 s.d. 19!
Indonesia, Citilink, Sriwijaya menurunkan harga Walaupun khasiat susu bagi tubuh kita sudah tidak
tiket pesawat. diragukan lagi, tidak setiap orang dapat mengonsumsi air susu
E. Tiga puluh empat maskapai yang tergabung dalam dengan baik. Hal ini disebabkan oleh dua kemungkinan,
INACA, termasuk di dalamnya Lion Air, Garuda yakni sifat yang terkandung dalam susu yang tidak disukai
Indonesia, Citilink, Sriwijaya menurunkan harga orang atau sifat biologis orang yang bersangkutan (intoleran).
tiket pesawat dan tarif bagasi. Penyebab pertama dapat diatasi dengan penambahan sari
jeruk, markisa, apel, atau lainnya sehingga rasa asli susu yang
11. Berdasarkan paragraf dua, manakah pernyataan di memualkan dapat dihilangkan. Sementara itu, penyebab
bawah ini yang PALING MUNGKIN benar mengenai kedua dapat diatasi dengan menggantinya dengan air susu
INACA? F. mengalami perlakuan khusus, yaitu fermentasi.
yang telah
A. Indonesia National Air Carriers Association atau Secara biologis, penderita intoleran susu tidak mampu
INACA merupakan sebuiah Asosisasi Perusahaan mencerna laktosa dari makanan atau minuman dalam susu
Penerbangan Nasional Indonesia yang didirikan sehingga terjadi penimbunan laktosa dalam usus. Penderita
oleh para pengusaha perusahaan penerbangan. yang demikian dapat minum susu bubuk dengan kadar laktosa

171
SSE SIDIKALANG : Jl. A. Yani no 56 & 58 Bimbel Sse Sidikalang bimbelsse
BIMBINGAN BELAJAR
“STAR SCIENCE EDUCATION SIDIKALANG” FOKUS LULUS UTBK SBMPTN
Be a Star With The Star

rendah atau air susu fermentasi, seperti yoghurt, kefur, dan C. cara mengatasi laktosa penderita intoleran susu,
G.
koumis. pentingnya susu kadar laktosa rendah, dan ke-
Susu fermentasi juga bermanfaat bagi manula (manusia usia untungan susu fermentasi
lanjut) yang mengalami kesukaran dalam mencerna makanan D. kemungkinan orang tidak suka susu, penderita
yang berprotein linggi. Susu fermentasi, seperti yoghurt, yang intoleran susu, dan cara mengatasi masalah penderita
dapat dicerna dalam waktu satu jam dapat mengatasi yang mengalami kesulitan mencerna makanan
kesukaran ini. Keuntungan lain dari susu fermentasi dapat berprotein tinggi.
menurunkan kadar kolesterol darah dan mencegah diare bagi E. khasiat susu bagi tubuh, kandungan laktosa dalam
penderita kekurangan laktosa. susu, dan berbagai keuntungan mengonsumsi susu
15. Masalah pokok yang dibahas dalam bacaan di atas fermentasi.
adalah…
A. khasiat susu bagi manula 20. Kata berimbuhan bercetak miring berikut digunakan
B. manfaat susu fermentasi bagi konsumen intoleran secara tepat dalam konteks kalimat....
susu A. Setiap mandor di perusahaan ini membawahkan
C. kelebihan susu fermentasi sepuluh orang kuli.
D. penanganan kendala mengonsumsi susu B. Pimpinan menugaskan stafnya untuk mengikuti
E. kandungan gizi susu bubuk dengan kadar laktosa rapat.
rendah C. Semoga Anda mendapatkan rahmat dari Tuhan
Yang Maha Esa.
16. Pertanyaan yang jawabannya dapat ditemukan pada D. Dia berusaha keras mencarikan pekerjaan untuk
bacaan di atas adalah .... temannya.
A. Mengapa penderita intoleran tidak mau E. Kepala sekolah menanyakan guru BP tentang anak-
mengonsumsi susu? anak yang berkasus.
B. Bagaimanakah mengurangi kadar laktosa pada susu?
C. Bagaimanakah proses fermentasi susu? 21. Kata berimbuhan me-kan yang memiliki kesamaan
D. Benarkah bahwa semua susu bubuk berkadar laktosa dalam pembentukan dan makna kata dengan kata
rendah? membersihkan pada kalimat "Satu hal yang juga
E. Apakah susu fermentasi lebih baik daripada susu dilakukan adalah membersihkan jalur pedestrian dari
yang tidak terfermentasi? parkir-parkir kendaraan."terdapat pada... .
A. Hewan vertebrata memiliki tali yang merupakan
17. Yang dimaksud dengan penderita intoleran susu susunan tempat terkumpulnya sel-sel saraf dan
dalam bacaan di atas adalah .... memiliki perpanjangan kumpulan saraf dari otak.
A. orang yang tidak suka minum susu karena tidak suka B. Pembebasan bersyarat ini membuat masyarakat
dengan rasa atau bau susu mulai meragukan komitmen pemerintah dalam hal
B. orang yang secara biologis menghindari minum susu pemberantasan korupsi
karena kandungan laktosa susu C. Pada 2009, banjir bandang bahkan telah
C. orang yang secara biologis tidak dapat mencerna melumpuhkan sebagian wilayah Kuala Lumpur,
kandungan laktosa susu Malaysia.
D. orang yang suka minum susu yang sudah diubah D. Akhirnya, Real Madrid mampu memenangkan
rasa dan baunya pertandingan dengan skor telak.
E. orang yang secara biologis hanya boleh minum susu E. Beberapa ratus spesies ikan memiliki organ penghasil
yang sudah difermentasi listrik, namun hanya sedikit yang dapat
menghasilkan daya listrik yang kuat.
18. Simpulan yang sesuai dengan isi bacaan di atas adalah….
A. orang yang sudah berusia lanjut harus minum susu 22. Gates memang dikenal aktif berkampanye tentang isu
yang berfermentasi. kesehatan dan kemanusiaan. Ia mendanai proyek inovatif
B. orang yang menderita gangguan pencernaan tidak ini melalui Gates Foundation, yayasan filantropi yang
boleh minum susu didirikannya bersama sang istri, Melinda Gates. Pendiri
C. orang yang tidak suka minum susu akan mudah Microsoft ini telah lama berkomitmen membagikan air
terkena penyakit. bersih kepada 700 juta penduduk miskin yang kesulitan
D. penderita intoleran susu dapat mengonsumsi susu
mengakses air bersih. Kalimat manakah yang memuat
yang sudah mengalami perlakuan khusus
kata bermakna gramatikal sama dengan kata mendanai?
E. orang yang kegemukan karena terlalu banyak
A. Setelah menuangkan air segelas, dia menggulai
kolesterol tidak boleh mengonsumsi susu
tehnya sehingga menjadi manis.
B. Tinja justru bisa mencemari sumber dan pasokan air.
19. Secara runtun masalah pokok yang dibicarakan bacaan
C. Menurut Gates, sekitar 40 persen populasi dunia atau
di atas adalah .... 2,5 miliar penduduk perkotaan tidak memiliki akses
A. penyebab orang tidak menyukai susu, penderita terhadap sanitasi yang memadai.
intoleran susu, dan manfaat susu fermentasi D. Dia menyisipkan sehelai kertas dalam buku itu
B. khasiat susu bagi tubuh, penderita intoleran susu, untuk membatasi halaman buku yang sudah
dan masalah kesehatan manula dibacanya.
172
SSE SIDIKALANG : Jl. A. Yani no 56 & 58 Bimbel Sse Sidikalang bimbelsse
BIMBINGAN BELAJAR
“STAR SCIENCE EDUCATION SIDIKALANG” FOKUS LULUS UTBK SBMPTN
Be a Star With The Star

E. Dengan alat ini, mereka dapat hidup secara sehat dan D. Tempat penyelenggaraan kegiatan yakni Ladang Nan
menjalani kualitas hidup yang lebih baik. Jombang.
E. Secara keseluruhan, angin segar dan kebaruan
23. Penulisan bilangan dengan huruf yang tepat ditemukan diembuskan oleh kelompok-kelompok musik Asia.
dalam kalimat-kalimat berikut, KECUALI... .
A. Empat puluh saham yang dibeli perusahaan 28. Telah berabad-abad lamanya manusia menyadari bahwa
konglomerat itu merupakan saham dengan IHSG musik memiliki dampak spiritual yang besar bagi
yang stabil selama ini. kesejahteraan hidup manusia. (2) Musik lahir dari
B. Seorang narasumber mengatakan ada sekitar dua kecintaan manusia pada kehidupan dan dilandasi oleh
ratus empat puluh bangunan di kawasan kota lama ingatan manusia akan pengalaman-pengalaman
dengan tidak lebih dari dua ratus telah dimanfaatkan. hidupnya. (3) ... . (4) Di selatan Perancis, di daerah
C. Berbeda dengan tiga tahun yang lalu, dua tahun pegunungan Pirenia, serta di sejumlah situs arkeologi di
terakhir ini kondisi perdagangan produk bahari Rusia juga ditemukan sejumlah bentuk alat musik yang
meningkat. menyerupai seruling yang diperkirakan berusia
D. Sementara itu, dua orang awak hilang setelah perahu 30.000tahun.
kecil yang mereka tumpangi terbalik diterjang Kalimat manakah yang dapat menjadi kalimat 3 sehingga
gelombang. (E) Sejumlah lima persen mahasiswa paragraf menjadi lengkap?
berindeks prestasi terbaik mendapat beasiswa A. Musik bersumber dari akar kata muse dalam bahasa
sepanjang studi. Inggris yang berarti renungan.
B. Meskipun demikian, di museum di Belanda, terdapat
24. Kata hasil penyerapan digunakan secara tidak tepat koleksi artefak kuno yang terkait degan musik.
dalam kalimat... . C. Menurut data arkeologi, dibenua Afrika dan Eropa
A. Ironisnya, Jakarta tetap macet meskipun beberapa diperkirakan ada sekitar 40.000 bentuk tatahan pada
upaya telah dilakukan. dinding batu yang menggambarkan aktivitas manusia
B. Kalau kondisi tersebut dibiarkan, Jakarta akan dengan musik.
menjadi lebih parah, bahkan mungkin lalu lintas di D. Sementara itu, dalam mitologi Yunani, Muse
Jakarta menjadi stagnasi. (Muses) adalah sembilan putri cantik hasil
C. Pada tahun 1980 jumlah transportasi massal 18 % perkawinan Dewa Zeus dan Dewi Mnemosyne.
dari jumlah kendaraan. E. Ia bagian dari manusia dan seumur dengan
D. Sekarang sistem transportasi trem digantikan dengan kehidupan manusia itu sendiri.
kendaraan jenis bus.
E. Sistem transportasi yang baik adalah yang pertama 29. Fasilitas kesehatan tingkat pertama yang dituntut mampu
bus, kedua trem, ketiga monorail, dan keempat baru mendiagnosis 155 penyakit belum berperan maksimal.
subway. Pola kalimat di atas sama dengan pola kalimat... .
A. Manfaat layanan kesehatan yang diberikan akan
25. Tanda koma (,) yang tidak tepat digunakan pada ditambah.
kalimat... . B. Realisasi penambahan peserta ternyata tidak hanya
A. Gajah itu, kata Rusli, ditemukan, sedang merusak Sembilan juta seperti yang ditargetkan.
ladang. C. Terkait rujukan berjenjang, perbedaan tarif yang
B. Di bagian pangkal pipa, tersambung kaleng minyak jauh antartipe rumah sakit juga ditengarai menjadi
rem secara vertikal. penyebab terjadinya penumpukan pasien.
C. Menurut Rusli, warga mulai mendiami Dusun D. Setiap peserta berhak mendapat layanan kesehatan
Selamat sejak awal 1980-an. bermutu dimana pun mereka berada.
D. Namun, banjir bandang pada 1987 membuat mereka E. Besaran kapital yang ada saat ini dinilai oleh
harus kembali ke kampong halamannyadi Desa beberapa pengamat tidak adil bagi tenaga kesehatan.
Masen, desa asalny.
E. Dia menjual hasil kopinya di pusat kecamatan atau
ke kota terdekat di Calang, ibukota Kabupaten Aceh
Jaya.

26. Kalimat yang seluruhnya menggunakan ragam bahasa


standar, baik dari segi ejaan, pilihan kata, dan struktur
kalimat adalah... .
A. Sebagian besar kegiatan berlangsung di sanggar yang
menyebut dirinya Ladang Nan Jombang.
B. Hampir semua dari para peserta menampilkan karya
berupa eksperimen-eksperimen yang berhubungan
dengan tubuh.
C. Di tengah kepanikan terhadap gempa bumi, Kota
Manado diguyur hujan deras yang amat lebat.

173
SSE SIDIKALANG : Jl. A. Yani no 56 & 58 Bimbel Sse Sidikalang bimbelsse
BIMBINGAN BELAJAR
“STAR SCIENCE EDUCATION SIDIKALANG” FOKUS LULUS UTBK SBMPTN
Be a Star With The Star

Bacaan 2.
(1) Saat ini makin banyak kebudayaan Betawi yang tidak
dapat dinikmati masyarakat globalisasi. (2) Contohnya
adalah seni pertunjukkan ondel-ondel. (3) Seni pertunjukkan
ondel-ondel adalah seni pertunjukkan dengan boneka besar
setinggi dua meter. (4) Pada awalnya, boneka ini berfungsi
sebagai penolak bala dari gangguan roh halus. (5) Dalam
Bacaan 1. perkembangan selanjutnya, pertunjukan ondel-ondel juga
(1) Musik, bagi banyak orang, telah menjadi bagian dari digunakan untuk menambah semarak pesta-pesta rakyat,
kehidupan sehari-hari. (2) Tidak hanya dapat membuat peresmian gedung baru, atau menyambut tamu terhormat.
seseorang bersenandung dan menggerakkan badan, juga (6) Contoh lain kebudayaan Betawi yang terancam tergerus
dapat membantu kita mengatasi masalah kesehatan. (3) globalisasi adalah Lenong. (7) Lenong merupakan teater
Terdapat beberapa manfaat musik yang telah terbukti tradisional Betawi yang diiringi musik gambang kromong.
melalui penelitian terkait hubungan musik dengan kesehatan (8) Gambang kromong adalah musik tradisional Betawi
yang dimuat dalam berbagai jurnal ilmiah internasional. yang mendapat pengaruh dari suku Tionghoa. (9) Lakon
(4) Sebuah studi yang dimuat dalam jurnal Frontiers in atau scenario lenong umumnya mengandung pesan moral,
Psychology menemukan bahwa ketika pasien Fibromialgia seperti membela kaum lemah. (10) Pertunjukan itu biasanya
mendengarkan musik yang mereka sukai selama 10 menit, diadakan di lapangan terbuka tanpa panggung. (11) Ketika
detak jantung mereka menjadi lebih lambat dari 120 denyut pertunjukan tersebut berlangsung, salah seorang aktor atau
per menit dan rasa sakit mereka berkurang. (5) Selain itu, aktris mengitari penonton untuk meminta sumbangan secara
mobilitas para pasien juga meningkat dengan mendengarkan sukarela. (12) Pada saat ini, lenong mulai dipertunjukkan
musik. (6) Selanjutnya, jurnal Ergonomics pada tahun 2013 atas permintaan pelanggan dalam acara-acara di panggung
mempublikasikan sebuah studi yang mununjukkan bahwa hajatan, seperti resepsi pernikahan. (13) Selain kedua seni
musik dapat membantu pikiran seseorang lebih terfokus pertunjukan tersebut, budaya arsitektur dan pertanian Betawi
dalam bekerja dan juga membantu meningkatkan juga mengalami kemunduran. (14) Contohnya rumah-rumah
kemampuan mengemudi kendaraan. (7) Musik yang sesuai asli Betawi di kawasan Condet saat ini terabaikan. (15)
untuk keduanya adalah yang mempunyai irama pelan dan Tambahan lagi, rumah-rumah baru orang Betawi saat ini
tenang. seperti musik pada zaman Barok. (8) Manfaat musik tidak menggunakan gaya arsitektur Betawi.
lainnya, menurut sebuah studi pada jurnal Nature 04. Gagasan pokok bacaan di atas adalah ….
Neuroscience pada tahun 2011, adalah meningkatkan (A) kemunduran kesenian Betawi
performa olahraga dan memerbaiki suasana hati. (B) seni pertunjukan budaya Betawi
01. Pemakaian huruf yang salah pada bacaan di atas adalah (C) pelestarian budaya Betawi
…. (D) kondisi budaya Betawi
(A) mobilitas (E) seni musik tradisional Betawi
(B) Fibromialgia
(C) jurnal Ergonomics 05. Kata tergerus pada kalimat (6) bermakna ….
(A) punah
(D) zaman Barok
(B) hilang
(E) performa
(C) hancur
(D) lenyap
02. Gagasan utama pada kalimat (6) adalah ….
(E) musnah
(A) musik membantu meningkatkan kemampuan
mengemudi
06. Kalimat perbaikan dari kalimat (5) adalah ….
(B) sebuah studi menunjukkan sesuatu (A) Dalam perkembangan selanjutnya, pertunjukkan
(C) jurnal Ergonomics memublikasikan studi ondel-ondel yang juga digunakan untuk
(D) musik membantu pikiran seseorang lebih terfokus menyemarakkan pesta-pesta rakyat, meresmikan
(E) jurnal Ergonomics menunjukkan musik membantu gedung baru, atau menyambut tamu terhormat
pikiran (B) Selanjutnya, pertunjukan ondel-ondel juga
digunakan untuk menyemarakkan pesta-pesta
03. Pernyataan yang tidak sesuai dengan isi bacaan di atas rakyat, peresmian gedung baru, atau menyambut
adalah …. tamu terhormat
(A) musik bermanfaat bagi kesehatan manusia (C) Dalam perkembangan selanjutnya,
(B) musik dapat mengurangi rasa sakit mempertunjukkan ondel-ondel adalah untuk
(C) musik dapat meningkatkan daya konsentrasi menyemarakkan pesta-pesta rakyat, meresmian
(D) musik meningkatkan kemampuan menyetir mobil gedung baru, atau menyambut tamu terhormat
(E) musik dengan irama tenang dapat memengaruhi (D) Dalam perkembangan selanjutnya, pertunjukan
suasana hati ondel-ondel juga digunakan untuk menyemarakkan
174
SSE SIDIKALANG : Jl. A. Yani no 56 & 58 Bimbel Sse Sidikalang bimbelsse
BIMBINGAN BELAJAR
“STAR SCIENCE EDUCATION SIDIKALANG” FOKUS LULUS UTBK SBMPTN
Be a Star With The Star

pesta-pesta rakyat, meresmikan gedung baru, atau (E) (12)


menyambut tamu terhormat
(E) Pertunjukan ondel-ondel juga digunakan untuk 10. Gagasan utama kalimat (2) adalah ….
pesta-pesta rakyat, peresmian gedung baru, atau (A) harganya mahal
menyambut tamu terhormat (B) biaya operasional tidak besar karena areal
07. Kata tersebut pada kalimat (11) merujuk pada…. tangkapan lobster
(A) jenis musik tradisional Betawi yang dipengaruhi (C) biaya operasional nelayan tidak besar
suku Tionghoa (D) areal tangkapan terletak di sekitar bukit karang
(B) seni pertunjukan untuk menyambut tamu-tamu (E) areal tangkapan lobster terletak di tepi pantai
terhormat
(C) gaya arsitektur rumah Betawi yang terdapat di 11. Pernyataan-pernyataan berikut sesuai dengan isi bacaan
Condet di atas, kecuali …
(D) pertunjukan dua boneka besar untuk menolak bala (A) hasil tangkapan lobster berkisar antara 2-7 kg
(E) teater tradisional Betawi dengan gambang kromong sekali melaut
(B) panen lobster terjadi pada saat gelombang laut
08. Simpulan bacaan di atas adalah …. tinggi
(A) kebudayaan Betawi saat ini mengalami (C) nelayan beralih mencari lobster karena hasil
kemunduran tangkapannya lebih banyak daripada ikan
(B) fungsi pertunjukan ondel -ondel mengalami
(D) selain lobster, nelayan dapat juga mencari
perubahan
rajungan dan ubur-ubur di daerah pinggir pantai
(C) musik tradisional Betawi adalah gambang kromong
(E) mencari lobster membutuhkan bahan bakar tidak
(D) orang-orang Betawi tidak menggunakan arsitektur
sebanyak untuk mencari ikan laut
Betawi
(E) seni pertunjukan Betawi mulai mengalami
kepunahan 12. (1) Dalam upaya pencegahan pencemaran udara, hutan
mampu menangkal  polutan gas ataupun butiran padat.
(2) Hasil penelitian menunjukkan bahwa volume udara
Bacaan I
yang mengandung polusi gas zon sebesar 150 ppm gas
(1) Nelayan di pantai selatan Kabupaten Kebumen memilih
ternyata 99% terserap oleh tegukan hutan dalam waktu
mencari lobster selama cuaca buruk melanda perairan itu.
delapan jam. (3) Komplek industri  yang mengeluarkan
(2) Selain harganya mahal, biaya operasional nelayan juga
polutan belerang dioksida di Uni Rusia ternyata
tidak besar karena area tangkapan lobster hanya terletak di
berkurang dengan adanya jalur vegetasi kayu selebar
sekitar bukit-bukit karang tepi pantai. (3) Muslimin (45),
500 m yang mengelilingi kawasan industri tersebut. (4)
nelayan Pantai Pasir, mengatakan bahwa masa panen lobster
Tumbuhan berkayu ataupun pohon memang diandalkan
berlangsung pada saat gelombang laut tinggi menyebabkan
dalam penyelamatan keadaan lingkungan seperti tanah,
nelayan tidak bisa melaut. (4) Dalam sekali tangkapan,
air, dan udara walaupun peran pohon tersebut sebatas
paling sedikit nelayan membawa lobster 2 kg. (5) “Untuk
pada lingkungan, yang belum akut. (5) Pohon memang
sekali melaut, satu perahu bisa mendapat 4-5 kg, tetapi jika
tidak akan mampu menetralisasi polusi, terutama pada
sepi hanya mendapat 2 kg. (6) Bahkan, beberapa perahu bisa
kawasan industri besar. 
mendapat 7 kg," ujarnya.
Kata tidak baku terdapat pada kalimat ….
(7) Dia menuturkan bahwa pendapatan dari menangkap
(A) (1
lobster juga lebih besar daripada ikan laut. (8) Satu kilogram
(B) (2)
bisa terjual Rp250.000,00 hingga Rp450.000,00. (9) Lobster
(C) (3)
merah dihargakan Rp250.000,00- Rp350.000,00 per kg,
(D) (4)
lobster hijau Rp250.000,00-Rp450.000,00 per kg, dan
(E) (5)
lobster mutiara Rp600.000,00- Rp1.000.000,00 per kg.
(10) Eko Wardi (33), nelayan di Pantai Menganti,
13. Penulisan kata serapan yang benar terdapat pada
mengatakan bahwa selain nilai ekonomi lebih tinggi, jarak
kalimat ...
dan waktu tangkap lobster lebih pendek. (11) Biasanya
(A) Sisi interkonektifitas videocard ini masih standar.
untuk melaut, nelayan membutuhkan 20 liter solar, tetapi
(B) Produk ini merupakan modifikasi, terutama pada
untuk mencari lobster, mereka hanya membutuhkan 3-4
penggunaan HFS nonstandar.
liter. (12) Selain lobster, kata Eko, beberapa komoditi laut
(C) Produsen Saphire menghadirkan seri videocard
lainnya yang dapat dicari di daerah pinggir pantai adalah
Toxid yang ekstrim.
rajungan dan ubur-ubur.
(D) Disain papan juga berbeda dari yang biasanya.
09. Penulisan kata yang salah terdapat dalam kalimat ….
(E) PCMAV 4.0 secara automatis akan mendeteksi
(A) (2)
virus yang mengganggu computer anda.
(B) (4)
(C) (6)
(D) (11)
175
SSE SIDIKALANG : Jl. A. Yani no 56 & 58 Bimbel Sse Sidikalang bimbelsse
BIMBINGAN BELAJAR
“STAR SCIENCE EDUCATION SIDIKALANG” FOKUS LULUS UTBK SBMPTN
Be a Star With The Star

14. Kalimat yang tidak mengandung unsur mubazir adalah (E) Kemahirannya dalam berkomunikasi dianggap luar
… biasa sehingga sampai hari ini namanya tidak
(A) Sambil memanfaatkan situasi, para anak-anak tenggelam walaupun tidak menjadi pejabat lagi.
petani cidekat Deniliquin bermain di Kanal
Mulwala. 18. (1) Indonesia mau mengadopsi model yang
(B) Majalah Waktu memasukkan Jakarta ke dalam dikembangkan oleh Amerika Serikat dalam ekonomi
senarai daftar kota yang paling tinggi tingkat digital. (2) Model tersebut bernama plug and play,
pencemarannva. yakni perusahaan yang menyiapkan perkembangan
(C) Salah satu rute dengan pemandangan yang sangat perusahaan rintisan. (3) Pemerintah secara serius
luar biasa sekali di Amerika Serikat adalah jalur menyiapkan inkubator dan permodalan. (4) Hal tersebut
Bay. dapat memudahkan pelaku usaha rintisan. (5) Dengan
(D) Semua peserta lomba balap sepeda itu mendapat berbagai upaya itu, ekosistem ekonomi digital yang
cenderamata. kompetitif akan terbentuk.
(E) Hadiah yang diperebutkan dalam balap sepeda itu Kata beragam nonformal terdapat di dalam kalimat ….
antara lain telepon genggam, computer jinjing, dan (A) (1)
lain-lain. (B) (2)
(C) (3)
15. Jika mengamati rentang waktu tersebut, dikhawatirkan (D) (4)
banyak penumpang tewas ditengah ganasnya (E) (5)
gelombang laut pada malam yang gelap gulita itu.
Kalimat tersebut menjadi baku jika diperbaiki dengan 19. (1) Salah satu tempat wisata terkenal di Indonesia
cara ... adalah Wakatobi. (2) Wakatobi sudah dikenal luas
(A) Mengganti mengamati dengan diamati. hingga mancanegara. (3) Perairan yang berada di bawah
(B) Memindahkan posisi dikhawatirkan ke posisi segitiga terumbu karang ini memiliki macam-macam
sesudah penumpang. terumbu karang dan hayati tertinggi di dunia. (4) Di
(C) Mengganti di tengah dengan di tengah-tengah. sana terdapat lebih dari 112 jenis karang serta 93 jenis
(D) Menambahkan rang sesudah penumpang. ikan. (5) Itulah yang menyebabkan Wakatobi dijuluki
(E) Menghilangkan-nya pada ganasnya. surga wisata.
Kata beragam nonformal dalam paragraf di atas terdapat
16. Menguatnya sikap pragmatisme mengejar kekuasaan dalam kalimat ….
politik dengan mengandalkan kekuatan kapital/finansial (A) (1)
tersebut berimplikasi pada sikap mental rakyat, yaitu (B) (2)
merefleksi tumbuhnya budaya politik transaksional (jual (C) (3)
beli) suara rakyat sebagai pemilik mandat demokrasi (D) (4)
melalui kompetisi pemilu secara reguler. (E) (5)
Ada dua bentukan atau pilihan kata yang salah
padakalimat di atas, yaitu ... . 20. Kalimat yang termasuk baik dan benar adalah....
(A) transaksional dan finansial (A) Orang-orang yang menduduki lapisan atas yang
(B) finansial dan regular banyak memiliki pengaruh besar dalam
(C) berimplikasi dan mental menentukan arah kebijakan politik negara
(D) merefleksi dan pragmatism (B) Kerja pada tingkat praktik sosial menampakkan
(E) transaksional dan kompetisi heterogenitas dan tersedia sistem klasifikasinya
dalam terminologi setempat
17. Mana di antara kata bercetak miring yang tidak (C) Setelah mempersandingkan konsep serta kategori
bermakna kiasan? berdasarkan perspektif keil-muan penelitian ini
(A) Pada masa pemerintahan kolonial Belanda, menemukan suatu tipologi budaya kerja petani
pembesar-pembesar duduk sejajar dengan para (D) Heterogenitas budaya kerja petani tak sepenuhnya
sultan. dikarenakan pertimbangan-pertimbangan ekonomi
(B) Repatriasi TKI korban konflik di Suriah sebagaimana yang diduga oleh sejumlah ahli
membuktikan bahwa negara hadir dalam rangka (E) Pada tingkat arus utama, memperlihatkan
memberikan perlindungan kepada warga negara. kecendrungan bahwa latar belakang posisi ekonomi
(C) Berita tentang pesawat yang jatuh itu sempat memainkan peran yang mempengaruhi budaya
membuat kami waswas karena kerabat kami sedang petani
terbang pada saat yang sama.
(D) Mereka selalu turun bersama seperti pada setiap 21. Penulisan unsur serapan yang tidak sesuai dengan
pesta dansa yang mereka ikuti sejak kelas tujuh. pedoman ejaan terdapat dalam kalimat....

176
SSE SIDIKALANG : Jl. A. Yani no 56 & 58 Bimbel Sse Sidikalang bimbelsse
BIMBINGAN BELAJAR
“STAR SCIENCE EDUCATION SIDIKALANG” FOKUS LULUS UTBK SBMPTN
Be a Star With The Star

(A) duduk di bawah pohon rindang dapat membuat hati (C) Dia tidak mau bertanggung jawab atas
dan pikiran menjadi lebih tenang sehingga stress perbuatannya.
pun hilang (D) Tidak benar membebas tugaskan karyawan tanpa
(B) cara tersebut cukup efektif bila dilakukan oleh alasan.
perempuan yang menderita gangguan kecemasan (E) Ada juga pengusaha non pribumi yang mau
(C) ragam aktivitas di luar ruang berpengaruh besar menjadi orangtua asuh.
terhadap para remaja
(D) untuk mendapatkan sensasi rileks yang optimal, 27. Penggunaan kata daripada yang tidak pada tempatnya
segala macam gawai sebaiknya ditinggalkan terdapat pada kelompok kata, kecuali…
(E) Jurnalis sains, Florence Williams, melaporkan (A) perumahan daripada pegawai negeri
bahwa beristirahat di bawah pohon rindang sangat (B) bebas daripada rasa takut
bermanfaat (C) menanggapi daripada usul ketua
(D) dibuat daripada plastik
22. Insiden pesawat keluar landasan merupakan (E) Deli Serdang lebih luas daripada Medan
kecelakaan yang paling sering terjadi.
Kata yang tepat untuk menggantikan kata insiden dalam 28. Pemisahan bagian-bagian kata yang sesuai dengan
kalimat di atas adalah .... kaidah ejaan kita adalah…
(A) tindakan (A) pel – a – jar ; Ap – ril
(B) kejadian (B) tem – ba – kan ; pu – la
(C) keadaan (C) ko – pra ; tek – nik
(D) kecelakaan (D) peng – em – bang – an ; se – rang – ga
(E) kenyataan (E) film ; pe – ngu – sul – an

29. Penulisan kata bercetak miring yang benar terdapat


23. Meskipun beberapa link yang dituliskan sudah berkali-
pada kalimat…
kali dicermati, informasi yang diperlukannya belum
(A) Para mahasiswa diwajibkan membayar SPP
memadai.
persemester.
Padanan kata yang tepat untuk kata link dalam kalimat
(B) Rumah itu terpaksa di jual karena perlu uang.
di atas adalah ....
(C) Pertandingan basket antarmahasiswa berlangsung
(A) pranala
seru.
(B) saltik
(D) Siapakah diantara Anda yang bersedia membantu
(C) luring
kami?
(D) portofan
(E) Sekalipun kami belum pernah ke sana
(E) warganet
30. Terdapat penulisan kata yang salah dalam kalimat
24. Konfiks yang benar pada gabungan kata berikut
berikut, kecuali…
terdapat pada…
(A) Seseorang dapat dipastikan menderita suatu
(A) pertanggungjawaban
penyakit tertentu setelah melalui hasil analisa
(B) keikut sertaan
pemeriksaan laboratorium.
(C) pengalihan bahasa
(B) Setiap perusahaan akan selalu berusaha untuk
(D) pendayaan guna
meningkatkan produktifitas para pegawainya agar
(E) dibebaskan tugas
perusahaan itu dapat meningkatkan layanan kepada
konsumen secara maksimal.
25. Penulisan kata depan, yang benar terdapat pada
(C) Setelah menyelesaikan program strata satu di
kalimat….
Universitas Udayana, Guntur Saputra pergi ke
(A) Dimana pintu keluar ?
Tokyo untuk mengambil program pasca sarjana di
(B) Lebih baik belajar di dalam negeri dari pada di luar
kota itu.
negeri.
(D) Fotokopi ijazah harus disertakan pada formulir
(C) Surat itu ditujukan ke pada keluarganya.
pendaftaran untuk ke universitas.
(D) Setiap minggu, Pak Ahmad pergi keluar kota.
(E) Hanya akibat dari persoalan yang sangat
(E) Seminar itu bertempat di Lembaga Pengabdian
sederhana, pertikaian – bahkan perkelahian antar
kepada Masyarakat Unair.
wilayah seringkali terjadi pada tahun – tahun
terakhir ini.
26. Penulisan gabungan kata berikut salah, kecuali…
(A) Kita harus pandai mendaya-gunakan segala yang
kita miliki.
(B) Atas perhatian Anda, kami sampaikan terimakasih.

177
SSE SIDIKALANG : Jl. A. Yani no 56 & 58 Bimbel Sse Sidikalang bimbelsse
BIMBINGAN BELAJAR
“STAR SCIENCE EDUCATION SIDIKALANG” FOKUS LULUS UTBK SBMPTN
Be a Star With The Star

(C) Pak Karim Hasam menjadi duta besar di Swedia.


(D) Program Keluarga Berencana menekankan
terbentuknya catur warga.
(E) Kegiaan ekstra kurikuler dapat meningkatkan
kreativitas siswa.

06. Penulisan yang benar terdapat pada kata berikut,


yaitu…
(A) hakikat, izin, konkret
(B) apotik, sistem, analisa
(C) joang, metode, nasehat
(D) khotbah, karir, kaidah
(E) varietas, teoritis, rizki

07. Penulisan gabungan kata yang benar terdapat pada …


(A) intra kurikuler
01. Di antara cara penulisan berikut yang sesuai dengan (B) tataterbib
kaidah ejaan yang berlaku adalah… (C) sumberdaya
(A) mengambing hitamkan (D) tanda tangan
(B) mengambinghitamkan (E) dayabeli
(C) mengambing – hitamkan
(D) meng – ambing hitamkan 08. Penulisan kata maha yang benar adalah…
(E) meng – ambing hitam-kan (A) maha adil
(B) maha tahu
02. Kata serapan yang berasal dari bahasa asing digunakan (C) maha esa
secara tepat dalam kalimat… (D) maha siswa
(A) Para wakil rakyat yang duduk di DPR harus dapat (E) maha guru
menyampaikan inspirasi masyarakat kepada
pemerintah. 09. Penulisan partikel per yang tepat adalah, kecuali…
(B) Tampaknya isi pasal-pasal dalam perundangan itu (A) Pengunjung dikenai tiket masuk Rp 1.500,00 per
tidak efektif untuk menanggulangi kejahatan. kepala.
(C) Kami tidak suka membeli barang-barang yang (B) Harap masuk satu per satu.
kuantitasnya kurang baik. (C) Dua per lima anggota belum hadir.
(D) Kita dapat menjangkau desa terpencil itu dengan (D) Kenaikan gaji pegawai negeri akan di bayar per
menggunakan sarana komunikasi sepeda motor. satu April 2002.
(E) Orang itu dapat memanfaatkan waktu secara (E) Ia menghabiskan seperlima hidupnya di penjara.
efektif sehingga sukses dalam usahanya.
10. Penulisan kata-kata berikut ini yang sudah tepat,
03. Pemisahan kata berikut benar, kecuali … kecuali …
(A) me – ne – lu – sur – i (A) dasawarsa, antikomunis
(B) me – nga – lam – i (B) antarkota, tataniaga
(C) men – de – kat – kan (C) pascasarjana, nonblok
(D) mem – pel – a – jar – i (D) tuna rungu, swadaya
(E) men – de – ri – ta (E) pancasila, inframerah

04. Penulisan gabungan kata berikut benar, kecuali… 11. Penulisan partikel pun yang salah adalah …
(A) tunawicara (A) sekalipun
(B) non aktif (B) meskipun
(C) swalayan (C) maupun
(D) prasejarah (D) biarpun
(E) se-Indonesia (E) apapun

05. Penulisan gabungan kata yang benar terdapat dalam 12. Pemisahan suku kata di bawah ini yang tidak tepat
kalimat… adalah…
(A) Permasalahan tuna wisma itu harus segera (A) me – nu – li – si
ditangani. (B) ke – la – par – an
(B) Perkelahian antar pelajar sering terjadi di kota (C) ber – la – ri – an
besar. (D) di – ka – lah – kan
178
SSE SIDIKALANG : Jl. A. Yani no 56 & 58 Bimbel Sse Sidikalang bimbelsse
BIMBINGAN BELAJAR
“STAR SCIENCE EDUCATION SIDIKALANG” FOKUS LULUS UTBK SBMPTN
Be a Star With The Star

(E) men – de – ngar – kan Kalimat tersebut dapat dijadikan kalimat efektif
dengan…
13. Setiap kata ditulis secara benar dalam kalimat… (A) mengubah menuntut menjadi dituntut
(A) Disamping itu, ia juga diminta membantu (B) meletakkan para pendidik pada awal kalimat
menyelesaikan laporan proyek. (C) menghilangkan kata dengan
(B) Diantara sepuluh orang, hanya tidak orang yang (D) menghilangkan kata telah
berani masuk kedalam arena pertandingan. (E) meletakkan dengan perubahan zaman pada akhir
(C) Kerjasama diantara mereka sudah di antisipasi oleh kalimat
kelompok lawannya.
(D) Ia tidak pernah lupa bersyukur ke hadirat Tuhan 19. Presiden membicarakan tentang serangan Amerika
Yang Mahakuasa. terhadap Afganistan dalam sidang kabinet kemarin.
(E) Dari duapuluh sembilan korban, lima diantaranya Kalimat tersebut akan menjadi kalimat yang baku jika
adalah anak pejabat. diperbaiki menjadi…
(A) Presiden membicarakan tentang serangan Amerika
14. Dalam rapat itu membicarakan masalah yang terhadap Afganistan dalam sidang kabinet kemarin.
berhubungan dengan kesejahteraan anggota. (B) Presiden membicarakan serangan tentang Amerika
Kalimat di atas dapat dijadikan kalimat baku dengan… terhadap Afganistan dalam sidang kabinet kemarin.
(A) menghilangkan kata yang (C) Dalam sidang kabinet, presiden membicarakan
(B) mengubah bentuk kata membicarakan menjadi tentang serangan Amerika terhadap Afganistan.
dibicarakan (D) Presiden membicarakan serangan Amerika
(C) menghilangkan kata anggota terhadap Afganistan kemarin di depan sidang
(D) meletakkan dalam rapat pada akhir kalimat kabinet.
(E) meletakkan klausa masalah yang berhubungan (E) Presiden membicarakan tentang serangan Amerika
dengan kesejahteraan anggota ke akhir kalimat terhadap Afganistan kemarin dalam sidang kabinet.
15. Kalimat berikut ini merupakan contoh kalimat baku, 20. Wawasan Nusantara tidak hanya bertujuan untuk
kecuali…
mewujudkan kesejahteraan bagi bangsa Indonesia saja,
(A) Kita harus dapat membuktikan bahwa kita mampu
tetapi juga ikut serta dalam mewujudkan kebahagiaan
melaksanakan tugas ini.
(B) Marilah kita memulai pertemuan ini! dalam umat manusia.
(C) Dalam pertemuan ini kita akan membicarakan Penggunaan kata di atas dapat dihemat dengan
rencana perjalanan wisata ke Bandung. menghilangkan kata…
(D) Akan kita diskusikan lagi masalah ini besok! (A) untuk dan saja
(E) Kita akan bicarakan lagi masalah ini. (B) untuk dan tetapi
(C) untuk, saja dan tetapi
16. Semua kalimat berikut adalah kalimat yang tidak (D) untuk, saja, dan bagi
menggunakan ragam baku, kecuali… (E) untuk, tetapi, dan bagi
(A) Dalam rapat itu membicarakan masalah hasil
evaluasi belajar mahasiswa. 21. Kalimat berikut yang termasuk kalimat baku adalah…
(B) Buku itu bermanfaat bagi penulisan karya ilmiah. (A) Ia menceritakan tentang peristiwa kepada teman-
(C) Pada bahasa Indonesia yang baik dan benar temannya.
membicarakan ejaan dan ragam. (B) Surat itu memberitakan mengenai keadaan
(D) Bagi pegawai negeri sipil tidak boleh ikut partai. neneknya yang sedang sakit.
(E) Rencana undang-undang disyahkan oleh DPR. (C) Kita harus saling percaya akan kemampuan orang
lain, tidak boleh saling curiga.
17. Pembangunan pertanian tidak hanya membatasi pada (D) Kita harus dapat melawan akan hawa nafsu yang
pengembangan tanaman tradisional. dapat menjerumuskan ke lembah kehinaan.
Kalimat di atas dianggap tidak efektif karena (E) Demi untuk mempertahankan semangat korps, kita
penggunaan kata yang salah. Kata yang salah itu tidak boleh saling menghujat.
adalah…
(A) pembangunan 22. Kalimat berikut ini yang termasuk kalimat baku
(B) tidak adalah…
(C) membatasi (A) Menurut Yeo bahwa birokrasi di Indonesia terlalu
(D) tanaman rumit sehingga menimbulkan banyak masalah bagi
(E) tradisional investor yang mau bergabung.
(B) Pihak biro perjalanan mengeluhkan, tingginya
18. Dengan perubahan zaman telah menuntut para kenaikan harga BBM berpengaruh besar terhadap
pendidik untuk mencari metode mengajar yang baru. minat wisatawan untuk melakukan kunjungan.

179
SSE SIDIKALANG : Jl. A. Yani no 56 & 58 Bimbel Sse Sidikalang bimbelsse
BIMBINGAN BELAJAR
“STAR SCIENCE EDUCATION SIDIKALANG” FOKUS LULUS UTBK SBMPTN
Be a Star With The Star

(C) Berdasarkan hasil survei menunjukkan bahwa (A) Jika berpidato, ia senantiasa selalu
sampai saat ini wisata pantai masih tetap diminati, memperhitungkan waktu yang tersedia.
terutama oleh kaum muda. (B) Menjaga lingkungan agar tetap sehat adalah
(D) Menurut pengamat lingkungan, krisis air bersih dan merupakan tanggung jawab seluruh warga.
masalah udara segar mulai merisaukan penduduk di (C) Hal-hal yang yang telah dikemukakan itu memang
kota-kota besar. merupakan kendala bagi kita semua.
(E) Dengan adanya bencana alam yang menelan korban (D) Segala peraturan-peraturan yang lebih rendah tidak
ratusan ribu nyawa manusia menggugah nurani dan boleh bertentangan dengan peraturan yang lebih
sikap kedermawanan semua orang. tinggi kedudukannya.
(E) Kedua masalah itu saling berkaitan.
23. Jika bekerja di stasiun radio, Anda penyiar radio.
Kalimat di atas salah nalar dan dapat diperbaiki 28. Dalam berwiraswasta memerlukan kreativitas untuk
menjadi… menyiasati apa yang ada dan melahirkan peluang
(A) Anda bekerja di stasiun radio sehingga menjadi bisnis.
penyiar radio. Kalimat tersebut akan menjadi benar apabila…
(B) Bekerjalah di stasiun radio supaya anda menjadi (A) kata memerlukan diubah menjadi diperlukan
penyiar radio. (B) Kata dalam diganti dengan kata dengan
(C) Jika Anda penyiar radio, Anda bekerja di stasiun (C) kata kreativitas diubah menjadi kreasi
radio. (D) kata kreativitas diubah menjadi kreatifas
(D) Anda bekerja di stasiun radio, maka Anda penyiar (E) kata menyiasati diubah menjadi mensiasati
radio.
(E) Oleh karena bekerja di stasiun radio, anda penyiar 29. Yang merupakan kalimat tidak efektif adalah
radio. (A) Masing-masing mahasiswa harus membayar iuran
lima ribu rupiah.
24. Penulisan kata serapan yang benar terdapat pada (B) Tiap-tiap dermawan mendapat tanda penghargaan
kalimat … dari yayasan itu.
(A) Soal itu direfisi di Jakarta bersama guru-guru (C) Mereka masing-masing menunjukkan
daerah. kebolehannya bersilat.
(B) Pengambilan raport dilakukan oleh orang tua atau (D) Setiap hari kujumpai perempuan tua itu berdiri di
wali murid. pintu keluar pintu stasiun.
(C) Lima tahun terakhir ini, industri tekstil mengalami (E) Masing-masing membawa hadiah kecil untuk
kelesuan. nyonya kecil.
(D) Bila tidak ditemukan sumber energi alternative,
dunia mengalami krisis energi. 30. Semua kata di bawah ini yang tidak termasuk kata
(E) Pemerintah diharapkan dapat menetapkan baku, kecuali…
kebijakan eksport dan import. (A) komplex
(B) risiko
25. Ragam baku terdapat dalam kalimat… (C) standard
(A) Kenaikan harga barang-barang karena kenaikan (D) atlit
biaya produksi. (E) karir
(B) Dalam rapat itu membicarakan kenaikan SPP.
(C) Meninggalnya orang itu sebab kecelakaan.
(D) Bagi saya, soal itu sangat mudah.
(E) Dengan cara ini akan memudahkan komunikasi
antaranggota.

26. Kalimat berikut merupakan contoh kalimat baku,


kecuali…
(A) Kita harus dapat membuktikan bahwa kita mampu
melaksanakan tugas ini.
(B) Marilah kita memulai pertemuan ini.
(C) Dalam pertemuan ini kita akan membicarakan
rencana perjalanan ke Bandung.
(D) Akan kita diskusikan lagi masalah ini besok.
(E) Kita akan bicarakan lagi masalah ini besok.

27. Contoh kalimat baku adalah…

180
SSE SIDIKALANG : Jl. A. Yani no 56 & 58 Bimbel Sse Sidikalang bimbelsse
BIMBINGAN BELAJAR
“STAR SCIENCE EDUCATION SIDIKALANG” FOKUS LULUS UTBK SBMPTN
Be a Star With The Star

(C) 3
(D) 4
(E) 5

05. Kalimat berikut ini yang mengikuti kaidah yang benar


adalah…
(A) Di Indonesia sekarang ini sedang giat mengadakan
pelatihan untuk peningkatan kualitas pembelajaran.
(B) Berdasarkan uraian di atas menunjukkan
pentingnya pendidikan emosi bagi orang dewasa.
(C) Dengan penelitian ini dapat meningkatkan kualitas
pembelajaran di sekolah terpencil.
(D) Di Kalimantan terdapat binatang langka yang
kehidupannya nyaris punah karena kebakaran.
(E) Dengan menggunakan pupuk tablet dapat
menghemat biaya tanam mencapai 50 %.
01. Kalimat di bawah ini yang tidak termasuk kalimat
baku, kecuali… 06. Yang bukan kalimat baku adalah…
(A) Cuti besar diberikan kepada yang telah bekerja (A) Berdoa pada dasarnya untuk berbicara atau
selama tiga tahun. berdialog dengan Tuhannya, yakni Sang Maha
(B) Sudah selayaknya kita harus saling tolong- Pencipta.
menolong. (B) Dikatakannya bahwa mistik sebagai tahapan yang
(C) Kepada saudara-saudara sekalian diminta untuk sangat mendalam dalam kehidupan manusia.
berdiri. (C) Dalam hal ini, terutama berkaitan dengan relasi
(D) Yang telah kami selesaikan adalah masalah tentang manusia dengan sesamanya dan dengan Tuhannya.
masalah narkoba. (D) Relasi yang sedemikian rupa untuk dapat saling
(E) Mereka mendiskusikan tentang bahaya banjir. memahami sehingga setiap orang dapat saling
menyalahkan.
02. Kalimat di bawah ini yang termasuk kalimat efektif (E) Karena itu, perlu mengubah cara hidup dan
adalah… pandangan hidup orang sehingga mereka dapat
(A) Dalam pertemuan itu membicarakan kehadiran berpikir maju.
orang asing di lingkungan kita.
(B) Kenapa mereka tidak peduli dengan keadaan anak- 07. Dalam perjalanan menuju lokasi Festival Seni di
anaknya. Jepang. Saya melihat tulisan besar. Tulisan ini terus
(C) Kami harus menyerahkan penjahat itu kepada terang menggelitik imajinatif dan pikiran saya. Saya
polisi. bertanya pada teman apa artinya “Osama”. Ternyata
(D) Kepada hadirin dipersilahkan duduk kembali. berarti raja. Jepang memang mempunyai beberapa
(E) Mereka sudah laporkan hal itu kepada polisi. obyek wisata menarik.
Perbaikan kata-kata tidak baku dalam paragraf tersebut
03. Kalimat yang baku di bawah ini adalah… adalah …
(A) Untuk masalah keluarga berencana adalah masalah (A) festival, imaji, obyek
kita. (B) festival, imajinasi, objek.
(B) Anaknya Pak Marto yang kedua lahir lewat operasi. (C) pestipal, imaji, objek.
(C) Beberapa ibu-ibu dipaksa bekerja melampaui batas. (D) festival, imajinasi, obyek.
(D) Ia anak yang baik karena selalu mengacuhkan (E) festifal, imajinasi, objek.
orang tua.
(E) Pada pertemuan itu membahas masalah UN. 08. Karena keluarga menjadi korban bencana gempa dan
tsunami, maka mereka menjadi sangat menderita.
04. (1) Rumah adik saya jauh sekali dari Kalimat tersebut termasuk dalam kalimat tidak baku
keramaian. seperti kalimat-kalimat berikut, kecuali…
(2) Bagi yang tidak berkepentingan dilarang masuk. (A) Walaupun sumbangan datang dari berbagai negara,
(3) Banyak anak kecil bermain di jalan raya. tetapi rakyat Aceh masih tetap menderita.
(4) Ia telah diperingatkan dokter agar tidak merokok. (B) Untuk keperluan biaya pembangunan Aceh,
(5) Pendidikan agama perlu diberikan kepada siswa memerlukan bantuan dari berbagai pihak.
SLB. (C) Dalam rapat tersebut membahas tentang bagaimana
Kalimat yang tidak baku terdapat pada nomor… membangun Aceh kembali.
(A) 1
(B) 2
181
SSE SIDIKALANG : Jl. A. Yani no 56 & 58 Bimbel Sse Sidikalang bimbelsse
BIMBINGAN BELAJAR
“STAR SCIENCE EDUCATION SIDIKALANG” FOKUS LULUS UTBK SBMPTN
Be a Star With The Star

(D) Kami sangat berduka melihat anak-anak menjadi 14. Semenjak suaminya berangkat ke medan perang, tak
korban bencana itu, jika diizinkan kami ingin pernah tentram hatinya. Jabatan anak kalimat tersebut
menjadi orang tua asuh. adalah…
(E) Karena sudah banyak orang yang memberikan (A) K – P – S
bantuan, sehingga kini mereka tidak perlu gelisah (B) S – P
dan merasa takut lagi. (C) S – P – K
(D) P – S
09. Kalimat di bawah ini yang menunjukkan relasi implisit (E) S – P – O – K
adalah…
(A) Ketika mereka datang, semuanya sudah lengkap. 15. Kalimat majemuk bertingkat perluasan subjek terdapat
(B) Suasana semakin ramai saat penyanyi itu tampil di pada…
atas panggung (A) Walaupun hujan turun, ia tetap datang.
(C) Kakak bermain di halaman ; adik membaca di (B) Bahwa kamu sangat cerdas, sudah diketahui semua
kamar orang.
(D) Mereka pergi ke ladang ketika cuaca sangat cerah. (C) Rumah itu dibeli ayah setelah pulang dari luar
(E) Salomo bandel, tetapi ia cerdas seperti Einstein. negeri.
(D) Polisi menangkap orang yang mencuri ayam itu.
10. (1) Sapu tangan orang itu putih. (E) Sewaktu polisi datang, para penjudi berlarian
(2) Sapu tangan orang itu lebar. menyembunyikan diri.
Gabungan kedua kalimat di atas yang tepat adalah…
(A) Orang yang bersapu tangan itu lebar. 16. Saat ia duduk di teras, ia melihat tukang bakso lewat.
(B) Sapu tangan orang itu putih dan lebar. Kalimat di atas memiliki anak kalimat berpola…
(C) Sapu tangan itu putih, orang itu lebar. (A) S – P
(D) Putih dan lebar bersapu tangan itu. (B) S – P – O
(E) Lebar dan putih orang bersapu tangan itu. (C) K – S – P – O
(D) S – K
11. Kalimat di bawah ini yang termasuk kalimat majemuk (E) S – P – K
koordinatif adalah…
(A) Saat mereka tiba, kami menyambutnya dengan O1
gembira.
(B) Rumah yang dibeli ayah sangat besar. 17. Kalimat yang berpola S – P – SPO 2 adalah…
(C) Saya mengetahui bahwa mereka mencintainya. (A) Ketika itu hari amat panas, namun kami tak perduli.
(D) Paman baru tiba dari Jakarta. (B) Boy terkenal bandal, tetapi ia pandai.
(E) Ibu pergi ke pasar sedangkan ayah pergi ke kantor. (C) Adik menangis karena terjatuh dari pohon mangga.
(D) Anton membaca buku yang dibeli kakek.
12. Semua penumpang tertidur ketika bus melaju kencang (E) Erwin akan pulang ke Indonesia setelah ia lulus
di jalan tol itu. ujian.
Pola anak kalimat yang terdapat pada kalimat di atas
adalah… 18. Bu Karta memilih kain itu dan Pak Karta
(A) S – P – O membayarnya.
(B) S – P – K Pola kalimat majemuk tersebut sama dengan pola
(C) S – P – O – K kalimat majemuk berikut, kecuali ...
(D) P – S – K A. Adik tidur saja di sini atau adik ikut pulang juga.
(E) S – P – O – K B. Hasan tidak mau belajar serius, melainkan
bermain-main sepanjang hari.
13. Kalimat di bawah ini yang bukan kalimat majemuk C. Para mahasiswa aktif mengikuti, hanya Andi yang
adalah… pasif.
(A) Tugas itu akan kukerjakan setelah saya selesai D. Kakak makan di rumah atau kakak makan di
makan. warung.
(B) Harga barang-barang pokok naik karena biaya E. Miryati dan kepala regu penyiar mengetahui hal itu.
transportasi naik.
(C) Narapidana yang melarikan diri dari penjara belum 19. Kalimat majemuk bertingkat dengan anak kalimat
tertangkap. yang berpola S – P adalah …
(D) Sesudah semua perlengkapan kami periksa, kami A. Rusli belum bangun meskipun hari sudah terang.
pun berangkat. B. Gadis yang memetik bunga itu sudah pergi.
(E) Kepada Anto kakak berjanji membeli boneka. C. Meskipun ia belum membaca surat itu, ia sudah
tahu apa isinya.

182
SSE SIDIKALANG : Jl. A. Yani no 56 & 58 Bimbel Sse Sidikalang bimbelsse
BIMBINGAN BELAJAR
“STAR SCIENCE EDUCATION SIDIKALANG” FOKUS LULUS UTBK SBMPTN
Be a Star With The Star

D. Ketika ayah pulang dari kantor, adik sedang B. Kamu ikut saya atau ikut mereka.
menangis. C. Walaupun dia kaya, dia kikir.
E. Karena permintaannya tidak dikabulkan ibunya, ia D. Ayah mengatakan bahwa kau akan datang.
marah – marah. E. Kau boleh berangkat bila hujan berhenti.

20. (1) Serangga memfosil di dalam batu. 24. Kalimat majemuk di bawah ini yang menggunakan
(2) Serangga itu diperkirakan telah ada relasi implisit adalah…
55 juta tahun sebelum munculnya dinosaurus. A. Adik cantik, tetapi dia suka menangis.
(3) Serangga itu ditemukan pada B. Sewaktu kakak makan, kami pergi.
sebuah tambang batu bara di Ohio. C. Shania tidak pergi, tetapi dia tidur.
Rangkaian yang tepat dari ketiga kalimat di atas D. Saat Dina tidur, ibu memasak di dapur.
adalah… E. Adik membaca buku, kakak menulis surat.
A. Serangga yang ditemukan pada sebuah tambang
batu bara di Ohio itu diperkirakan telah ada 55 juta 25. Saya terharu ketika Elton Jhon menyanyikan lagu
tahun sebelum munculnya dinosaurus dan kini “candle in the wind” yang dipersembahkan kepada
memfosil di dalam batu. Princes of Wales. Kalimat di atas termasuk kalimat
B. Serangga itu diperkirakan telah ada 55 juta tahun majemuk koordinatif
sebelum munculnya dinosaurus yang ditemukan SEBAB
pada sebuah tambang batu bara di Ohio yang Salah satu unsur kalimat di atas diperluas
memfosil di dalam batu.
C. Serangga yang memfosil di dalam batu 26. Mereka belajar dengan tekun ketika saya datang.
diperkirakan telah ada 55 juta tahun sebelum Pola induk kalimat dan anak kalimat di atas adalah...
munculnya dinosaurus dan ditemukan pada sebuah (1) Induk Kalimat : S-P-K
tambang batu bara di Ohio. (2) Anak Kalimat : S-P-K
D. Serangga yang memfosil di dalam batu ditemukan (3) Anak Kalimat : S-P
pada sebuah tambang batu bara di Ohio dan (4) Induk Kalimat : S-P
diperkirakan telah ada 55 juta tahun sebelum
munculnya dinosaurus. O3
E. Serangga yang memfosil di dalam batu itu S − P − O1 −
diperkirakan telah ada 55 juta tahun sebelum 27. Kalimat yang berpola S −P −K adalah …
munculnya dinosaurus yang ditemukan pada (1) Paman memberikan saya hadiah sepatu yang baru.
sebuah tambang batu bara di Ohio. (2) Anton meminjamkan buku kepada Aminah kawan
sekelasnya.
21. Ketika guru itu datang, murid - murid ternyata belum (3) Pedagang itu memberikan diskon kepada pembeli
lengkap. yang sudah lama menjadi langganannya
Kalimat di atas tergolong… (4) Kakak membelikan boneka untuk adik yang
A. Kalimat sederhana. menangis di kamar.
B. Kalimat luas setara.
C. Kalimat luas tak setara bersubjek sama. 28. Kalimat-kalimat di bawah ini merupakan kalimat
D. Kalimat luas tak setara dengan klausa bawahan nominal, kecuali…
pengganti keterangan. (A) Rekomendasi itu lahir karena timbulnya
E. Kalimat luas tak setara dengan klausa bawahan kekhawatiran akan kurangnya rasa cinta pelajar
pengganti objek. akan kebudayaannya sendiri.
(B) HPBI adalah wadah yang menampung kegiatan
22. Paket pertama mengambil tema puasa dan krisis para guru dan ahli bahasa.
moral. (C) Urbanisasi adalah masalah sosial yang tidak mudah
Paket kedua mengambil tema prospek ekonomi rakyat. dipecahkan.
Kedua kalimat di atas dapat dijadikan kalimat majemuk (D) Roket itulah kendaraan satu-satunya yang dapat
setara mempertentangkan dengan konjungsi… mencapai bulan.
A. dan (E) Ruang tempat belajar kami tidak luas.
B. bahkan
C. padahal 29. Semua kalimat berikut adalah kalimat aktif transitif,
D. sedangkan kecuali…
E. Lalu (A) Adik membaca buku di kamar.
(B) Polisi menembak penjahat tersebut.
(C) Ayah mengisap rokok gudang garam.
23. Kalimat di bawah ini yang termasuk kalimat majemuk
(D) Wanita yang memakai baju putih itu berbelanja di
setara mempertentangkan adalah…
pasar.
A. Dia ini bukan adikku, melainkan dia itu sahabatku.
183
SSE SIDIKALANG : Jl. A. Yani no 56 & 58 Bimbel Sse Sidikalang bimbelsse
BIMBINGAN BELAJAR
“STAR SCIENCE EDUCATION SIDIKALANG” FOKUS LULUS UTBK SBMPTN
Be a Star With The Star

(E) Perjalanan itu membutuhkan waktu yang cukup


lama.

30. Saya akan menyelesaikan tugas-tugas itu nanti malam.


Kalimat pasif yang tepat dari kalimat aktif di atas
adalah…
(A) Tugas itu akan diselesaikan oleh saya nanti malam.
(B) Tugas itu saya akan selesaikan nanti malam.
(C) Tugas itu akan saya selesaikan nanti malam.
(D) Nanti malam tugas itu akan diselesaikan oleh saya.
(E) Oleh saya tugas itu akan diselesaikan nanti malam.

184
SSE SIDIKALANG : Jl. A. Yani no 56 & 58 Bimbel Sse Sidikalang bimbelsse

Anda mungkin juga menyukai